Download as pdf or txt
Download as pdf or txt
You are on page 1of 554

See discussions, stats, and author profiles for this publication at: https://www.researchgate.

net/publication/329474187

Mathematical Inequalities Volume 5 Other Recent Methods for Creating and Solving
Inequalities

Book · December 2018

CITATIONS READS

0 512

1 author:

VASILE CIRTOAJE
Petroleum - Gas University of Ploiesti
41 PUBLICATIONS   107 CITATIONS   

SEE PROFILE

All content following this page was uploaded by VASILE CIRTOAJE on 28 January 2020.

The user has requested enhancement of the downloaded file.



     
      
  !
      "  #"   #  $ 
       ""  #         #     !
   !  #"      % &
# ' #       (  #   
)        *  #   ) #   $ +  
          #      
    % !       ,   $    !
   -  . / 01          2  3   #
   /   
             4
"                  
56'  1      0     +
 "     5          "
 $             !    %      
               #    
 $
 # "                " " 7   
         $


 

    7  '  



  
8"  +     "
 9  # '     $5   

   #! !    
   !       
      
  "    $  





 




 

 




 

 
 
   
 



 !"#


  

 
  
 


   
  
   
 
   
 



   
 


  
  
 
   
  

 


  
   


 
       
  
 

 
  
 

   

  

  
 
  
 
    

 


 
 
 


 

 !"#$
 
 
 
%
 " !
&

 

 ' &   
( 
)*!
 &

"
" *)+,-!




  


  

 ./ 
 

  . 0,)1 %


  "  !
 &

 

 
' &   ( 
Contents

1 Arithmetic Mean Method and Arithmetic Compensation Method 1


1.1 Theoretical Basis . . . . . . . . . . . . . . . . . . . . . . . . . . . . . . . . 1
1.2 Applications . . . . . . . . . . . . . . . . . . . . . . . . . . . . . . . . . . . 9
1.3 Solutions . . . . . . . . . . . . . . . . . . . . . . . . . . . . . . . . . . . . . 17

2 pqr Method 99
2.1 Theoretical Basis . . . . . . . . . . . . . . . . . . . . . . . . . . . . . . . . 99
2.2 Applications . . . . . . . . . . . . . . . . . . . . . . . . . . . . . . . . . . . 103
2.3 Solutions . . . . . . . . . . . . . . . . . . . . . . . . . . . . . . . . . . . . . 109

3 Highest Coefficient Cancellation Method for Symmetric Homogeneous


Inequalities in Real Variables 185
3.1 Theoretical Basis . . . . . . . . . . . . . . . . . . . . . . . . . . . . . . . . 185
3.2 Applications . . . . . . . . . . . . . . . . . . . . . . . . . . . . . . . . . . . 201
3.3 Solutions . . . . . . . . . . . . . . . . . . . . . . . . . . . . . . . . . . . . . 213

4 Highest Coefficient Cancellation Method for Symmetric Homogeneous


Inequalities in Nonnegative Variables 359
4.1 Theoretical Basis . . . . . . . . . . . . . . . . . . . . . . . . . . . . . . . . 359
4.2 Applications . . . . . . . . . . . . . . . . . . . . . . . . . . . . . . . . . . . 371
4.3 Solutions . . . . . . . . . . . . . . . . . . . . . . . . . . . . . . . . . . . . . 379

5 On Popoviciu’s Inequality 499


5.1 Theoretical Basis . . . . . . . . . . . . . . . . . . . . . . . . . . . . . . . . 499
5.2 Applications . . . . . . . . . . . . . . . . . . . . . . . . . . . . . . . . . . . 503
5.3 Solutions . . . . . . . . . . . . . . . . . . . . . . . . . . . . . . . . . . . . . 507

A Glosar 523

B Index 537

i
ii Vasile Cîrtoaje
Chapter 1

Arithmetic Mean Method and


Arithmetic Compensation Method

1.1 Theoretical Basis


The Arithmetic Mean - Theorem (AM-Theorem) is useful to prove some symmetric
inequalities of n real variables a1 , a2 , . . . , an , where the equality occurs when n or
n − 1 variables are equal.
AM-THEOREM. Let

F (a1 , a2 , . . . , an ) :  → ,  ∈ n ,

be a symmetric continuous function satisfying


a + a a1 + an 
1 n
F (a1 , a2 , . . . , an−1 , an ) ≥ F , a2 , . . . , an−1 ,
2 2
for all a1 , a2 , . . . , an ∈  such that a1 ≤ a2 ≤ · · · ≤ an or a1 ≥ a2 ≥ · · · ≥ an . Then,
for all a1 , a2 , . . . , an ∈ , the following inequality holds:
a1 + a2 + · · · + an
F (a1 , a2 , . . . , an ) ≥ F (a, a, . . . , a), a= .
n

Proof. Let
A j = (a j1 , a j2 , . . . , a jn ), j = 0, 1, 2, ... ,
where each A j is constructed from the preceding A j−1 by replacing its smallest and
largest elements with their arithmetic mean, and

A0 = (a01 , a02 , . . . , a0n ) = (a1 , a2 , . . . , an ).

By hypothesis, we have

F (a1 , a2 , . . . , an ) ≥ F (a11 , a12 , . . . , a1n ) ≥ F (a21 , a22 , . . . , a2n ) ≥ · · · ,

1
2 Vasile Cîrtoaje

and by Lemma below it follows that

F (a1 , a2 , . . . , an ) ≥ lim F (a j1 , a j2 , . . . , a jn ) = F (a, a, . . . , a).


j→∞

Lemma. Let
A0 , A1 , A2 , . . .
be an infinite sequence of n-tuples A j = (a j1 , a j2 , . . . , a jn ) ∈ n , where each A j is
constructed from the preceding A j−1 by replacing its smallest and largest elements with
their arithmetic mean. Then

a j1 + a j2 + · · · + a jn = a01 + a02 + · · · + a0n

and
a01 + a02 + · · · + a0n
lim A j = (a, a, . . . , a), a= .
j→∞ n
Proof. For any n-tuple A j , define the closed interval  j = [a j , b j ], where

a j = min{a j1 , a j2 , . . . , a jn }, b j = max{a j1 , a j2 , . . . , ain },

and denote by | j | the length of the closed interval  j :

| j | = b j − a j .

Clearly, we have

0 ⊇ 1 ⊇ 2 ⊇ · · · , |0 | ≥ |1 | ≥ |2 | ≥ · · · .

We infer that for any integer j there exists an integer k j such that

n 2
kj ≤ , | j+k j | ≤ | j |.
2 3
Under this assumption, we have

lim | j | = 0,
j→∞

therefore
a01 + a02 + · · · + a0n
lim A j = (a, a, . . . , a), a= .
j→∞ n
To end the proof, let
     
2a j + b j 2a j + b j a j + 2b j a j + 2b j
j = aj, , j = , , j = , bj .
3 3 3 3

Consider that A j has k j1 elements in  j , k j2 elements in  j and n−k j1 −k j2 elements


in  j . Let
k j = min{k j1 , k j2 }.
Arithmetic Mean Method and Arithmetic Compensation Method 3

If c1 ∈  j and c2 ∈  j , then

c1 + c2 a j + (a j + 2b j )/3 2a j + b j
> = ,
2 2 3
c1 + c2 (2a j + b j )/3 + b j a j + 2b j
< = ,
2 2 3
therefore
c 1 + c2
∈ j.
2
As a consequence, A j+k j has k j1 − k j elements in  j , k j2 − k j elements in  j , and the
other elements in  j . Since

(k j1 − k j )(k j2 − k j ) = 0,

all elements of A j+k j belong to


 
a j + 2b j
j ∪ j = aj,
3
or  
2a j + b j
j ∪ j = , bj .
3
Because
2(b j − a j ) 2
| j ∪  j | = | j ∪  j | = = | j |,
3 3
it follows that
2
| j+k j | ≤
| j |.
3
This Lemma is a known result (see, for example, problem 2389 in Crux Mathe-
maticorum, 1999, page 171 and page 520).
Assuming that an = min{a1 , a2 , . . . , an } or an = max{a1 , a2 , . . . , an } and fixing it,
we get
AM-Corollary (Vasile Cîrtoaje, 2005). Let

F (a1 , a2 , . . . , an ) :  → ,  ∈ n ,

be a symmetric continuous function satisfying


a + a a1 + an−1 
1 n−1
F (a1 , a2 , . . . , an−2 , an−1 , an ) ≥ F , a2 , . . . , an−2 , , an
2 2
for all a1 , a2 , . . . , an ∈  such that a1 ≤ a2 ≤ · · · ≤ an (or a1 ≥ a2 ≥ · · · ≥ an ). Then,
for all a1 , a2 , . . . , an ∈  such that a1 ≤ a2 ≤ · · · ≤ an (or a1 ≥ a2 ≥ · · · ≥ an ), the
following inequality holds:
a1 + a2 + · · · + an−1
F (a1 , a2 , . . . , an−1 , an ) ≥ F (t, t, . . . , t, an ), t= .
n−1
4 Vasile Cîrtoaje

From the AM-Theorem and the AM-Corollary, we get the AM1-Theorem and
the AM1-Corollary, respectively.

AM1-THEOREM (Vasile Cîrtoaje, 2005). Let

G(a1 , a2 , . . . , an ) :  → ,  ∈ +n ,

be a symmetric continuous function satisfying




a2 + a2
a2 + a2
1 n 1 n
G(a1 , a2 , . . . , an−1 , an ) ≥ G , a2 , . . . , an−1 ,
2 2

for a1 , a2 , . . . , an ∈  such that 0 ≤ a1 ≤ a2 ≤ · · · ≤ an . Then, for a1 , a2 , . . . , an ∈ ,


the following inequality holds:


a2 + a2 + · · · + a2
1 2 n
G(a1 , a2 , . . . , an ) ≥ F (t, t, . . . , t), t= .
n

Proof. Let
b1 = a12 , b2 = a22 , . . . , bn = an2
and 
F (b1 , b2 , . . . , bn ) = G b1 , b2 , . . . , bn .
Since
a12 + a22 + · · · + an2 = b1 + b2 + · · · + bn ,

G(a1 , a2 , . . . , an ) = G b1 , b2 , . . . , bn = F (b1 , b2 , . . . , bn ),

  

a2 + a2
a2 + a2
b + b
b + b
1 n 1 n 1 n 1 n
G , a2 , . . . , an−1 , =G , b2 , . . . , bn−1 ,
2 2 2 2
 
b1 + b n b1 + b n
=F , b2 , . . . , bn−1 , ,
2 n
the AM1-Theorem follows immediately from the AM-Theorem.

AM1-Corollary (Vasile Cîrtoaje, 2005). Let

G(a1 , a2 , . . . , an ) :  → ,  ∈ +n ,

be a symmetric continuous function satisfying




a2 + a2
a2 + a2
1 n−1 1 n−1
G(a1 , a2 , . . . , an−2 , an−1 , an ) ≥ G , a2 , . . . , an−2 , , an
2 2
Arithmetic Mean Method and Arithmetic Compensation Method 5

for all a1 , a2 , . . . , an ∈  such that a1 ≤ a2 ≤ · · · ≤ an (or a1 ≥ a2 ≥ · · · ≥ an ). For all


a1 , a2 , . . . , an ∈  such that a1 ≤ a2 ≤ · · · ≤ an (or a1 ≥ a2 ≥ · · · ≥ an ), the following
inequality holds:


a2 + a2 + · · · + a2
1 2 n−1
G(a1 , a2 , . . . , an−1 , an ) ≥ F (t, t, . . . , t, an ), t= .
n−1

***

The Arithmetic Compensation - Theorem (AC-Theorem) is a power tool for solving


some symmetric inequalities of the form F (a1 , a2 , . . . , an ) ≥ 0, where a1 , a2 , . . . , an
are nonnegative real variables satisfying a1 + a2 + · · · + an = s, s > 0. Notice that
the AC-method can be applied especially to those inequalities where the equality
occurs when n − k variables are zero and the other k are equal to s/k, where k ∈
{1, 2, . . . , n}.
AC-THEOREM (Vasile Cîrtoaje, 2005). Let s > 0 and let F be a symmetric continuous
function on the compact set in n

S = {(a1 , a2 , . . . , an ) : a1 + a2 + · · · + an = s, ai ≥ 0, i = 1, 2, . . . , n}.

If
F (a1 , a2 , a3 , . . . , an ) ≥
 a + a a + a  
1 2 1 2
≥ min F , , a3 , . . . , an , F (0, a1 + a2 , a3 , . . . , an ) (*)
2 2
for all (a1 , a2 , . . . , an ) ∈ S, then
 s s
F (a1 , . . . , an−k , an−k+1 , . . . , an ) ≥ min F 0, . . . , 0, , . . . ,
1≤k≤n k k
for all (a1 , a2 , . . . , an ) ∈ S.

Proof The AC-Theorem is clearly true for n = 2. Consider further n ≥ 3. Since the
function F is continuous on the compact set S, F achieves its minimum at one or
more points of the set. We need to show that among these global minimum points
there is one having n − k coordinates equal to zero and k coordinates equal to
s/k, where k ∈ {1, 2, . . . , n}. Using the mathematical induction, it is easy to prove
that this is true if among the global minimum points there is one having either a
coordinate equal to zero or all coordinates equal to s/n. Let

B0 = (b1 , b2 , . . . , bn ), b1 ≤ b2 ≤ · · · ≤ bn ,

be a global minimum point of F over the set S. If b1 = 0 or b1 = b2 = · · · = bn ,


then the proof is completed. Consider further that

0 < b1 < bn .
6 Vasile Cîrtoaje

From the hypothesis (*), it follows that at least one of

A1 = (0, b2 , . . . , bn−1 , b1 + bn )

and  
b1 + bn b1 + b n
B1 = , b2 , . . . , bn−1 ,
2 2
is also a global minimum point of F over the set S. In the first case, the proof is
completed. In the second case, starting from B1 as a global minimum point of F ,
we repeat the process infinitely to get A∞ with a coordinate equal to zero and, by
Lemma above, s s s
B∞ = , ,..., .
n n n
Since at least one of A∞ and B∞ is a global minimum point of F , the conclusion
follows.

AC-Corollary. Let s > 0 and let F be a symmetric continuous function on the compact
set in n

S = {(a1 , a2 , . . . , an ) : a1 + a2 + · · · + an = s, ai ≥ 0, i = 1, 2, . . . , n}.

If
F (a1 , a2 , a3 , . . . , an ) ≥ F (0, a1 + a2 , a3 , . . . , an )
for all (a1 , a2 , . . . , an ) ∈ S satisfying
a + a a + a 
1 2 1 2
F (a1 , a2 , a3 , . . . , an ) < F , , a3 , . . . , a n , a1 = a2 ,
2 2
then F (a1 , a2 , . . . , an ) is minimum when n − k of the variables a1 , a2 , . . . , an are zero
s
and the other k variables are equal to , where k ∈ {1, 2, . . . , n}.
k

Proof. Consider the following two possible cases:


a + a a + a 
1 2 1 2
F (a1 , a2 , a3 , . . . , an ) ≥ F , , a3 , . . . , a n
2 2
and a + a a + a 
1 2 1 2
F (a1 , a2 , a3 , . . . , an ) < F , , a3 , . . . , a n .
2 2
If the first case, the condition (*) in AC-Theorem is obviously satisfied. In the
second case, which implies a1 = a2 , the hypothesis in the AC-Corollary gives

F (a1 , a2 , a3 , . . . , an ) ≥ F (0, a1 + a2 , a3 , . . . , an ),

and the condition (*) in AC-Theorem is also satisfied.


Arithmetic Mean Method and Arithmetic Compensation Method 7

AC1-Corollary. Let s > 0 and let G be a symmetric continuous function on the com-
pact set in n

S = {(a1 , a2 , . . . , an ) : a12 + a22 + · · · + an2 = s, ai ≥ 0, i = 1, 2, . . . , n}.

If  
G(a1 , a2 , a3 , . . . , an ) ≥ G 0, a12 + a22 , a3 , . . . , an
for all (a1 , a2 , . . . , an ) ∈ S satisfying


a2 + a2
a2 + a2
1 2 1 2
G(a1 , a2 , a3 , . . . , an ) < G , , a3 . . . , an , a1 = a2 ,
2 2

then G(a1 , a2 , . . . , an ) is minimum when


 n − k of the variables a1 , a2 , . . . , an are zero
s
and the other k variables are equal to , where k ∈ {1, 2, . . . , n}.
k
8 Vasile Cîrtoaje
Arithmetic Mean Method and Arithmetic Compensation Method 9

1.2 Applications

1.1. If a, b, c, d are nonnegative real numbers such that a + b + c + d = 4, then



9(a3 + b3 + c 3 + d 3 ) + 28 ≤ 2(a2 + b2 + c 2 + d 2 ).

1.2. If a, b, c, d, e, f ≥ 0 such that a + b + c + d + e + f = 6, then

5(a3 + b3 + c 3 + d 3 + e3 + f 3 ) + 36 ≥ 11(a2 + b2 + c 2 + d 2 + e2 + f 2 ).

1.3. If a, b, c, d ≥ 0 such that a + b + c + d = 4 , then

a bc + bcd + cd a + d a b + a2 b2 c 2 + b2 c 2 d 2 + c 2 d 2 a2 + d 2 a2 b2 ≤ 8.

1.4. If a, b, c, d ≥ 0 such that a + b + c + d = 4 , then

1 1 1 1
+ + + ≤ 1.
5 − a bc 5 − bcd 5 − cd a 5 − d a b

1.5. Let a1 , a2 , . . . , an (n ≥ 3) be nonnegative real numbers such that

a1 + a2 + · · · + an = n,

and let  n−1


1 (n − 1)en−1
en−1 = 1 + , k≥ .
n−1 n − en−1
Then,

1 1 1 n
+ + ··· + ≤ .
k − a1 a2 · · · an−1 k − a2 a3 · · · an k − an a1 · · · an−2 k−1

1.6. If a, b, c, d ≥ 0 such that a + b + c + d = 4 , then

1 1 1 1 15
+ + + ≤ .
4 − a bc 4 − bcd 4 − cd a 4 − d a b 11
10 Vasile Cîrtoaje

1.7. Let a1 , a2 , . . . , an (n ≥ 3) be nonnegative real numbers such that

a1 + a2 + · · · + an = n,

and let  n−1


1 (n − 1)en−1
en−1 = 1 + , en−1 < k ≤ .
n−1 n − en−1
Then,
1 1 1 n−1 1
+ + ··· + ≤ + .
k − a1 a2 · · · an−1 k − a2 a3 · · · an k − an a1 · · · an−2 k k − en−1

1.8. If a, b, c, d ≥ 0 such that a + b + c + d = 4 , then


1 1 1 1 1 1 2
+ + + + + ≤ .
10 − a b 10 − bc 10 − cd 10 − d a 10 − ac 10 − bd 3

1.9. If a, b, c, d ≥ 0 such that a + b + c + d = 4 , then

3(a3 + b3 + c 3 + d 3 − 4) ≥ (a2 + b2 + c 2 + d 2 − 4)(a2 + b2 + c 2 + d 2 − 1).

1.10. If a, b, c, d ≥ 0 such that a + b + c + d = 4 , then

7(a3 + b3 + c 3 + d 3 − 4) ≥ (a2 + b2 + c 2 + d 2 − 4)(a2 + b2 + c 2 + d 2 + 11).

1.11. Let a1 , a2 , . . . , an (n ≥ 3) be nonnegative real numbers such that

a1 + a2 + · · · + an = n.

Then
2 3   n 2 
a1 + a23 + · · · + an3 + ai1 ai2 ai3 ≥ a + a22 + · · · + an2 .
3 1≤i <i <i
3 1
1 2 3 ≤n

1.12. Let a1 , a2 , . . . , an (n ≥ 4) be nonnegative real numbers such that

a1 + a2 + · · · + an = n.

Then
6   
a13 + a23 + · · · + an3 + ai1 ai2 ai3 ≥ 2 a12 + a22 + · · · + an2 .
n(n − 3) 1≤i <i <i
1 2 3 ≤n
Arithmetic Mean Method and Arithmetic Compensation Method 11

1.13. Let a1 , a2 , . . . , an (n ≥ 3) be nonnegative real numbers such that

a1 + a2 + · · · + an = n.

Then
  6   
(n + 1) a13 + a23 + · · · + an3 + ai1 ai2 ai3 ≥ 2n a12 + a22 + · · · + an2 .
n − 2 1≤i <i <i
1 2 3 ≤n

1.14. Let a1 , a2 , ..., an be nonnegative real numbers such that a1 + a2 + · · · + an = 1.


If
m ∈ {1, 2, . . . , n + 1},
then
   
m(m − 1) a13 + a23 + · · · + an3 + 1 ≥ (2m − 1) a12 + a22 + · · · + an2 .

1.15. If a1 , a2 , . . . , an are nonnegative real numbers such that a1 + a2 + · · · + an = n,


then  
(n + 1) a12 + a22 + · · · + an2 ≥ n2 + a13 + a23 + · · · + an3 .

1.16. If a1 , a2 , . . . , an are nonnegative real numbers such that a1 + a2 + · · · + an = n,


then  
(n2 + n + 1) a12 + a22 + · · · + an2 − n ≥ a14 + a24 + · · · + an4 − n.

1.17. Let a1 , a2 , . . . , an be nonnegative real numbers such that a1 + a2 +· · ·+ an = n.


If m ≥ 3 is an integer, then

nm−1 − 1  2 
a1 + a22 + · · · + an2 − n ≥ a1m + a2m + · · · + anm − n.
n−1

1.18. If a, b, c, d, e are nonnegative real numbers such that a + b + c + d + e = 5,


then
5
a2 + b2 + c 2 + d 2 + e2 − 5 ≥ (1 − a bcd e).
4

1.19. If a1 , a2 , . . . , an are nonnegative real numbers such that a1 + a2 + · · · + an = n,


then
n
a12 + a22 + · · · + an2 − n ≥ (1 − a1 a2 · · · an ).
n−1
12 Vasile Cîrtoaje

1.20. If a, b, c, d, e are nonnegative real numbers such that a + b + c + d + e = 5,


then
45
a3 + b3 + c 3 + d 3 + e3 − 5 ≥ (1 − a bcd e).
16

1.21. If a1 , a2 , . . . , an are nonnegative real numbers such that a1 + a2 + · · · + an = n,


then
n(2n − 1)
a13 + a23 + · · · + an3 − n ≥ (1 − a1 a2 · · · an ).
(n − 1)2

1.22. If a1 , a2 , . . . , an are nonnegative real numbers such that a1 + a2 + · · · + an = n,


then
 
1 3  n−2 2
a1 + a23 + · · · + an3 − n ≥ a12 + a22 + · · · + an2 − n + (a1 a2 · · · an − 1).
n 2

1.23. If a1 , a2 , . . . , an are nonnegative real numbers such that a1 + a2 + · · · + an = n,


then

n(n − 1)  
a13 + a23 + · · · + an3 − n ≥ a12 + a22 + · · · + an2 − n.
n+1

1.24. If a, b, c, d are nonnegative real numbers such that a + b + c + d = 4, then




313 5
2 (a4 + b4 + c 4 + d 4 ) + + ≥ a2 + b2 + c 2 + d 2 .
81 9

1.25. If a, b, c, d are nonnegative real numbers such that a + b + c + d = 4, then




a4 + b4 + c 4 + d 4 − 4
2 ≥ a2 + b2 + c 2 + d 2 − 4.
7

1.26. If a, b, c, d are nonnegative real numbers such that a + b + c + d = 4, then


  
16 4 3  2 
a4 + b4 + c 4 + d 4 +
− 8 ≥
− a + b2 + c 2 + d 2 .
3 3 2
Arithmetic Mean Method and Arithmetic Compensation Method 13

1.27. If a, b, c, d are nonnegative real numbers such that a + b + c + d = 4, then


  

1  2 
a4 + b4 + c 4 + d 4 + 8(2 − 2) ≥ 2 −
a + b2 + c 2 + d 2 .
2

1.28. If a, b, c, d are nonnegative real numbers such that a + b + c + d = 4, then


    
1 1 3 2  2 
a4 + b4 + c 4 + d 4 + 16



a + b2 + c 2 + d 2 .
2 3 2 3

1.29. If a, b, c, d are nonnegative real numbers such that a2 + b2 + c 2 + d 2 = 1, then

a3 + b3 + c 3 + d 3 + a bc + bcd + cd a + d a b ≤ 1.

1.30. If a, b, c, d are nonnegative real numbers such that a2 + b2 + c 2 + d 2 = 2, then

a3 + b3 + c 3 + d 3 + a bc + bcd + cd a + d a b ≥ 2.

1.31. If a, b, c, d are nonnegative real numbers such that a2 + b2 + c 2 + d 2 = 3, then

3(a3 + b3 + c 3 + d 3 ) + 2(a bc + bcd + cd a + d a b) ≥ 11.

1.32. If a, b, c, d are nonnegative real numbers such that a2 + b2 + c 2 + d 2 = 4, then

4(a3 + b3 + c 3 + d 3 ) + a bc + bcd + cd a + d a b ≥ 20.

1.33. If a, b, c, d are nonnegative real numbers such that a2 + b2 + c 2 + d 2 = 4, then

28
a3 + b3 + c 3 + d 3 + 3(a + b + c + d) ≤
.
3

1.34. If a, b, c, d are nonnegative real numbers such that a2 + b2 + c 2 + d 2 = 4, then

a3 + b3 + c 3 + d 3 + 4(a + b + c + d) ≤ 20.
14 Vasile Cîrtoaje

1.35. If a, b, c, d are nonnegative real numbers such that a2 + b2 + c 2 + d 2 = 4, then



a3 + b3 + c 3 + d 3 + 2 2(a + b + c + d) ≥ 4(2 + 2 ).

1.36. If a, b, c, d are nonnegative real numbers such that a2 + b2 + c 2 + d 2 = 4, then


 

2
2
a + b +c +d +2
3 3 3 3
(a + b + c + d) ≥ 4 2+
.
3 3

1.37. If a, b, c, d are nonnegative real numbers such that a2 + b2 + c 2 + d 2 = 4, then


2
a3 + b3 + c 3 + d 3 − 4 +
(a + b + c + d − 4) ≥ 0.
3

1.38. If a1 , a2 , . . . , an are real numbers such that

a1 + a2 + · · · + an = n,

then
 2 n(n − 1)  4 
a12 + a22 + · · · + an2 − n2 ≥ a + a24 + · · · + an4 − n .
n2 − n + 1 1

1.39. If a, b, c, d are real numbers such that a + b + c + d = 4, then


 2  26

a + b2 + c 2 + d 2 − 4 a2 + b2 + c 2 + d 2 + ≥ a4 + b4 + c 4 + d 4 − 4.
5

1.40. If a, b, c, d are nonnegative real numbers such that a + b + c + d = 4, then


 2  11

3 4 
a + b2 + c 2 + d 2 − 4 a2 + b2 + c 2 + d 2 + ≥ a + b4 + c 4 + d 4 − 4 .
6 4

1.41. If a, b, c, d are nonnegative real numbers such that a + b + c + d = 4, then


 2  
a + b2 + c 2 + d 2 − 4 2a2 + 2b2 + 2c 2 + 2d 2 − 1 ≥ a4 + b4 + c 4 + d 4 − 4.

1.42. If a, b, c, d are nonnegative real numbers such that a + b + c + d = 4, then


   4 4 
a2 + b2 + c 2 + d 2 − 4 a2 + b2 + c 2 + d 2 + 12 ≥ a + b4 + c 4 + d 4 − 4 .
3
Arithmetic Mean Method and Arithmetic Compensation Method 15

1.43. If a, b, c, d are nonnegative real numbers such that a + b + c + d = 4, then


 2  76

12  4 
a + b2 + c 2 + d 2 − 4 a2 + b2 + c 2 + d 2 + ≥ a + b4 + c 4 + d 4 − 4 .
11 11

1.44. If a1 , a2 , . . . , an are nonnegative real numbers such that

a1 + a2 + · · · + an = m, m ∈ {1, 2, . . . , n},

then
1 1 1 m
+ + ··· + ≥ n− .
1 + a12 1 + a22 1 + an2 2

1.45. If a, b, c, d are nonnegative real numbers such that a + b + c + d = 2, then

1 1 1 1 16
+ + + ≥ .
1 + 3a2 1 + 3b2 1 + 3c 2 1 + 3d 2 7

1.46. If a, b, c, d are nonnegative real numbers such that a + b + c + d = 4, then

(1 + a2 )(1 + b2 )(1 + c 2 )(1 + d 2 ) ≤ 25.

1.47. If a, b, c, d are nonnegative real numbers such that a + b + c + d = 1, then

(1 + 2a)(1 + 2b)(1 + 2c)(1 + 2d) 125


≥ .
(1 − a)(1 − b)(1 − c)(1 − d) 8

2
1.48. If a1 , a2 , . . . , an are nonnegative real numbers such that a1 + a2 + · · · + an = ,
3
then  ai a j 1
≤ .
1≤i≤ j≤n
(1 − a i )(1 − a j ) 4

1.49. If a1 , a2 , . . . , an are nonnegative real numbers such that a1 + a2 + · · · + an = 1


and no one of which is 1, then
 ai a j n
≥ .
1≤i≤ j≤n
(1 − a i )(1 − a j ) 2(n − 1)
16 Vasile Cîrtoaje

1.50. If a, b, c, d are nonnegative real numbers such that a + b + c + d = 4, then

(1 + 3a)(1 + 3b)(1 + 3c)(1 + 3d) ≤ 125 + 131a bcd.

1.51. If a, b, c, d are nonnegative real numbers such that a + b + c + d = 4, then

(1 + 3a2 )(1 + 3b2 )(1 + 3c 2 )(1 + 3d 2 ) ≤ 255 + a2 b2 c 2 d 2 .

1.52. If a1 , a2 , . . . , an (n ≥ 3) are nonnegative real numbers, then


  4n(n − 2) 
a12 + 2 a1 a2 a3 + ≥2 a1 a2 .
s ym
3(n − 1) 2
s ym

1.53. If a, b, c, d are nonnegative real numbers such that a + b + c + d = 3 , then

a b(a + 2b + 3c) + bc(b + 2c + 3d) + cd(c + 2d + 3a) + d a(d + 2a + 3b) ≤ 2.


Arithmetic Mean Method and Arithmetic Compensation Method 17

1.3 Solutions

P 1.1. If a, b, c, d are nonnegative real numbers such that a + b + c + d = 4, then



9(a3 + b3 + c 3 + d 3 ) + 28 ≤ 2(a2 + b2 + c 2 + d 2 ).

(Vasile C., 2006)

Solution. Assume that


a ≤ b ≤ c ≤ d,
and write the inequality as F (a, b, c, d) ≥ 0, where

F (a, b, c, d) = 4(a2 + b2 + c 2 + d 2 )2 − 9(a3 + b3 + c 3 + d 3 ) − 28.

First Solution. If
F (a, b, c, d) ≥ F (t, b, c, t)
for
a+d
t= ,
2
then, by the AM-Theorem, we have

F (a, b, c, d) ≥ F (1, 1, 1, 1) = 0.

Using the identities

a2 + d 2 − 2t 2 = 2(t 2 − ad), a3 + d 3 − 2t 3 = 3(t 2 − ad)(a + d),

we may write the inequality F (a, b, c, d) ≥ F (t, b, c, t) as follows:


    
4 a2 + d 2 − 2t 2 2t 2 + a2 + 2b2 + 2c 2 + d 2 − 9 a3 + d 3 − 2t 3 ≥ 0,
 
8(t 2 − ad) 2t 2 + a2 + 2b2 + 2c 2 + d 2 − 27(t 2 − ad)(a + d) ≥ 0.
Since
1
t 2 − ad = (a − d)2 ≥ 0,
4
this inequality is true if

8(2t 2 + a2 + 2b2 + 2c 2 + d 2 ) − 28(a + d) ≥ 0,

which is equivalent to the homogeneous inequalities

12(a2 + d 2 ) + 8ad + 16(b2 + c 2 ) − 7(a + d)(a + b + c + d) ≥ 0,

5(a2 + d 2 ) − 6ad + 16(b2 + c 2 ) − 7(a + d)(b + c) ≥ 0.


18 Vasile Cîrtoaje

Since
2(b2 + c 2 ) ≥ (b + c)2 ,
it suffices to show that

5(a2 + d 2 ) − 6ad + 8(b + c)2 − 7(a + d)(b + c) ≥ 0.

Using the substitution


b+c
x= , a ≤ x ≤ d,
2
we may write the inequality as E(a, x, d) ≥ 0, where

E(a, x, d) = 5(a2 + d 2 ) − 6ad + 32x 2 − 14x(a + d).

We will show that


E(a, x, d) ≥ E(x, x, d) ≥ 0.
We have
E(a, x, d) − E(x, x, d) = (x − a)(9x + 6d − 5a) ≥ 0
and
E(x, x, d) = 5d 2 − 20d x + 23x 2 = 5(d − 2x)2 + 3x 2 > 0.
The equality holds for a = b = c = d = 1.
Second Solution. We will apply the AM-Corollary. Thus, we need to show that
a+c
F (a, b, c, d) ≥ F (t, b, t, d), t= ≤ d.
2
As shown at the first solution, this inequality is true if

5(a2 + c 2 ) − 6ac + 16(b2 + d 2 ) − 7(a + c)(b + d) ≥ 0,

which can be written as


 
7a + 7c 2 49(a + c)2
5(a2 + c 2 ) − 6ac + 16d 2 + 4b − − − 7(a + c)d ≥ 0.
8 64

It suffices to show that

3(a + c)2
5(a2 + c 2 ) − 6ac + 16d 2 − − 7(a + c)d ≥ 0.
2
Since
5(a2 + c 2 ) − 6ac ≥ (a + c)2 = 4t 2 ,
it suffices to show that

4t 2 + 16d 2 − 6t 2 − 14t d ≥ 0,
Arithmetic Mean Method and Arithmetic Compensation Method 19

which is equivalent to the obvious inequality

(d − t)(8d + t) ≥ 0.

By the AM-Corollary, it suffices to prove the original inequality for a = b = c. That


is, to show that 3a + d = 4 involves
 
4 (3a2 + d 2 )2 − 16 − 9(3a3 + d 3 − 4) ≥ 0,

which is equivalent to

4(3a2 + d 2 − 4)(3a2 + d 2 + 4) − 9(3a3 + d 3 − 4) ≥ 0.

Since
3a2 + d 2 − 4 = 12(a − 1)2 , 3a2 + d 2 + 4 = 4(3a2 − 6a + 5),
3a + d − 4 = 12(a − 1)2 (5 − 2a),
3 3

the inequality reduces to

(a − 1)2 (48a2 − 78a + 35) ≥ 0,

which is true because

48a2 − 78a + 35 > 3(16a2 − 26a + 11)

and  
13 2 7
16a2 − 26a + 11 = 4a − + > 0.
4 16

P 1.2. If a, b, c, d, e, f ≥ 0 such that a + b + c + d + e + f = 6, then

5(a3 + b3 + c 3 + d 3 + e3 + f 3 ) + 36 ≥ 11(a2 + b2 + c 2 + d 2 + e2 + f 2 ).

(Vasile C., 2006)

Solution. Write the inequality as F (a, b, c, d, e, f ) ≥ 0, where

F (a, b, c, d, e, f ) = 5(a3 + b3 + c 3 + d 3 + e3 + f 3 ) + 36− 11(a2 + b2 + c 2 + d 2 + e2 + f 2 ).

First Solution. Apply the AM-Corollary. We will show first that

F (a, b, c, d, e, f ) ≥ F (t, b, c, d, t, f )

for
a+e
a ≥ b ≥ c ≥ d ≥ e ≥ f, t= .
2
20 Vasile Cîrtoaje

Using the identities

a2 + e2 − 2t 2 = 2(t 2 − ae), a3 + e3 − 2t 3 = 3(a + e)(t 2 − ae),

we write the desired inequality F (a, b, c, d, e, f ) ≥ F (t, b, c, d, t, f ) as

5(a3 + e3 − 2t 3 ) − 11(a2 + e2 − 2t 2 ) ≥ 0,

15(a + e)(t 2 − ae) − 22(t 2 − ae) ≥ 0.


Since
1
t 2 − ae = (a − e)2 ≥ 0,
4
we only need to show that
15(a + e) − 22 ≥ 0.
This inequality is equivalent to

45(a + e) − 11(a + b + c + d + e + f ) ≥ 0,

34a − 11(b + c + d) + 34e − 11 f ≥ 0,


a + 11(3a − b − c − d) + 23e + 11(e − f ) ≥ 0.
Clearly, the last inequality is true. By the AM-Corollary, it suffices to prove the
original inequality for a = b = c = d = e. Thus, we need to show that 5a + f = 6
involves
5(5a3 + f 3 ) + 36 ≥ 11(5a2 + f 2 ),
which is equivalent to
6 − 17a + 16a2 − 5a3 ≥ 0,
(1 − a)2 (6 − 5a) ≤ 0,
(1 − a)2 b ≥ 0.
The equality holds for a = b = c = d = e = f = 1, and also for

6
a=b=c=d=e= , f =0
5
(or any cyclic permutation).
Second Solution. Apply the AC-Corollary. We will show that

F (a, b, c, d, e, f ) < F (t, t, c, d, e, f )

involves
F (a, b, c, d, e, f ) ≥ F (0, 2t, c, d, e, f ),
where
a+b
t= , a = b.
2
Arithmetic Mean Method and Arithmetic Compensation Method 21

As shown at the first solution, the hypothesis F (a, b, c, d, e, f ) < F (t, t, c, d, e, f )


involves
15(a + b) − 22 < 0. (*)
Write now the required inequality F (a, b, c, d, e, f ) ≥ F (0, a+b, c, d, e, f ) as follows:

5[a3 + b3 − (a + b)3 ] − 11[a2 + b2 − (a + b)2 ] ≥ 0,

−15a b(a + b) + 22a b ≥ 0,


a b[15(a + b) − 22] ≤ 0.
Clearly, this inequality follows immediately from (*).
By the AC-Corollary, it suffices to prove that F (a, b, c, d, e, f ) ≥ 0 when 6 − k of
6
the variables a, b, c, d, e, f are zero and the other k variables are equal to , where
k
k ∈ {1, 2, 3, 4, 5, 6}; that is,
 3  2
6 6
5k + 36 − 11k ≥ 0,
k k

which is equivalent to the obvious inequality

(k − 5)(k − 6) ≥ 0.

P 1.3. If a, b, c, d ≥ 0 such that a + b + c + d = 4 , then

a bc + bcd + cd a + d a b + a2 b2 c 2 + b2 c 2 d 2 + c 2 d 2 a2 + d 2 a2 b2 ≤ 8.

(Vasile C., 2006)

Solution. Write the desired inequality as F (a, b, c, d) + 8 ≥ 0, where

F (a, b, c, d) = −(a bc + bcd + cd a + d a b + a2 b2 c 2 + b2 c 2 d 2 + c 2 d 2 a2 + d 2 a2 b2 ).

First Solution. According to the AM-Corollary, we need to show that

F (a, b, c, d) ≥ F (t, b, t, d)

for
a+c
a ≥ b ≥ c ≥ d, t= .
2
Taking into account that

F (a, b, c, d) = −ac(b + d) − (a + c)bd − a2 c 2 (b2 + d 2 ) − (a2 + c 2 )b2 d 2 ,


22 Vasile Cîrtoaje

we write the desired inequality F (a, b, c, d) ≥ F (t, b, t, d) as

(t 2 − ac)(b + d) + (t 4 − a2 c 2 )(b2 + d 2 ) − (a2 + c 2 − 2t 2 )b2 d 2 ≥ 0,


 
(t 2 − ac) b + d + (t 2 + ac)(b2 + d 2 ) − 2b2 d 2 ≥ 0.
Since
1
t 2 − ac = (a − c)2 ≥ 0,
4
this inequality is true if
ac(b2 + d 2 ) − 2b2 d 2 ≥ 0.
We have

ac(b2 + d 2 ) − 2b2 d 2 ≥ bd(b2 + d 2 ) − 2b2 d 2 = bd(b − d)2 ≥ 0.

By the AM-Corollary, it suffices to prove the original inequality for a = b = c;


that is, to show that 3a + d = 4 involves

a3 + 3a2 d + a6 + 3a4 d 2 ≤ 8,

which is equivalent to

7a6 − 18a5 + 12a4 − 2a3 + 3a2 − 2 ≤ 0,

(a − 1)2 (7a4 − 4a3 − 3a2 − 4a − 2) ≤ 0.


It suffices to show that
2
7a4 − 4a3 − 3a2 − 4a − 2 + ≤ 0,
81
which is equivalent to

567a4 − 324a3 − 243a2 − 324a − 160 ≤ 0,

(3a − 4)(189a3 + 144a2 + 111a + 40) ≤ 0,


d(189a3 + 144a2 + 111a + 40) ≥ 0.
The equality holds for a = b = c = d = 1.
Second Solution. Apply the AC-Corollary. First, we write F (a, b, c, d) in the form

F (a, b, c, d) = −a b(c + d) − (a + b)cd − a2 b2 (c 2 + d 2 ) − (a2 + b2 )c 2 d 2 .

We will show that


F (a, b, c, d) < F (t, t, c, d)
involves
F (a, b, c, d) ≥ F (0, 2t, c, d),
Arithmetic Mean Method and Arithmetic Compensation Method 23

where
a+b
t= , a = b.
2
Write the hypothesis F (a, b, c, d) < F (t, t, c, d) as follows:

(t 2 − a b)(c + d) + (t 4 − a2 b2 )(c 2 + d 2 ) − (a2 + b2 − 2t 2 )c 2 d 2 < 0,


 
(t 2 − a b) c + d + (t 2 + a b)(c 2 + d 2 ) − 2c 2 d 2 < 0.
Dividing by the positive factor t 2 − a b, the inequality becomes

c + d + (t 2 + a b)(c 2 + d 2 ) − 2c 2 d 2 < 0. (*)

Write now the required inequality F (a, b, c, d) ≥ F (0, 2t, c, d) as follows:


 
−a b(c + d) − a2 b2 (c 2 + d 2 ) + (a + b)2 − a2 − b2 c 2 d 2 ≥ 0,
 
−a b c + d + a b(c 2 + d 2 ) − 2c 2 d 2 ≥ 0,
 
a b c + d + a b(c 2 + d 2 ) − 2c 2 d 2 ≤ 0.
Clearly, this inequality follows immediately from (*).
By the AC-Corollary, it suffices to prove the original inequality for a = b = c =
d = 1, and for a = 0 and b + c + d = 4. In the first case, the equality occurs. In the
second case, the inequality becomes

bcd + b2 c 2 d 2 ≤ 8.

By the AM-GM inequality, we have


 3
b+c+d 64
bcd ≤ = ,
3 27

therefore  
64 64 8
bcd + b2 c 2 d 2 ≤ 1+ =8− < 8.
27 27 729

P 1.4. If a, b, c, d ≥ 0 such that a + b + c + d = 4 , then

1 1 1 1
+ + + ≤ 1.
5 − a bc 5 − bcd 5 − cd a 5 − d a b
24 Vasile Cîrtoaje

Solution. By the AM-GM inequality, we have


 3  3
a+b+c 4 71
5 − a bc ≥ 5 − ≥5− = > 0.
3 3 27

Write the desired inequality as F (a, b, c, d) ≥ 0, where

1 1 1 1
F (a, b, c, d) = 1 − − − − ,
5 − a bc 5 − bcd 5 − cd a 5 − d a b
and apply the AC-Corollary. First, we will show that

F (a, b, c, d) < F (t, t, c, d)

involves
F (a, b, c, d) ≥ F (0, 2t, c, d),
a+b
where t = , a = b. Write the hypothesis F (a, b, c, d) < F (t, t, c, d) as
2
   
1 1 1 1 2(5 − cd t) 2
− + − < − .
5 − ct 2 5 − a bc 5 − dt 2 5 − dab (5 − bcd)(5 − cd a) 5 − cd t

Dividing by the positive factor t 2 − a b, the inequality becomes

c d 2c 2 d 2
+ < .
(5 − a bc)(5 − c t ) (5 − d a b)(5 − d t ) (5 − bcd)(5 − cd a)(5 − cd t)
2 2

Since
c d c d
+ ≥ + ,
(5 − a bc)(5 − c t 2 ) (5 − d a b)(5 − d t 2 ) 5(5 − a bc) 5(5 − d a b)
we get
c d 10c 2 d 2
+ < . (*)
5 − a bc 5 − d a b (5 − bcd)(5 − cd a)(5 − cd t)
Similarly, write the required inequality F (a, b, c, d) ≥ F (0, 2t, c, d) as follows:
     
1 1 1 1 1 1 1 1
+ − + ≥ − + − ,
5 5 − 2cd t 5 − bcd 5 − cd a 5 − a bc 5 5 − dab 5

2(5 − cd t) 10(5 − cd t) a bc dab


− ≥ + ,
5 − 2cd t (5 − bcd)(5 − cd a) 5 − a bc 5 − d a b
2c 2 d 2 (5 − cd t) c d
≥ + .
(5 − bcd)(5 − cd a)(5 − 2cd t) 5 − a bc 5 − d a b
Since
5 − cd t 5
≥ ,
5 − 2cd t 5 − cd t
Arithmetic Mean Method and Arithmetic Compensation Method 25

it suffices to show that

10c 2 d 2 c d
≥ + .
(5 − bcd)(5 − cd a)(5 − cd t) 5 − a bc 5 − d a b

Taking into account (*), the conclusion follows.


By the AC-Corollary, it suffices to prove the original inequality for a = b = c =
d = 1, and for a = 0 and b + c + d = 4. In the first case, the equality occurs. In the
second case, since
 3  3
b+c+d 4 71
5 − bcd ≥ 5 − =5− = ,
3 3 27

we have
1 1 1 1 1 3 27 3
+ + + = + ≤ + < 1.
5 − a bc 5 − bcd 5 − cd a 5 − d a b 5 − bcd 5 71 5
The equality holds for a = b = c = d = 1.
Remark. In the same manner, we can prove the following generalization:
• If a1 , a2 , . . . , an (n ≥ 4) are nonnegative real numbers such that

a1 + a2 + · · · + an = n,

then
1 1 1
+ + ··· + ≤ 1,
n + 1 − a1 a2 · · · an−1 n + 1 − a2 a3 · · · an n + 1 − an a1 · · · an−2

with equality for a1 = a2 = · · · = an = 1.

P 1.5. Let a1 , a2 , . . . , an (n ≥ 3) be nonnegative real numbers such that

a1 + a2 + · · · + an = n,

and let  n−1


1 (n − 1)en−1
en−1 = 1 + , k≥ .
n−1 n − en−1
Then,

1 1 1 n
+ + ··· + ≤ .
k − a1 a2 · · · an−1 k − a2 a3 · · · an k − an a1 · · · an−2 k−1

(Vasile C., 2005)


26 Vasile Cîrtoaje

Solution. By the AM-GM inequality, we have


 a + a + · · · + a n−1  n n−1
1 2 n−1
a1 a2 · · · an−1 ≤ ≤ = en−1 < k.
n−1 n−1
Write the desired inequality as
n
F (a1 , a2 , . . . , an ) + ≥ 0,
k−1
where
 
1 1 1
F (a1 , a2 , . . . , an ) = − + + ··· + .
k − a2 a3 · · · an k − a1 a3 · · · an k − a1 a2 · · · an−1

We assert that
F (a1 , a2 , a3 , . . . , an ) < F (t, t, a3 , . . . , an ) (1)
involves
F (a1 , a2 , a3 , . . . , an ) ≥ F (0, 2t, a3 , . . . , an ) (2)
for any k > en−1 , where
a1 + a2
t= , a1 = a2 .
2
Let us denote
b = a3 a4 · · · an .
Since
1 1  ai
n
−F (a1 , a2 , a3 , . . . , an ) = + + ,
k − a1 b k − a2 b i=3 kai − a1 a2 b
and
2  n
ai
−F (t, t, a3 , . . . , an ) = + ,
k − t b i=3 kai − t 2 b
the hypothesis (1) is equivalent to
 
2b 
n
ai
b(t − a1 a2 )
2
− > 0,
(k − t b)(k − a1 b)(k − a2 b) i=3 (kai − t 2 b)(kai − a1 a2 b)

which involves

2b  n
ai
> .
(k − t b)(k − a1 b)(k − a2 b) i=3
(kai − t 2 b)(kai − a1 a2 b)

Because
ai 1
≥ ,
kai − t 2 b k
Arithmetic Mean Method and Arithmetic Compensation Method 27

this inequality involves

2k b  n
1
> . (3)
(k − t b)(k − a1 b)(k − a2 b) i=3
ka i − a1 a2 b

On the other hand, since

1 1  n
ai
−F (a1 , a2 , a3 , . . . , an ) = + + ,
k − a1 b k − a2 b i=3 kai − a1 a2 b

and
n−1 1
−F (0, 2t, a3 , . . . , an ) =
+ ,
k k − 2t b
the required inequality (2) can be written as follows:
       n  
1 1 1 1 1 1 ai 1
− − − − − ≥ − ,
k − 2t b k k − a1 b k k − a2 b k i=3
kai − a1 a2 b k

2t b 2(kt − a1 a2 b)b  n
a1 a2 b
− ≥ ,
k − 2t b (k − a1 b)(k − a2 b) i=3
ka i − a1 a2 b
 
2b(k − t b) n
1
a1 a2 b − ≥ 0.
(k − 2t b)(k − a1 b)(k − a2 b) i=3 kai − a1 a2 b
Because
k − tb k
≥ ,
k − 2t b k − tb
this inequality is true if

2k b  n
1
> ,
(k − t b)(k − a1 b)(k − a2 b) i=3
kai − a1 a2 b

which is exactly (3).


According to the AC-Corollary, it suffices to prove the original inequality for a1 =
a2 = · · · = an = 1, and for a1 = 0 and a2 + a3 + · · · + an = n. For the first case,
the original inequality is an equality. For the second case, the original inequality
becomes
1 n−1 n
+ ≤ ,
k − a2 a3 · · · an k k−1
which is equivalent to
kn
a2 a3 · · · an ≤.
k+n−1
Since a2 a3 · · · an ≤ en−1 , it suffices to show that

kn
en−1 ≤ ,
k+n−1
28 Vasile Cîrtoaje

which is equivalent to the hypothesis

(n − 1)en−1
k≥ .
n − en−1

(n − 1)en−1
The equality holds for a1 = a2 = · · · = an = 1. In addition, if k = , then
n − en−1
the equality holds also for
n
a1 = 0, a2 = a3 = · · · = an =
n−1
(or any cyclic permutation).
Remark. For n = 4, we get the following statement:
• If a, b, c, d are nonnegative real numbers such that a + b + c + d = 4 , then
1 1 1 1 4
+ + + ≤ ,
48 − 11a bc 48 − 11bcd 48 − 11cd a 48 − 11d a b 37
4
with equality for a = b = c = d = 1, and also for a = 0 and b = c = d = (or any
3
cyclic permutation).

P 1.6. If a, b, c, d ≥ 0 such that a + b + c + d = 4 , then


1 1 1 1 15
+ + + ≤ .
4 − a bc 4 − bcd 4 − cd a 4 − d a b 11

Solution. The proof is similar to the one of the preceding P 1.5. Finally, it suffices to
prove the original inequality for a = b = c = d = 1, and for a = 0 and b + c + d = 4.
The first case is trivial, while the second case leads to the inequality
1 3 15
+ ≤ ,
4 − bcd 4 11
which is equivalent to
64
bcd ≤ .
27
By the AM-GM inequality, we have
 
b + c + d 3 64
bcd ≤ = .
3 27
The equality holds for
4
a = 0, b=c=d=
3
Arithmetic Mean Method and Arithmetic Compensation Method 29

(or any cyclic permutation).


Remark. In the same manner, we can prove the following generalization:
• Let a1 , a2 , . . . , an (n ≥ 4) be nonnegative real numbers such that

a1 + a2 + · · · + an = n.

and let  n−1


1
en−1 = 1 + .
n−1
Then,

1 1 1 en−1
+ + ··· + ≤1+ ,
n − a1 a2 · · · an−1 n − a2 a3 · · · an n − an a1 · · · an−2 n(n − en−1 )
n
with equality for a1 = 0 and a2 = · · · = an = (or any cyclic permutation).
n−1

P 1.7. Let a1 , a2 , . . . , an (n ≥ 3) be nonnegative real numbers such that

a1 + a2 + · · · + an = n,

and let  n−1


1 (n − 1)en−1
en−1 = 1 + , en−1 < k ≤ .
n−1 n − en−1
Then,

1 1 1 n−1 1
+ + ··· + ≤ + .
k − a1 a2 · · · an−1 k − a2 a3 · · · an k − an a1 · · · an−2 k k − en−1

(Vasile C., 2005)

Solution. Write the desired inequality as

n−1 1
F (a1 , a2 , . . . , an ) + + ≥ 0,
k k − en−1

where
 
1 1 1
F (a1 , a2 , . . . , an ) = − + + ··· + .
k − a1 a2 · · · an−1 k − a2 a3 · · · an k − an a1 · · · an−2

As shown in the proof of P 1.5, the inequality

F (a1 , a2 , a3 , . . . , an ) < F (t, t, a3 , . . . , an )


30 Vasile Cîrtoaje

involves
F (a1 , a2 , a3 , . . . , an ) ≥ F (0, 2t, a3 , . . . , an )
for any k > en−1 , where
a1 + a2
t= , a1 = a2 .
2
According to the AC-Corollary, it suffices to prove the original inequality for a1 =
a2 = · · · = an = 1, and for a1 = 0 and a2 + a3 + · · · + an = n. In the first case, the
original inequality becomes

n n−1 1
≤ + ,
k−1 k k − en−1

which is equivalent to the hypothesis

(n − 1)en−1
k≤ .
n − en−1

In the second case, the original inequality becomes

1 1
≤ ,
k − a2 a3 · · · an k − en−1

which is equivalent to
a2 a3 · · · an ≤ en−1 .
By the AM-GM inequality, we have
 a + a + · · · + a n−1  n n−1
1 2 n−1
a1 a2 · · · an−1 ≤ ≤ = en−1 .
n−1 n−1
The equality holds for
n
a1 = 0, a2 = a3 = · · · = an =
n−1
(n − 1)en−1
(or any cyclic permutation). In addition, if k = , then the equality holds
n − en−1
also for a1 = a2 = · · · = an = 1.

P 1.8. If a, b, c, d ≥ 0 such that a + b + c + d = 4 , then

1 1 1 1 1 1 2
+ + + + + ≤ .
10 − a b 10 − bc 10 − cd 10 − d a 10 − ac 10 − bd 3
Arithmetic Mean Method and Arithmetic Compensation Method 31

Solution. By the AM-GM inequality, we have


 2
a+b
10 − a b ≥ 10 − ≥ 10 − 4 > 0.
2
Write the desired inequality as
2
F (a, b, c, d) + ≥ 0,
3
where
 
1 1 1 1 1 1
F (a, b, c, d) = − + + + + + ,
10 − a b 10 − ac 10 − ad 10 − bc 10 − bd 10 − cd
and apply the AC-Corollary. First, we will show that

F (a, b, c, d) < F (t, t, c, d)

involves
F (a, b, c, d) ≥ F (0, 2t, c, d),
a+b
where t = , a = b. Write the hypothesis F (a, b, c, d) < F (t, t, c, d) as
2
   
1 1 2 1 1 2
+ − + + − >
10 − ac 10 − bc 10 − t c 10 − ad 10 − bd 10 − t d
1 1
> − .
10 − t 2 10 − a b
Dividing by the positive factor t 2 − a b, the inequality becomes

2c 2 2d 2 1
+ > .
(10 − ac)(10 − bc)(10 − t c) (10 − ad)(10 − bd)(10 − t d) (10 − a b)(10 − t 2 )
Since
1 1
≥ ,
(10 − a b)(10 − t 2 ) 10(10 − a b)
we get

20c 2 20d 2 1
+ > . (*)
(10 − ac)(10 − bc)(10 − t c) (10 − ad)(10 − bd)(10 − t d) 10 − a b

Similarly, write the required inequality F (a, b, c, d) ≥ F (0, 2t, c, d) as follows:


     
1 1 1 1 1 1
− + + − +
10 − a b 10 10 − ac 10 − bc 10 10 − 2t c
   
1 1 1 1
+ + − + ≤ 0,
10 − ad 10 − bd 10 10 − 2t d
32 Vasile Cîrtoaje


ab 2(10 − t c) 2(10 − t c)
+ −
10(10 − a b) (10 − ac)(10 − bc) 10(10 − 2t c)

2(10 − t d) 2(10 − t d)
+ − ≤ 0,
(10 − ad)(10 − bd) 10(10 − 2t d)

a bc 2 (10 − t c) a bd 2 (10 − t d) ab
+ ≥ .
(10 − ac)(10 − bc)(5 − t c) (10 − ad)(10 − bd)(5 − t d) 10 − a b
This is true if

c 2 (10 − t c) d 2 (10 − t d) 1
+ ≥ .
(10 − ac)(10 − bc)(5 − t c) (10 − ad)(10 − bd)(5 − t d) 10 − a b

Since
10 − t c 20 10 − t d 20
≥ , ≥ ,
5 − tc 10 − t c 5 − td 10 − t d
it suffices to show that

20c 2 20d 2 1
+ ≥ .
(10 − ac)(10 − bc)(10 − t c) (10 − ad)(10 − bd)(10 − t d) 10 − a b

According to (*), the conclusion follows.


By the AC-Corollary, it suffices to prove the original inequality for a = b = c =
d = 1, for a = 0 and b = c = d = 4/3, and for a = b = 0 and c + d = 4. In the first
case, the equality holds. In the second case, the inequality becomes

3 27 2
+ ≤ .
10 74 3
We have
2 3 27 11 27 4
− − = − = > 0.
3 10 74 30 74 2220
In the third case, the inequality becomes

1 1
≤ ,
10 − cd 6
cd ≤ 4.
We have  2
c+d
cd ≤ = 4.
2
The equality holds for a = b = c = d = 1, and for a = b = 2 and c = d = 0 (or any
permutation).
Remark. In the same manner, we can prove the following generalizations (Vasile
Cîrtoaje, 2005):
Arithmetic Mean Method and Arithmetic Compensation Method 33

• Let a1 , a2 , . . . , an (n ≥ 3) be nonnegative numbers such that a1 + a2 +· · ·+ an = n.


n(n + 1)
(a) If k ≥ , then
2
 1 n(n − 1)
≤ ,
1≤i< j≤n
k − a a
i j 2(k − 1)

with equality for a1 = a2 = · · · = an = 1;


n2 n(n + 1)
(b) If <k≤ , then
4 2
 1 (n − 2)(n + 1) 4
≤ + ,
1≤i< j≤n
k − ai a j 2k 4k − n2

with equality for


n
a1 = a2 = , a3 = · · · = a n = 0
2
(or any permutation).

• Let a1 , a2 , . . . , an (n ≥ 3) be nonnegative numbers such that a1 + a2 +· · ·+ an = n,


and let  1
G(a1 , a2 , . . . , an ) = .
1≤i <···<i ≤n
k − a i1 a i2 · · · a i m
1 m

 
n
−1
m
(a) If k ≥    m , then
n m
−1
m n
G(a1 , a2 , . . . , an ) ≤ G(1, 1, . . . , 1) ;
 
n
 m m −1
m
(b) If < k ≤    m , then
n n m
−1
m n
n n
G(a1 , a2 , . . . , an ) ≤ G( , . . . , , 0 . . . , 0) .
m m

P 1.9. If a, b, c, d ≥ 0 such that a + b + c + d = 4 , then

3(a3 + b3 + c 3 + d 3 − 4) ≥ (a2 + b2 + c 2 + d 2 − 4)(a2 + b2 + c 2 + d 2 − 1).

(Vasile C., 2005)


34 Vasile Cîrtoaje

Solution. Write the inequality as

F (a, b, c, d) ≥ 0,

where
 
5 2 9
F (a, b, c, d) = 3(a3 + b3 + c 3 + d 3 − 4) − a2 + b2 + c 2 + d 2 − + ,
2 4
and apply the AC-Corollary. We will show that

F (a, b, c, d) < F (t, t, c, d) (*)

involves
F (a, b, c, d) ≥ F (0, 2t, c, d), (**)
where
a+b
t= , a = b.
2
Using the identities

a2 + b2 − 2t 2 = 2(t 2 − a b), a3 + b3 − 2t 3 = 3(t 2 − a b)(a + b),

we may write (*) as

3(a3 + b3 − 2t 3 ) − (a2 + b2 − 2t 2 )(2t 2 + a2 + b2 + 2c 2 + 2d 2 − 5) < 0,

9(t 2 − a b)(a + b) − 2(t 2 − a b)(2t 2 + a2 + b2 + 2c 2 + 2d 2 − 5) < 0,


9(a + b) − 2(2t 2 + a2 + b2 + 2c 2 + 2d 2 − 5) < 0. (A)
Since

a2 + b2 − (a + b)2 = −2a b, a3 + b3 − (a + b)3 = −3a b(a + b),

we may write (**) as


    
3 a3 + b3 − (a + b)3 − a2 + b2 − (a + b)2 (a + b)2 + a2 + b2 + 2c 2 + 2d 2 − 5 ≥ 0,

−9a b(a + b) + 2a b(4t 2 + a2 + b2 + 2c 2 + 2d 2 − 5) ≥ 0,


which is true if

9(a + b) − 2(4t 2 + a2 + b2 + 2c 2 + 2d 2 − 5) ≤ 0.

Clearly, this inequality follows immediately from (A). As a consequence, (*) implies
(**). Thus, by the AC-Corollary, we have

F (a, b, c, d) ≥ min f (k),


1≤k≤4
Arithmetic Mean Method and Arithmetic Compensation Method 35

where
16(k − 1)(4 − k)
f (k) =
k2
is the value of F for the case where 4 − k of the variables a, b, c, d are zero and the
other k variables are equal to 4/k. We have
32
f (1) = f (4) = 0, f (2) = 8, f (3) = ,
9
therefore F (a, b, c, d) ≥ 0.
The equality holds for a = b = c = 0 and d = 4 (or any cyclic permutation), and
also for a = b = c = d = 1.

P 1.10. If a, b, c, d ≥ 0 such that a + b + c + d = 4 , then

7(a3 + b3 + c 3 + d 3 − 4) ≥ (a2 + b2 + c 2 + d 2 − 4)(a2 + b2 + c 2 + d 2 + 11).

(Vasile C., 2005)

Solution. Write the inequality as F (a, b, c, d) ≥ 0, where


 
7 2 225
F (a, b, c, d) = 7(a3 + b3 + c 3 + d 3 − 4) − a2 + b2 + c 2 + d 2 + + ,
2 4

First Solution. Apply the AM-Corollary. We will show first that

F (a, b, c, d) ≥ F (t, b, t, d)

for
a+c
a ≥ b ≥ c ≥ d, t= .
2
Using the identities

a2 + c 2 − 2t 2 = 2(t 2 − ac), a3 + c 3 − 2t 3 = 3(a + c)(t 2 − ac),

we write the desired inequality F (a, b, c, d) ≥ F (t, b, t, d) as

7(a3 + c 3 − 2t 3 ) − (a2 + c 2 − 2t 2 )(2t 2 + a2 + c 2 + 2b2 + 2d 2 + 7) ≥ 0,

21(t 2 − ac)(a + c) − 2(t 2 − ac)(2t 2 + a2 + c 2 + 2b2 + 2d 2 + 7) ≥ 0.


This is true if

21(a + c) − (4t 2 + 2a2 + 2c 2 + 4b2 + 4d 2 + 14) ≥ 0.

Since

4t 2 +2a2 +2c 2 +4b2 +4d 2 = (a + c)2 +2(a2 + c 2 )+4(b2 + d 2 ) ≤ 3(a + c)2 +4(b + d)2 ,
36 Vasile Cîrtoaje

it suffices to show that

21(a + c) − 3(a + c)2 − 4(b + d)2 − 14 ≥ 0.

Substituting
x = a + c, y = b + d,
we need to prove that
21x − 3x 2 − 4 y 2 − 14 ≥ 0
for x + y = 4, x ≥ y. We have

21x(x + y) 7(x + y)2


21x − 3x 2 − 4 y 2 − 14 = − 3x 2 − 4 y 2 −
4 8
11x 2 + 28x y − 39 y 2
=
8
(x − y)(11x + 39 y)
= ≥ 0.
8
By the AM-Corollary, it suffices to prove the original inequality for a = b = c; that
is, to show that 3a + d = 4 involves

7(3a3 + d 3 − 4) ≥ (3a2 + d 2 − 4)(3a2 + d 2 + 11),

which is equivalent to

84(1 − a)2 (5 − 2a) ≥ 36(1 − a)2 (4a2 − 8a + 9),

(1 − a)2 (1 + 2a)(4 − 3a) ≥ 0,


(1 − a)2 (1 + 2a)d ≥ 0,
The equality holds for a = b = c = d = 1, and also for a = b = c = 4/3 and
d = 0 (or any cyclic permutation).

Second Solution. Apply the AC-Corollary. We will show that

F (a, b, c, d) < F (t, t, c, d) (*)

involves
F (a, b, c, d) ≥ F (0, 2t, c, d), (**)
where
a+b
t= , a = b.
2
As shown at the first solution, (*) involves

21(a + b) − 2(2t 2 + a2 + b2 + 2c 2 + 2d 2 + 7) < 0. (A)


Arithmetic Mean Method and Arithmetic Compensation Method 37

Since

a2 + b2 − (a + b)2 = −2a b, a3 + b3 − (a + b)3 = −3a b(a + b),

we may write (**) as


    
7 a3 + b3 − (a + b)3 − a2 + b2 − (a + b)2 (a + b)2 + a2 + b2 + 2c 2 + 2d 2 + 7 ≥ 0,

−21a b(a + b) + 2a b(4t 2 + a2 + b2 + 2c 2 + 2d 2 + 7) ≥ 0,


which is true if

21(a + b) − 2(4t 2 + a2 + b2 + 2c 2 + 2d 2 + 7) ≤ 0.

Clearly, this inequality follows immediately from (A). As a consequence, (*) implies
(**). Thus, by the AC-Corollary, we have

F (a, b, c, d) ≥ min f (k),


1≤k≤n

where
16(k − 3)(k − 4)
f (k) =
k2
is the value of F for the case where 4 − k of the variables a, b, c, d are zero and the
other k variables are equal to 4/k. We have

f (1) = 96, f (2) = 8, f (3) = f (4) = 0,

therefore F (a, b, c, d) ≥ 0.

P 1.11. Let a1 , a2 , . . . , an (n ≥ 3) be nonnegative real numbers such that

a1 + a2 + · · · + an = n.

Then
2 3   n 2 
a1 + a23 + · · · + an3 + ai1 ai2 ai3 ≥ a + a22 + · · · + an2 .
3 1≤i <i <i
3 1
1 2 3 ≤n

(Vasile C., 2005)

Solution. Let
b = a3 + · · · + a n
and
  
F (a1 , a2 , . . . , an ) = k1 a13 + · · · + an3 + ai1 ai2 ai3 − k2 n(a12 + · · · + an2 ),
1≤i1 <i2 <i3 ≤n
38 Vasile Cîrtoaje

where k1 and k2 are fixed real numbers. We claim that

F (a1 , a2 , a3 , . . . , an ) < F (t, t, a3 , . . . , an )

involves
F (a1 , a2 , a3 , . . . , an ) ≥ F (0, 2t, a3 , . . . , an )
where
a1 + a2
t= , a1 = a2 .
2
Since
  
ai1 ai2 ai3 = a1 a2 b + (a1 + a2 ) a i1 a i2 + a i1 a i2 a i3 ,
1≤i1 <i2 <i3 ≤n 3≤i1 <i2 ≤n 3≤i1 <i2 <i3 ≤n

a12 + a22 − 2t 2 = 2(t 2 − a1 a2 ), a13 + a23 − 2t 3 = 3(a1 + a2 )(t 2 − a1 a2 ),


the inequality F (a1 , a2 , a3 , . . . , an ) < F (t, t, a3 , . . . , an ) is equivalent to

k1 (a13 + a23 − 2t 3 ) + (a1 a2 − t 2 )b − k2 n(a12 + a22 − 2t 2 ) < 0,

(t 2 − a1 a2 )[3k1 (a1 + a2 ) − b − 2k2 n] < 0,


3k1 (a1 + a2 ) − b − 2k2 n < 0. (*)
On the other hand, since

a12 + a22 − (a1 + a2 )2 = −2a1 a2 , a13 + a23 − (a1 + a2 )3 = −3a1 a2 (a1 + a2 ),

the required inequality F (a1 , a2 , a3 , . . . , an ) ≥ F (0, 2t, a3 , . . . , an ) is equivalent to


   
k1 a13 + a23 − (a1 + a2 )3 + a1 a2 b − k2 n a12 + a22 − (a1 + a2 )2 ≥ 0,

a1 a2 [3k1 (a1 + a2 ) − b − 2k2 n] ≤ 0.


Clearly, this inequality follows immediately from (*).
According to the AC-Corollary, F (a1 , a2 , . . . , an ) is minimum when n − k of the
variables a1 , a2 , . . . , an are zero and the other k variables are equal to n/k, where
k ∈ {1, 2, . . . , n}. Thus,
 n n  n3
F (a1 , . . . , an−k , an−k+1 , . . . , an ) ≥ min F 0, . . . , 0, , . . . , = min f (k),
1≤k≤n k k 6 1≤k≤n
where    3
n3 n n k1 n3 k n3
f (k) = + 2 − 2 ,
6 3 k k k
3(2k2 + 1) 2(3k1 + 1)
f (k) = 1 − + .
k k2
We only need to show that f (k) ≥ 0 for all k ∈ {1, 2, . . . , n}.
Arithmetic Mean Method and Arithmetic Compensation Method 39

For the original inequality, we have


2 1
k1 = , k2 = ,
3 3
hence
5 6 (k − 2)(k − 3)
f (k) = 1 − + = .
k k2 k2
Clearly, for k ∈ {1, 2, . . . , n}, we have

f (k) ≥ 0,

with equality when k = 2 and k = 3. The original inequality is an equality for


n
a1 = a2 = , a3 = · · · = a n = 0
2
(or any permutation), and also for
n
a1 = a2 = a3 = , a4 = · · · = a n = 0
3
(or any permutation).
Remark. We can rewrite the inequality in the following homogeneous forms:
  
2 a13 + a23 + · · · + an3 + 3 ai1 ai2 ai3 ≥ (a1 + a2 + · · · + an )(a12 + a22 + · · · + an2 )
1≤i1 <i2 <i3 ≤n

and
 
a13 + a23 + · · · + an3 + 3 a i1 a i2 a i3 ≥ ai1 ai2 (ai1 + ai2 ).
1≤i1 <i2 <i3 ≤n 1≤i1 <i2 ≤n

For n = 3, we get the third degree Schur’s inequality

a13 + a23 + a33 + 3a1 a2 a3 ≥ a1 a2 (a1 + a2 ) + a2 a3 (a2 + a3 ) + a3 a1 (a3 + a1 ),

with equality for a1 = a2 and a3 = 0 (or any cyclic permutation), and also for
a1 = a 2 = a 3 .

P 1.12. Let a1 , a2 , . . . , an (n ≥ 4) be nonnegative real numbers such that

a1 + a2 + · · · + an = n.

Then
6   
a13 + a23 + · · · + an3 + ai1 ai2 ai3 ≥ 2 a12 + a22 + · · · + an2 .
n(n − 3) 1≤i <i <i
1 2 3 ≤n

(Vasile C., 2005)


40 Vasile Cîrtoaje

Solution. As shown at the preceding P 1.11, it suffices to show that f (k) ≥ 0 for
all k ∈ {1, 2, . . . , n}, where

3(2k2 + 1) 2(3k1 + 1)
f (k) = 1 − + ,
k k2
with
n(n − 3) n−3
k1 = , k2 = .
6 3
We get

2n − 3 (n − 1)(n − 2) (k − n + 1)(k − n + 2)
f (k) = 1 − + = .
k k2 k2
Clearly, for k ∈ {1, 2, . . . , n}, we have

f (k) ≥ 0,

with equality when k = n − 2 and k = n − 1. The original inequality is an equality


for
n
a1 = · · · = an−2 = , an−1 = an = 0
n−2
(or any permutation), and also for
n
a1 = · · · = an−1 = , an = 0
n−1
(or any cyclic permutation).

P 1.13. Let a1 , a2 , . . . , an (n ≥ 3) be nonnegative real numbers such that

a1 + a2 + · · · + an = n.

Then
  6   
(n + 1) a13 + a23 + · · · + an3 + ai1 ai2 ai3 ≥ 2n a12 + a22 + · · · + an2 .
n − 2 1≤i <i <i
1 2 3 ≤n

(Vasile C., 2005)

Solution. As shown at P 1.11, it suffices to show that f (k) ≥ 0 for all k ∈ {1, 2, . . . , n},
where
3(2k2 + 1) 2(3k1 + 1)
f (k) = 1 − + ,
k k2
with
(n + 1)(n − 2) n−2
k1 = , k2 = .
6 3
Arithmetic Mean Method and Arithmetic Compensation Method 41

We get
2n − 1 n(n − 1) (k − n)(k − n + 1)
f (k) = 1 − + = .
k k2 k2
Clearly, for k ∈ {1, 2, . . . , n}, we have

f (k) ≥ 0,

with equality when k = n − 1 and k = n. The original inequality is an equality for


n
a1 = · · · = an−1 = , an = 0
n−1
(or any cyclic permutation), and also for

a1 = a2 = · · · = an = 1.

Remark. For n = 4, we get the homogeneous inequality

2 2
a3 + b3 + c 3 + d 3 + a bc + bcd + cd a + d a b ≥ a (b + c + d),
3
where a, b, c, d are nonnegative real numbers. The equality holds for a = b = c
and d = 0 (or any cyclic permutation), and also for a = b = c = d.

P 1.14. Let a1 , a2 , . . . , an be nonnegative real numbers such that a1 + a2 +· · ·+ an = 1.


If
m ∈ {1, 2, . . . , n + 1},
then
   
m(m − 1) a13 + a23 + · · · + an3 + 1 ≥ (2m − 1) a12 + a22 + · · · + an2 .

(Vasile C., 2005)

Solution. We need to prove that F (a1 , a2 , . . . , an ) ≥ 0, where


   
F (a1 , a2 , . . . , an ) = m(m − 1) a13 + a23 + · · · + an3 + 1 − (2m − 1) a12 + a22 + · · · + an2 .

Assume that
F (a1 , a2 , a3 , . . . , an ) < F (t, t, a3 , . . . , an ) (*)
involves
F (a1 , a2 , a3 , . . . , an ) ≥ F (0, 2t, a3 , . . . , an ), (**)
where
a1 + a2
t= , a1 = a2 .
2
42 Vasile Cîrtoaje

Then, by AC-Corollary, we have

F (a1 , a2 , . . . , an ) ≥ min f (k),


1≤k≤n

where
(k − m)(k − m + 1)
f (k) =
k2
is the value of F for a1 = · · · = an−k = 0 and an−k+1 = · · · = an = 1/k. Obviously,
for k ∈ {1, 2, . . . , n}, we have
f (k) ≥ 0,
with equality when k = m − 1 (m ≥ 2) and k = m (m ≤ n). From f (k) ≥ 0 for
k ∈ {1, 2, . . . , n}, it follows that F (a1 , a2 , . . . , an ) ≥ 0.
To prove that (*) implies (**), we write (*) as

m(m − 1)(a13 + a23 − 2t 3 ) − (2m − 1)(a12 + a22 − 2t 2 ) < 0,

(t 2 − a1 a2 )[3m(m − 1)(a1 + a2 ) − 2(2m − 1)] < 0,


3m(m − 1)(a1 + a2 ) − 2(2m − 1) < 0, (A)
and (**) as

m(m − 1)(a13 + a23 − 8t 3 ) − (2m − 1)(a12 + a22 − 4t 2 ) ≥ 0,

a1 a2 [3m(m − 1)(a1 + a2 ) − 2(2m − 1)] ≤ 0. (B)


Clearly, (A) implies (B), hence (*) implies (**). This completes the proof.
For m = 1, the equality holds when n − 1 of the numbers ai are zero. For m ∈
{2, 3, . . . , n}, the equality holds when m or m − 1 of the numbers ai are equal and
the other numbers are zero. For m = n + 1, the equality holds when all numbers
a1 , a2 , ..., an are equal.

Remark. Actually, the inequality holds for all m ∈ [0, 1] ∪ {2, 3, . . . , n} ∪ [n+ 1, ∞),
but it does not hold for m ∈ (1, 2) ∪ (2, 3) ∪ · · · ∪ (n, n + 1).

P 1.15. If a1 , a2 , . . . , an are nonnegative real numbers such that a1 + a2 + · · · + an = n,


then  
(n + 1) a12 + a22 + · · · + an2 ≥ n2 + a13 + a23 + · · · + an3 .
(Vasile C., 2002)

Solution. Write the desired inequality as F (a1 , a2 , . . . , an ) ≥ 0, where


 
F (a1 , a2 , . . . , an ) = (n + 1) a12 + a22 + · · · + an2 − n2 − a13 − a23 − · · · − an3 .
Arithmetic Mean Method and Arithmetic Compensation Method 43

First Solution. Apply the AM-Corollary. First, we will show that

F (a1 , a2 , . . . , an−2 , an−1 , an ) ≥ F (t, a2 , . . . , an−2 , t, an )

for
a1 + an−1
a1 ≤ a2 ≤ · · · ≤ an−1 ≤ an , t= .
2
Write this inequality as follows:

(n + 1)(a12 + an−1
2
− 2t 2 ) − (a13 + an−1
3
− 2t 3 ) ≥ 0,

(t 2 − a1 an−1 )[2(n + 1) − 3(a1 + an−1 )] ≥ 0.


It is true if
2(n + 1) − 3(a1 + an−1 ) ≥ 0,
which is equivalent to the homogeneous inequalities

2(n + 1)(a1 + a2 + · · · + an ) − 3n(a1 + an−1 ) ≥ 0,

2(n + 1)(a2 + · · · + an−2 + an ) − (n − 2)(a1 + an−1 ) ≥ 0.


It suffices to show that

2(n + 1)an − (n − 2)(a1 + an−1 ) ≥ 0.

We have

2(n + 1)an − (n − 2)(a1 + an−1 ) > (n − 2)(2an − a1 − an−1 ) ≥ 0.

By the AM-Corollary, it suffices to prove the original inequality for a1 = a2 = · · · =


an−1 ; that is, to show that (n − 1)a1 + an = n involves
 
(n + 1) (n − 1)a12 + an2 ≥ n2 + (n − 1)a13 + an3 .

This inequality is equivalent to

n(n − 1)(n − 2)a1 (a1 − 1)2 ≥ 0.

The equality holds when a1 = a2 = · · · = an = 1, and also when

a1 = · · · = an−1 = 0, an = n

(or any cyclic permutation).


Second Solution. Apply the AC-Corollary. Assume that

F (a1 , a2 , a3 , . . . , an ) < F (t, t, a3 , . . . , an ) (*)

involves
F (a1 , a2 , a3 , . . . , an ) ≥ F (0, 2t, a3 , . . . , an ), (**)
44 Vasile Cîrtoaje

where
a1 + a2
t= , a1 = a2 .
2
Then, by AC-Corollary, we have

F (a1 , a2 , . . . , an ) ≥ min f (k),


1≤k≤n

where
n2 n3 n2 (k − 1)(n − k)
f (k) = (n + 1) − n2 − 2 =
k k k2
is the value of F for a1 = · · · = an−k = 0 and an−k+1 = · · · = an = n/k. Obviously,

f (k) ≥ 0

for k ∈ {1, 2, . . . , n}.


To prove that (*) implies (**), we write (*) as

(n + 1)(a12 + a22 − 2t 2 ) − (a13 + a23 − 2t 3 ) < 0,

(t 2 − a1 a2 )[2(n + 1) − 3(a1 + a2 )] < 0,


2(n + 1) − 3(a1 + a2 ) < 0, (A)
and (**) as
   
(n + 1) a12 + a22 − (a1 + a2 )2 − a13 + a23 − (a1 + a2 )3 ≥ 0,

a1 a2 [2(n + 1) − 3(a1 + a2 )] ≤ 0. (B)


Clearly, (A) implies (B), hence (*) implies (**). This completes the proof.

P 1.16. If a1 , a2 , . . . , an are nonnegative real numbers such that a1 + a2 + · · · + an = n,


then  
(n2 + n + 1) a12 + a22 + · · · + an2 − n ≥ a14 + a24 + · · · + an4 − n.
(Vasile C., 2002)

Solution. We need to prove that F (a1 , a2 , . . . , an ) ≥ 0, where


   
F (a1 , a2 , . . . , an ) = (n2 + n + 1) a12 + a22 + · · · + an2 − n − a14 + a24 + · · · + an4 − n .

First Solution. Apply the AM-Corollary. Let us show that

F (a1 , a2 , . . . , an−2 , an−1 , an ) ≥ F (t, a2 , . . . , an−2 , t, an )

for
a1 + an−1
a1 ≤ a2 ≤ · · · ≤ an−1 ≤ an , t= .
2
Arithmetic Mean Method and Arithmetic Compensation Method 45

Since
a12 + an−1
2
− 2t 2 = 2(t 2 − a1 an−1 ),
1
a14 + an−1
4
− 2t 4 = (t 2 − a1 an−1 )(7a12 + 7an−1 2
+ 10a1 an−1 ),
2
the inequality F (a1 , a2 , . . . , an−2 , an−1 , an ) ≥ F (t, a2 , . . . , an−2 , t, an ) can be written
as follows:

(n2 + n + 1)(a12 + an−1


2
− 2t 2 ) − (a14 + an−1
4
− 2t 3 ) ≥ 0,

4(n2 + n + 1)(t 2 − a1 an−1 ) − (t 2 − a1 an−1 )(7a12 + 7an−1


2
+ 10a1 an−1 ) ≥ 0.
Since t 2 − a1 an−1 ≥ 0, we need to show that

4(n2 + n + 1) ≥ 7a12 + 7an−1


2
+ 10a1 an−1 .

It suffices to prove that

4n2 ≥ 7a12 + 7an−1


2
+ 10a1 an−1

for n ≥ 3, which is equivalent to the homogeneous inequality

4(a1 + a2 + · · · + an )2 ≥ 7a12 + 7an−1


2
+ 10a1 an−1 .

Since

a1 + a2 + · · · + an ≥ (n − 2)a1 + an−1 + an ≥ (n − 2)a1 + 2an−1 ≥ a1 + 2an−1 ,

we only need to prove that

4(a1 + 2an−1 )2 ≥ 7a12 + 7an−1


2
+ 10a1 an−1 ,

which reduces to the obvious inequality


2
3an−1 + 2a1 an−1 ≥ a12 .

By the AM-Corollary, it suffices to prove the original inequality for a1 = a2 = · · · =


an−1 ; that is, to show that (n − 1)a1 + an = n involves

(n2 + n + 1)[(n − 1)a12 + an2 − n] ≥ (n − 1)a14 + an4 − n.

Since
(n − 1)a12 + an2 − n = n(n − 1)(a1 − 1)2 ,
 
(n−1)a14 +an4 −n = n(n−1)(a1 −1)2 (n2 − 3n + 3)a12 − 2(n2 − n − 1)a1 + n2 + n + 1 ,
the inequality is equivalent to

n(n − 1)a1 (a1 − 1)2 [2(n2 − n − 1) − (n2 − 3n + 3)a1 ] ≥ 0.


46 Vasile Cîrtoaje

It is true because
(n2 − 3n + 3)n n3 − n2 − 3n + 2
2(n2 −n−1)−(n2 −3n+3)a1 ≥ 2(n2 −n−1)− = > 0.
n−1 n−1
The equality holds when a1 = a2 = · · · = an = 1, and also when

a1 = · · · = an−1 = 0, an = n

(or any cyclic permutation).


Second Solution. Apply the AC-Corollary. Assume that

F (a1 , a2 , a3 , . . . , an ) < F (t, t, a3 , . . . , an ) (*)

involves
F (a1 , a2 , a3 , . . . , an ) ≥ F (0, 2t, a3 , . . . , an ), (**)
where
a1 + a2
t= , a1 = a2 .
2
Then, by AC-Corollary, we have

F (a1 , a2 , . . . , an ) ≥ min f (k),


1≤k≤n

where
   4 
n2 n n2 (k − 1)(n − k)(nk + n + k)
f (k) = (n2 + n + 1) −n − 3 −n =
k k k3
is the value of F for a1 = · · · = an−k = 0 and an−k+1 = · · · = an = n/k. Obviously,

f (k) ≥ 0

for k ∈ {1, 2, . . . , n}.


Let us show now that (*) involves (**). Since
1 2
a12 + a22 − 2t 2 = 2(t 2 − a1 a2 ), a14 + a24 − 2t 4 = (t − a1 a2 )(7a12 + 7a22 + 10a1 a2 ),
2
we may write (*) as follows:

(n2 + n + 1)(a12 + a22 − 2t 2 ) − (a14 + a24 − 2t 4 ) < 0,

4(n2 + n + 1)(t 2 − a1 a2 ) − (t 2 − a1 a2 )(7a12 + 7a22 + 10a1 a2 ) < 0,


7a12 + 7a22 + 10a1 a2 > 4(n2 + n + 1). (A)
Since

a12 + a22 − (a1 + a2 )2 = −2a1 a2 , a14 + a24 − (a1 + a2 )4 = −a1 a2 (4a12 + 4a22 + 6a1 a2 ),
Arithmetic Mean Method and Arithmetic Compensation Method 47

we may write (**) as follows:


   
(n2 + n + 1) a12 + a22 − (a1 + a2 )2 − a14 + a24 − (a1 + a2 )4 ≥ 0,

a1 a2 [−2(n2 + n + 1) + (4a12 + 4a22 + 6a1 a2 )] ≥ 0.


This is true if
2(4a12 + 4a22 + 6a1 a2 ) ≥ 4(n2 + n + 1). (B)
Taking intp account (A), it suffices to show that

2(4a12 + 4a22 + 6a1 a2 ) ≥ 7a12 + 7a22 + 10a1 a2 ,

which is equivalent to (a1 − a2 )2 ≥ 0.

P 1.17. Let a1 , a2 , . . . , an be nonnegative real numbers such that a1 + a2 +· · ·+ an = n.


If m ≥ 3 is an integer, then

nm−1 − 1  2 
a1 + a22 + · · · + an2 − n ≥ a1m + a2m + · · · + anm − n.
n−1
(Vasile C., 2002)

Solution. Let
nm−1 − 1
r= .
n−1
We need to prove that F (a1 , a2 , . . . , an ) ≥ 0, where
   
F (a1 , a2 , . . . , an ) = r a12 + a22 + · · · + an2 − n − a1m + a2m + · · · + anm − n .

Assume that
F (a1 , a2 , a3 , . . . , an ) < F (t, t, a3 , . . . , an ) (*)
involves
F (a1 , a2 , a3 , . . . , an ) ≥ F (0, 2t, a3 , . . . , an ), (**)
where
a1 + a2
t= , a1 = a2 .
2
Then, by AC-Corollary, we have

F (a1 , a2 , . . . , an ) ≥ min f (k),


1≤k≤n

where  
n2 nm
f (k) = r − n − m−1 + n
k k
48 Vasile Cîrtoaje

is the value of F for a1 = · · · = an−k = 0 and an−k+1 = · · · = an = n/k. We need to


show that f (k) ≥ 0 for k ∈ {1, 2, . . . , n − 1}. Write the inequality f (k) ≥ 0 in the
form
nm−1 − k m−1
r ≥ m−2 .
k (n − k)
We have
nm−1 − 1  n m−2  n m−3 nm−1 − k m−1
r= = nm−2 + nm−3 + · · · + 1 ≥ + + · · · + 1 = m−2 .
n−1 k k k (n − k)
Let us show now that (*) involves (**). We may write (*) as
a1m + a2m − 2t m
> r,
a12 + a22 − 2t 2

a1m + a2m − 2t m
> 2r. (A)
t 2 − a1 a2
For the nontrivial case a1 a2 = 0, the desired inequality (**) is equivalent to
a1m + a2m − (a1 + a2 )m
≤ r,
a12 + a22 − (a1 + a2 )2

a1m + a2m − (2t)m


≥ 2r.
a1 a2
This is true if
a1m + a2m − (2t)m a1m + a2m − 2t m
≥ ,
a1 a2 t 2 − a1 a2
a1m + a2m + (2m − 2)a1 a2 t m−2 − (2t)m ≤ 0.
For m = 3, the inequality is an identity. Consider next that m ≥ 4. Due to homo-
geneity, we may set t = 1. Using the substitution

a1 = 1 + x, a2 = 1 − x, 0 ≤ x < 1,

we need to show that f (x) ≤ 0, where

f (x) = (1 + x)m + (1 − x)m + (2m − 2)(1 − x 2 ) − 2m , 0 ≤ x ≤ 1.

We have  
f (x) = m (1 + x)m−1 − (1 − x)m−1 − (2m+1 − 4)x,
 
f (t) = m(m − 1) (1 + x)m−2 + (1 − x)m−2 − 2m+1 + 4,
 
f (x) = m(m − 1)(m − 2) (1 + x)m−3 − (1 − x)m−3 .
Since f > 0, f is strictly increasing. Because

f (0) = 2(m2 − m + 2 − 2m ) < 0, f (1) = 2m−2 (m2 − m − 8) + 4 > 0,


Arithmetic Mean Method and Arithmetic Compensation Method 49

there exists x 1 ∈ (0, 1) such that f (x 1 ) = 0, f (x) < 0 for x ∈ [0, x 1 ) and
f (x) > 0 for x ∈ (x 1 , 1]. Consequently, f’ is strictly decreasing on [0, x 1 ] and
strictly increasing on [x 1 , 1]. Since

f (0) = 0, f (1) = 4 + (m − 4)2m−1 > 0,

there exists x 2 ∈ (x 1 , 1) such that f (x 2 ) = 0, f (x) < 0 for x ∈ (0, x 2 ) and f (x) >
0 for x ∈ (x 2 , 1]. Thus, f is strictly decreasing on [0, x 2 ] and strictly increasing on
[x 2 , 1]. Since f (0) = f (1) = 0, it follows that f (x) ≤ 0 for 0 ≤ x ≤ 1.
The equality holds when a1 = a2 = · · · = an = 1, and also when

a1 = · · · = an−1 = 0, an = n

(or any cyclic permutation).

P 1.18. If a, b, c, d, e are nonnegative real numbers such that a + b + c + d + e = 5,


then
5
a2 + b2 + c 2 + d 2 + e2 − 5 ≥ (1 − a bcd e).
4
(Vasile C., 2005)

Solution. We apply the AM-Corollary to the function

5
F (a, b, c, d, e) = a2 + b2 + c 2 + d 2 + e2 − 5 − (1 − a bcd e).
4
So, we need to show that

F (a, b, c, d, e) ≥ F (t, b, c, t, e)

for
a+d
a ≥ b ≥ c ≥ d ≥ e, t= .
2
We have
5
F (a, b, c, d, e) − F (t, b, c, t, e) = a2 + d 2 − 2t 2 − bce(t 2 − ad)
4
5
= 2(t 2 − ad) − bce(t 2 − ad)
4
1
= (t 2 − ad)(8 − 5bce).
4
It suffices to show that
8 − 5bce ≥ 0,
50 Vasile Cîrtoaje

which is equivalent to the homogeneous inequality

8(a + b + c + d + e)3 ≥ 625bce.

By the AM-GM inequality, we have



3

3
a + b + c + d + e ≥ 2b + c + 2e ≥ 3 (2b)c(2e) = 3 4bce,

hence
8(a + b + c + d + e)3 ≥ 8 · 27 · (4bce) = 864bce ≥ 625bce.
According to AM-Corollary, it suffices to prove the original inequality for a = b =
c = d. Thus, we need to show that 4a + e = 5 implies
5
4a2 + e2 − 5 ≥ (1 − a4 e),
4
which is equivalent to
5
20(1 − a)2 ≥ (1 − a)2 (4a3 + 3a2 + 2a + 1),
4
(1 − a)2 (15 − 2a − 3a2 − 4a3 ) ≥ 0,
(1 − a)2 (3 + 2a + a2 )(5 − 4a) ≥ 0,
(1 − a)2 (3 + 2a + a2 )e ≥ 0.
5
The equality holds for a = b = c = d = e = 1, and also for a = b = c = d =
4
and e = 0 (or any cyclic permutation).

P 1.19. If a1 , a2 , . . . , an are nonnegative real numbers such that a1 + a2 + · · · + an = n,


then
n
a12 + a22 + · · · + an2 − n ≥ (1 − a1 a2 · · · an ).
n−1
Solution. Write the inequality as F (a1 , a2 , . . . , an ) ≥ 0, where
n
F (a1 , a2 , . . . , an ) = a12 + a22 + · · · + an2 − n − (1 − a1 a2 · · · an ).
n−1
Assume that
F (a1 , a2 , a3 , . . . , an ) < F (t, t, a3 , . . . , an ) (*)
involves
F (a1 , a2 , a3 , . . . , an ) ≥ F (0, 2t, a3 , . . . , an ), (**)
where
a1 + a2
t= , a1 = a2 .
2
Arithmetic Mean Method and Arithmetic Compensation Method 51

On this assumption, by the AC-Corollary, we have

F (a1 , a2 , . . . , an ) ≥ min f (k),


1≤k≤n

where
f (n) = F (1, 1, . . . , 1) = 0
and
n2 n2
f (k) = − , k ∈ {1, 2, . . . , n − 1},
k n−1
is the value of F for a1 = · · · = an−k = 0 and an−k+1 = · · · = an = n/k. Since
f (k) ≥ 0 for k ∈ {1, 2, . . . , n − 1} (with equality when k = n − 1), we have

F (a1 , a2 , . . . , an ) ≥ 0.

To prove that (*) implies (**), we write these inequalities as


 n 
(t 2 − a1 a2 ) 2 − a3 · · · an < 0 (A)
n−1
and  n 
a1 a2 2 − a3 · · · an ≤ 0, (B)
n−1
respectively. Clearly, (A) implies (B), therefore (*) implies (**). This completes the
proof.
The equality holds when a1 = a2 = · · · = an = 1, and also when
n
a1 = 0, a2 = · · · = an =
n−1
(or any cyclic permutation).

P 1.20. If a, b, c, d, e are nonnegative real numbers such that a + b + c + d + e = 5,


then
45
a3 + b3 + c 3 + d 3 + e3 − 5 ≥ (1 − a bcd e).
16
(Vasile C., 2005)

Solution. We apply the AM-Corollary to the function

45
F (a, b, c, d, e) = a3 + b3 + c 3 + d 3 + e3 − 5 − (1 − a bcd e).
16
So, we need to show that

F (a, b, c, d, e) ≥ F (t, b, c, t, e)
52 Vasile Cîrtoaje

for
a+d
a ≥ b ≥ c ≥ d ≥ e, t= .
2
We have
45
F (a, b, c, d, e) − F (t, b, c, t, e) = a3 + d 3 − 2t 3 −bce(t 2 − ad)
16
45
= 3(t 2 − ad)(a + d) − bce(t 2 − ad)
16
3 2
= (t − ad)(16a + 16d − 15bce).
16
It suffices to show that
a + d ≥ bce,
which is equivalent to the homogeneous inequality

(a + d)(a + b + c + d + e)2 ≥ 16bce.

Let 
x= be.
Since

a + d ≥ b + e ≥ 2x, a + b + c + d + e ≥ 2b + c + 2e ≥ 4x + c,

it suffices to show that


2x(4x + c)2 ≥ 16c x 2 ,
which is equivalent to
2x(16x 2 + c 2 ) ≥ 0.
According to AM-Corollary, it suffices to prove the original inequality for a = b =
c = d. Thus, we need to show that 4a + e = 5 implies
45
4a3 + e3 − 5 ≥ (1 − a4 e),
16
which is equivalent to
45
60(1 − a)2 (2 − a) ≥ (1 − a)2 (1 + 2a + 3a2 + 4a3 ),
16
(1 − a)2 (125 − 70a − 9a2 − 12a3 ) ≥ 0,
(1 − a)2 (25 + 6a + 3a2 )(5 − 4a) ≥ 0,
(1 − a)2 (25 + 6a + 3a2 )e ≥ 0.
5
The equality holds for a = b = c = d = e = 1, and also for a = b = c = d =
4
and e = 0 (or any cyclic permutation).
Arithmetic Mean Method and Arithmetic Compensation Method 53

P 1.21. If a1 , a2 , . . . , an are nonnegative real numbers such that a1 + a2 + · · · + an = n,


then
n(2n − 1)
a13 + a23 + · · · + an3 − n ≥ (1 − a1 a2 · · · an ).
(n − 1)2
Solution. Write the inequality as F (a1 , a2 , . . . , an ) ≥ 0, where
n(2n − 1)
F (a1 , a2 , . . . , an ) = a13 + a23 + · · · + an3 − n − (1 − a1 a2 · · · an ).
(n − 1)2
We claim that
F (a1 , a2 , a3 , . . . , an ) < F (t, t, a3 , . . . , an ) (*)
involves
F (a1 , a2 , a3 , . . . , an ) ≥ F (0, 2t, a3 , . . . , an ), (**)
where
a1 + a2
t=
, a1 = a2 .
2
On this assumption, by the AC-Corollary, we have

F (a1 , a2 , . . . , an ) ≥ min f (k),


1≤k≤n

where
f (n) = F (1, 1, . . . , 1) = 0
and
n3 n3
f (k) = − , k ∈ {1, 2, . . . , n − 1},
k 2 (n − 1)2
is the value of F for a1 = · · · = an−k = 0 and an−k+1 = · · · = an = n/k. Since
f (k) ≥ 0 for k ∈ {1, 2, . . . , n − 1} (with equality when k = n − 1), we have

F (a1 , a2 , . . . , an ) ≥ 0.

To prove that (*) implies (**), we write these inequalities as



n(2n − 1)
(t 2 − a1 a2 ) 3(a1 + a2 ) − a3 · · · an < 0 (A)
(n − 1)2
and 
n(2n − 1)
a1 a2 3(a1 + a2 ) − a3 · · · an ≤ 0, (B)
(n − 1)2
respectively. Clearly, (A) implies (B), therefore (*) implies (**). This completes the
proof.
The equality holds when a1 = a2 = · · · = an = 1, and also when
n
a1 = 0, a2 = · · · = an =
n−1
(or any cyclic permutation).
54 Vasile Cîrtoaje

P 1.22. If a1 , a2 , . . . , an are nonnegative real numbers such that a1 + a2 + · · · + an = n,


then
 
1 3  n−2 2
a1 + a23 + · · · + an3 − n ≥ a12 + a22 + · · · + an2 − n + (a1 a2 · · · an − 1).
n 2

(Vasile C., 2005)

Solution. Write the inequality as F (a1 , a2 , . . . , an ) ≥ 0, where

1 3   
F (a1 , a2 , . . . , an ) = a + a23 + · · · + an3 − n − a12 + a22 + · · · + an2 − n
n 1
 
n−2 2
− (a1 a2 · · · an − 1).
2

If
F (a1 , a2 , a3 , . . . , an ) < F (t, t, a3 , . . . , an ) (*)
involves
F (a1 , a2 , a3 , . . . , an ) ≥ F (0, 2t, a3 , . . . , an ), (**)
where
a1 + a2
, t= a1 = a2 ,
2
then, by the AC-Corollary, we have

F (a1 , a2 , . . . , an ) ≥ min f (k),


1≤k≤n

where
f (n) = F (1, 1, . . . , 1) = 0
and
   2   
1 n3 n n − 2 2 (k − 2)2 n2
f (k) = − n − − n + = , k ∈ {1, 2, . . . , n − 1},
n k2 k 2 4k2

is the value of F for a1 = · · · = an−k = 0 and an−k+1 = · · · = an = n/k. Since


f (k) ≥ 0 (with equality when k = 2), we have

F (a1 , a2 , . . . , an ) ≥ 0.

To prove that (*) implies (**), we write these inequalities as



3 (n − 2)2
(t 2 − a1 a2 ) (a1 + a2 ) − 2 + a3 · · · a n < 0 (A)
n 4
and 
3 (n − 2)2
a1 a2 (a1 + a2 ) − 2 + a3 · · · an ≤ 0, (B)
n 4
Arithmetic Mean Method and Arithmetic Compensation Method 55

respectively. Clearly, (A) implies (B), therefore (*) implies (**). This completes the
proof.
The equality holds when a1 = a2 = · · · = an = 1, and also when

a1 = · · · = an−2 = 0, an−1 = an = n/2

(or any permutation).

P 1.23. If a1 , a2 , . . . , an are nonnegative real numbers such that a1 + a2 + · · · + an = n,


then

n(n − 1)  
a13 + a23 + · · · + an3 − n ≥ a12 + a22 + · · · + an2 − n.
n+1
(Vasile C., 2006)

Solution. Write the inequality as F (a1 , a2 , . . . , an ) ≥ 0, where

n(n − 1)  3 
F (a1 , a2 , . . . , an ) = a1 + a23 + · · · + an3 − n − (a12 + a22 + · · · + an2 − n)2 .
n+1
If
F (a1 , a2 , a3 , . . . , an ) < F (t, t, a3 , . . . , an ) (*)
involves
F (a1 , a2 , a3 , . . . , an ) ≥ F (0, 2t, a3 , . . . , an ), (**)
where
a1 + a2
, t= a1 = a2 ,
2
then, by the AC-Corollary, we have

F (a1 , a2 , . . . , an ) ≥ min f (k),


1≤k≤n

where f (k) is the value of F for a1 = · · · = an−k = 0 and an−k+1 = · · · = an = n/k;


that is,
   2 2
n(n − 1) n3 n 2n3 (k − 1)(n − k)
f (k) = − n − − n = .
n+1 k2 k k2 (n + 1)

Since f (k) ≥ 0 (with equality when k = 1 and k = n), we have

F (a1 , a2 , . . . , an ) ≥ 0.

Using the identities

a12 + a22 − 2t 2 = 2(t 2 − a1 a2 ), a13 + a23 − 2t 3 = 3(a1 + a2 )(t 2 − a1 a2 ),


56 Vasile Cîrtoaje

we may write (*) as


 
n(n − 1) 3 a12 + a22
(a1 + a23 − 2t 3 ) − 2(a12 + a22 − 2t 2 ) t 2 + + a32 + · · · + an2 − n < 0,
n+1 2
  
3n(n − 1) a1 + a2
2 2
(t − a1 a2 )
2
(a1 + a2 ) − 4 t + 2
+ a3 + · · · + an − n < 0,
2 2
n+1 2
 
3n(n − 1) a12 + a22
(a1 + a2 ) − 4 t 2 + + a32 + · · · + an2 − n < 0. (A)
n+1 2
Since

a12 + a22 − (a1 + a2 )2 = −2a1 a2 , a13 + a23 − (a1 + a2 )3 = −3a1 a2 (a1 + a2 ),

we may write (**) as


n(n − 1) 3
[a1 + a23 − (a1 + a2 )3 ] −
n+1
 
a12 + a22
−2[a1 + a2 − (a1 + a2 ) ] 2t +
2 2 2 2
+ a3 + · · · + an − n ≥ 0,
2 2
2
  
3n(n − 1) a12 + a22
a1 a2 (a1 + a2 ) − 4 2t 2 + + a32 + · · · + an2 − n ≤ 0. (B)
n+1 2
Clearly, (A) implies (B), therefore (*) implies (**). This completes the proof.
The equality holds when a1 = a2 = · · · = an = 1, and also when

a1 = · · · = an−1 = 0, an = n

(or any cyclic permutation).

P 1.24. If a, b, c, d are nonnegative real numbers such that a + b + c + d = 4, then




313 5
2 (a4 + b4 + c 4 + d 4 ) + + ≥ a2 + b2 + c 2 + d 2 .
81 9
(Vasile C., 2006)
Solution. Write the inequality in the form
  313  2 
5 2
2 a4 + b4 + c 4 + d 4 + ≥ a + b2 + c 2 + d 2 − .
81 9
Consider the more general inequality F (a, b, c, d) ≥ 0, where
   2
F (a, b, c, d) = k1 a4 + b4 + c 4 + d 4 + k2 − k3 (a2 + b2 + c 2 + d 2 ) − k4 ,
Arithmetic Mean Method and Arithmetic Compensation Method 57

with
2k32 ≥ k1 > 0, 4k3 ≥ k4 k3 > 0.
We will show that
F (a, b, c, d) < F (t, t, c, d) (*)
involves
F (a, b, c, d) ≥ F (0, 2t, c, d), (**)
where
a+b
t= , a = b.
2
Using the identities
1
a2 + b2 − 2t 2 = 2(t 2 − a b), a4 + b4 − 2t 4 = (t 2 − a b)(7a2 + 7b2 + 10a b),
2
we may write (*) as
 
a2 + b2
k1 (a4 + b4 − 2t 4 ) − 2k3 (a2 + b2 − 2t 2 ) k3 (t 2 + + c 2 + d 2 ) − k4 < 0,
2
  
k1 (7a + 7b + 10a b)
2 2
a + b2
2
(t 2 − a b) − 4k3 k3 (t 2 + + c 2 + d 2 ) − k4 < 0,
2 2

k1 (7a2 + 7b2 + 10a b) a 2
+ b 2
− 4k3 k3 (t 2 + + c 2 + d 2 ) − k4 < 0. (A)
2 2
Since

a2 + b2 − (a + b)2 = −2a b, a4 + b4 − (a + b)4 = −a b(4a2 + 4b2 + 6a b),

we may write (**) as


  a2 + b2
k1 [a4 +b4 −(a+b)4 ]−2k3 a2 + b2 − (a + b)2 k3 (2t 2 + + c 2 + d 2 ) − k4 ≥ 0,
2

a2 + b2
a bk1 (4a2 + 4b2 + 6a b) − 4k3 a b k3 (2t 2 + + c 2 + d 2 ) − k4 ≤ 0.
2
This inequality is true if

a2 + b2
k1 (4a2 + 4b2 + 6a b) − 4k3 k3 (2t 2 + + c 2 + d 2 ) − k4 ≤ 0,
2
which is equivalent to

k1 (7a2 + 7b2 + 10a b) a2 + b2
+ 2k1 t 2 − 4k3 k3 (2t 2 + + c 2 + d 2 ) − k4 ≤ 0. (B)
2 2
Clearly, (A) implies (B) if
2k1 t 2 − 4k32 t 2 ≤ 0,
58 Vasile Cîrtoaje

which is true because k1 ≤ 2k32 . As a consequence, (*) implies (**). Thus, by the
AC-Corollary, we have
F (a, b, c, d) ≥ min f (k),
1≤k≤n

where
   2 2
4 4 44 k1 4 k3
f (k) = min F 0, . . . , 0, , . . . , = 3 + k2 − − k4
1≤k≤4 k k k k

is the value of F for the case where 4 − k of the variables a, b, c, d are zero and the
other k variables are equal to 4/k. Since

42 k3
− k4 ≥ 4k3 − k4 ≥ 0,
k
we only need to show that g(k) ≥ 0 for k ∈ {1, 2, 3, 4}, where


256k 16k3
1
g(k) = + k2 − + k4 .
k3 k
For the original inequality, we have
313 5
k1 = 2, k2 = , k3 = 1, k4 = ,
81 9
5
2k32 − k1 = 0, 4k3 − k4 = 4 − >0, k3 > 0,
9


512 313 16 5
g(k) = + − + , k ∈ {1, 2, 3, 4}.
k3 81 k 9
Since

313 5

g(1) = 512 + − 16 + > 512 − 16 > 0,


81 9


313 5 
g(2) = 64 + − 8 + > 64 − 8 = 0,
81 9


512 313 16 5 43 16 5
g(3) = + − + = − + = 0,
27 81 3 9 9 3 9


313 5 31 5
g(4) = 8 + −4+ = − 4 + = 0,
81 9 9 9
the conclusion follows.
The equality holds for a = 0 and b = c = d = 4/3 (or any cyclic permutation),
and also for a = b = c = d = 1.
Arithmetic Mean Method and Arithmetic Compensation Method 59

P 1.25. If a, b, c, d are nonnegative real numbers such that a + b + c + d = 4, then




a4 + b4 + c 4 + d 4 − 4
2 ≥ a2 + b2 + c 2 + d 2 − 4.
7

(Vasile C., 2006)

Solution. Write the inequality as F (a, b, c, d) ≥ 0, where


   2
F (a, b, c, d) = k1 a4 + b4 + c 4 + d 4 + k2 − k3 (a2 + b2 + c 2 + d 2 ) − k4 ,

with
4 16
k1 = , k2 = − , k3 = 1, k4 = 4,
7 7
4
2k32 − k1 = 2 − > 0, 4k3 − k4 = 0 k3 > 0.
7
As shown in the proof of the preceding P 1.24, we only need to show that g(k) ≥ 0
for k = 1, 2, 3, 4, where


256k 16k3 4
64 − k3 16
1
g(k) = + k2 − + k4 = − + 4.
k3 k k 7k k

We have

63
g(1) = 4 − 12 = 0,
7


56
g(2) = 2 − 4 = 0,
14

4
37 16 4 16
g(3) = − +4> − + 4 = 0,
3 21 3 3 3
g(4) = 0.
The equality holds for a = b = c = 0 and d = 4 (or any cyclic permutation), for
a = b = 0 and c = d = 2 (or any permutation), and also for a = b = c = d = 1.

P 1.26. If a, b, c, d are nonnegative real numbers such that a + b + c + d = 4, then


  
16 4 3  2 
a4 + b4 + c 4 + d 4 +
− 8 ≥
− a + b2 + c 2 + d 2 .
3 3 2

(Vasile C., 2006)


60 Vasile Cîrtoaje

Solution. Write the inequality as F (a, b, c, d) ≥ 0, where


   2
F (a, b, c, d) = k1 a4 + b4 + c 4 + d 4 + k2 − k3 (a2 + b2 + c 2 + d 2 ) − k4 ,

with
4 3 16
k1 = 1, k2 = 0, k3 =
− , k4 =
− 8,
3 2 3

91 − 48 3

2k3 − k1 =
2
> 4(3 − 2 3 ) > 0, 4k3 − k4 = 2 > 0.
6
As shown in the proof of P 1.24, we only need to show that g(k) ≥ 0 for k =
1, 2, 3, 4, where


256k 16k3
1
g(k) = + k2 − + k4 = 8h(k),
k3 k
 
2 2 4 3
h(k) =

− 1 + − 1.
k k 3 k k
We have

h(1) = 2(2 − 3 ) > 0,


1 2 1
h(2) =

+ > 0,
2 3 2
h(3) = 0,
h(4) = 0.
The equality holds when a = 0 and b = c = d = 4/3 (or any cyclic permutation),
and also when a = b = c = d = 1.

P 1.27. If a, b, c, d are nonnegative real numbers such that a + b + c + d = 4, then


  

1  2 
a4 + b4 + c 4 + d 4 + 8(2 − 2) ≥ 2 −
a + b2 + c 2 + d 2 .
2
(Vasile C., 2006)

Solution. Write the inequality as F (a, b, c, d) ≥ 0, where


   2
F (a, b, c, d) = k1 a4 + b4 + c 4 + d 4 + k2 − k3 (a2 + b2 + c 2 + d 2 ) − k4 ,

with
1

k1 = 1, k2 = 0, k3 = 2 −
, k4 = 8(2 − 2 ),
2

2k32 − k1 > 2 − 1 > 0, 4k3 − k4 = 2(3 2 − 4) > 0.


Arithmetic Mean Method and Arithmetic Compensation Method 61

As shown in the proof of P 1.24, we only need to show that g(k) ≥ 0 for k =
1, 2, 3, 4, where


256k 16k3
1
g(k) = + k2 − + k4 = 16h(k),
k3 k
 
1 2 1 1
h(k) =
+ 1 − −
1 − .
k k k 2 k
We have
h(1) = 0,
h(2) = 0,
 
1 1

h(3) = 1 +
− 2 > 0,
3 3

5−3 2
h(4) = > 0.
8
The equality holds when a = b = c = 0 and d = 4 (or any cyclic permutation),
and also when a = b = 0 and c = d = 2 (or any permutation).

P 1.28. If a, b, c, d are nonnegative real numbers such that a + b + c + d = 4, then


    
1 1 3 2  2 
a4 + b4 + c 4 + d 4 + 16



a + b2 + c 2 + d 2 .
2 3 2 3
(Vasile C., 2006)

Solution. Write the inequality as F (a, b, c, d) ≥ 0, where


   2
F (a, b, c, d) = k1 a4 + b4 + c 4 + d 4 + k2 − k3 (a2 + b2 + c 2 + d 2 ) − k4 ,

with  
3 2 1 1
k1 = 1, k2 = 0, k3 =

, k4 = 16

,
2 3 2 3
 
2 1
2k32 − k1 > 0, 4k3 − k4 = 4

> 0.
3 2
As shown in the proof of P 1.24, we only need to show that g(k) ≥ 0 for k =
1, 2, 3, 4, where


256k 16k3
1
g(k) = + k2 − + k4 = 16h(k),
k3 k
   
1 1 3 1 2
h(k) =
+
1 − −
1− .
k k 2 k 3 k
62 Vasile Cîrtoaje

We have

1 3 1
h(1) = 1 − 2 +
> 1 − + = 0,
3 2 2
h(2) = 0,
h(3) = 0,
 
1 1 1 1
h(4) = +

> 0.
2 4 2 2 3
The equality holds when a = b = 0 and c = d = 2 (or any permutation), and
also when a = 0 and b = c = d = 4/3 (or any cyclic permutation).

P 1.29. If a, b, c, d are nonnegative real numbers such that a2 + b2 + c 2 + d 2 = 4, then

a3 + b3 + c 3 + d 3 + a bc + bcd + cd a + d a b ≤ 8.

(Vasile C., 2006)

Solution. Write the inequality as G(a, b, c, d) ≥ 0, where

G(a, b, c, d) = 8 − ac(b + d) − (a + c)bd − (a3 + b3 + c 3 + d 3 ),

and apply the AM1-Corollary. First, we will show that

G(a, b, c, d) ≥ G(t, b, t, d)

for

a2 + c 2
a ≤ b ≤ c ≤ d, t= .
2
Write this inequality as follows:

(t 2 − ac)(b + d) + (2t − a − c)bd − (a3 + c 3 − 2t 3 ) ≥ 0,

4t 2 − (a + c)2 (a3 + c 3 )2 − 4t 6
(t 2 − ac)(b + d) + bd − 3 ≥ 0,
2t + a + c a + c 3 + 2t 3
(a − c)2 (b + d) (a − c)2 (a − c)2 [(a2 + ac + c 2 )2 − 3a2 c 2 ]
+ bd − ≥ 0,
2 2t + a + c 2(a3 + c 3 + 2t 3 )
which is true if
2bd (a2 + ac + c 2 )2
b+d+ ≥ 3 .
2t + a + c a + c 3 + 2t 3
Since
b + d ≥ a + c, bd ≥ ac,
2t + a + c ≤ 3(a + c)
Arithmetic Mean Method and Arithmetic Compensation Method 63

and
1
a3 + c 3 + 2t 3 ≥ a3 + c 3 + (a + c)(a2 + c 2 ) ≥ a3 + c 3 + ac(a + c) = (a + c)(a2 + c 2 ),
2
it suffices to show that

2ac (a2 + ac + c 2 )2
a+c+ ≥ ,
3(a + c) (a + c)(a2 + c 2 )

which is equivalent to

3(a2 + ac + c 2 )2
3(a2 + c 2 ) + 8ac ≥ ,
a2 + c 2

2ac(a2 + c 2 ) ≥ 3a2 c 2 ,
2ac(a − c)2 + a2 c 2 ≥ 0.
By the AM1-Corollary, it suffices to prove the original inequality for a = b = c;
that is, to show that 3a2 + d 2 = 4 involves

4a3 + d 3 + 3a2 d ≤ 8.

This inequality is equivalent to

4a3 + 4d ≤ 8,

2 − a3 ≥ d.
Since 3a2 ≤ 4, we have
8
2 − a3 ≥ 2 −
> 0.
3 3
Thus, we only need to show that

(2 − a3 )2 ≥ d 2 ,

which is equivalent to
(2 − a3 )2 ≥ 4 − 3a2 ,
a2 (a4 − 4a + 3) ≥ 0,
a2 (a − 1)2 (a2 + 2a + 3) ≥ 0.
The equality holds for a = b = c = d = 1, and also for a = b = c = 0 and d = 2 (or
any cyclic permutation).
64 Vasile Cîrtoaje

P 1.30. If a, b, c, d are nonnegative real numbers such that a2 + b2 + c 2 + d 2 = 2, then

a3 + b3 + c 3 + d 3 + a bc + bcd + cd a + d a b ≥ 2.

(Vasile C., 2006)

Solution. Write the inequality as G(a, b, c, d) ≥ 0, where

G(a, b, c, d) = a3 + b3 + c 3 + d 3 + ka b(c + d) + k(a + b)cd − m, k = 1, m = 2,

and apply the AC1-Corollary. First, we will show that

G(a, b, c, d) < G(t, t, c, d)

involves

G(a, b, c, d) ≥ G(0, 2 t, c, d)
for any k > 0 and real m, where


a2 + b2
t= , a = b.
2
Write the hypothesis G(a, b, c, d) < G(t, t, c, d) as

(a3 + b3 − 2t 3 ) − k(t 2 − a b)(c + d) − k(2t − a − b)cd < 0,

(a3 + b3 )2 − 4t 6 4t 2 − (a + b)2
− k(t 2 − a b)(c + d) − k · cd < 0,
a + b + 2t
3 3 3 2t + a + b
(a − b) [a + b + 2a b(a + b )] k(a − b) (c + d)
2 4 4 2 2 2
k(a − b)2
− − cd < 0,
2(a + b + 2t )
3 3 3 2 2t + a + b
 
a4 + b4 + 2a b(a2 + b2 ) 2cd
<k c+d+ . (*)
a + b + 2t
3 3 3 2t + a + b

Write now the required inequality G(a, b, c, d) ≥ G(0, 2 t, c, d) as follows:



ka b(c + d) + k(a + b − 2 t)cd ≥ 2 2 t 3 − a3 − b3 ,

(a + b)2 − 2t 2 8t 6 − (a3 + b3 )2
ka b(c + d) + k ·
cd ≥
,
a+b+ 2 t 2 2 t 3 + a3 + b3
2ka bcd a2 b2 (3a2 + 3b2 − 2a b)
ka b(c + d) +

,
a+b+ 2 t 2 2 t 3 + a3 + b3
2kcd a b(3a2 + 3b2 − 2a b)
k(c + d) +

.
a+b+ 2t 2 2 t 3 + a3 + b3
This inequality is true if

2kcd a b(3a2 + 3b2 − 2a b)


k(c + d) + ≥ .
a + b + 2t 2t 3 + a3 + b3
Arithmetic Mean Method and Arithmetic Compensation Method 65

Having in view (*), it suffices to show that

a b(3a2 + 3b2 − 2a b) ≤ a4 + b4 + 2a b(a2 + b2 ),

which is equivalent to
(a2 + b2 )(a2 + b2 − a b) ≥ 0.
By the AC1-Corollary, it suffices to prove that




  
2 2 2 2
2
2
2
G , , , ≥ 0, G 0, , , ≥ 0,
2 2 2 2 3 3 3

G(0, 0, 1, 1) ≥ 0, G(0, 0, 0, 2) ≥ 0.
We have




  

2 2 2 2

2
2
2 8 6
G , , , = 4 2 − 2, G 0, , , = − 2,
2 2 2 2 3 3 3 9

G(0, 0, 1, 1) = 0, G(0, 0, 0, 2) = 2 2 − 2.
The equality holds for a = b = 0 and c = d = 1 (or any permutation).

P 1.31. If a, b, c, d are nonnegative real numbers such that a2 + b2 + c 2 + d 2 = 3, then

3(a3 + b3 + c 3 + d 3 ) + 2(a bc + bcd + cd a + d a b) ≥ 11.

(Vasile C., 2006)

Solution. Write the inequality as G(a, b, c, d) ≥ 0, where

G(a, b, c, d) = 3(a3 + b3 + c 3 + d 3 ) + 2a b(c + d) + 2(a + b)cd − 11,

and apply the AC1-Corollary. As shown at the preceding P 1.30, if

G(a, b, c, d) < G(t, t, c, d),

then

G(a, b, c, d) ≥ G(0, 2 t, c, d),


where

a2 + b2
t= , a = b.
2
By the AC1-Corollary, it suffices to prove that





3 3 3 3
G , , , ≥ 0, G (0, 1, 1, 1) ≥ 0,
2 2 2 2
66 Vasile Cîrtoaje

 

3
3

G 0, 0, , ≥ 0, G(0, 0, 0, 3) ≥ 0.
2 2
We have






3 3 3 3 15 3
G , , , = − 11 > 0, G (0, 1, 1, 1) = 0,
2 2 2 2 2
 

3
3
3

G 0, 0, , =9 − 11 > 0, G(0, 0, 0, 3) = 9 3 − 11 > 0.


2 2 2
The equality holds for a = 0 and b = c = d = 1 (or any cyclic permutation).

P 1.32. If a, b, c, d are nonnegative real numbers such that a2 + b2 + c 2 + d 2 = 4, then

4(a3 + b3 + c 3 + d 3 ) + a bc + bcd + cd a + d a b ≥ 20.

(Vasile C., 2006)

Solution. Write the inequality as G(a, b, c, d) ≥ 0, where

G(a, b, c, d) = 4(a3 + b3 + c 3 + d 3 ) + ac(b + d) + (a + c)bd − 20.

First Solution. Apply the AM1-Corollary. First, we will show that

G(a, b, c, d) ≥ G(t, b, t, d)

for

a2 + c 2
a ≥ b ≥ c ≥ d, t= .
2
Write this inequality as follows:

4(a3 + c 3 − 2t 3 ) − (t 2 − ac)(b + d) − (2t − a − c)bd ≥ 0,

4[(a3 + c 3 )2 − 4t 6 ] 4t 2 − (a + c)2
− (t 2 − ac)(b + d) − bd ≥ 0
a + c + 2t
3 3 3 2t + a + c
4(a − c)2 [a4 + b4 + 2a b(a2 + b2 )] (a − c)2 (b + d) (a − c)2
− − bd ≥ 0,
2(a + c + 2t )
3 3 3 2 2t + a + c
which is true if
4a4 + 4c 4 + 8ac(a2 + c 2 ) 2bd
≥ b+d+ .
a3 + c 3 + 2t 3 2t + a + c
Since
2t 3 ≤ a3 + c 3 , 2t ≥ a + c,
Arithmetic Mean Method and Arithmetic Compensation Method 67

it suffices to show that

4a4 + 4c 4 + 8ac(a2 + c 2 ) 2bd


≥ b+d+ ,
2(a3 + c 3 ) 2(a + c)

which is equivalent to

2a4 + 2c 4 + 4ac(a2 + c 2 )
≥ (a + c)(b + d) + bd,
a2 + c 2 − ac
Since b + d ≤ a + c and bd ≤ ac, we only need to prove that

2a4 + 2c 4 + 4ac(a2 + c 2 )
≥ (a + c)2 + ac,
a2 + c 2 − ac
which is equivalent to

2a4 + 2c 4 + 4ac(a2 + c 2 ) ≥ (a2 + c 2 − ac)(a2 + c 2 + 3ac),

a4 + c 4 + 2ac(a2 + c 2 ) + a2 c 2 ≥ 0.
By the AM1-Corollary, it suffices to prove the original inequality for a = b = c;
that is, to show that 3a2 + d 2 = 4 involves

4(3a3 + d 3 ) + a3 + 3a2 d ≥ 20,

which is equivalent to
13a3 + 4d 3 + 3a2 d ≥ 20,
13a3 + 4d(4 − 3a2 ) + 3a2 d ≥ 20,
(16 − 9a2 )d ≥ 20 − 13a3 . (*)
Since 3a ≤ 4 involves 16 − 9a > 0, we only need to show that
2 2

(16 − 9a2 )2 d 2 ≥ (20 − 13a3 )2

20
for a3 ≤ , which is equivalent to
13

312 − 960a2 + 260a3 + 594a4 − 206a6 ≥ 0,

312
a(1 − a)2 f (a) ≥ 0, f (a) =+ 624 − 24a − 412a2 − 206a3 .
a
20
Since f is decreasing, it suffices to show that f (a) ≥ 0 for a3 = . This is true
13
20
because (*) holds for a =
3
.
13
The equality holds for a = b = c = d = 1.
68 Vasile Cîrtoaje

Second Solution. Apply the AC1-Corollary. As shown at P 1.30, if

G(a, b, c, d) < G(t, t, c, d),

then

G(a, b, c, d) ≥ G(0, 2 t, c, d),


where

a2 + b2
t= , a = b.
2
Therefore, by the AC1-Corollary, it suffices to prove that
 
2 2 2
G(1, 1, 1, 1) ≥ 0, G 0,
,
,
≥ 0,
3 3 3

G(0, 0, 2, 2) ≥ 0, G(0, 0, 0, 2) ≥ 0.
We have
 
2 2 2 104
G(1, 1, 1, 1) = 0,G 0,
,
,
=
− 20 > 0,
3 3 3 3 3


G(0, 0, 2, 2 ) = 16 2 − 20 > 0, G(0, 0, 0, 2) = 12.

P 1.33. If a, b, c, d are nonnegative real numbers such that a2 + b2 + c 2 + d 2 = 4, then


28
a3 + b3 + c 3 + d 3 + 3(a + b + c + d) ≤
.
3
(Vasile C., 2006)
28
Solution. Write the inequality as G(a, b, c, d) +
≥ 0, where
3

G(a, b, c, d) = −(a3 + b3 + c 3 + d 3 ) − k(a + b + c + d), k = 3,

and apply the AM1-Corollary. First, we will show that

G(a, b, c, d) ≥ G(t, b, t, d)

for k ≥ 3 and

a2 + c 2
a ≤ b ≤ c ≤ d, t= .
2
Write this inequality as follows:

k(2t − a − c) − (a3 + c 3 − 2t 3 ) ≥ 0,
Arithmetic Mean Method and Arithmetic Compensation Method 69

k[4t 2 − (a + c)2 ] (a3 + c 3 )2 − 4t 6


− 3 ≥ 0,
2t + a + c a + c 3 + 2t 3
k(a − c) 2
(a − c) [a + c + 2ac(a + c 2 )]
2 4 4 2
− ≥ 0,
2t + a + c 2(a3 + c 3 + 2t 3 )
which is true if
2k a4 + c 4 + 2ac(a2 + c 2 )
≥ 3 .
2t + a + c a + c 3 + (a2 + c 2 )t
Write this inequality in the homogeneous form

k(a2 + b2 + c 2 + d 2 ) a4 + c 4 + 2ac(a2 + c 2 )
≥ 3 ,
2(2t + a + c) a + c 3 + (a2 + c 2 )t
Since
k(a2 + b2 + c 2 + d 2 ) ≥ 2k(a2 + c 2 ) ≥ 6(a2 + c 2 ),
it suffices to show that
3(a2 + c 2 ) a4 + c 4 + 2ac(a2 + c 2 )
≥ 3 ,
2t + a + c a + c 3 + (a2 + c 2 )t
which is equivalent to

(a − c)4 t + (a − c)2 (2a2 + 2c 2 − ac) ≥ 0.

By the AM1-Corollary, it suffices to prove the original inequality for a = b = c;


that is, to show that 3a2 + d 2 = 4 involves
28
3a3 + d 3 + 3(3a + d) ≤
.
3
Using the substitution
x y
a=
, c=
, x, y ≥ 0,
3 3
we need to prove that
3x 2 + y 2 = 12, x ≤2
involves
y3
x3 + + 9x + 3 y ≤ 28,
3
which is equivalent to

x 3 + (4 − x 2 ) y + 9x + 3 y ≤ 28,

(7 − x 2 ) y ≤ 28 − 9x − x 3 .
Since x ≤ 2 implies

28 − 9x − x 3 = 2 + 9(2 − x) + (8 − x 3 ) > 0,
70 Vasile Cîrtoaje

we only need to show that

(7 − x 2 )2 y 2 ≤ (28 − 9x − x 3 )2 ,

which is equivalent to

(7 − x 2 )2 (12 − 3x 2 ) ≤ (28 − 9x − x 3 )2 ,

x 6 − 9x 4 − 14x 3 + 99x 2 − 126x + 49 ≥ 0,


(x − 1)2 f (x) ≥ 0,
where

f (x) = x 4 + 2x 3 − 6x 2 − 28x + 49
= (x − 2)2 (x 2 + 6x + 14) + 4x − 7
= (2 − x)(24 − 2x − 4x 2 − x 3 ) + 1.

Since f (x) ≥ 0 for 4x − 7 ≥ 0, it suffices to show that 24 − 2x − 4x 2 − x 3 ≥ 0 for


7
0 ≤ x ≤ . Indeed, we have
4
1 3 105
24 − 2x − 4x 2 − x 3 = (4 − 2x) + (13 − 4x 2 ) + (7 − x 3 ) ≥ + + > 0.
2 4 64
1

The equality holds for a = b = c =


and d = 3 (or any cyclic permutation).
3

P 1.34. If a, b, c, d are nonnegative real numbers such that a2 + b2 + c 2 + d 2 = 4, then

a3 + b3 + c 3 + d 3 + 4(a + b + c + d) ≤ 20.

(Vasile C., 2006)

Solution. Write the inequality as G(a, b, c, d) + 20 ≥ 0, where

G(a, b, c, d) = −(a3 + b3 + c 3 + d 3 ) − k(a + b + c + d), k = 4,

and apply the AM1-Corollary. As shown at the preceding P 1.33, we have

G(a, b, c, d) ≥ G(t, b, t, d)

for k ≥ 3 and

a2 + c 2
a ≤ b ≤ c ≤ d, t= .
2
Arithmetic Mean Method and Arithmetic Compensation Method 71

By the AM1-Corollary, it suffices to prove the original inequality for a = b = c; that


is, to show that 3a2 + d 2 = 4 involves

3a3 + d 3 + 4(3a + d) ≤ 20,

which is equivalent to

3a3 + (4 − 3a2 )d + 12a + 4d ≤ 20,

(8 − 3a2 )d ≤ 20 − 12a − 3a3 .


Since 3a2 ≤ 4 implies

20 − 12a − 3a3 ≥ 20 − 12a − 4a = 4(5 − 4a) > 0,

we only need to show that

(8 − 3a2 )2 d 2 ≤ (20 − 12a − 3a3 )2 ,

which is equivalent to

(8 − 3a2 )2 (4 − 3a2 ) ≤ (20 − 12a − 3a3 )2 ,

3a6 − 9a4 − 10a3 + 44a2 − 40a + 12 ≥ 0,


(a − 1)2 f (a) ≥ 0, f (a) = 3a4 + 6a3 − 16a + 12.
It suffices to show that f (a) ≥ 0. We have

f (a) ≥ 6a3 − 18a + 12 = 6(a − 1)2 (a + 2) ≥ 0.

The equality holds for a = b = c = d = 1.

P 1.35. If a, b, c, d are nonnegative real numbers such that a2 + b2 + c 2 + d 2 = 4, then



a3 + b3 + c 3 + d 3 + 2 2 (a + b + c + d) ≥ 4(2 + 2 ).

(Vasile C., 2006)

Solution. Write the inequality as G(a, b, c, d) ≥ 0, where



G(a, b, c, d) = a3 + b3 + c 3 + d 3 + k(a + b + c + d) − m, k = 2 2, m = 4(2 + 2 ),

and apply the AC1-Corollary. First, we will show that

G(a, b, c, d) < G(t, t, c, d)


72 Vasile Cîrtoaje

involves

G(a, b, c, d) ≥ G(0, 2 t, c, d)
for any k > 0 and real m, where


a2 + b2
t= , a = b.
2

Write the hypothesis G(a, b, c, d) < G(t, t, c, d) as

(a3 + b3 − 2t 3 ) − k(2t − a − b) < 0,

(a3 + b3 )2 − 4t 6 4t 2 − (a + b)2
−k· cd < 0,
a + b + 2t
3 3 3 2t + a + b
(a − b)2 [a4 + b4 + 2a b(a2 + b2 )] k(a − b)2
− < 0,
2(a + b + 2t )
3 3 3 2t + a + b
[a4 + b4 + 2a b(a2 + b2 )](2t + a + b)
< 2k. (*)
a3 + b3 + 2t 3

Write now the required inequality G(a, b, c, d) ≥ G(0, 2 t, c, d) as follows:



k(a + b − 2 t) ≥ 2 2 t 3 − a3 − b3 ,

k[(a + b)2 − 2t 2 ] 8t 6 − (a3 + b3 )2




,
a+b+ 2 t 2 2 t 3 + a3 + b3
2ka b a2 b2 (3a2 + 3b2 − 2a b)


,
a+b+ 2 t 2 2 t 3 + a3 + b3

a b(a + b + 2 t)(3a2 + 3b2 − 2a b)


2k ≥
.
2 2 t 3 + a3 + b3
Having in view (*), it suffices to show that

[a4 + b4 + 2a b(a2 + b2 )](2t + a + b) (a + b + 2 t)[a b(3a2 + 3b2 − 2a b)]



.
a3 + b3 + 2t 3 2 2 t 3 + a3 + b3
We can get this inequality by multiplying the inequalities

2[a4 + b4 + 2a b(a2 + b2 )] ≥ 3[a b(3a2 + 3b2 − 2a b)]

and

3(2t + a + b) 2(a + b + 2 t)

.
a3 + b3 + 2t 3 2 2 t 3 + a3 + b3
The first inequality is equivalent to

(a − b)2 (2a2 + 2b2 − a b) ≥ 0,


Arithmetic Mean Method and Arithmetic Compensation Method 73

and the second inequality to




8 2t 4 + 2(3 2 − 2)(a + b)t 3 + 2(3 − 2)(a3 + b3 )t + (a + b)(a3 + b3 ) ≥ 0.

By the AC1-Corollary, it suffices to prove that


 
2 2 2
G(1, 1, 1, 1) ≥ 0, G 0,
,
,
≥ 0,
3 3 3

G(0, 0, 2, 2) ≥ 0, G(0, 0, 0, 2) ≥ 0.
We have

G(1, 1, 1, 1) = 4( 2 − 1) > 0,
   
2 2 2

2
G 0,
,
,
= 4( 3 − 1) 2−
> 0,
3 3 3 3

G(0, 0, 2, 2) = 0, G(0, 0, 0, 2) = 0.

The equality holds for a = b = 0 and c = d = 2 (or any permutation), and also
for a = b = c = 0 and d = 2 (or any cyclic permutation).

P 1.36. If a, b, c, d are nonnegative real numbers such that a2 + b2 + c 2 + d 2 = 4, then


 

2
2
a3 + b3 + c 3 + d 3 + 2 (a + b + c + d) ≥ 4 2+
.
3 3

(Vasile C., 2006)

Solution. Write the inequality as G(a, b, c, d) ≥ 0, where


 

2
2
G(a, b, c, d) = a +b +c +d +k(a+b+c+d)−m,
3 3 3 3
k=2 , m=4 2+
,
3 3

and apply the AC1-Corollary. As shown at the preceding P 1.35, if

G(a, b, c, d) < G(t, t, c, d),

then

G(a, b, c, d) ≥ G(0, 2 t, c, d)
for any k > 0 and real m, where


a2 + b2
t= , a = b.
2
74 Vasile Cîrtoaje

As a consequence, by the AC1-Corollary, it suffices to prove that


 
2 2 2
G(1, 1, 1, 1) ≥ 0, G 0,
,
,
≥ 0,
3 3 3

G(0, 0, 2, 2) ≥ 0, G(0, 0, 0, 2) ≥ 0.
We have
   

2 2 2 2
G(1, 1, 1, 1) = 4( 2 − 1)
− 1 > 0, G 0,
,
,
=0
3 3 3 3
 



1
G(0, 0, 2, 2) = 0, G(0, 0, 0, 2) = 4(2 − 2) 1 −
> 0.
3
2
The equality holds for a = 0 and b = c = d =
(or any cyclic permutation),

3
and also for a = b = 0 and c = d = 2 (or any permutation).

P 1.37. If a, b, c, d are nonnegative real numbers such that a2 + b2 + c 2 + d 2 = 4, then


2
a3 + b3 + c 3 + d 3 − 4 +
(a + b + c + d − 4) ≥ 0.
3
(Vasile C., 2006)

Solution. Write the inequality as G(a, b, c, d) ≥ 0, where


2
G(a, b, c, d) = a3 + b3 + c 3 + d 3 − 4 + k(a + b + c + d − 4), k=
.
3

First Solution. Apply the AM1-Corollary. First, we will show that

G(a, b, c, d) ≥ G(t, b, t, d)
4
for k ≤ and
3

a2 + c 2
a ≥ b ≥ c ≥ d, t= .
2
Write this inequality as follows:

(a3 + c 3 − 2t 3 ) − k(2t − a − c) ≥ 0,

(a3 + c 3 )2 − 4t 6 k[4t 2 − (a + c)2 ]


− ≥ 0,
a3 + c 3 + 2t 3 2t + a + c
(a − c) [a + c + 2ac(a + c )]
2 4 4 2 2
k(a − c)2
− ≥ 0,
2(a + c + 2t )
3 3 3 2t + a + c
Arithmetic Mean Method and Arithmetic Compensation Method 75

which is true if
a4 + c 4 + 2ac(a2 + c 2 ) 2k
≥ .
a3 + c 3 + (a2 + c 2 )t 2t + a + c
Write this inequality in the homogeneous form

a4 + c 4 + 2ac(a2 + c 2 ) k(a2 + b2 + c 2 + d 2 )
≥ ,
a3 + c 3 + (a2 + c 2 )t 2(2t + a + c)

Since
8(a2 + c 2 )
k(a2 + b2 + c 2 + d 2 ) ≤ 2k(a2 + c 2 ) ≤ ,
3
it suffices to show that

a4 + c 4 + 2ac(a2 + c 2 ) 4(a2 + c 2 )
≥ . (*)
a3 + c 3 + (a2 + c 2 )t 3(2t + a + c)

This inequality can be written in the form 2At + B ≥ 0, where

A = a4 + c 4 + 6ac(a2 + c 2 ) − 4a2 c 2 .
a+c a+c
Since A > 0 and t ≥ , it suffices to prove the inequality (*) for t = . That
2 2
means to show that

a4 + c 4 + 2ac(a2 + c 2 ) 4(a2 + c 2 )
≥ ,
a3 + c + (a + c )(a + c)/2
3 2 2 6(a + c)

which is true if
a4 + c 4 + 2ac(a2 + c 2 ) 2(a2 + c 2 )
≥ ,
a2 + c − ac + (a + c )/2
2 2 2 3
which is equivalent to

a4 + c 4 + 2ac(a2 + c 2 ) a2 + c 2
≥ ,
3(a2 + c 2 ) − 2ac 3

ac(4a2 + 4c 2 − 3ac) ≥ 0.
According to the AM1-Corollary, it suffices to prove the original inequality for a =
b = c. Write the original inequality

2
a3 + b3 + c 3 + d 3 + k(a + b + c + d) ≥ 4(1 + k), k=
,
3
in the homogeneous inequality

4(a3 + b3 + c 3 + d 3 ) 
+ k(a + b + c + d) ≥ 2(1 + k) a2 + b2 + c 2 + d 2 .
a2 + b2 + c 2 + d 2
76 Vasile Cîrtoaje

We only need to prove this inequality for a = b = c = 0 and for a = b = c = 1. The


first case reduces to (2 − k)d ≥ 0, while the second case to
4(d 3 + 3) 
+ k(d + 3) ≥ 2(1 + k) d 2 + 3.
d +3
2

By squaring, the inequality becomes


Ak2 + 2Bk + C ≥ 0,
where
3 3d(d − 1)2 3(d − 1)2 (d 4 + 2d 3 + 6d + 3)
A = − (d − 1)2 , B= , C= .
4 d2 + 3 (d 2 + 3)2
Thus, we need to show that
3
− k2 (d 2 + 3)2 + 6kd(d 2 + 3)2 + 3(d 4 + 2d 3 + 6d + 3) ≥ 0,
4
which is equivalent to
−(d 2 + 3)2 + 6kd(d 2 + 3) + 3(d 4 + 2d 3 + 6d + 3) ≥ 0,
6kd(d 2 + 3) + 2d(d 3 + 3d 2 − 3d + 9) ≥ 0.
2
The equality holds for a = b = c = d = 1, and also for a = b = c =
and
3
d = 0 (or any cyclic permutation).

Second Solution. Apply the AC1-Corollary. As shown at P 1.35, if


G(a, b, c, d) < G(t, t, c, d),
then

G(a, b, c, d) ≥ G(0, 2 t, c, d)
for any k > 0, where

a2 + b2
t= , a = b.
2
Therefore, by the AC1-Corollary, it suffices to prove that
 
2 2 2
G(1, 1, 1, 1) ≥ 0, G 0,
,
,
≥ 0,
3 3 3

G(0, 0, 2, 2) ≥ 0, G(0, 0, 0, 2) ≥ 0.
We have  
2 2 2
G(1, 1, 1, 1) = 0,
G 0,
,
,
=0
3 3 3
   



1 1 1
G(0, 0, 2, 2 ) = 4(2 − 2 )

> 0, G(0, 0, 0, 2) = 4 1 −
> 0.
2 3 3
Arithmetic Mean Method and Arithmetic Compensation Method 77

P 1.38. If a1 , a2 , . . . , an are real numbers such that

a1 + a2 + · · · + an = n,

then
 2 n(n − 1)  4 
a12 + a22 + · · · + an2 − n2 ≥ a + a24 + · · · + an4 − n .
n2 − n + 1 1
(Vasile C., 2006)

Solution. Write the inequality as F (a1 , a2 , . . . , an ) ≥ 0, where


 2  
F (a1 , a2 , . . . , an ) = a12 + a22 + · · · + an2 − n2 − k a14 + a24 + · · · + an4 − n ,

with
n(n − 1)
. k=
n2 − n + 1
Without loss of generality, assume that

a1 ≤ a2 ≤ · · · ≤ an

or
a1 ≥ a2 ≥ · · · ≥ an ,
such that
an2 = max{a12 , a22 , . . . , an2 }.
If
a1 + an−1
F (a1 , a2 , . . . , an−2 , an−1 , an ) ≥ F (t, a2 , . . . , an−2 , t, an ) , t= ,
2
then, by the AM-Corollary, it suffices to prove the original inequality for

a1 = a2 = · · · = an−1 .

That means to show that


(n − 1)a + an = n
involves  2  
(n − 1)a2 + an2 − n2 ≥ k (n − 1)a4 + an4 − n .
Since
 2   
(n − 1)a2 + an2 − n2 = (n − 1)a2 + an2 − n (n − 1)a2 + an2 + n
 
= n2 (n − 1)(a − 1)2 (n − 1)a2 − 2(n − 1)a + n + 1

and
 
(n−1)a4 +an4 −n = n(n−1)(a−1)2 (n2 − 3n + 3)a2 − 2(n2 − n − 1)a + n2 + n + 1 ,
78 Vasile Cîrtoaje

the desired inequality can be written in the obvious form

(a − 1)2 [(n − 1)a − 1]2 ≥ 0.

Using the identities

a12 + an−1
2
− 2t 2 = 2(t 2 − a1 an−1 ),

1
a14 + an−1
4
− 2t 4 = (t 2 − a1 an−1 )(7a12 + 7an−1 2
+ 10a1 an−1 ),
2
we may write the inequality F (a1 , a2 , . . . , an−2 , an−1 , an ) ≥ F (t, a2 , . . . , an−2 , t, an ) as
follows:

(a12 + an−1
2
− 2t 2 )(2t 2 + a12 + 2a22 + · · · + 2an−2
2
+ an−1
2
+ 2an2 ) ≥ k(a14 + an−1
4
− 2t 4 ),

2(t 2 − a1 an−1 )(2t 2 + a12 + 2a22 + · · · + 2an−2


2
+ an−1
2
+ 2an2 ) ≥
k 2
(t − a1 an−1 )(7a12 + 7an−1
2
+ 10a1 an−1 ).
2
Since t 2 − a1 an−1 ≥ 0 and 2a22 + · · · + 2an−2
2
≥ 0, we only need to prove that
 2 
4 2t + a12 + an−1
2
+ 2an2 ≥ k(7a12 + 7an−12
+ 10a1 an−1 ).

Since
2an2 ≥ a12 + an−1
2
,
it suffices to show that
 
8 t 2 + a12 + an−1
2
≥ k(7a12 + 7an−1
2
+ 10a1 an−1 2).

This is true because k < 1 and


 
8 t 2 + a12 + an−1
2
= 10a12 + 10an−1
2
+ 4a1 an−1 ≥ 7a12 + 7an−1
2
+ 10a1 an−1 .

The equality holds when a1 = a2 = · · · = an = 1, and also when


1
a1 = · · · = an−1 = , an = n − 1
n−1
(or any cyclic permutation).

P 1.39. If a, b, c, d are real numbers such that a + b + c + d = 4, then


 2  26

a + b2 + c 2 + d 2 − 4 a2 + b2 + c 2 + d 2 + ≥ a4 + b4 + c 4 + d 4 − 4.
5

(Vasile C., 2006)


Arithmetic Mean Method and Arithmetic Compensation Method 79

Solution. Write the inequality as F (a, b, c, d) ≥ 0, where


   2  
3 2 17
F (a, b, c, d) = a2 + b2 + c 2 + d 2 + − − a4 + b4 + c 4 + d 4 − 4 .
5 5
Without loss of generality, assume that a ≤ b ≤ c ≤ d or a ≥ b ≥ c ≥ d, such that
d 2 = max{a2 , b2 , c 2 , d 2 }. We will show that
a+c
F (a, b, c, d) ≥ F (t, b, t, d), t= .
2
Using the identities
1
a2 + c 2 − 2t 2 = 2(t 2 − ac), a4 + c 4 − 2t 4 = (t 2 − ac)(7a2 + 7c 2 + 10ac),
2
we may write the inequality F (a, b, c, d) ≥ F (t, b, t, d) as follows:
 
6
(a2 + c 2 − 2t 2 ) 2t 2 + a2 + 2b2 + c 2 + 2d 2 + − (a4 + c 4 − 2t 4 ) ≥ 0,
5
 
6 1
2(t 2 − ac) 2t 2 + a2 + 2b2 + c 2 + 2d 2 + − (t 2 − ac)(7a2 + 7c 2 + 10ac) ≥ 0.
5 2
The inequality is true if
 
6
4 2t 2 + a2 + 2b2 + c 2 + 2d 2 + − (7a2 + 7c 2 + 10ac) ≥ 0,
5
which is equivalent to

−5(a2 + c 2 ) − 30ac + 40(b2 + d 2 ) + 24 ≥ 0.

It suffices to show that

−5(a2 + c 2 ) − 30ac + 40d 2 ≥ 0.

Since 2d 2 ≥ a2 + c 2 , we have

−5(a2 + c 2 ) − 30ac + 40d 2 ≥ −5(a2 + c 2 ) − 30ac + 20(a2 + c 2 ) = 15(a − c)2 ≥ 0.

By the AM-Corollary, it suffices to prove the original inequality for a = b = c. That


is, to show that 3a + d = 4 implies
 
26
(3a2 + d 2 − 4) 3a2 + d 2 + ≥ 3a4 + d 4 − 4.
5
Since
3a2 + d 2 − 4 = 12(a − 1)2 ,
26 2(30a2 − 60a + 53)
3a2 + d 2 + =
5 5
80 Vasile Cîrtoaje

and
3a4 + d 4 − 4 = 12(a − 1)2 (7a2 − 22a + 21),
the desired inequality can be written in the obvious form

(a − 1)2 (5a − 1)2 ≥ 0.


1 17
The equality holds for a = b = c = d = 1, and also for a = b = c = and d =
5 5
(or any cyclic permutation).

P 1.40. If a, b, c, d are nonnegative real numbers such that a + b + c + d = 4, then


 2  11

3 4 
a + b2 + c 2 + d 2 − 4 a2 + b2 + c 2 + d 2 + ≥ a + b4 + c 4 + d 4 − 4 .
6 4
(Vasile C., 2006)

Solution. Write the inequality as F (a, b, c, d) ≥ 0, where


   2
13 2 35 3 4 
F (a, b, c, d) = a2 + b2 + c 2 + d 2 − − − a + b4 + c 4 + d 4 − 4 ,
12 12 4
Assume that a ≤ b ≤ c ≤ d and show that
a+c
F (a, b, c, d) ≥ F (t, b, t, d), t= .
2
Using the identities
1
a2 + c 2 − 2t 2 = 2(t 2 − ac), a4 + c 4 − 2t 4 = (t 2 − ac)(7a2 + 7c 2 + 10ac),
2
we may write the inequality F (a, b, c, d) ≥ F (t, b, t, d) as follows:
 
13 3
(a2 + c 2 − 2t 2 ) 2t 2 + a2 + 2b2 + c 2 + 2d 2 − − (a4 + c 4 − 2t 4 ) ≥ 0,
6 4
 
13 3
2(t 2 − ac) 2t 2 + a2 + 2b2 + c 2 + 2d 2 − − (t 2 − ac)(7a2 + 7c 2 + 10ac) ≥ 0.
6 8
The inequality is true if
 
13
16 2t 2 + a2 + 2b2 + c 2 + 2d 2 − − 3(7a2 + 7c 2 + 10ac) ≥ 0,
6
which is equivalent to
13
3(a2 + c 2 ) − 14ac + 32(b2 + d 2 ) − (a + b + c + d)2 ≥ 0,
6
Arithmetic Mean Method and Arithmetic Compensation Method 81

18(a2 + c 2 ) − 84ac + 192(b2 + d 2 ) − 13(a + b + c + d)2 ≥ 0,


18(a2 + c 2 ) − 84ac + 179b2 + 192d 2 − 26b(a + c + d) − 13(a + c + d)2 ≥ 0,
18(a2 + c 2 ) − 84ac + 10b2 + 192d 2 + (13b − a − c − d)2 − 14(a + c + d)2 ≥ 0.
The inequality is true if

18(a2 + c 2 ) − 84ac + 192d 2 − 16(a + c + d)2 ≥ 0,

which is equivalent to

9(a2 + c 2 ) − 42ac + 96d 2 − 8(a + c + d)2 ≥ 0.

Since
9(a2 + c 2 ) − 42ac ≥ −6(a + c)2 = −24t 2 ,
it suffices to show that

−24t 2 + 96d 2 − 8(2t + d)2 ≥ 0,

which is equivalent to the obvious inequality

(d − t)(11d + 7t) ≥ 0.

By the AM-Corollary, it suffices to prove the original inequality for a = b = c. That


is, to show that 3a + d = 4 implies
 
11 3 4 
(3a2 + d 2 − 4) 3a2 + d 2 + ≥ 3a + d 4 − 4 .
6 4

Since
3a2 + d 2 − 4 = 12(a − 1)2 ,
11 72a2 − 144a + 107
3a2 + d 2 + =
6 6
and
3a4 + d 4 − 4 = 12(a − 1)2 (7a2 − 22a + 21),
the desired inequality can be written in the obvious form

(a − 1)2 (9a − 5)2 ≥ 0.

5 7
The equality holds for a = b = c = d = 1, and also for a = b = c = and d =
9 3
(or any cyclic permutation).
82 Vasile Cîrtoaje

P 1.41. If a, b, c, d are nonnegative real numbers such that a + b + c + d = 4, then


 2  
a + b2 + c 2 + d 2 − 4 2a2 + 2b2 + 2c 2 + 2d 2 − 1 ≥ a4 + b4 + c 4 + d 4 − 4.

(Vasile C., 2006)

Solution. Write the inequality as F (a, b, c, d) ≥ 0, where


 
9 2 49  4 
F (a, b, c, d) = 2 a2 + b2 + c 2 + d 2 − − − a + b4 + c 4 + d 4 − 4 .
4 8
Assume that a ≤ b ≤ c ≤ d and show that
a+c
F (a, b, c, d) ≥ F (t, b, t, d), t= .
2
Using the identities
1
a2 + c 2 − 2t 2 = 2(t 2 − ac), a4 + c 4 − 2t 4 = (t 2 − ac)(7a2 + 7c 2 + 10ac),
2
we may write the inequality F (a, b, c, d) ≥ F (t, b, t, d) as follows:
 
9
2(a2 + c 2 − 2t 2 ) 2t 2 + a2 + 2b2 + c 2 + 2d 2 − − (a4 + c 4 − 2t 4 ) ≥ 0,
2
 
9
8(t 2 − ac) 2t 2 + a2 + 2b2 + c 2 + 2d 2 − − (t 2 − ac)(7a2 + 7c 2 + 10ac) ≥ 0.
2
The inequality is true if
 
9
8 2t 2 + a2 + 2b2 + c 2 + 2d 2 − − (7a2 + 7c 2 + 10ac) ≥ 0,
2
which is equivalent to

5(a2 + c 2 ) − 2ac + 16(b2 + d 2 ) − 36 ≥ 0,

20(a2 + c 2 ) − 8ac + 64(b2 + d 2 ) − 9(a + b + c + d)2 ≥ 0,


20(a + c 2 ) − 8ac + 64d 2 + 55b2 − 18b(a + c + d) − 9(a + c + d)2 ≥ 0,
2

 2
9 522
20(a2 + c 2 ) − 8ac + 64d 2 + 6b2 + 7b − (a + c + d) − (a + c + d)2 ≥ 0.
7 49
The inequality is true if
522
20(a2 + c 2 ) − 8ac + 64d 2 − (a + c + d)2 ≥ 0,
49
which is equivalent to
261
10(a2 + c 2 ) − 4ac + 32d 2 − (a + c + d)2 ≥ 0.
49
Arithmetic Mean Method and Arithmetic Compensation Method 83

Since
10(a2 + c 2 ) − 4ac ≥ 4(a + c)2 = 16t 2
and
261 16
< ,
49 3
it suffices to show that
16
16t 2 + 32d 2 − (2t + d)2 ≥ 0,
3
which is equivalent to the obvious inequality

(d − t)(5d + t) ≥ 0.

According to the AM-Corollary, it suffices to prove the original inequality for a =


b = c. That is, to show that 3a + d = 4 implies

(3a2 + d 2 − 4)(6a2 + 2d 2 − 1) ≥ 3a4 + d 4 − 4.

Since
3a2 + d 2 − 4 = 12(a − 1)2 ,

6a2 + 2d 2 − 1 = 24a2 − 48a + 31


and
3a4 + d 4 − 4 = 12(a − 1)2 (7a2 − 22a + 21),
the desired inequality can be written as

(a − 1)2 (17a2 − 26a + 10) ≥ 0.

This is true because

17(17a2 − 26a + 10) = (17a − 13)2 + 1 > 0.

The equality holds for a = b = c = d = 1.

P 1.42. If a, b, c, d are nonnegative real numbers such that a + b + c + d = 4, then


   4 4 
a2 + b2 + c 2 + d 2 − 4 a2 + b2 + c 2 + d 2 + 12 ≥ a + b4 + c 4 + d 4 − 4 .
3

(Vasile C., 2006)


84 Vasile Cîrtoaje

Solution. Write the inequality as F (a, b, c, d) ≥ 0, where

4
F (a, b, c, d) = (a2 + b2 + c 2 + d 2 + 4)2 − 64 − (a4 + b4 + c 4 + d 4 − 4).
3
Assume that a ≤ b ≤ c ≤ d and show that
a+c
F (a, b, c, d) ≥ F (t, b, t, d), t= .
2
Using the identities

1
a2 + c 2 − 2t 2 = 2(t 2 − ac), a4 + c 4 − 2t 4 = (t 2 − ac)(7a2 + 7c 2 + 10ac),
2
we may write the inequality F (a, b, c, d) ≥ F (t, b, t, d) as

4
(a2 + c 2 − 2t 2 )(2t 2 + a2 + 2b2 + c 2 + 2d 2 + 8) − (a4 + c 4 − 2t 4 ) ≥ 0,
3
2
2(t 2 − ac)(2t 2 + a2 + 2b2 + c 2 + 2d 2 + 8) − (t 2 − ac)(7a2 + 7c 2 + 10ac) ≥ 0,
3
(t 2 − ac)(−5a2 − 5c 2 − 14ac + 12b2 + 12d 2 + 48) ≥ 0.
This inequality is true if

−5a2 − 5c 2 − 14ac + 12b2 + 12d 2 + 48 ≥ 0,

which is equivalent to

−5a2 − 5c 2 − 14ac + 12b2 + 12d 2 + 3(a + b + c + d)2 ≥ 0.

It suffices to show that

−5a2 − 5c 2 − 14ac + 12d 2 + 3(a + c)2 ≥ 0.

Since 2d 2 ≥ a2 + c 2 , we only need to show that

−5a2 − 5c 2 − 14ac + 6(a2 + c 2 ) + 3(a + c)2 ≥ 0,

which reduces to
4(a − c)2 ≥ 0.
By the AM-Corollary, it suffices to prove the original inequality for a = b = c. That
is, to show that 3a + d = 4 involves

3(3a2 + d 2 − 4)(3a2 + d 2 + 12) ≥ 4(3a4 + d 4 − 4).

Since
3a2 + d 2 − 4 = 12(a − 1)2 ,
Arithmetic Mean Method and Arithmetic Compensation Method 85

3a2 + d 2 + 12 = 4(3a2 − 6a + 7)
and
3a4 + d 4 − 4 = 12(a − 1)2 (7a2 − 22a + 21),
the desired inequality can be written in the obvious form

a(a − 1)2 (a + 2) ≥ 0.

The equality holds for a = b = c = d = 1, and also for a = b = c = 0 and d = 4


(or any cyclic permutation).

P 1.43. If a, b, c, d are nonnegative real numbers such that a + b + c + d = 4, then


 2  76

12  4 
a + b2 + c 2 + d 2 − 4 a2 + b2 + c 2 + d 2 + ≥ a + b4 + c 4 + d 4 − 4 .
11 11

(Vasile C., 2006)

Solution. Write the inequality as F (a, b, c, d) ≥ 0, where


   2
16 2 60 12  4 
F (a, b, c, d) = a2 + b2 + c 2 + d 2 + − − a + b4 + c 4 + d 4 − 4 .
11 11 11

Assume that a ≤ b ≤ c ≤ d and show that


a+c
F (a, b, c, d) ≥ F (t, b, t, d), t= .
2
Using the identities

1
a2 + c 2 − 2t 2 = 2(t 2 − ac), a4 + c 4 − 2t 4 = (t 2 − ac)(7a2 + 7c 2 + 10ac),
2
we may write the inequality F (a, b, c, d) ≥ F (t, b, t, d) as follows:
 2  32

12  4 
a + c 2 − 2t 2 2t 2 + a2 + 2b2 + c 2 + 2d 2 + − a + c 4 − 2t 4 ≥ 0,
11 11
 
32
22(t 2 − ac) 2t 2 + a2 + 2b2 + c 2 + 2d 2 + − 6(t 2 − ac)(7a2 + 7c 2 + 10ac) ≥ 0,
11
(t 2 − ac)(−9a2 − 9c 2 − 38ac + 44b2 + 44d 2 + 64) ≥ 0.
This inequality is true if

−9a2 − 9c 2 − 38ac + 44b2 + 44d 2 + 4(a + b + c + d)2 ≥ 0.


86 Vasile Cîrtoaje

It suffices to show that

−9a2 − 9c 2 − 38ac + 44d 2 + 4(a + c)2 ≥ 0.

Since 2d 2 ≥ a2 + c 2 , we only need to show that

−9a2 − 9c 2 − 38ac + 22(a2 + c 2 ) + 4(a + c)2 ≥ 0.

which reduces to
2(a2 + c 2 ) + 15(a − c)2 ≥ 0.
By the AM-Corollary, it suffices to prove the original inequality for a = b = c. That
is, to show that 3a + d = 4 involves
   
(3a2 + d 2 − 4) 33a2 + 11d 2 + 76 ≥ 12 3a4 + d 4 − 4 .

Since
3a2 + d 2 − 4 = 12(a − 1)2 ,
33a + 11d 2 + 76 = 12(11a2 − 22a + 21)
2

and
3a4 + d 4 − 4 = 12(a − 1)2 (7a2 − 22a + 21),
the desired inequality can be written in the obvious form

a2 (a − 1)2 ≥ 0.

The equality holds for a = b = c = d = 1, and also for a = b = c = 0 and d = 4


(or any cyclic permutation).

P 1.44. If a1 , a2 , . . . , an are nonnegative real numbers such that

a1 + a2 + · · · + an = m, m ∈ {1, 2, . . . , n},

then
1 1 1 m
+ + ··· + ≥ n− .
1 + a12 1 + a22 1 + an2 2
(Vasile C., 2005)

Solution. We need to prove that F (a1 , a2 , . . . , an ) ≥ 0, where

1 1 1 m
F (a1 , a2 , . . . , an ) = + + ··· + −n+ .
1 + a12 1 + a22 1 + an2 2

Assume that
F (a1 , a2 , a3 , . . . , an ) < F (t, t, a3 , . . . , an ) (*)
Arithmetic Mean Method and Arithmetic Compensation Method 87

involves
F (a1 , a2 , a3 , . . . , an ) ≥ F (0, a1 + a2 , a3 , . . . , an ), (**)
where
a1 + a2
t= , a1 = a2 .
2
Then, by AC-Corollary, we have

F (a1 , a2 , . . . , an ) ≥ min f (k),


1≤k≤n

where
k m m(m − k)2
f (k) = n − k + −n+ =
1 + (m/k) 2 2 2(m2 + k2 )
m
is the value of F for a1 = · · · = an−k = 0 and an−k+1 = · · · = an = . Obviously, we
k
have f (k) ≥ 0 (with equality for k = m), therefore F (a1 , a2 , . . . , an ) ≥ 0.
To prove that (*) implies (**), we write (*) as

1 1 2
+ − < 0,
1 + a12 1 + a22 1 + t 2

(a1 − a2 )2 (a12 + a22 + 4a1 a2 − 2)


< 0,
(1 + a12 )(1 + a22 )(1 + t 2 )
a12 + a22 + 4a1 a2 − 2 < 0,
(a1 + a2 )2 < 2 − 2a1 a2 , (A)
and (**) as
1 1 1
+ −1− ≥ 0,
1 + a12 1 + a22 1 + (a1 + a2 )2
a1 a2 [2 − 2a1 a2 − a1 a2 (a1 + a2 )2 ]
≥ 0.
(1 + a12 )(1 + a22 )(1 + 4t 2 )
We need to show that

2 − 2a1 a2 − a1 a2 (a1 + a2 )2 ≥ 0.

Using (A), we have

2 − 2a1 a2 − a1 a2 (a1 + a2 )2 ≥ 2 − 2a1 a2 − a1 a2 (2 − 2a1 a2 ) = 2(a1 a2 − 1)2 ≥ 0.

The equality holds for a1 = · · · = an−m = 0 and an−m+1 = · · · = an = 1 (or any


permutation).
88 Vasile Cîrtoaje

P 1.45. If a, b, c, d are nonnegative real numbers such that a + b + c + d = 2, then


1 1 1 1 16
+ + + ≥ .
1 + 3a2 1 + 3b2 1 + 3c 2 1 + 3d 2 7
(Vasile C., 2005)

Solution. Write the inequality as F (a, b, c, d) ≥ 0, where


1 1 1 1 16
F (a, b, c, d) = + + + − .
1 + 3a2 1 + 3b2 1 + 3c 2 1 + 3d 2 7
Assume that
F (a, b, c, d) < F (t, t, c, d) (*)
involves
F (a, b, c, d) ≥ F (0, a + b, c, d), (**)
where
a+b
t= , a = b.
2
Then, by AC-Corollary, we have

F (a, b, c, d) ≥ min f (k),


1≤k≤4

where
k 16 12(k − 3)(k − 4)
f (k) = 4 − k + − =
1 + 3(2/k)2 7 7(k2 + 12)
is the value of F when 4 − k of the numbers a, b, c, d are zero, and the other k
2
numbers are equal to . Obviously, we have f (k) ≥ 0 for k ∈ {1, 2, 3, 4} (with
k
equality for k = 3 and k = 4), therefore F (a1 , a2 , . . . , an ) ≥ 0.
To prove that (*) implies (**), we write (*) as
1 1 2
+ − < 0,
1 + 3a2 1 + 3b2 1 + 3t 2
(t 2 − a b)(6t 2 − 1 + 3a b)
< 0,
(1 + 3a2 )(1 + 3b2 )(1 + 3t 2 )
6t 2 < 1 − 3a b, (A)
and (**) as
1 1 1
+ −1− ≥ 0,
1 + 3a2 1 + 3b2 1 + 3(a + b)2
a b(1 − 3a b − 18a bt 2 )
≥ 0.
(1 + 3a2 )(1 + 3b2 )(1 + 12t 2 )
For the nontrivial case a b = 0, this is true if
1
6t 2 ≤ − 1.
3a b
Arithmetic Mean Method and Arithmetic Compensation Method 89

Using (A), we only need to show that

1
1 − 3a b ≤ − 1,
3a b
which is equivalent to
(3a b − 1)2 ≥ 0.
1 2
The equality holds for a = b = c = d = , and also for a = 0 and b = c = d =
2 3
(or any cyclic permutation).

P 1.46. If a, b, c, d are nonnegative real numbers such that a + b + c + d = 4, then

(1 + a2 )(1 + b2 )(1 + c 2 )(1 + d 2 ) ≤ 25.

(Vasile C., 2005)

Solution. Write the inequality as F (a, b, c, d) ≥ 0, where

F (a, b, c, d) = 25 − (1 + a2 )(1 + b2 )(1 + c 2 )(1 + d 2 ).

Assume that
F (a, b, c, d) < F (t, t, c, d) (*)
involves
F (a, b, c, d) ≥ F (0, a + b, c, d), (**)
where
a+b
t= , a = b.
2
By the AC-Corollary, it suffices to prove that
 
4 4 4
F (1, 1, 1, 1) ≥ 0, F 0, , , ≥ 0,
3 3 3
F (0, 0, 2, 2) ≥ 0, F (0, 0, 0, 4) ≥ 0.
We have  
4 4 4 2600
F (1, 1, 1, 1) = 9, F 0, , , = ,
3 3 3 729

F (0, 0, 2, 2 ) = 0, F (0, 0, 0, 4) = 8.
To prove that (*) implies (**), we write (*) as
 
(1 + t 2 )2 − (1 + a2 )(1 + b2 ) (1 + c 2 )(1 + d 2 ) < 0,

(1 + t 2 )2 − (1 + a2 )(1 + b2 ) < 0,
90 Vasile Cîrtoaje

(t 2 − a b)(t 2 + a b − 2) < 0,
2 − a b > t 2, (A)
and (**) as
 
1 + (a + b)2 − (1 + a2 )(1 + b2 ) (1 + c 2 )(1 + d 2 ) ≥ 0,

1 + (a + b)2 − (1 + a2 )(1 + b2 ) ≥ 0,
a b(2 − a b) ≥ 0.
This inequality is true if 2 − a b ≥ 0. Using (A), we have

2 − a b > t 2 ≥ 0.

The equality holds for a = b = 0 and c = d = 2 (or any permutation).

P 1.47. If a, b, c, d are nonnegative real numbers such that a + b + c + d = 1, then

(1 + 2a)(1 + 2b)(1 + 2c)(1 + 2d) 125


≥ .
(1 − a)(1 − b)(1 − c)(1 − d) 8

Solution. Write the inequality as F (a, b, c, d) ≥ 0, where


125
F (a, b, c, d) = (1+2a)(1+2b)(1+2c)(1+2d)−k(1−a)(1−b)(1−c)(1−d), k= .
8
If
F (a, b, c, d) < F (t, t, c, d) (*)
involves
F (a, b, c, d) ≥ F (0, a + b, c, d), (**)
where
a+b
t= , a = b.
2
then, by the AC-Corollary, it suffices to prove that
 
1 1
F (0, 0, 0, 1) ≥ 0, F 0, 0, , ≥ 0,
2 2
   
1 1 1 1 1 1 1
F 0, , , ≥ 0, F , , , ≥ 0.
3 3 3 4 4 4 4
We have  
1 1 3
F (0, 0, 0, 1) = 3, F 0, 0, , = ,
2 2 32
Arithmetic Mean Method and Arithmetic Compensation Method 91
   
1 1 1 1 1 1 1 243
F 0, , , = 0, F , , , = .
3 3 3 4 4 4 4 2048
The inequality (*) is equivalent to
 
(1 + 2a)(1 + 2b) − (1 + 2t)2 (1 + 2c)(1 + 2d)−
 
−k (1 − a)(1 − b) − (1 − t)2 (1 − c)(1 − d) < 0,
−4(t 2 − a b)(1 + 2c)(1 + 2d) + k(t 2 − a b)(1 − c)(1 − d) < 0,
4(1 + 2c)(1 + 2d) > k(1 − c)(1 − d), (A)
and (**) to

[(1 + 2a)(1 + 2b) − (1 + 2a + 2b)] (1 + 2c)(1 + 2d)−

−k [(1 − a)(1 − b) − (1 − a − b)] (1 − c)(1 − d) ≥ 0,


4a b(1 + 2c)(1 + 2d) − ka b(1 − c)(1 − d) ≥ 0.
This inequality is true if

4(1 + 2c)(1 + 2d) ≥ k(1 − c)(1 − d) ≥ 0,

which follows immediately from (A); therefore, (*) implies (**).


1
The equality holds for a = 0 and b = c = d = (or any cyclic permutation).
3

2
P 1.48. If a1 , a2 , . . . , an are nonnegative real numbers such that a1 + a2 +· · ·+ an = ,
3
then  ai a j 1
≤ .
1≤i≤ j≤n
(1 − a i )(1 − a j ) 4

(Vasile C., 2005)

Solution. We need to prove that F (a1 , a2 , . . . , an ) ≥ 0, where


1  ai a j
F (a1 , a2 , . . . , an ) = − .
4 1≤i≤ j≤n (1 − ai )(1 − a j )

Assume that
F (a1 , a2 , a3 , . . . , an ) < F (t, t, a3 , . . . , an ) (*)
involves
F (a1 , a2 , a3 , . . . , an ) ≥ F (0, a1 + a2 , a3 , . . . , an ), (**)
where
a1 + a2
t= , a1 = a2 .
2
92 Vasile Cîrtoaje

Then, by AC-Corollary, we only need to prove the original inequality for the case
when n−k of the variables a1 , a2 , . . . , an are zero and the other k variables are equal
2
to , where k ∈ {1, 2, · · · , n}; that is, to show that
3k
   2 2
k 3k 1
  ≤ ,
2 1− 2 2 4
3k

which is equivalent to
2k(k − 1) 1
≤ ,
(3k − 2)2 4
(k − 2)2 ≥ 0.
To prove that (*) implies (**), we write the inequality (*) as
  n
t2 a1 a2 2t a1 a2 ai
− + − − < 0,
(1 − t) 2 (1 − a1 )(1 − a2 ) 1 − t 1 − a1 1 − a2 i=3 1 − ai
 
t 2 − a1 a2 n
ai
1 − 2t − 2(1 − t) < 0,
(1 − a1 )(1 − a2 )(1 − t)2 i=3
1 − ai

n
ai
1 − 2t − 2(1 − t) < 0, (A)
i=3
1 − ai
and (**) as
 n
−a1 a2 2t a1 a2 ai
+ − − ≥ 0,
(1 − a1 )(1 − a2 ) 1 − 2t 1 − a1 1 − a2 i=3 1 − ai
 
a1 a2 n
ai
1 − 2t − 2(1 − t) ≤ 0.
(1 − a1 )(1 − a2 )(1 − 2t) i=3
1 − ai
Clearly, this inequality follows immediately from (A), therefore (*) implies (**).
This completes the proof.
1
The equality holds when a1 = a2 = · · · = an−2 = 0 and an−1 = an = (or any
3
permutation).

P 1.49. If a1 , a2 , . . . , an are nonnegative real numbers such that a1 + a2 + · · · + an = 1


and no one of which is 1, then
 ai a j n
≥ .
1≤i≤ j≤n
(1 − a i )(1 − a j ) 2(n − 1)

(Gabriel Dospinescu, 2005)


Arithmetic Mean Method and Arithmetic Compensation Method 93

Solution. For n = 2, the inequality is an equality. Consider next that n ≥ 3. We


need to show that F (a1 , a2 , . . . , an ) ≥ 0, where
 ai a j n
F (a1 , a2 , . . . , an = − ,
1≤i≤ j≤n
(1 − ai )(1 − a j ) 2(n − 1)

Assume that
F (a1 , a2 , a3 , . . . , an ) < F (t, t, a3 , . . . , an ) (*)
involves
F (a1 , a2 , a3 , . . . , an ) ≥ F (0, a1 + a2 , a3 , . . . , an ), (**)
where
a1 + a2
t= , a1 = a2 .
2
Then, by AC-Corollary, we only need to prove the original inequality for the case
when n−k of the variables a1 , a2 , . . . , an are zero and the other k variables are equal
1
to , where k ∈ {1, 2, · · · , n}; that is, to show that
k
   1 2
k k n
  ≥ ,
2 1− 1 2 2(n − 1)
k

which is equivalent to n − k ≥ 0. As shown at the preceding P 1.49, the inequalities


(*) and (**) are equivalent to


n
ai
2t − 1 + 2(1 − t) <0
i=3
1 − ai

and  
a1 a2 n
ai
2t − 1 + 2(1 − t) ≤ 0,
(1 − a1 )(1 − a2 )(1 − 2t) i=3
1 − ai

respectively. Thus, (*) involves (**).


1
The equality holds when a1 = a2 = · · · = an = .
n

P 1.50. If a, b, c, d are nonnegative real numbers such that a + b + c + d = 4, then

(1 + 3a)(1 + 3b)(1 + 3c)(1 + 3d) ≤ 125 + 131a bcd.

(Pham Kim Hung, 2005)


94 Vasile Cîrtoaje

Solution. Write the inequality as F (a, b, c, d) ≥ 0, where

F (a, b, c, d) = 125 − (1 + 3a)(1 + 3b)(1 + 3c)(1 + 3d) + 131a bcd.

If
F (a, b, c, d) < F (t, t, c, d) (*)
involves
F (a, b, c, d) ≥ F (0, a + b, c, d), (**)
where
a+b
t= , a = b.
2
then, by the AC-Corollary, it suffices to prove that

F (0, 0, 0, 4) ≥ 0, F (0, 0, 2, 2) ≥ 0,
 
4 4 4
F 0, , , ≥ 0, F (1, 1, 1, 1) ≥ 0.
3 3 3
We have
F (0, 0, 0, 4) = 112, F (0, 0, 2, 2) = 76,
 
4 4 4
F 0, , , = 0, F (1, 1, 1, 1) = 0.
3 3 3
The inequality (*) is equivalent to
 
(1 + 3t)2 − (1 + 3a)(1 + 3b) (1 + 3c)(1 + 3d) − 131(t 2 − a b)cd < 0,

9(t 2 − a b)(1 + 3c)(1 + 3d) − 131(t 2 − a b)cd < 0,


9(1 + 3c)(1 + 3d) − 131cd < 0, (A)
and (**) to

[(1 + 3a + 3b) − (1 + 3a)(1 + 3b)] (1 + 3c)(1 + 3d) + 131a bcd ≥ 0,

−9a b(1 + 3c)(1 + 3d) + 131a bcd ≥ 0,


a b[9(1 + 3c)(1 + 3d) − 131cd] ≤ 0,
This inequality follows immediately from (A); therefore, (*) implies (**).
4
The equality holds for a = b = c = d = 1, and also for a = 0 and b = c = d =
3
(or any cyclic permutation).

P 1.51. If a, b, c, d are nonnegative real numbers such that a + b + c + d = 4, then

(1 + 3a2 )(1 + 3b2 )(1 + 3c 2 )(1 + 3d 2 ) ≤ 255 + a2 b2 c 2 d 2 .

(Vasile C., 2005)


Arithmetic Mean Method and Arithmetic Compensation Method 95

Solution. Write the inequality as F (a, b, c, d) ≥ 0, where


F (a, b, c, d) = 255 − (1 + 3a2 )(1 + 3b2 )(1 + 3c 2 )(1 + 3d 2 ) + a2 b2 c 2 d 2 .
If
F (a, b, c, d) < F (t, t, c, d) (*)
involves
F (a, b, c, d) ≥ F (0, a + b, c, d), (**)
where
a+b
t= , a = b.
2
then, by the AC-Corollary, it suffices to prove that
F (0, 0, 0, 4) ≥ 0, F (0, 0, 2, 2) ≥ 0,
 
4 4 4
F 0, , , ≥ 0, F (1, 1, 1, 1) ≥ 0.
3 3 3
We have
F (0, 0, 0, 4) = 206, F (0, 0, 2, 2) = 86,
 
4 4 4 404
F 0, , , = , F (1, 1, 1, 1) = 0.
3 3 3 3
The inequality (*) is equivalent to
 
(1 + 3t 2 )2 − (1 + 3a2 )(1 + 3b2 ) (1 + 3c 2 )(1 + 3d 2 ) − (t 4 − a2 b2 )c 2 d 2 < 0,
3(t 2 − a b)(3t 2 + 3a b − 2)(1 + 3c 2 )(1 + 3d 2 ) − (t 2 − a b)(t 2 + a b)c 2 d 2 < 0,
3(3t 2 + 3a b − 2)(1 + 3c 2 )(1 + 3d 2 ) − (t 2 + a b)c 2 d 2 < 0,
3t 2 + 3a b − 2 c2 d 2
< , (A)
t + ab
2 3(1 + 3c 2 )(1 + 3d 2 )
and (**) to
 
1 + 3(a + b)2 − (1 + 3a2 )(1 + 3b2 ) (1 + 3c 2 )(1 + 3d 2 ) + a2 b2 c 2 d 2 ≥ 0,
−3a b(3a b − 2)(1 + 3c 2 )(1 + 3d 2 ) + a2 b2 c 2 d 2 ≥ 0.
This inequality is true if
−3(3a b − 2)(1 + 3c 2 )(1 + 3d 2 ) + a bc 2 d 2 ≥ 0,
c2 d 2 3a b − 2
≥ .
3(1 + 3c 2 )(1 + 3d 2 ) ab
Having in view (A), it suffices to show that
3t 2 + 3a b − 2 3a b − 2
≥ ,
t2 + ab ab
which reduces to t 2 ≥ 0; therefore, (*) implies (**).
The equality holds for a = b = c = d = 1.
96 Vasile Cîrtoaje

P 1.52. If a1 , a2 , . . . , an (n ≥ 3) are nonnegative real numbers, then


  4n(n − 2) 
a12 + 2 a1 a2 a3 + ≥2 a1 a2 .
s ym
3(n − 1)2 s ym

(Vasile C., 2005)

Solution. Let us denote


s = a1 + a2 + · · · + an .
Since  
2 a1 a2 = s2 − a12 ,
s ym

we need to show that F (a1 , a2 , . . . , an ) ≥ 0, where


  2n(n − 2) s2
F (a1 , a2 , . . . , an ) = a12 + a1 a2 a3 + − .
s ym
3(n − 1)2 2

Assume that
F (a1 , a2 , a3 , . . . , an ) < F (t, t, a3 , . . . , an ) (*)
involves
F (a1 , a2 , a3 , . . . , an ) ≥ F (0, a1 + a2 , a3 , . . . , an ), (**)
where
a1 + a2
t= , a1 = a2 .
2
Then, by AC-Corollary, we only need to prove the original inequality for the case
when n−k of the variables a1 , a2 , . . . , an are zero and the other k variables are equal
s
to , where k ∈ {1, 2, · · · , n}; that is, to show that
k
 s 2    3    3
k s 4n(n − 2) k s
k +2 + ≥ 2 ,
k 3 k 3(n − 1)2 2 k
which is equivalent to

(k − 1)(k − 2)s3 8n(n − 2) 6(k − 2)s2


2· + ≥ .
k2 (n − 1)2 k
This inequality is clearly true for k = 1 and k = 2. For k ∈ {3, . . . , n}, using the
AM-GM inequality to three numbers, it suffices to show that


3 (k − 1) (k − 2) s
2 2 6 8n(n − 2) 6(k − 2)s2
3 · ≥ ,
k4 (n − 1)2 k
which is equivalent to
n(n − 2) k(k − 2)
≥ ,
(n − 1)2 (k − 1)2
Arithmetic Mean Method and Arithmetic Compensation Method 97

(n − 1)2 − 1 (k − 1)2 − 1
≥ ,
(n − 1)2 (k − 1)2
1 1
≥ .
(k − 1)2 (n − 1)2
The inequality (*) is equivalent to

(a12 + a22 − 2t 2 ) − (t 2 − a1 a2 )(a3 + · · · + an ) < 0,

2(t 2 − a1 a2 ) − (t 2 − a1 a2 )(a3 + · · · + an ) < 0,


2 < a3 + · · · + an , (A)
and the inequality (**) to

a12 + a22 − (a1 + a2 )2 + a1 a2 (a3 + · · · + an ) ≥ 0,

−2a1 a2 + a1 a2 (a3 + · · · + an ) ≥ 0,
a1 a2 (2 − a3 − · · · − an ) ≤ 0.
This inequality follows from (A), therefore (*) involves (**).
2
The equality holds when a1 = a2 = · · · = an = .
n−1

P 1.53. If a, b, c, d are nonnegative real numbers such that a + b + c + d = 3 , then

a b(a + 2b + 3c) + bc(b + 2c + 3d) + cd(c + 2d + 3a) + d a(d + 2a + 3b) ≤ 2.

Solution. Write the inequality as F (a, b, c, d) ≤ 2, where

F (a, b, c, d) = a b(a + 2b + 3c) + bc(b + 2c + 3d) + cd(c + 2d + 3a) + d a(d + 2a + 3b).

From
F (a, b, c, d) − f (a + c, b, 0, d) = c(b − d)(a + c − b − d)
and
F (a, b, c, d) − f (0, b, a + c, d) = a(d − b)(c + a − d − b),
we get

[F (a, b, c, d) − f (a + c, b, 0, d)] [F (a, b, c, d) − f (0, b, a + c, d)] =

= −ac(b − d)2 (a + c − b − d)2 ≤ 0,


hence
F (a, b, c, d) ≤ max{ f (a + c, b, 0, d), f (0, b, a + c, d)}.
98 Vasile Cîrtoaje

Similarly, we have

F (a, b, c, d) ≤ max{ f (a, b + d, c, 0), f (a, 0, c, b + d)}.

Therefore, f (a, b, c, d) is maximum when one of a and c is zero, and one of b and d
is zero. Thus, it suffices to consider

one of these cases. For instance, for c = d = 0,
we need to show that a + b = 3 involves

a b(a + 2b) ≤ 2.

We have

2 − a b(a + 2b) = 2 − b(a2 + 2a b) = 2 − b[(a + b)2 − b2 ]


= 2 − 3b + b3 = (1 − b)2 (2 + b) ≥ 0.

The equality holds for a = 3 − 1, b = 1, c = d = 0 (or any cyclic permutation).


Chapter 2

pqr Method

2.1 Theoretical Basis


The pqr method is applicable to prove symmetric and cyclic polynomial inequalities
of the form
P(a, b, c) ≥ 0.

If P(a, b, c) is a symmetric polynomial having the degree not very large (practically,
less than or equal to eight), then the inequality can be written in the polynomial
form
P1 (p, q, r) ≥ 0,

where
p = a + b + c, q = a b + bc + ca, r = a bc.

In addition, the pqr method can be applied to prove inequalities where P1 (p, q, r)
is not a polynomial function.
The pqr method enables to prove an inequality P1 (p, q, r) ≥ 0 using the theorems
below.

Theorem 1 (see P 2.53 in Volume 1). If a ≥ b ≥ c are real numbers such that

a + b + c = p, a b + bc + ca = q,

where p and q are fixed real numbers satisfying p2 ≥ 3q, then the product r = a bc is
minimum for a = b, and maximum for b = c.

Theorem 2 (see P 3.57 in Volume 1). If a ≥ b ≥ c are real nonnegative numbers


such that
a + b + c = p, a b + bc + ca = q,

where p and q are fixed real numbers satisfying p2 ≥ 3q, then the product r = a bc is
minimum for a = b or c = 0, and maximum for b = c.

99
100 Vasile Cîrtoaje

Theorem 3 (see P 3.83 in Volume 1). If a ≥ b ≥ c are real nonnegative numbers


such that
a + b + c = p, a bc = r,
where p and r are fixed real numbers satisfying p3 ≥ 27r, then q = a b + bc + ca is
minimum for b = c, and maximum for a = b.
For cyclic polynomial inequalities, the pqr method can be applied by using the
formulae
   
2C a2 b + 2D a b2 = (C + D) a b(a + b) + (C − D) a b(a − b)
= (C + D)(pq − 3r) − (C − D)(a − b)(b − c)(c − a),

   
2C a3 b + 2D a b3 = (C + D) a b(a2 + b2 ) + (C − D) a b(a2 − b2 )
= (C + D)(p q − 2q − pr) − (C − D)p(a − b)(b − c)(c − a),
2 2


|(a − b)(b − c)(c − a)| = (a − b)2 (b − c)2 (c − a)2


4(p2 − 3q)3 − (2p3 − 9pq + 27r)2
= , (*)
27

and

Lemma. If α, β, x, y are real numbers such that

α ≥ 0, x ≥ 0, x 2 ≥ y 2,

then 
βy +α x2 − y2 ≤ x α2 + β 2 ,
with equality for

βx = y α2 + β 2 .

Proof. Since

x α2 + β 2 − β y ≥ |β|x − β y ≥ |β|| y| − β y ≥ 0,
we can write the desired inequality as follows:

α x2 − y2 ≤ x α2 + β 2 − β y,
2
α2 (x 2 − y 2 ) ≤ x α2 + β 2 − β y ,
2
βx − y α2 + β 2 ≥ 0.
pqr Method 101

Using (*) and Lemma above for

1
α=
, x = 2(p2 − 3q)3/2 , y = 2p3 − 9pq + 27r,
27
we get the following theorem:
Theorem 4. Let a, b, c be real numbers. For any real β, the following inequality holds


1  3/2
27β r + |(a − b)(b − c)(c − a)| ≤ 9β pq − 2β p3 + 2 + β 2 p2 − 3q ,
27

with equality for




1  
2β(p2 − 3q)3/2 = + β 2 2p3 − 9pq + 27r .
27

From Theorem 4, we get


Corollary 1. Let a, b, c be real numbers such that a + b + c = 1 For any real β, the
following inequality holds:


1 + 27β 2
27β r + |(a − b)(b − c)(c − a)| ≤ 9βq − 2β + 2 (1 − 3q)3/2 , (**)
27

with equality for




1 + 27β 2
2β(1 − 3q) 3/2
= (2 − 9q + 27r). (***)
27
102 Vasile Cîrtoaje
pqr Method 103

2.2 Applications

2.1. If a, b, c and k are real numbers, then



(a − b)(a − c)(a − k b)(a − kc) ≥ 0.

2.2. If a, b, c are real numbers such that

a + b + c > 0, a b + bc + ca ≥ 0,

then
(a) 3(a3 + b3 + c 3 ) ≥ (a + b + c)(a2 + b2 + c 2 );
15
(b) a3 + b3 + c 3 + a bc ≥ 0;
4
(c) 4(a3 + b3 + c 3 ) + 15a bc ≥ 3(a + b + c)(a b + bc + ca).

2.3. If a, b, c are real numbers such that

a + b + c > 0, a2 + b2 + c 2 + 3(a b + bc + ca) ≥ 0,

then
2(a3 + b3 + c 3 ) ≥ a b(a + b) + bc(b + c) + ca(c + a).

2.4. If a, b, c are real numbers such that

a + b + c > 0, 33(a b + bc + ca) ≥ 8(a2 + b2 + c 2 ),

then
8(a3 + b3 + c 3 ) + 39a bc ≥ 7(a + b + c)(a b + bc + ca).

2.5. If a, b, c are real numbers such that

a + b + c > 0, a b + bc + ca ≥ 0,

then
11
(a) a4 + b4 + c 4 + a bc(a + b + c) ≥ 0;
4
(b) 4(a4 + b4 + c 4 ) + a bc(a + b + c) ≥ 5(a2 b2 + b2 c 2 + c 2 a2 ).
104 Vasile Cîrtoaje

2.6. If a, b, c are real numbers such that

a + b + c > 0, a b + bc + ca ≥ 0,

then
81(a3 + b3 + c 3 )2 ≥ 25(a2 + b2 + c 2 )3 .

2.7. If a, b, c are real numbers such that

a + b + c > 0, a b + bc + ca ≥ 0,

then
a2 (a − 2b + c) + b2 (b − 2c + a) + c 2 (c − 2a + b) ≥ 0.

2.8. If a, b, c are real numbers such that

a + b + c > 0, a b + bc + ca ≥ 0,

then
(a) 2(a3 + b3 + c 3 ) + 7a bc ≥ (a − b)(b − c)(c − a);
(b) a3 + b3 + c 3 + 2a bc ≥ a b2 + bc 2 + ca2 ;
! !
(c) 9(a3 + b3 + c 3 ) + 12a bc ≥ 2 a2 b + 11 a b2 .

2.9. If a, b, c are real numbers such that

a + b + c > 0, a b + bc + ca ≥ 0,

then
11
(a) a3 + b3 + c 3 + a bc ≥ (a − b)(b − c)(c − a);
4
! !
(b) 4(a3 + b3 + c 3 ) + 5a bc + 2 a2 b ≥ 6 a b2 ;
! !
(c) 36(a3 + b3 + c 3 ) + 30a bc + 13 a2 b ≥ 59 a b2 .

2.10. If a, b, c are real numbers such that

a + b + c > 0, a b + bc + ca ≥ 0,

then
1
a3 + b3 + c 3 − a bc ≥ 2(a − b)(b − c)(c − a).
4
pqr Method 105

2.11. If a, b, c are real numbers such that

a + b + c > 0, a b + bc + ca ≥ 0,

then

(a) 2(a3 + b3 + c 3 ) + 3(3 3 − 2)a bc + 6 3 (a2 b + b2 c + c 2 a) ≥ 0;


3 3
(b) a3 + b3 + c 3 − 3a bc ≥ (a − b)(b − c)(c − a).
2

2.12. If a, b, c are real numbers such that

a + b + c > 0, 2(a2 + b2 + c 2 ) + 7(a b + bc + ca) ≥ 0,

then
a2 (a − b) + b2 (b − c) + c 2 (c − a) ≥ 0.

2.13. If a, b, c are real numbers such that

a + b + c > 0, 3(a b + bc + ca) ≥ a2 + b2 + c 2 ,

then
a2 (a + 2b − 3c) + b2 (b + 2c − 3a) + c 2 (c + 2a − 3b) ≥ 0.

2.14. If a, b, c are real numbers, then

(a2 + b2 + c 2 )2 ≥ 3(a3 b + b3 c + c 3 a).

2.15. If a, b, c are real numbers, then

a3 (a − 2b + c) + b3 (b − 2c + a) + c 3 (c − 2a + b) ≥ 0.

2.16. If a, b, c are real numbers, then


a4 + b4 + c 4 − a bc(a + b + c) ≥ 3 (a + b + c)(a − b)(b − c)(c − a).

2.17. If a, b, c are real numbers, then


2
a3 (a + b) + b3 (b + c) + c 3 (c + a) ≥ (a b + bc + ca)2 .
3
106 Vasile Cîrtoaje

2.18. If a, b, c are real numbers such that a b + bc + ca = 3, then

a3 (a − 2b) + b3 (b − 2c) + c 3 (c − 2a) + 3 ≥ 0.

2.19. If a, b, c are real numbers such that a b + bc + ca ≥ 0, then


15
a4 + b4 + c 4 − a bc(a + b + c) ≥ (a + b + c)(a − b)(b − c)(c − a).
2

2.20. If a, b, c are real numbers such that

2(a b + bc + ca) ≥ a2 + b2 + c 2 ,

then

39
a4 + b4 + c 4 − a bc(a + b + c) ≥ (a + b + c)(a − b)(b − c)(c − a).
2

2.21. If a, b, c are real numbers, then

a4 + b4 + c 4 + 2a bc(a + b + c) ≥ a b3 + bc 3 + ca3 .

2.22. If a, b, c are real numbers, then


a4 + b4 + c 4 + 2 (a3 b + b3 c + c 3 a) ≥ 0.

2.23. If a, b, c are real numbers, then

(a2 + b2 + c 2 )2 + 2(a3 b + b3 c + c 3 a) ≥ 3(a b3 + bc 3 + ca3 ).

2.24. If a, b, c are real numbers, then


8
(a2 + b2 + c 2 )2 +
(a3 b + b3 c + c 3 a) ≥ 0.
7

2.25. If a, b, c are real numbers such that a b + bc + ca ≤ 0, then


(a2 + b2 + c 2 )2 ≥ (2 7 − 1)(a b3 + bc 3 + ca3 ).


pqr Method 107

2.26. If a, b, c are real numbers such that a b + bc + ca ≥ 0, then


(a2 + b2 + c 2 )2 + (1 + 2 7 )(a3 b + b3 c + c 3 a) ≥ 0.

2.27. If a, b, c are real numbers such that a b + bc + ca ≤ 0, then


a4 + b4 + c 4 ≥ (2 3 − 1)(a b3 + bc 3 + ca3 ).

2.28. If a, b, c are real numbers such that a b + bc + ca ≥ 0, then


a4 + b4 + c 4 + (1 + 2 3 )(a3 b + b3 c + c 3 a) ≥ 0.

2.29. If a, b, c are real numbers such that a b + bc + ca ≥ 0, then


a4 + b4 + c 4 + 2 2 (a3 b + b3 c + c 3 a) ≥ a b3 + bc 3 + ca3 .

2.30. If a, b, c are real numbers such that a b + bc + ca ≥ 0, then

(a + b + c)(a3 + b3 + c 3 ) + 5(a3 b + b3 c + c 3 a) ≥ 0.

2.31. If a, b, c are real numbers such that




1+ 21 + 8 7
k(a b + bc + ca) = a + b + c ,
2 2 2
k≥ ≈ 3.7468,
2
then
a3 b + b3 c + c 3 a ≥ 0.

2.32. If a, b, c are nonnegative real numbers, then

3(a4 + b4 + c 4 ) + 4(a3 b + b3 c + c 3 a) ≥ 7(a b3 + bc 3 + ca3 ).

2.33. If a, b, c are nonnegative real numbers, then

16(a4 + b4 + c 4 ) + 52(a3 b + b3 c + c 3 a) ≥ 47(a b3 + bc 3 + ca3 ).


108 Vasile Cîrtoaje

2.34. If a, b, c are nonnegative real numbers, then

a4 + b4 + c 4 + 5(a3 b + b3 c + c 3 a) ≥ 6(a2 b2 + b2 c 2 + c 2 a2 ).

2.35. If a, b, c are nonnegative real numbers such that a + b + c = 4, then


473
a3 b + b3 c + c 3 a + a bc ≤ 27.
64

2.36. If a, b, c are positive real numbers such that a + b + c = 3, then

a2 b2 c 2
+ + + 5(a b + bc + ca) ≥ 18.
b c a

2.37. If a, b, c are positive real numbers, then

a b c 5(a2 + b2 + c 2 − a b − bc − ca)
+ + −3≥ .
b c a a2 + b2 + c 2 + a b + bc + ca

2.38. If a, b, c are positive real numbers, then

a b c 16(a2 + b2 + c 2 − a b − bc − ca)
+ + −3≥ 2 .
b c a a + b2 + c 2 + 6(a b + bc + ca)

2.39. If a, b, c are real numbers such that a b + bc + ca ≥ 0, then

(a2 + b2 + c 2 )(a3 + b3 + c 3 ) + 5(a4 b + b4 c + c 4 a) ≥ 0.

2.40. If a, b, c are real numbers such that

a + b + c = 3, a b + bc + ca ≥ 0,

then

a3 b + b3 c + c 3 a + 18 3 ≥ a b3 + bc 3 + ca3 .

2.41. If a, b, c are real numbers such that a2 + b2 + c 2 = 3, then


81 2
a3 b + b3 c + c 3 a + ≥ a b3 + bc 3 + ca3 .
32
pqr Method 109

2.3 Solutions

P 2.1. If a, b, c and k are real numbers, then



(a − b)(a − c)(a − k b)(a − kc) ≥ 0.

(Vasile C., 2005)

Solution. Let

p = a + b + c, q = a b + bc + ca, r = a bc.

Write the inequality as



(a2 + 2bc − q)[a2 + k(k + 1)bc − kq] ≥ 0,

which is equivalent to
A4 − A2 q + 3kq2 ≥ 0,
where
  
A4 = (a2 + 2bc)[a2 + k(k + 1)bc], A2 = (k + 1) a2 + k(k + 3) bc.

We have

a4 + b4 + c 4 = (a2 + b2 + c 2 )2 − 2(a2 b2 + b2 c 2 + c 2 a2 )
= (p2 − 2q)2 − 2(q2 − 2pr)
= p4 − 4p2 q + 2q2 + 4pr,

therefore
  
A4 = a4 + (k2 + k + 2)a bc a + 2k(k + 1) b2 c 2
= p − 4p q + 2q + 4pr + (k + k + 2)pr + 2k(k + 1)(q2 − 2pr)
4 2 2 2

= p4 − 4p2 q + 2(k2 + k + 1)q2 − 3(k2 + k − 2)pr,

For given p and q, the left hand side of the desired inequality A4 − A2 q + 3kq2 ≥ 0
is a linear function of r. Therefore, this function is minimum when r is either
minimum or maximum. Thus, according to Theorem 1, it suffices to prove the
original inequality for b = c. In this case, the inequality becomes

(a − b)2 (a − k b)2 ≥ 0.
a
The equality holds for a = b = c, and also for = b = c (or any cyclic permuta-
k
tion). If k = 0, then the equality holds for a = b = c, and also for a = 0 and b = c
(or any cyclic permutation).
110 Vasile Cîrtoaje

P 2.2. If a, b, c are real numbers such that

a + b + c > 0, a b + bc + ca ≥ 0,

then
(a) 3(a3 + b3 + c 3 ) ≥ (a + b + c)(a2 + b2 + c 2 );
15
(b) a3 + b3 + c 3 + a bc ≥ 0;
4
(c) 4(a3 + b3 + c 3 ) + 15a bc ≥ 3(a + b + c)(a b + bc + ca).
(Vasile C., 2006)

Solution. (a) Write the inequality as

3(3r + p3 − 3pq) ≥ p(p2 − 2q),

2p3 − 7pq + 9r ≥ 0,
where
p = a + b + c, q = a b + bc + ca, r = a bc.
According to Theorem 1, for fixed p and q, the product r is minimum when

a = b ≥ c.

Thus, it suffices to prove the original inequality for a = b ≥ c. Since

a + b + c = 2a + c, a b + bc + ca = a(a + 2c),

we need to show that

a ≥ c, 2a + c > 0, a + 2c ≥ 0

involve
3(2a3 + c 3 ) ≥ (2a + c)(2a2 + c 2 ),
which is equivalent to
(a − c)2 (a + c) ≥ 0.
This inequality is true because

3(a + c) = (2a + c) + (a + 2c) > 0.

The equality holds for a = b = c > 0.


(b) Write the inequality as

15
3r + p3 − 3pq + r ≥ 0,
4
pqr Method 111

4p3 − 12pq + 27r ≥ 0.


According to Theorem 1, for fixed p and q, the product r is minimum when a =
b ≥ c. Thus, it suffices to prove the original inequality for a = b ≥ c. Since
a + b + c = 2a + c, a b + bc + ca = a(a + 2c),
we need to show that
a ≥ c, 2a + c > 0, a + 2c ≥ 0
involve
15 2
2a3 + c 3 + a c ≥ 0,
4
which is equivalent to
(a + 2c)(8a2 − ac + 2c 2 ) ≥ 0.
The equality holds for a = b = −2c > 0 (or any cyclic permutation).
(c) Write the inequality as
4(3r + p3 − 3pq) + 15r ≥ 3pq,
4p3 − 15pq + 27r ≥ 0.
According to Theorem 1, it suffices to prove the original inequality for a = b ≥ c.
Since
a + b + c = 2a + c, a b + bc + ca = a(a + 2c),
we need to show that
a ≥ c, 2a + c > 0, a + 2c ≥ 0
involve
4(2a3 + c 3 ) + 15a2 c ≥ 3(2a + c)(a2 + 2ac),
which is equivalent to
a3 − 3ac 2 + 2c 3 ≥ 0,
(a + 2c)(a − c)2 ≥ 0.
The equality holds for a = b = c > 0, and also for a = b = −2c > 0 (or any cyclic
permutation).
Remark. The inequality in (c) is sharper than the inequalities in (a) and (b) because
these last inequalities can be obtained by summing the inequality in (c) to the
obvious inequalities
2(a3 + b3 + c 3 − 3a bc) ≥ 0
and
3(a + b + c)(a b + bc + ca) ≥ 0,
respectively. The inequality a3 + b3 + c 3 − 3a bc ≥ 0 is equivalent to
(a + b + c)(a2 + b2 + c 2 − a b − bc − ca) ≥ 0.
112 Vasile Cîrtoaje

P 2.3. If a, b, c are real numbers such that

a + b + c > 0, a2 + b2 + c 2 + 3(a b + bc + ca) ≥ 0,

then
2(a3 + b3 + c 3 ) ≥ a b(a + b) + bc(b + c) + ca(c + a).
(Vasile C., 2006)

First Solution. Write the inequality as

2(3r + p3 − 3pq) ≥ pq − 3r,

2p3 − 7pq + 9r ≥ 0.
According to Theorem 1, it suffices to prove the original inequality for a = b ≥ c.
Since

a + b + c = 2a + c, a2 + b2 + c 2 + 3(a b + bc + ca) = (a + c)(5a + c),

we need to show that

a ≥ c, 2a + c > 0, a+c ≥0

involve
2(2a3 + c 3 ) ≥ 2a(a2 + ac + c 2 ),
which is equivalent to
a3 − a2 c − ac 2 + c 3 ≥ 0,
(a + c)(a − c)2 ≥ 0.
The equality holds for a = b = c > 0, and also for a = b = −c > 0 (or any cyclic
permutation).

Second Solution. Assume that a ≥ b ≥ c, and write the inequality as follows:



(a3 + b3 − a2 b − a b2 ) ≥ 0,

(a + b)(a − b)2 ≥ 0.

(a + b)(a − b)2 + (b + c)(b − c)2 + (c + a)(c − a)2 ≥ 0,


(a + b)(a − b)2 + (b + c)(b − c)2 + (a + c)[(a − b) + (b − c)]2 ≥ 0,
(2a + b + c)(a − b)2 + (a + b + 2c)(b − c)2 + 2(a + c)(a − b)(b − c) ≥ 0.

Case 1: a + b + 2c ≥ 0. The AM-GM inequality yields



(2a+ b+c)(a− b)2 +(a+ b+2c)(b−c)2 ≥ 2 (2a + b + c)(a + b + 2c) (a− b)(b−c).
pqr Method 113

Thus, it suffices to show that



(2a + b + c)(a + b + 2c) + a + c ≥ 0,

which is true if
(2a + b + c)(a + b + 2c) ≥ (a + c)2 .
This inequality is equivalent to the hypothesis a2 + b2 + c 2 + 3(a b + bc + ca) ≥ 0.
Case 2: a + b + 2c < 0. This case is not possible because, as shown at the preceding
case 1, the hypothesis a2 + b2 + c 2 + 3(a b + bc + ca) ≥ 0 involves

(a + c)2
a + b + 2c ≥ ≥ 0.
2a + b + c

P 2.4. If a, b, c are real numbers such that

a + b + c > 0, 33(a b + bc + ca) ≥ 8(a2 + b2 + c 2 ),

then
8(a3 + b3 + c 3 ) + 39a bc ≥ 7(a + b + c)(a b + bc + ca).
(Vasile C., 2006)
First Solution. Write the inequality as

8(3r + p3 − 3pq) + 39r ≥ 7pq,

8p3 − 31pq + 63r ≥ 0.


According to Theorem 1, it suffices to prove the original inequality for a = b ≥ c.
Since

a + b + c = 2a + c, 33(a b + bc + ca) − 8(a2 + b2 + c 2 ) = (17a − 2c)(a + 4c),

we only need to show that

a ≥ c, a + 4c ≥ 0

involve
8(2a3 + c 3 ) + 39a2 c ≥ 7(2a + c)(a2 + 2ac),
which is equivalent to
a3 + 2a2 c − 7ac 2 + 4c 3 ≥ 0,
(a + 4c)(a − c)2 ≥ 0.
The equality holds for a = b = c > 0, and also for a = b = −4c > 0 (or any cyclic
permutation).
114 Vasile Cîrtoaje

P 2.5. If a, b, c are real numbers such that

a + b + c > 0, a b + bc + ca ≥ 0,

then
11
(a) a4 + b4 + c 4 + a bc(a + b + c) ≥ 0;
4
(b) 4(a4 + b4 + c 4 ) + a bc(a + b + c) ≥ 5(a2 b2 + b2 c 2 + c 2 a2 ).
(Vasile C., 2006)

Solution. We have

a4 + b4 + c 4 = (a2 + b2 + c 2 )2 − 2(a2 b2 + b2 c 2 + c 2 a2 )
= (p2 − 2q)2 − 2(q2 − 2pr)
= p4 − 4p2 q + 2q2 + 4pr.

(a) Write the inequality as

27
p4 − 4p2 q + 2q2 + pr ≥ 0.
4
According to Theorem 1, it suffices to prove the original inequality for a = b ≥ c.
Clearly, the original inequality is true for a = b ≥ c ≥ 0. Consider further that
a = b > 0 > c. Since

a + b + c = 2a + c, a b + bc + ca = a(a + 2c),

we need to show that

a > 0 > c, 2a + c > 0, a + 2c ≥ 0

involve
11 2
2a4 + c 4 + a c(2a + c) ≥ 0,
4
which is equivalent to

8a4 + 22a3 c + 11a2 c 2 + 4c 4 ≥ 0,

(a + 2c)(8a3 + 6a2 c − ac 2 + 2c 3 ) ≥ 0.
The last inequality is true since a + 2c ≥ 0 and

8a3 + 6a2 c − ac 2 + 2c 3 > 8a3 + 6a2 c − ac 2 + 38c 3


= (a + 2c)(8a2 − 10ac + 19c 2 ) ≥ 0.

The equality holds for a = b = −2c > 0 (or any cyclic permutation).
pqr Method 115

(b) Write the inequality as

4(p4 − 4p2 q + 2q2 + 4pr) + pr ≥ 5(q2 − 2pr),

4p4 − 16p2 q + 3q2 + 27pr ≥ 0.


According to Theorem 1, it suffices to prove the original inequality for a = b ≥ c.
Since
a + b + c = 2a + c, a b + bc + ca = a(a + 2c),
we need to show that

a ≥ c, 2a + c > 0, a + 2c ≥ 0

involve
4(2a4 + c 4 ) + a2 c(2a + c) ≥ 5(a4 + 2a2 c 2 ),
which is equivalent to

3a4 + 2a3 c − 9a2 c 2 + 4c 4 ≥ 0,

(a − c)2 (a + 2c)(3a + 2c) ≥ 0.


The equality holds for a = b = c > 0, and also for a = b = −2c > 0 (or any cyclic
permutation).

Remark. The inequality in (b) is sharper than the inequalities in (a) because this
last inequality can be obtained by summing the inequality in (b) to the obvious
inequalities
(a b + bc + ca)2 ≥ 0.

P 2.6. If a, b, c are real numbers such that

a + b + c > 0, a b + bc + ca ≥ 0,

then
81(a3 + b3 + c 3 )2 ≥ 25(a2 + b2 + c 2 )3 .
(Vasile C., 2006)

Solution. Write the inequality as

81(3r + p3 − 3pq)2 ≥ 25(p2 − 2q)3 ,

27
p4 − 4p2 q + 2q2 + pr ≥ 0.
4
116 Vasile Cîrtoaje

According to Theorem 1, it suffices to prove the original inequality for a = b ≥ c.


Since
a + b + c = 2a + c, a b + bc + ca = a(a + 2c),
we need to show that
a ≥ c, 2a + c > 0, a + 2c ≥ 0
involve
81(2a3 + c 3 )2 ≥ 25(2a2 + c 2 )3 .
Since the inequality is trivial for c = 0, consider further the cases c > 0 and c < 0.
Case 1: c > 0. Due to homogeneity, we may set c = 1. So, we need to show that
a ≥ c = 1 involves
81(2a3 + 1)2 ≥ 25(2a2 + 1)3 ,
which is equivalent to
62a6 − 150a4 + 162a3 − 75a2 + 28 ≥ 0.
It suffices to show that
25(a6 − 6a4 + 6a3 − 3a2 + 1) ≥ 0,
which is equivalent to
(a − 1)2 (2a4 + 4a3 + 2a + 1) ≥ 0.

Case 1: c < 0. Due to homogeneity, we may set c = −1. So, we need to show that
a−2≥0
involves
81(2a3 − 1)2 ≥ 25(2a2 + 1)3 ,
which is equivalent to
62a6 − 150a4 − 162a3 − 75a2 + 28 ≥ 0.
Since
62a6 − 150a4 − 162a3 ≥ 248a4 − 150a4 − 162a3
= 98a4 − 162a3 ≥ 196a3 − 162a3 = 34a3 ,
it suffices to show that
34a3 − 75a2 + 28 ≥ 0, .
which is equivalent to
(a − 2)(34a2 − 7a − 14) ≥ 0.
This is true because
34a2 − 7a − 14 > 7a2 − 7a − 14 = 7(a + 1)(a − 2) ≥ 0.
The equality holds for a = b = −2c > 0 (or any cyclic permutation).
pqr Method 117

P 2.7. If a, b, c are real numbers such that

a + b + c > 0, a b + bc + ca ≥ 0,

then
a2 (a − 2b + c) + b2 (b − 2c + a) + c 2 (c − 2a + b) ≥ 0.
(Vasile C., 2006)

Solution. Let

p = a + b + c, q = a b + bc + ca, r = a bc.

Due to homogeneity, we may set

p2 1
p = 1, 0≤q≤ = .
3 3

Write the inequality as follows:


 
2(a3 + b3 + c 3 ) ≥ a b(a + b) + 3 a b(a − b),

2(a3 + b3 + c 3 ) + 3a bc ≥ (a + b + c)(a b + bc + ca) − 3(a − b)(b − c)(c − a),


2(3r + p3 − 3pq) + 3r ≥ pq − 3(a − b)(b − c)(c − a),
2p3 − 7pq + 9r ≥ −3(a − b)(b − c)(c − a),
2 − 7q ≥ −9r − 3(a − b)(b − c)(c − a).
This inequality is true if

2 − 7q
≥ −3r + |(a − b)(b − c)(c − a)|.
3
Applying Corollary 1 for
−1
β= ,
9
we have
2 4
−3r + |(a − b)(b − c)(c − a)| ≤ −q + + (1 − 3q)3/2 ,
9 9
with equality for

(1 − 3q)3/2 = 9q − 27r − 2. (*)

Therefore, it suffices to show that

2 − 7q 2 4
≥ −q + + (1 − 3q)3/2 ,
3 9 9
118 Vasile Cîrtoaje

which is equivalent to
1 − 3q ≥ (1 − 3q)3/2 ,

(1 − 3q)(1 − 1 − 3q) ≥ 0,
3q(1 − 3q)
 ≥ 0.
1+ 1 − 3q
The last inequality is true, with equality for q = 1/3 and q = 0. According to (*),
q = 1/3 involves r = 1/27, and q = 0 involves r = −1/9. Thus, the original
inequality is an equality for a = b = c > 0, and also for

a b+ bc +ca = 0, (a+ b+c)3 +9a bc = 0, a+ b+c > 0, (a− b)(b−c)(c −a) < 0.

The last equality conditions are equivalent to the condition that a, b, c satisfy

(a − b)(b − c)(c − a) < 0

and are proportional to the roots of the equation

9w3 − 9w2 + 1 = 0.

P 2.8. If a, b, c are real numbers such that

a + b + c > 0, a b + bc + ca ≥ 0,

then
(a) 2(a3 + b3 + c 3 ) + 7a bc ≥ (a − b)(b − c)(c − a);
(b) a3 + b3 + c 3 + 2a bc ≥ a b2 + bc 2 + ca2 ;
! !
(c) 9(a3 + b3 + c 3 ) + 12a bc ≥ 2 a2 b + 11 a b2 .
(Vasile C., 2006)

Solution. Denote

p = a + b + c, q = a b + bc + ca, r = a bc.

Due to homogeneity, we may set

p2 1
p = 1, 0≤q≤ = .
3 3

(a) Write the inequality as

2(3r + p3 − 3pq) + 7r ≥ (a − b)(b − c)(c − a),


pqr Method 119

2p3 − 6pq ≥ −13r + (a − b)(b − c)(c − a),


2 − 6q ≥ −13r + (a − b)(b − c)(c − a).
It suffices to show that

2 − 6q ≥ −13r + |(a − b)(b − c)(c − a)|.

−13
Applying Corollary 1 for β = , we have
27
−13q 26 28
−13r + |(a − b)(b − c)(c − a)| ≤ + + (1 − 3q)3/2 ,
3 27 27
with equality for

13(1 − 3q)3/2 = 7(9q − 27r − 2). (*)

Therefore, it suffices to show that

−13q 26 28
2 − 6q ≥ + + (1 − 3q)3/2 ,
3 27 27
which is equivalent to
28 − 45q ≥ 28(1 − 3q)3/2 .
Using the substitution 
t= 1 − 3q, 0 ≤ t ≤ 1,
the inequality becomes
28 − 15(1 − t 2 ) ≥ 28t 3 ,
13 + 15t 2 − 28t 3 ≥ 0.
We have
13 + 15t 2 − 28t 3 = (1 − t)(13 + 13t + 28t 2 ) ≥ 0,
with equality for t = 1. Notice that t = 1 involves q = 0, and (*) gives r = −1/7.
Thus, if p = 1, then the original inequality is an equality for q = 0, r = −1/7 and
(a − b)(b − c)(c − a) > 0. More general, the equality holds for

a b+ bc+ca = 0, (a+ b+c)3 +7a bc = 0, a+ b+c > 0, (a− b)(b−c)(c−a) > 0.

These equality conditions are equivalent to the condition that a, b, c satisfy

(a − b)(b − c)(c − a) > 0

and are proportional to the roots of the equation

7w3 − 7w2 + 1 = 0.
120 Vasile Cîrtoaje

(b) Write the inequality as


 
2(a3 + b3 + c 3 ) + 4a bc ≥ a b(a + b) − a b(a − b),

2(a3 + b3 + c 3 ) + 4a bc − a b(a + b) ≥ (a − b)(b − c)(c − a),

2(a + b + c ) + 7a bc − (a + b + c)(a b + bc + ca) ≥ (a − b)(b − c)(c − a),


3 3 3

2(3r + p3 − 3pq) + 7r − pq ≥ (a − b)(b − c)(c − a),


2p3 − 7pq ≥ −13r + (a − b)(b − c)(c − a),
2 − 7q ≥ −13r + (a − b)(b − c)(c − a).
It suffices to show that

2 − 7q ≥ −13r + |(a − b)(b − c)(c − a)|.

As shown at (a), we have


−13q 26 28
−13r + |(a − b)(b − c)(c − a)| ≤ + + (1 − 3q)3/2 ,
3 27 27
Therefore, it suffices to show that
−13q 26 28
2 − 7q ≥ + + (1 − 3q)3/2 ,
3 27 27
which is equivalent to
7 − 18q ≥ 7(1 − 3q)3/2 .
Using the substitution 
t= 1 − 3q, 0 ≤ t ≤ 1,
the inequality becomes
7 − 6(1 − t 2 ) ≥ 7t 3 ,
1 + 6t 2 − 7t 3 ≥ 0.
We have
1 + 6t 2 − 7t 3 = (1 − t)(1 + 7t + 7t 2 ) ≥ 0,
with equality for t = 1. The original inequality is an equality for

a b+ bc+ca = 0, (a+ b+c)3 +7a bc = 0, a+ b+c > 0, (a− b)(b−c)(c−a) > 0.

These equality conditions are equivalent to the condition that a, b, c satisfy

(a − b)(b − c)(c − a) > 0

and are proportional to the roots of the equation

7w3 − 7w2 + 1 = 0.
pqr Method 121

(c) Write the inequality as


 
18(a3 + b3 + c 3 ) + 24a bc ≥ 13 a b(a + b) − 9 a b(a − b),

18(a3 + b3 + c 3 ) + 24a bc − 13 a b(a + b) ≥ 9(a − b)(b − c)(c − a),

18(a3 + b3 + c 3 ) + 63a bc − 13(a + b + c)(a b + bc + ca) ≥ 9(a − b)(b − c)(c − a),

18(3r + p3 − 3pq) + 63r − 13pq ≥ 9(a − b)(b − c)(c − a),

18p3 − 67pq ≥ −117r + 9(a − b)(b − c)(c − a),

18 − 67q ≥ −117r + 9(a − b)(b − c)(c − a).


It suffices to show that
67q
2− ≥ −13r + |(a − b)(b − c)(c − a)|.
9
As shown at (a), we have

−13q 26 28
−13r + |(a − b)(b − c)(c − a)| ≤ + + (1 − 3q)3/2 .
3 27 27

Therefore, it suffices to show that

67q −13q 26 28
2− ≥ + + (1 − 3q)3/2 ,
9 3 27 27
which is equivalent to
1 − 3q ≥ (1 − 3q)3/2 ,

(1 − 3q)(1 − 1 − 3q ) ≥ 0,
3q(1 − 3q)
 ≥ 0.
1− 1 − 3q
The last inequality is true, with equality for q = 1/3 and q = 0. The original
inequality is an equality for a = b = c > 0, and also for

a b+ bc+ca = 0, (a+ b+c)3 +7a bc = 0, a+ b+c > 0, (a− b)(b−c)(c−a) > 0.

The last equality conditions are equivalent to the condition that a, b, c satisfy

(a − b)(b − c)(c − a) > 0

and are proportional to the roots of the equation

7w3 − 71w2 + 1 = 0.
122 Vasile Cîrtoaje

Remark. The inequality (c) is sharper than the inequalities (b), and the inequality
(b) is sharper than the inequalities (a). Indeed, the inequalities (b) can be obtained
by summing the inequality in (c) to the obvious inequality

2(a + b + c)(a b + bc + ca) ≥ 0,

and the inequalities (a) can be obtained by summing the inequality in (b) to the
inequality
1
(a + b + c)(a b + bc + ca) ≥ 0.
2

P 2.9. If a, b, c are real numbers such that

a + b + c > 0, a b + bc + ca ≥ 0,

then
11
(a) a3 + b3 + c 3 + a bc ≥ (a − b)(b − c)(c − a);
4
! !
(b) 4(a3 + b3 + c 3 ) + 5a bc + 2 a2 b ≥ 6 a b2 ;
! !
(c) 36(a3 + b3 + c 3 ) + 30a bc + 13 a2 b ≥ 59 a b2 .
(Vasile C., 2006)

Solution. Consider
p2 1
p = 1, 0≤q≤ = .
3 3
(a) Write the inequality as
11
3r + p3 − 3pq + r ≥ (a − b)(b − c)(c − a),
4
23
p3 − 3pq + r ≥ (a − b)(b − c)(c − a),
4
−23
1 − 3q ≥ r + (a − b)(b − c)(c − a).
4
It suffices to show that
−23r
1 − 3q ≥ + |(a − b)(b − c)(c − a)|.
4
−23
Applying Corollary 1 for β = , we have
108
−23 −23q 23 31
r + |(a − b)(b − c)(c − a)| ≤ + + (1 − 3q)3/2 ,
4 12 54 54
pqr Method 123

with equality for

46(1 − 3q)3/2 = 31(9q − 27r − 2). (*)

Therefore, it suffices to show that


−23q 23 31
1 − 3q ≥ + + (1 − 3q)3/2 ,
12 54 54
which is equivalent to
62 − 117q ≥ 62(1 − 3q)3/2 .
Using the substitution 
t= 1 − 3q, 0 ≤ t ≤ 1,
the inequality becomes
62 − 39(1 − t 2 ) ≥ 62t 3 ,
23 + 39t 2 − 62t 3 ≥ 0.
We have
23 + 39t 2 − 62t 3 = (1 − t)(23 + 23t + 62t 2 ) ≥ 0,
with equality for t = 1. Notice that t = 1 involves q = 0, and (*) gives r = −4/31.
Thus, if p = 1, then the original inequality is an equality for q = 0, r = −4/31 and
(a − b)(b − c)(c − a) > 0. More general, the equality holds for

a b+bc+ca = 0, 4(a+b+c)3 +31a bc = 0, a+b+c > 0, (a−b)(b−c)(c−a) > 0.

These equality conditions are equivalent to the condition that a, b, c satisfy

(a − b)(b − c)(c − a) > 0

and are proportional to the roots of the equation

31w3 − 31w2 + 4 = 0.

(b) Write the inequality as


 
4(a3 + b3 + c 3 ) + 5a bc ≥ 2 a b(a + b) − 4 a b(a − b),

4(a3 + b3 + c 3 ) + 5a bc ≥ 2 a b(a + b) + 4(a − b)(b − c)(c − a),

4(a + b + c ) + 11a bc ≥ 2(a + b + c)(a b + bc + ca) + 4(a − b)(b − c)(c − a),


3 3 3

4(3r + p3 − 3pq) + 11r ≥ 2pq + 4(a − b)(b − c)(c − a),


7 −23
p3 − pq ≥ r + (a − b)(b − c)(c − a),
2 4
7q −23r
1− ≥ + (a − b)(b − c)(c − a).
2 4
124 Vasile Cîrtoaje

It suffices to show that


7q −23
1− ≥ r + |(a − b)(b − c)(c − a)|.
2 4
As shown at (a), we have
−23 −23q 23 31
r + |(a − b)(b − c)(c − a)| ≤ + + (1 − 3q)3/2 .
4 12 54 54
Therefore, it suffices to show that
7q −23q 23 31
1− ≥ + + (1 − 3q)3/2 ,
2 12 54 54
which is equivalent to
62 − 171q ≥ 62(1 − 3q)3/2 .
Using the substitution 
t= 1 − 3q, 0 ≤ t ≤ 1,
the inequality becomes
62 − 57(1 − t 2 ) ≥ 62t 3 ,
5 + 57t 2 − 62t 3 ≥ 0.
We have
5 + 57t 2 − 62t 3 = (1 − t)(5 + 5t + 62t 2 ) ≥ 0,
with equality for t = 1. The equality holds for

a b+bc+ca = 0, 4(a+b+c)3 +31a bc = 0, a+b+c > 0, (a−b)(b−c)(c−a) > 0.

These equality conditions are equivalent to the condition that a, b, c satisfy

(a − b)(b − c)(c − a) > 0

and are proportional to the roots of the equation

31w3 − 31w2 + 4 = 0.

(c) Write the inequality as


 
36(a3 + b3 + c 3 ) + 30a bc ≥ 23 a b(a + b) − 36 a b(a − b),

36(a3 + b3 + c 3 ) + 30a bc ≥ 23 a b(a + b) + 36(a − b)(b − c)(c − a),

36(a + b + c ) + 99a bc − 23(a + b + c)(a b + bc + ca) ≥ 36(a − b)(b − c)(c − a),


3 3 3

36(3r + p3 − 3pq) + 99r − 23pq ≥ 36(a − b)(b − c)(c − a),


36p3 − 131pq ≥ −207r + 36(a − b)(b − c)(c − a),
pqr Method 125

36 − 131q ≥ −207r + 36(a − b)(b − c)(c − a).


It suffices to show that
36 − 131q −23r
≥ + |(a − b)(b − c)(c − a)|.
36 4
As shown at (a), we have
−23 −23q 23 31
r + |(a − b)(b − c)(c − a)| ≤ + + (1 − 3q)3/2 .
4 12 54 54
Therefore, it suffices to show that
36 − 131q −23q 23 31
≥ + + (1 − 3q)3/2 ,
36 12 54 54
which is equivalent to
1 − 3q ≥ (1 − 3q)3/2 ,

(1 − 3q)(1 − 1 − 3q ) ≥ 0,
3q(1 − 3q)
 ≥ 0.
1− 1 − 3q
The last inequality is true, with equality for q = 1/3 and q = 0. The original
inequality is an equality for a = b = c > 0, and also for

a b+bc+ca = 0, 4(a+b+c)3 +31a bc = 0, a+b+c > 0, (a−b)(b−c)(c−a) > 0.

The last equality conditions are equivalent to the condition that a, b, c satisfy

(a − b)(b − c)(c − a) > 0

and are proportional to the roots of the equation

31w3 − 31w2 + 4 = 0.

Remark. The inequality (c) is sharper than the inequalities (b), and the inequality
(b) is sharper than the inequalities (a). Indeed, the inequalities (b) can be obtained
by summing the inequality in (c) to the obvious inequality

5(a + b + c)(a b + bc + ca) ≥ 0,

and the inequalities (a) can be obtained by summing the inequality in (b) to the
inequality
2(a + b + c)(a b + bc + ca) ≥ 0.
126 Vasile Cîrtoaje

P 2.10. If a, b, c are real numbers such that

a + b + c > 0, a b + bc + ca ≥ 0,

then
1
a3 + b3 + c 3 − a bc ≥ 2(a − b)(b − c)(c − a).
4
(Vasile C., 2006)

Solution. Assume
p2 1
p = 1, 0≤q≤ = ,
3 3
and write the inequality as

1
3r + p3 − 3pq − r ≥ 2(a − b)(b − c)(c − a),
4
11
p3 − 3pq + r ≥ 2(a − b)(b − c)(c − a),
4
−11
1 − 3q ≥ r + 2(a − b)(b − c)(c − a).
4
It suffices to show that
1 3 −11r
− q≥ + |(a − b)(b − c)(c − a)|.
2 2 8
−11
Applying Corollary 1 for β = , we have
216
−11 −11q 11 43
r + |(a − b)(b − c)(c − a)| ≤ + + (1 − 3q)3/2 ,
8 24 108 108
with equality for

22(1 − 3q)3/2 = 43(9q − 27r − 2). (*)

Therefore, it suffices to show that


1 3 −11q 11 43
− q≥ + + (1 − 3q)3/2 ,
2 2 24 108 108
which is equivalent to
86 − 225q ≥ 86(1 − 3q)3/2 .
Using the substitution 
t= 1 − 3q, 0 ≤ t ≤ 1,
the inequality becomes
86 − 75(1 − t 2 ) ≥ 86t 3 ,
pqr Method 127

11 + 75t 2 − 86t 3 ≥ 0.
We have
11 + 75t 2 − 86t 3 = (1 − t)(11 + 11t + 86t 2 ) ≥ 0,
with equality for t = 1. Notice that t = 1 involves q = 0, and (*) gives r = −4/43.
Thus, if p = 1, then the original inequality is an equality for q = 0, r = −4/43 and
(a − b)(b − c)(c − a) > 0. More general, the equality holds for

a b+bc+ca = 0, 4(a+b+c)3 +43a bc = 0, a+b+c > 0, (a−b)(b−c)(c−a) > 0.

These equality conditions are equivalent to the condition that a, b, c satisfy

(a − b)(b − c)(c − a) > 0

and are proportional to the roots of the equation

43w3 − 43w2 + 4 = 0.

P 2.11. If a, b, c are real numbers such that

a + b + c > 0, a b + bc + ca ≥ 0,

then

(a) 2(a3 + b3 + c 3 ) + 3(3 3 − 2)a bc + 6 3 (a2 b + b2 c + c 2 a) ≥ 0;


3 3
(b) a3 + b3 + c 3 − 3a bc ≥ (a − b)(b − c)(c − a).
2
(Vasile C., 2006)

Solution. Due to homogeneity, we may set

p2 1
p = 1, 0≤q≤ = .
3 3

(a) Write the inequality as






2(a3 + b3 + c 3 ) + 3(3 3 − 2)a bc + 3 3 a b(a + b) + 3 3 a b(a − b) ≥ 0,




2(a3 + b3 + c 3 ) + 3(3 3 − 2)a bc + 3 3 a b(a + b) ≥ 3 3 (a − b)(b − c)(c − a),



2(3r + p3 − 3pq) − 6r + 3 3 pq ≥ 3 3 (a − b)(b − c)(c − a),



2p3 − 3(2 − 3)pq ≥ 3 3 (a − b)(b − c)(c − a),



2 − 3(2 − 3)q ≥ 3 3(a − b)(b − c)(c − a).


128 Vasile Cîrtoaje

It suffices to show that



2 − 3(2 − 3)q ≥ 3 3|(a − b)(b − c)(c − a)|.

Applying Corollary 1 for β = 0, we have

2
|(a − b)(b − c)(c − a)| ≤
(1 − 3q)3/2 ,
3 3
with equality for

27r = 9q − 2. (*)

Therefore, it suffices to show that


2 − 3(2 − 3)q ≥ 2(1 − 3q)3/2 .

Using the substitution 


t= 1 − 3q, 0 ≤ t ≤ 1,
the inequality becomes

2 − (2 − 3)(1 − t 2 ) ≥ 2t 3 ,

3 + (2 − 3)t 2 − 2t 3 ≥ 0,

(1 − t)( 3 + 3 t + 2t 2 ) ≥ 0,
with equality for t = 1. Notice that t = 1 involves q = 0, and (*) gives r = −2/27.
Thus, if p = 1, then the original inequality is an equality for q = 0, r = −2/27 and
(a − b)(b − c)(c − a) > 0. More general, the equality holds for

a b+bc+ca = 0, 2(a+b+c)3 +27a bc = 0, a+b+c > 0, (a−b)(b−c)(c−a) > 0;

that is,
b c
a=
=
>0
1+ 3 1− 3
(or any cyclic permutation).
(b) Write the inequality as

3 3
p3 − 3pq ≥ (a − b)(b − c)(c − a),
2

3 3
1 − 3q ≥ (a − b)(b − c)(c − a).
2
It suffices to show that

3 3
1 − 3q ≥ |(a − b)(b − c)(c − a)|.
2
pqr Method 129

As shown at (a), we have


2
|(a − b)(b − c)(c − a)| ≤
(1 − 3q)3/2 .
3 3
Therefore, it suffices to show that

1 − 3q ≥ (1 − 3q)3/2 .

which is equivalent to   
(1 − 3q) 1 − 1 − 3q ≥ 0,
3q(1 − 3q)
 ≥ 0.
1+ 1 − 3q
The equality holds for q = 1/3 and q = 0. The original inequality is an equality for
a = b = c > 0, and also for

a b+bc+ca = 0, 2(a+b+c)3 +27a bc = 0, a+b+c > 0, (a−b)(b−c)(c−a) > 0;

this means that


b c
a=
=
>0
1+ 3 1− 3
(or any cyclic permutation).
Remark. The inequality (b) is sharper than the inequality (a), because the last
inequalities can be obtained by summing the inequality in (b) to the obvious in-
equality

3 3
(a + b + c)(a b + bc + ca) ≥ 0.
2

P 2.12. If a, b, c are real numbers such that

a + b + c > 0, 2(a2 + b2 + c 2 ) + 7(a b + bc + ca) ≥ 0,

then
a2 (a − b) + b2 (b − c) + c 2 (c − a) ≥ 0.
(Vasile C., 2006)

Solution. Due to homogeneity, we may set p = 1. From p2 ≥ 3q and

2(a2 + b2 + c 2 ) + 7(a b + bc + ca) = 2p2 + 3q = 2 + 3q ≥ 0,

we get
−2 1
≤q≤ .
3 3
130 Vasile Cîrtoaje

Write the inequality as


 
2(a3 + b3 + c 3 ) ≥ a b(a + b) + a b(a − b),

2(a3 + b3 + c 3 ) + 3a bc ≥ (a + b + c)(a b + bc + ca) − (a − b)(b − c)(c − a),


2(3r + p3 − 3pq) + 3r ≥ pq − (a − b)(b − c)(c − a),
2p3 − 7pq ≥ −9r − (a − b)(b − c)(c − a),
2 − 7q ≥ −9r − (a − b)(b − c)(c − a).
It suffices to show that

2 − 7q ≥ −9r + |(a − b)(b − c)(c − a)|.

Applying Corollary 1 for β = −1/3, we have


2 4
−9r + |(a − b)(b − c)(c − a)| ≤ −3q + +
(1 − 3q)3/2 ,
3 3 3
with equality for

3 (1 − 3q)3/2 = 9q − 27r − 2. (*)

Therefore, it suffices to show that


2 4
2 − 7q ≥ −3q + +
(1 − 3q)3/2 ,
3 3 3
which is equivalent to

3(1 − 3q) ≥ (1 − 3q)3/2 ,




(1 − 3q)( 3 − 1 − 3q) ≥ 0,
(1 − 3q)(2 + 3q)

 ≥ 0,
3 + 1 − 3q
with equality for q = 1/3 and q = −2/3. According to (*), q = 1/3 involves
r = 1/27, and q = −2/3 involves r = −17/27. Thus, the equality holds for a =
b = c > 0, and also for

17(a + b + c)3 + 27a bc = 0, 2(a + b + c)2 + 3(a b + bc + ca) = 0,

a + b + c > 0, (a − b)(b − c)(c − a) < 0.


The last equality conditions are equivalent to the condition that a, b, c satisfy

(a − b)(b − c)(c − a) < 0

and are proportional to the roots of the equation

27w3 − 27w2 − 18w + 17 = 0.


pqr Method 131

P 2.13. If a, b, c are real numbers such that

a + b + c > 0, 3(a b + bc + ca) ≥ a2 + b2 + c 2 ,

then
a2 (a + 2b − 3c) + b2 (b + 2c − 3a) + c 2 (c + 2a − 3b) ≥ 0.
(Vasile C., 2006)

Solution. Due to homogeneity, we may set p = 1. From p ≥ 3q and 2

3(a b + bc + ca) − (a2 + b2 + c 2 ) = 3q − (p2 − 2q) = 5q − 1 ≥ 0,

we get
1 1
≤q≤ .
5 3
Write the inequality as follows:
 
2(a3 + b3 + c 3 ) ≥ a b(a + b) − 5 a b(a − b),

2(a3 + b3 + c 3 ) + 3a bc ≥ (a + b + c)(a b + bc + ca) + 5(a − b)(b − c)(c − a),


2(3r + p3 − 3pq) + 3r ≥ pq + 5(a − b)(b − c)(c − a),
2p3 − 7pq ≥ −9r + 5(a − b)(b − c)(c − a),
2 − 7q ≥ −9r + 5(a − b)(b − c)(c − a).
It suffices to show that
2 − 7q −9r
≥ + |(a − b)(b − c)(c − a)|.
5 5
Applying Corollary 1 for β = −1/15, we have

−9r −3q 2 4
7
+ |(a − b)(b − c)(c − a)| ≤ + + (1 − 3q)3/2 ,
5 5 15 15 3

with equality for



3 (1 − 3q)3/2 = 7 (9q − 27r − 2). (*)

Therefore, it suffices to show that



2 − 7q −3q 2 4
7
≥ + + (1 − 3q)3/2 ,
5 5 15 15 3

which is equivalent to

3 (1 − 3q) ≥ 7 (1 − 3q)3/2 ,
132 Vasile Cîrtoaje
"
#
(1 − 3q) 3− 7(1 − 3q) ≥ 0.
This is true if
3 ≥ 7(1 − 3q),
4
which is equivalent to q ≥ . Indeed, we have
21
1 4
q≥ > .
5 21
The equality holds for a = b = c > 0.

P 2.14. If a, b, c are real numbers, then

(a2 + b2 + c 2 )2 ≥ 3(a3 b + b3 c + c 3 a).

(Vasile C., 1992)

Solution. Write the inequality as follows:


 
2(a2 + b2 + c 2 )2 ≥ 3 a b(a2 + b2 ) + 3 a b(a2 − b2 ),
 2       
2 a2 ≥3 ab a2 − 3a bc a−3 a (a − b)(b − c)(c − a),

2(p2 − 2q)2 − 3q(p2 − 2q) ≥ −3pr − 3p(a − b)(b − c)(c − a),


2p4 − 11p2 q + 14q2 ≥ −3pr − 3p(a − b)(b − c)(c − a).
It suffices to show that

2p4 − 11p2 q + 14q2 ≥ 3p [−r + |(a − b)(b − c)(c − a)|] .

Since the inequality remains unchanged by replacing a, b, c with −a, −b, −c, re-
spectively, we may assume that p ≥ 0. For the nontrivial case p = 0, we may set
p = 1 (due to homogeneity). Thus, we need to show that

2 − 11q + 14q2
≥ −r + |(a − b)(b − c)(c − a).
3
Applying Corollary 1 for β = −1/27, we have

−q 2 4 7
−r + |(a − b)(b − c)(c − a)| ≤ + + (1 − 3q)3/2 ,
3 27 27
with equality for

(1 − 3q)3/2 = 7 (9q − 27r − 2). (*)


pqr Method 133

Therefore, it suffices to show that


2 − 11q + 14q2 −q 2 4 7
≥ + + (1 − 3q)3/2 ,
3 3 27 27

which is equivalent to

8 − 45q + 63q2 ≥ 2 7 (1 − 3q)3/2 ,


(1 − 3q)(8 − 21q) ≥ 2 7 (1 − 3q)3/2 ,


" #
(1 − 3q) 8 − 21q − 2 7(1 − 3q) ≥ 0.

Since 1 − 3q ≥ 0, this is true if

(8 − 21q)2 ≥ 28(1 − 3q),

which is equivalent to
(7q − 2)2 ≥ 0.
According to (*), q = 1/3 involves r = 1/27, and q = 2/7 involves r = −1/49.
Thus, the original inequality is an equality for a = b = c, and also for

2p2 − 7q = 0, p3 + 49r = 0, (a + b + c)(a − b)(b − c)(c − a) ≤ 0.

The last equality conditions are equivalent to the condition that a, b, c satisfy

(a − b)(b − c)(c − a) ≤ 0

and are proportional to the roots of the equation

49w3 − 49w2 + 14w + 1 = 0;

this means that


a b c
= =
sin2 4π
7 sin2 2π
7
sin2 π7
(or any cyclic permutation).

P 2.15. If a, b, c are real numbers, then

a3 (a − 2b + c) + b3 (b − 2c + a) + c 3 (c − 2a + b) ≥ 0.

(Vasile C., 1998)


134 Vasile Cîrtoaje

Solution. Write the inequality as follows:


 
2(a4 + b4 + c 4 ) ≥ a b(a2 + b2 ) + 3 a b(a2 − b2 ),
       
2 a4 ≥ ab a2 − a bc a−3 a (a − b)(b − c)(c − a),

2(p4 − 4p2 q + 2q2 + 4pr) ≥ q(p2 − 2q) − pr − 3p(a − b)(b − c)(c − a),
2p4 − 9p2 q + 6q2 ≥ −9pr − 3p(a − b)(b − c)(c − a).
Since the inequality remains unchanged by replacing a, b, c with −a, −b, −c, re-
spectively, we may assume that p ≥ 0. For the nontrivial case p > 0, it suffices to
show that

2p4 − 9p2 q + 6q2 ≥ 3p [−3r + |(a − b)(b − c)(c − a)|] .

Due to homogeneity, we may set p = 1. Thus, we need to show that

2 − 9q + 6q2
≥ −3r + |(a − b)(b − c)(c − a).
3
Applying Corollary 1 for β = −1/9, we have
2 4
−3r + |(a − b)(b − c)(c − a)| ≤ −q + + (1 − 3q)3/2 ,
9 9
with equality for

(1 − 3q)3/2 = 9q − 27r − 2. (*)

Therefore, it suffices to show that

2 − 9q + 6q2 2 4
≥ −q + + (1 − 3q)3/2 ,
3 9 9
which is equivalent to
2 − 9q + 9q2 ≥ 2(1 − 3q)3/2 ,
(1 − 3q)(2 − 3q) ≥ 2(1 − 3q)3/2 ,
  
(1 − 3q) 2 − 3q − 2 1 − 3q ≥ 0.
Since 1 − 3q ≥ 0, this is true if

(2 − 3q)2 ≥ 4(1 − 3q),

which is equivalent to q2 ≥ 0. According to (*), q = 1/3 involves r = 1/27, and


q = 0 involves r = −1/9. Thus, the original inequality is an equality for a = b = c,
and also for

q = 0, p3 + 9r = 0, (a + b + c)(a − b)(b − c)(c − a) ≤ 0.


pqr Method 135

The last equality conditions are equivalent to the condition that a, b, c satisfy

(a − b)(b − c)(c − a) ≤ 0

and are proportional to the roots of the equation

9w3 − 9w2 + 1 = 0;

this means that


π 7π 13π
a sin = b sin = c sin
9 9 9
(or any cyclic permutation).

P 2.16. If a, b, c are real numbers, then


a4 + b4 + c 4 − a bc(a + b + c) ≥ 3 (a + b + c)(a − b)(b − c)(c − a).

(Vasile C., 2006)

Solution. Since the inequality remains unchanged by replacing a, b, c with −a, −b, −c,
respectively, we may assume that p ≥ 0. For the nontrivial case p > 0, it suffices to
show that

a4 + b4 + c 4 − a bc(a + b + c) ≥ 3 (a + b + c)|(a − b)(b − c)(c − a)|,

which is equivalent to

p4 − 4p2 q + 2q2 + 3pr ≥ 3 p|(a − b)(b − c)(c − a)|.

Due to homogeneity, we may set p = 1. Thus, we need to show that





1 − 4q + 2q2 ≥ 3 − 3 r + |(a − b)(b − c)(c − a)| .

Applying Corollary 1 for β = − 3/27, we have





− 3 q 2 3 2 10
− 3 r + |(a − b)(b − c)(c − a)| ≤ + +
(1 − 3q)3/2 ,
3 27 3 3
with equality for

2(1 − 3q)3/2 = 10 (9q − 27r − 2). (*)

Therefore, it suffices to show that


2 2 10
1 − 4q + 2q2 ≥ −q + + (1 − 3q)3/2 ,
9 9
136 Vasile Cîrtoaje

which is equivalent to

7 − 27q + 18q2 ≥ 2 10 (1 − 3q)3/2 ,


(1 − 3q)(7 − 6q) ≥ 2 10 (1 − 3q)3/2 ,


" #
(1 − 3q) 7 − 6q − 2 10(1 − 3q) ≥ 0.
Since 1 − 3q ≥ 0, this is true if

(7 − 6q)2 ≥ 40(1 − 3q),

which is equivalent to
(2q + 1)2 ≥ 0.
According to (*), q = 1/3 involves r = 1/27, and q = −1/2 involves r = −1/3.
Thus, the original inequality is an equality for a = b = c, and also for

(a+b+c)2 +2(a b+bc+ca) = 0, (a+b+c)3 +3a bc = 0, (a+b+c)(a−b)(b−c)(c−a) ≥ 0.

The last equality conditions are equivalent to the condition that a, b, c satisfy

(a − b)(b − c)(c − a) ≥ 0

and are proportional to the roots of the equation

6w3 − 6w2 − 3w + 2 = 0.

P 2.17. If a, b, c are real numbers, then

2
a3 (a + b) + b3 (b + c) + c 3 (c + a) ≥ (a b + bc + ca)2 .
3
(Vasile C., 2006)

Solution. Write the inequality as follows:


  4
2(a4 + b4 + c 4 ) + a b(a2 + b2 ) + a b(a2 − b2 ) ≥ (a b + bc + ca)2 ,
3
 4
2(a4 + b4 + c 4 )+ a b(a2 + b2 )− (a + b + c)(a − b)(b − c)(c − a) ≥ (a b + bc + ca)2 ,
3
4
2(p4 − 4p2 q + 2q2 + 4pr) + q(p2 − 2q) − pr − p(a − b)(b − c)(c − a) ≥ q2 ,
3
2 2
2p − 7p q + q + 7pr ≥ p(a − b)(b − c)(c − a).
4 2
3
pqr Method 137

Since the statement remains unchanged by replacing a, b, c with −a, −b, −c, re-
spectively, we may assume p ≥ 0. For the nontrivial case p > 0, it suffices to show
that
2
2p4 − 7p2 q + q2 + 7pr ≥ p|(a − b)(b − c)(c − a)|.
3
Due to homogeneity, we may set p = 1. From p2 ≥ 3q, we get

1
q≤ .
3
Thus, we need to show that
2
2 − 7q + q2 ≥ −7r + |(a − b)(b − c)(c − a)|.
3
−7
Applying Corollary 1 for β = , we have
27

−7q 14 4 19
−7r + |(a − b)(b − c)(c − a)| ≤ + + (1 − 3q)3/2 ,
3 27 27
with equality for

7(1 − 3q)3/2 = 19(9q − 27r − 2). (*)

Therefore, it suffices to show that


2 −7q 14 4 19
2 − 7q + q2 ≥ + + (1 − 3q)3/2 ,
3 3 27 27
which is equivalent to

20 − 63q + 9q2 ≥ 2 19(1 − 3q)3/2 ,

(1 − 3q)(20 − 3q) ≥ 2 19(1 − 3q)3/2 ,
" #
(1 − 3q) 20 − 3q − 2 19(1 − 3q) ≥ 0.
Since 1 − 3q ≥ 0, this is true if

(20 − 3q)2 ≥ 76(1 − 3q),

which is equivalent to
(q + 6)2 ≥ 0,
According to (*), q = 1/3 involves r = 1/27, and q = −6 involves r = −7. Thus,
the original inequality is an equality for a = b = c, and also for

a b+bc+ca+6(a+b+c)2 = 0, a bc+7(a+b+c)3 = 0, (a+b+c)(a−b)(b−c)(c−a) ≥ 0.


138 Vasile Cîrtoaje

The last equality conditions are equivalent to the condition that a, b, c satisfy

(a − b)(b − c)(c − a) ≥ 0

and are proportional to the roots of the equation

w3 − w2 − 6w + 7 = 0.

P 2.18. If a, b, c are real numbers such that a b + bc + ca = 3, then

a3 (a − 2b) + b3 (b − 2c) + c 3 (c − 2a) + 3 ≥ 0.

(Vasile C., 2006)


Solution. Write the inequality in the homogeneous form
  1
a4 + b4 + c 4 − a b(a2 + b2 ) − a b(a2 − b2 ) + (a b + bc + ca)2 ≥ 0,
3
which is equivalent to
 1 
a4 + b4 + c 4 − a b(a2 + b2 ) + (a b + bc + ca)2 ≥ a b(a2 − b2 ),
3
1
(p4 − 4p2 q + 2q2 + 4pr) − q(p2 − 2q) + pr + q2 ≥ −p(a − b)(b − c)(c − a),
3
13 2
p4 − 5p2 q + q + 5pr ≥ −p(a − b)(b − c)(c − a).
3
Since the inequality remains unchanged by replacing a, b, c with −a, −b, −c, re-
spectively, we may assume p ≥ 0. For the nontrivial case p > 0, it suffices to show
that
13 2
p4 − 5p2 q + q + 5pr ≥ p|(a − b)(b − c)(c − a)|.
3
Due to homogeneity, we may leave out the hypothesis a b + bc + ca = 3 for p = 1.
From p2 ≥ 3q, we get
1
q≤ .
3
Thus, we need to show that
13 2
1 − 5q + q ≥ −5r + |(a − b)(b − c)(c − a)|.
3
−5
Applying Corollary 1 for β = , we have
27

−5q 10 4 13
−5r + |(a − b)(b − c)(c − a)| ≤ + + (1 − 3q)3/2 ,
3 27 27
pqr Method 139

with equality for


5(1 − 3q)3/2 = 13(9q − 27r − 2). (*)

Therefore, it suffices to show that


13 2 −5q 10 4 13
1 − 5q + q ≥ + + (1 − 3q)3/2 ,
3 3 27 27
which is equivalent to

17 − 90q + 117q2 ≥ 4 13(1 − 3q)3/2 ,


(1 − 3q)(17 − 39q) ≥ 4 13(1 − 3q)3/2 ,


" #
(1 − 3q) 17 − 39q − 4 13(1 − 3q) ≥ 0.
Since 1 − 3q ≥ 0, this is true if

(17 − 39q)2 ≥ 208(1 − 3q),

which is equivalent to
(13q − 3)2 ≥ 0,
According to (*), q = 1/3 involves r = 1/27, and q = 3/13 involves r = −1/169.
Thus, the homogeneous inequality is an equality for a = b = c, and also for

13(a b + bc + ca) = 3(a + b + c)2 = 0, 169a bc + (a + b + c)3 = 0,

(a + b + c)(a − b)(b − c)(c − a) < 0.


Since a b + bc + ca = 3, the original inequality is an equality for

−1
a+b+c = 13, a bc =
, (a − b)(b − c)(c − a) < 0.
13
The last equality conditions are equivalent to the condition that a, b, c satisfy

(a − b)(b − c)(c − a) < 0

and are the roots of the equation



1
w3 − 13 w2 + 3w +
= 0.
13

Remark 1. The inequalities in P 2.14, ... , P2.18 are particular cases of the following
generalization:
• Let A, B, C, D be real numbers such that

1 + A + B + C + D = 0, 3(1 + A) ≥ C 2 + C D + D2 .
140 Vasile Cîrtoaje

Then, for any real a, b, c, the following inequality holds:


    
a4 + A a2 b2 + Ba bc a+C a3 b + D a b3 ≥ 0.

To prove this inequality, we write it in the form


      
a4 + A a2 b2 − a bc a − (1 + C + D)a bc a+C a3 b + D a b3 ≥ 0.
! !
Since a2 b2 − a bc a ≥ 0, it suffices to consider the case

3(1 + A) = C 2 + C D + D2 .

Proceeding as in the preceding problems (with p = 1), we need to show that

7 + 2A + C + D − 3(2 + A − C − D)q ≥

≥2 (1 − A)2 − (1 − A)(C + D) + C 2 − C D + D2 · 1 − 3q.
Since
C 2 + C D + D2 (C + D)2
1+A= ≥ ,
3 4
we have

(C + D)2
2 + A − C − D = (1 + A) + 1 − C − D ≥ +1−C −D
4
(C + D − 2)2
= ≥ 0,
4
hence

7 + 2A + C + D − 3(2 + A − C − D)q ≥ 7 + 2A + C + D − (2 + A − C − D)
= (1 + A) + 4 + 2(C + D)
(C + D)2
≥ + 4 + 2(C + D)
4
(C + D + 4)2
= ≥ 0.
4
Thus, we only need to show that

[7 + 2A + C + D − 3(2 + A − C − D)q]2 ≥
 
4 (1 − A)2 − (1 − A)(C + D) + C 2 − C D + D2 (1 − 3q),
which is equivalent to
 2
(2 + A − C − D)2 q − 4 − 5A + (1 + A)(C + D) + C 2 + D2 ≥ 0.
pqr Method 141

Remark 2. Replacing A, B, C, D with A/A0 , B/A0 , C/A0 , D/A0 , respectively, the state-
ment from Remark 1 becomes as follows:
• Let A0 , A, B, C, D be real numbers such that

A0 > 0, A0 + A + B + C + D = 0, 3A0 (A0 + A) ≥ C 2 + C D + D2 .

Then, for any real a, b, c, the following inequality holds:


    
A0 a4 + A a2 b2 + Ba bc a+C a3 b + D a b3 ≥ 0.

P 2.19. If a, b, c are real numbers such that a b + bc + ca ≥ 0, then


15
a4 + b4 + c 4 − a bc(a + b + c) ≥ (a + b + c)(a − b)(b − c)(c − a).
2
(Vasile C., 2006)

First Solution. Since the statement remains unchanged by replacing a, b, c with


−a, −b, −c, respectively, we may assume that p ≥ 0. For the nontrivial case p > 0,
it suffices to show that

15
a4 + b4 + c 4 − a bc(a + b + c) ≥ (a + b + c)|(a − b)(b − c)(c − a)|,
2
which is equivalent to

15
p4 − 4p2 q + 2q2 + 3pr ≥ p|(a − b)(b − c)(c − a)|.
2

Due to homogeneity, we may set p = 1. From p2 ≥ 3q, we get

1
0≤q≤ .
3
We need to show that


15 −6r
1 − 4q + 2q2 ≥
+ |(a − b)(b − c)(c − a)| .
2 15
−2
Applying Corollary 1 for β =
, we have
9 15
−6r −2q 4 14

+ |(a − b)(b − c)(c − a)| ≤
+
+
(1 − 3q)3/2 ,
15 15 9 15 9 15
142 Vasile Cîrtoaje

with equality for

4(1 − 3q)3/2 = 7(9q − 27r − 2). (*)

Therefore, it suffices to show that




15 −2q 4 14
1 − 4q + 2q2 ≥
+
+
(1 − 3q)3/2 ,
2 15 9 15 9 15
which is equivalent to

7 − 27q + 18q2 ≥ 7(1 − 3q)3/2 ,

(1 − 3q)(7 − 6q) ≥ 7(1 − 3q)3/2 ,


  
(1 − 3q) 7 − 6q − 7 1 − 3q ≥ 0.
Since 1 − 3q ≥ 0, this is true if

(7 − 6q)2 ≥ 49(1 − 3q),

which is equivalent to
q(4q + 7) ≥ 0,
According to (*), q = 1/3 involves r = 1/27, and q = 0 involves r = −2/21. Thus,
the original inequality is an equality for a = b = c, and also for

a b + bc + ca = 0, 21a bc + 2(a + b + c)3 = 0, (a + b + c)(a − b)(b − c)(c − a) ≥ 0.

The last equality conditions are equivalent to the condition that a, b, c satisfy

(a − b)(b − c)(c − a) ≥ 0

and are proportional to the roots of the equation

21w3 − 21w2 + 2 = 0.

Second Solution. We will find a stronger inequality

f (a, b, c) ≥ 0,

where


15  3 
f (a, b, c) =a4 + b4 + c 4 − a bc(a + b + c) + a b− a b3
2
− k(a b + bc + ca)(a2 + b2 + c 2 − a b − bc − ca), k > 0,

satisfies f (1, 1, 1) = 0. Since


   
(a b+ bc +ca)(a2 + b2 +c 2 −a b− bc −ca) = − a2 b2 −a bc a+ a3 b+ a b3 ,
pqr Method 143

the inequality can be written as


    
a4 + A a2 b2 + Ba bc a+C a3 b + D a b3 ≥ 0,

where

15 15
A = k, B = k − 1, C = −k + , D = −k − .
2 2
We see that 1 + A + B + C + D = 0. According to the statement in Remark 1 from P
2.18, if
3(1 + A) = C 2 + C D + D2 ,
then the inequality holds for all real a, b, c. It is easy to show that this condition
is satisfied for k = 1/2. Since the inequality f (a, b, c) ≥ 0 for k = 1/2 is stronger
than the original inequality, the proof is completed.

P 2.20. If a, b, c are real numbers such that

2(a b + bc + ca) ≥ a2 + b2 + c 2 ,

then

39
a4 + b4 + c 4 − a bc(a + b + c) ≥ (a + b + c)(a − b)(b − c)(c − a).
2
(Vasile C., 2006)

Solution. Since the statement remains unchanged by replacing a, b, c with −a, −b, −c,
respectively, we may assume that p ≥ 0. For the nontrivial case p > 0, it suffices to
show that

39
a4 + b4 + c 4 − a bc(a + b + c) ≥ (a + b + c)|(a − b)(b − c)(c − a)|,
2
which is equivalent to

39
p4 − 4p2 q + 2q2 + 3pr ≥ p|(a − b)(b − c)(c − a)|.
2
Due to homogeneity, we may set p = 1. From p2 ≥ 3q and 2(a b + bc + ca) ≥
a2 + b2 + c 2 , we get
1 1
≤q≤ .
4 3
We need to show that


39 −6r
1 − 4q + 2q2 ≥
+ |(a − b)(b − c)(c − a)| .
2 39
144 Vasile Cîrtoaje

−2
Applying Corollary 1 for β =
, we have
9 39
−6r −2q 4 22

+ |(a − b)(b − c)(c − a)| ≤
+
+
(1 − 3q)3/2 ,
39 39 9 39 9 39
with equality for

4(1 − 3q)3/2 = 11(9q − 27r − 2). (*)

Therefore, it suffices to show that




39 −2q 4 22
1 − 4q + 2q2 ≥
+
+
(1 − 3q)3/2 ,
2 39 9 39 9 39
which is equivalent to

7 − 27q + 18q2 ≥ 11(1 − 3q)3/2 ,

(1 − 3q)(7 − 6q) ≥ 11(1 − 3q)3/2 ,


  
(1 − 3q) 7 − 6q − 11 1 − 3q ≥ 0.
Since 1 − 3q ≥ 0, this is true if

(7 − 6q)2 ≥ 121(1 − 3q),

which is equivalent to
(4q − 1)(q + 8) ≥ 0,
According to (*), q = 1/3 involves r = 1/27, and q = 1/4 involves r = 1/132.
Thus, the original inequality is an equality for a = b = c, and also for

4(a b+bc+ca) = (a b+c)2 , 132a bc+(a+b+c)3 = 0, (a+b+c)(a−b)(b−c)(c−a) > 0.

The last equality conditions are equivalent to the condition that a, b, c satisfy

(a − b)(b − c)(c − a) ≥ 0

and are proportional to the roots of the equation

132w3 − 132w2 + 33w − 1 = 0.

Second Solution. We will find a stronger inequality

f (a, b, c) ≥ 0,

where

  
15
f (a, b, c) =a4 + b4 + c 4 − a bc(a + b + c) + a3 b − a b3
    2
 
−k 2 ab − a2 a2 − ab , k > 0,
pqr Method 145

satisfies f (1, 1, 1) = 0. Since


     
− 2 ab − a2 a2 − ab =

 2      2
= a2 − 3 ab a2 + 2 ab
    
= a4 + 4 a2 b2 + a bc a−3 a3 b − 3 a b3 ,

the inequality f (a, b, c) ≥ 0 can be written as


    
A0 a4 + A a2 b2 + Ba bc a+C a3 b + D a b3 ≥ 0,

where

39 39
A0 = k + 1, A = 4k, B = k − 1, C = −3k + , D = −3k − .
2 2
We see that A0 + A + B + C + D = 0. According to the statement in Remark 2 from
P 2.18, if A0 > 0 and
3A0 (A0 + A) = C 2 + C D + D2 ,
then the inequality holds for all real a, b, c. It is easy to show that this condition
is satisfied for k = 3/4. Since the inequality f (a, b, c) ≥ 0 for k = 3/4 is stronger
than the original inequality, the proof is completed.

P 2.21. If a, b, c are real numbers, then

a4 + b4 + c 4 + 2a bc(a + b + c) ≥ a b3 + bc 3 + ca3 .

(Vasile C., 2009)

First Solution. Write the inequality as follows:


 
2(a4 + b4 + c 4 ) + 4a bc(a + b + c) ≥ a b(a2 + b2 ) − a b(a2 − b2 ) ≥ 0,

2(p4 − 4p2 q + 2q2 + 4pr) + 4pr ≥ q(p2 − 2q) − pr + p(a − b)(b − c)(c − a) ≥ 0,
2p4 − 9p2 q + 6q2 ≥ −13pr + p(a − b)(b − c)(c − a) ≥ 0,
Since the statement remains unchanged by replacing a, b, c with −a, −b, −c, re-
spectively, we may assume p ≥ 0. For p = 0, the inequality is clearly true. For
p > 0, it suffices to show that

2p4 − 9p2 q + 6q2 ≥ −13pr + p|(a − b)(b − c)(c − a)| ≥ 0.


146 Vasile Cîrtoaje

Due to homogeneity, we may set p = 1. From p2 ≥ 3q, we get

1
q≤ .
3
Thus, we need to show that

2 − 9q + 6q2 ) ≥ −13r + |(a − b)(b − c)(c − a)| ≥ 0,

−13
Applying Corollary 1 for β = , we have
27
−13q 26 28
−13r + |(a − b)(b − c)(c − a)| ≤ + + (1 − 3q)3/2 ,
3 27 27
with equality for

13(1 − 3q)3/2 = 7(9q − 2 − 27r). (*)

Therefore, it suffices to show that

−13q 26 28
2 − 9q + 6q2 ≥ + + (1 − 3q)3/2 ,
3 27 27
which is equivalent to

14 − 63q + 81q2 ≥ 14(1 − 3q)3/2 .

Using the substitution 


t= 1 − 3q, t ≥ 0,
the inequality becomes
9t 4 − 14t 3 + 3t 2 + 2 ≥ 0,
which is equivalent to
(t − 1)2 (9t 2 + 4t + 2) ≥ 0.
The last inequality is true, with equality for t = 1, that is for q = 0. From (*), we
get r = −1/7. Thus, the original inequality is an equality when a, b, c satisfy

(a − b)(b − c)(c − a) ≥ 0

and are proportional to the roots of the equation

7w3 − 7w2 + 1 = 0.

The equality conditions are equivalent to

a b c
= =
sin 2π
7
sin 4π
7 sin 8π
7
pqr Method 147

(or any cyclic permutation).


Second Solution. We will find a stronger inequality

f (a, b, c) ≥ 0,

where
2 2
f (a, b, c) = a4 +b4 +c 4 +2a bc(a+b+c)−(a b3 +bc 3 +ca3 )− [p +m(p2 −3q)]2 , m ∈ ,
27
satisfies f (1, 1, 1) = 0. The inequality can be written as
    
a4 + A a2 b2 + Ba bc a+C a3 b + D a b3 ≥ 0,

where 1 + A + B + C + D = 0. According to the statement in Remark 1 from P 2.18,


if
3(1 + A) = C 2 + C D + D2 ,
then the inequality f (a, b, c) ≥ 0 holds for all real a, b, c. It is easy to show that
this condition is satisfied for m = −1, when
2
f (a, b, c) = a4 + b4 + c 4 + 2a bc(a + b + c) − (a b3 + bc 3 + ca3 ) − (a b + bc + ca)2 .
3

P 2.22. If a, b, c are real numbers, then


a4 + b4 + c 4 + 2 (a3 b + b3 c + c 3 a) ≥ 0.

(Vasile C., 2009)

Solution. We will prove a stronger inequality

f (a, b, c) ≥ 0,

where
1+k

f (a, b, c) = a4 + b4 + c 4 + k(a3 b + b3 c + c 3 a) − (a + b + c)4 , k= 2,


27
satisfies f (1, 1, 1) = 0. Since
    
(a + b + c)4 = a4 + 6 a2 b2 + 12a bc a+4 a3 b + 4 a b3 ,

the inequality can be written as


    
A0 a4 + A a2 b2 + Ba bc a+C a3 b + D a b3 ≥ 0,
148 Vasile Cîrtoaje

where

A0 = 26 − k, A = −6(1 + k), B = −12(1 + k), C = 23k − 4, D = −4(1 + k).

We see that A0 + A + B + C + D = 0. According to the statement in Remark 2 from


P 2.18, if A0 > 0 and
3A0 (A0 + A) ≥ C 2 + C D + D2 ,
then the inequality holds for all real a, b, c. We have

3A0 (A0 + A) − (C 2 + C D + D2 ) = 54(28 − 7k − 8k2 ) = 54(12 − 7 2) > 0.

Thus, the proof is completed. The equality holds for a = b = c = 0.


Remark. From the proof above, it follows that the inequality

a4 + b4 + c 4 + k(a3 b + b3 c + c 3 a) ≥ 0

holds for all real a, b, c and


3 105 − 7
−1 ≤ k ≤ ≈ 1.4838,
16

3 105 − 7
where k = is a root of the equation 28 − 7k − 8k2 = 0. Actually, the
16
inequality holds for
−1 ≤ k ≤ k0 ,
where k0 ≈ 1.4894 is a root of the equation

7k5 + 17k4 + 16k3 + 16k2 − 64k − 128 = 0.

P 2.23. If a, b, c are real numbers, then

(a2 + b2 + c 2 )2 + 2(a3 b + b3 c + c 3 a) ≥ 3(a b3 + bc 3 + ca3 ).

(Vasile C., 2009)

First Solution. Write the inequality as follows:


 
2(a2 + b2 + c 2 )2 ≥ a b(a2 + b2 ) − 5 a b(a2 − b2 ) ≥ 0,

2(p2 − 2q)2 ≥ q(p2 − 2q) − pr + 5p(a − b)(b − c)(c − a),


2p4 − 9p2 q + 10q2 ≥ −pr + 5p(a − b)(b − c)(c − a) ≥ 0,
pqr Method 149

Since the statement remains unchanged by replacing a, b, c with −a, −b, −c, re-
spectively, we may assume p ≥ 0. For p = 0, the inequality is clearly true. For
p > 0, it suffices to show that

2p4 − 9p2 q + 10q2 1


≥ − pr + p|(a − b)(b − c)(c − a)| ≥ 0.
5 5
Due to homogeneity, we may set p = 1. From p2 ≥ 3q, we get

1
q≤ .
3
Thus, we need to show that

2 − 9q + 10q2 1
≥ − r + |(a − b)(b − c)(c − a)| ≥ 0.
5 5
−1
Applying Corollary 1 for β = , we have
135
1 −5q 10 52
− pr + |(a − b)(b − c)(c − a)| ≤ + + (1 − 3q)3/2 ,
5 3 27 135
with equality for

(1 − 3q)3/2 = 13 (9q − 2 − 27r). (*)

Therefore, it suffices to show that

2 − 9q + 10q2 −q 2 52
≥ + + (1 − 3q)3/2 ,
5 15 135 135
which is equivalent to

26 − 117q + 135q2 ≥ 26(1 − 3q)3/2 .

Using the substitution 


t= 1 − 3q, t ≥ 0,
the inequality becomes

15(t 2 − 1)2 ≥ 13(2t 3 − 3t 2 + 1) ≥ 0,

which is equivalent to
(t − 1)2 (15t 2 + 4t + 2) ≥ 0.
The last inequality is true, with equality for t = 1, that is q = 0. From (*), we get
r = −1/13. Thus, the original inequality is an equality when a, b, c satisfy

(a − b)(b − c)(c − a) ≥ 0
150 Vasile Cîrtoaje

and are proportional to the roots of the equation

13w3 − 13w2 + 1 = 0.

Second Solution. We will find a stronger inequality

f (a, b, c) ≥ 0,

where
2 2
f (a, b, c) = (a2 +b2 +c 2 )2 +2(a3 b+b3 c+c 3 a)−3(a b3 +bc 3 +ca3 )− [p +m(p2 −3q)]2
27
satisfies f (1, 1, 1) = 0. The inequality can be written as
    
a4 + A a2 b2 + Ba bc a+C a3 b + D a b3 ≥ 0,

where 1 + A + B + C + D = 0. According to the statement in Remark 1 from P 2.18,


the inequality holds for all real a, b, c if

3(1 + A) = C 2 + C D + D2 .

It is easy to show that this condition is satisfied for m = −1, when


2
f (a, b, c) = (a2 + b2 +c 2 )2 +2(a3 b+ b3 c +c 3 a)−3(a b3 + bc 3 +ca3 )− (a b+ bc +ca)2 .
3

P 2.24. If a, b, c are real numbers, then


8
(a2 + b2 + c 2 )2 +
(a3 b + b3 c + c 3 a) ≥ 0.
7

Solution. Write the inequality as follows:



 
7 2
(a + b2 + c 2 )2 + a b(a2 + b2 ) + a b(a2 − b2 ) ≥ 0,
4

7 2
(p − 2q)2 + q(p2 − 2q) − pr − p(a − b)(b − c)(c − a) ≥ 0.
4
Since the statement remains unchanged by replacing a, b, c with −a, −b, −c, re-
spectively, we may assume p ≥ 0. For p = 0, the inequality is clearly true. For
p > 0, it suffices to show that

7 2
(p − 2q)2 + q(p2 − 2q) ≥ pr + p|(a − b)(b − c)(c − a)| ≥ 0.
4
pqr Method 151

Due to homogeneity, we may set p = 1. From p2 ≥ 3q, we get


1
q≤ .
3
Thus, we need to show that

7
(1 − 2q)2 + q(1 − 2q) ≥ r + |(a − b)(b − c)(c − a)|.
4
1
Applying Corollary 1 for β = , we have
27

q 2 4 7
r + |(a − b)(b − c)(c − a)| ≤ − + (1 − 3q)3/2 ,
3 27 27
with equality for

(1 − 3q)3/2 = 7(2 − 9q + 27r). (*)

Therefore, it suffices to show that



7 q 2 4 7
(1 − 2q)2 + q(1 − 2q) ≥ − + (1 − 3q)3/2 .
4 3 27 27
Using the substitution 
t= 1 − 3q, t ≥ 0,
(*) turns into

t3 = 7 (3t 2 − 1 + 27r), (**)

and the desired inequality becomes





12( 7 − 2)t 4 − 16 7 t 3 + 12( 7 + 2)t 2 + 3 7 + 8 ≥ 0,

which is equivalent to

2
3+ 7 


t− 6( 7 − 2)t 2 + 2(3 − 7 )t + 1 ≥ 0.
2

3+ 7
The last inequality is true, with equality for t = , hence for
2

1 − t2 2+ 7
q= =− .
3 2
From (**), we get

3+ 7
r =−
.
4 7
152 Vasile Cîrtoaje

Thus, the original inequality is an equality when a, b, c satisfy

(a − b)(b − c)(c − a) ≥ 0

and are proportional to the roots of the equation



3
4w3 − 4w2 − 2(2 + 7 )w + 1 +
= 0.
7

P 2.25. If a, b, c are real numbers such that a b + bc + ca ≤ 0, then


(a2 + b2 + c 2 )2 ≥ (2 7 − 1)(a b3 + bc 3 + ca3 ).

(Vasile C., 2009)


First Solution. Write the inequality as follows:

 
4 7+2 2
(a + b2 + c 2 )2 ≥ a b(a2 + b2 ) − a b(a2 − b2 ),
27

4 7+2 2
(p − 2q)2 ≥ q(p2 − 2q) − pr + p(a − b)(b − c)(c − a).
27
Since the statement remains unchanged by replacing a, b, c with −a, −b, −c, re-
spectively, we may assume p ≥ 0. For p = 0, the inequality is trivial. For p > 0, it
suffices to show that

4 7+2 2
(p − 2q)2 − q(p2 − 2q) ≥ −pr + p|(a − b)(b − c)(c − a)|.
27
Due to homogeneity, we may set p = 1. Thus, we need to show that

4 7+2
(1 − 2q)2 − q(1 − 2q) ≥ −r + |(a − b)(b − c)(c − a)|.
27
−1
Applying Corollary 1 for β = , we have
27

−q 2 4 7
−r + |(a − b)(b − c)(c − a)| ≤ + + (1 − 3q)3/2 ,
3 27 27
with equality for

(1 − 3q)3/2 = 7 (9q − 2 − 27r). (*)

Therefore, it suffices to show that



4 7+2 −q 2 4 7
(1 − 2q)2 − q(1 − 2q) ≥ + + (1 − 3q)3/2 ,
27 3 27 27
pqr Method 153

which can be written as


4 7  2
(1 − 2q)2 − (1 − 3q)3/2 − q(13 − 31q) ≥ 0.
27 27
Since q ≤ 0, this inequality is true if

4 7  2
(1 − 2q)2 − (1 − 3q)2 − q(13 − 31q) ≥ 0,
27 27
which is equivalent to

q[−13 + 4 7 + (31 − 10 7)q] ≥ 0,



(−q)[13 − 4 7 + (31 − 10 7)(−q)] ≥ 0.


The last inequality is true, with equality for q = 0. From (*), we get
 
−1 1
r= 2+
.
27 7
Thus, the original inequality is an equality when a, b, c satisfy

(a − b)(b − c)(c − a) ≥ 0

and are proportional to the roots of the equation


1
27w3 − 27w2 + 2 +
= 0.
7
Second Solution. We will find a stronger inequality of the form

(a2 + b2 + c 2 )2 + q[kp2 + m(p2 − 3q)] ≥ (2 7 − 1)(a b3 + bc 3 + ca3 ),

where k > 0, m > 0. Since


     2
q[kp2 + m(p2 − 3q)] = (k + m) ab a2 + (2k − m) ab
    
= (2k − m) a2 b2 + (5k − m)a bc a + (k + m) a3 b + a b3 ,
he inequality can be written as
    
a4 + A a2 b2 + Ba bc a+C a3 b + D a b3 ≥ 0,

where

A = 2k − m + 2, B = 5k − m, C = k + m, D = k + m + 1 − 2 7.

According to the statement in Remark 2 from P 2.18, the inequality holds for all
real a, b, c if
1 + A+ B + C + D = 0
154 Vasile Cîrtoaje

and
3(1 + A) = C 2 + C D + D2 .

2( 7 − 2) 7 7−5
These condition are satisfied for k = , and m = . Thus, the proof
9 9
is completed.

P 2.26. If a, b, c are real numbers such that a b + bc + ca ≥ 0, then


(a2 + b2 + c 2 )2 + (1 + 2 7 )(a3 b + b3 c + c 3 a) ≥ 0.

(Vasile C., 2009)

First Solution. Write the inequality as follows:



 
4 7−2 2
(a + b2 + c 2 )2 + a b(a2 + b2 ) + a b(a2 − b2 ) ≥ 0,
27

4 7−2 2
(p − 2q)2 + q(p2 − 2q) − pr − p(a − b)(b − c)(c − a) ≥ 0.
27
Since the statement remains unchanged by replacing a, b, c with −a, −b, −c, re-
spectively, we may assume p ≥ 0. For p = 0, the inequality is clearly true. For
p > 0, it suffices to show that

4 7−2 2
(p − 2q)2 + q(p2 − 2q) ≥ pr + p|(a − b)(b − c)(c − a)| ≥ 0.
27
Due to homogeneity, we may set p = 1. From p2 ≥ 3q, we get

1
0≤q≤ .
3
Thus, we need to show that

4 7−2
(1 − 2q)2 + q(1 − 2q) ≥ r + |(a − b)(b − c)(c − a)|.
27
1
Applying Corollary 1 for β = , we have
27

q 2 4 7
r + |(a − b)(b − c)(c − a)| ≤ − + (1 − 3q)3/2 ,
3 27 27
with equality for

(1 − 3q)3/2 = 7(2 − 9q + 27r). (*)


pqr Method 155

Therefore, it suffices to show that



4 7−2 q 2 4 7
(1 − 2q)2 + q(1 − 2q) ≥ − + (1 − 3q)3/2 ,
27 3 27 27
which can be written as

4 7  2   2
(1 − 2q)2 − (1 − 3q)3/2 + 1 − (1 − 2q)2 + q(1 − 3q) ≥ 0.
27 27 3
This is true if
(1 − 2q)2 − (1 − 3q)3/2 ≥ 0.
Using the substitution 
t= 1 − 3q, 0 ≤ t ≤ 1,
the inequality becomes
(1 + 2t 2 )2 ≥ 9t 3 ,
which is equivalent to

(1 − t)(1 + t + 5t 2 − 4t 3 ) ≥ 0.

Since

1 + t + 5t 2 − 4t 3 ≥ 1 + t + 2t 2 − 4t 3 = (1 − t)(1 + 2t + 4t 2 ) ≥ 0,

the inequality is true, with equality for t = 1, hence for q = 0. From (*), we get
 
1 1
r=
−2 .
27 7
Thus, the original inequality is an equality when a, b, c satisfy

(a − b)(b − c)(c − a) ≥ 0

and are proportional to the roots of the equation


1
27w3 − 27w2 + 2 −
= 0.
7

Second Solution. We will find a stronger inequality of the form


(a2 + b2 + c 2 )2 + (1 + 2 7 )(a3 b + b3 c + c 3 a) ≥ q[kp2 + m(p2 − 3q)],

where k > 0, m > 0. Since


     2
q[kp2 + m(p2 − 3q)] = (k + m) ab a2 + (2k − m) ab
    
= (2k − m) a2 b2 + (5k − m)a bc a + (k + m) a3 b + a b3 ,
156 Vasile Cîrtoaje

the inequality can be written as


    
a4 + A a2 b2 + Ba bc a+C a3 b + D a b3 ≥ 0,

where

A = −2k + m + 2, B = −5k + m, C = −k − m, D = −k − m + 1 + 2 7.

According to the statement in Remark 1 from P 2.18, the inequality holds for all
real a, b, c if
1 + A+ B + C + D = 0
and
3(1 + A) = C 2 + C D + D2 .

2(2 + 7) 5+7 7
These conditions are satisfied for k = and m = . Thus, the
9 9
proof is completed.

Remark. Because a b + bc + ca ≥ 0 involves

(a + b + c)2 ≥ a2 + b2 + c 2 ,

the inequality is sharper than the inequality


(a + b + c)4 + (1 + 2 7 )(a3 b + b3 c + c 3 a) ≥ 0. (A)

On the other hand, the inequality is weaker than the inequality


(a2 + b2 + c 2 − a b − bc − ca)2 + (1 + 2 7 )(a3 b + b3 c + c 3 a) ≥ 0. (B)

Also, the following inequality holds for a b + bc + ca ≥ 0:


(a2 + b2 + c 2 − a b − bc − ca)E + (1 + 2 7 )(a3 b + b3 c + c 3 a) ≥ 0, (C)

where

E = a2 + b2 + c 2 − 7 (a b + bc + ca).
We can prove the inequalities (B) and (C) in a similar way. The original inequality
and the inequalities (A), (B) and (C) are equalities in the same conditions.

P 2.27. If a, b, c are real numbers such that a b + bc + ca ≤ 0, then


a4 + b4 + c 4 ≥ (2 3 − 1)(a b3 + bc 3 + ca3 ).

(Vasile C., 2009)


pqr Method 157

First Solution. Let

1 1+2 3
k=
= .
2 3−1 11
Write the inequality as follows:
 
2k(a4 + b4 + c 4 ) ≥ a b(a2 + b2 ) − a b(a2 − b2 ),

2k(p4 − 4p2 q + 2q2 + 4pr) ≥ q(p2 − 2q) − pr + p(a − b)(b − c)(c − a).
Since the statement remains unchanged by replacing a, b, c with −a, −b, −c, re-
spectively, we may assume p ≥ 0. For p = 0, the inequality is true. For p > 0, it
suffices to show that

2k(p4 − 4p2 q + 2q2 + 4pr) ≥ q(p2 − 2q) − pr + p|(a − b)(b − c)(c − a)|.

Due to homogeneity, we may set p = 1. Thus, we need to show that

2k(1 − 4q + 2q2 + 4r) ≥ q(1 − 2q) − r + |(a − b)(b − c)(c − a)|,

which is equivalent to

2k(1 − 4q + 2q2 ) − q(1 − 2q) ≥ −(1 + 8k)r + |(a − b)(b − c)(c − a)|.

−(1 + 8k)
Applying Corollary 1 for β = , we have
27
−(1 + 8k)r + |(a − b)(b − c)(c − a)| ≤

−(1 + 8k)q 2(1 + 8k) 4(4 + 19 3)


≤ + + (1 − 3q)3/2
3 27 11 · 27
−(1 + 8k)q 2(1 + 8k) 2(19k − 1)
= + + (1 − 3q)3/2 ,
3 27 27
with equality for

2(1 + 8k)(1 − 3q)3/2 = (19k − 1)(9q − 2 − 27r). (*)

Therefore, it suffices to show that

−(1 + 8k)q 2(1 + 8k) 2(19k − 1)


2k(1 − 4q + 2q2 ) − q(1 − 2q) ≥ + + (1 − 3q)3/2 ,
3 27 27
which is equivalent to

k(19 − 72q + 54q2 ) + 27q2 − 9q − 1 ≥ (19k − 1)(1 − 3q)3/2 .

Using the substitution 


t= 1 − 3q, t ≥ 1,
158 Vasile Cîrtoaje

the inequality becomes


k(6t 4 + 12t 2 + 1) + 3t 4 − 3t 2 − 1 ≥ (19k − 1)t 3 ,
k(6t 4 − 19t 3 + 12t 2 + 1) + 3t 4 + t 3 − 3t 2 − 1 ≥ 0,
k(t − 1)(6t 3 − 13t 2 − t − 1) + (t − 1)(3t 3 + 4t 2 + t + 1) ≥ 0.
This is true if E ≥ 0, where
E = k(6t 3 − 13t 2 − t − 1) + (3t 3 + 4t 2 + t + 1).
We have
E > k(6t 3 − 13t 2 − t − 1) + k(3t 3 + 4t 2 + t + 1) = 9kt 2 (t − 1) ≥ 0.
The equality E = 0 occurs for t = 1, that means q = 0. From (*), we get

−2k −(2 3 + 1) −(10 + 3)


r= =
= .
19k − 1 19 3 + 4 97
Thus, the original inequality is an equality when a, b, c satisfy
(a − b)(b − c)(c − a) ≥ 0
and are proportional to the roots of the equation

97w3 − 97w2 + 10 + 3 = 0.
Second Solution. We will find a stronger inequality of the form

a4 + b4 + c 4 + q[kp2 + m(p2 − 3q)] ≥ (2 3 − 1)(a b3 + bc 3 + ca3 ),


where k > 0, m > 0. Since
     2
q[kp2 + m(p2 − 3q)] = (k + m) ab a2 + (2k − m) ab
    
= (2k − m) a2 b2 + (5k − m)a bc a + (k + m) a3 b + a b3 ,
the inequality can be written as
    
a4 + A a2 b2 + Ba bc a+C a3 b + D a b3 ≥ 0,
where

A = 2k − m, B = 5k − m, C = k + m, D = k + m + 1 − 2 3.
According to the statement in Remark 1 from P 2.18, the inequality holds for all
real a, b, c if
1 + A+ B + C + D = 0
and
3(1 + A) = C 2 + C D + D2 .

2( 3 − 1) 7( 3 − 1)
These conditions are satisfied for k = and m = . Thus, the
9 9
proof is completed.
pqr Method 159

P 2.28. If a, b, c are real numbers such that a b + bc + ca ≥ 0, then


a4 + b4 + c 4 + (1 + 2 3 )(a3 b + b3 c + c 3 a) ≥ 0.

(Vasile C., 2009)

Solution. Denoting

2 3−1
k= ,
11
we may write the inequality as follows:
 
2k(a4 + b4 + c 4 ) + a b(a2 + b2 ) + a b(a2 − b2 ) ≥ 0,

2k(p4 − 4p2 q + 2q2 + 4pr) + q(p2 − 2q) − pr − p(a − b)(b − c)(c − a) ≥ 0.


Since the statement remains unchanged by replacing a, b, c with −a, −b, −c, re-
spectively, we may assume p ≥ 0. For p = 0, the inequality is true. For p > 0, it
suffices to show that

2k(p4 − 4p2 q + 2q2 + 4pr) + q(p2 − 2q) − pr − p|(a − b)(b − c)(c − a)| ≥ 0.

Due to homogeneity, we may set

1
p = 1, 0≤q≤ .
3
Thus, we need to show that

2k(1 − 4q + 2q2 ) + q(1 − 2q) ≥ (1 − 8k)r + |(a − b)(b − c)(c − a)|.

1 − 8k
Applying Corollary 1 for β = , we have
27
(1 − 8k)r + |(a − b)(b − c)(c − a)| ≤

(1 − 8k)q 2(1 − 8k) 4(19 3 − 4)


≤ − + (1 − 3q)3/2
3 27 11 · 27
(1 − 8k)q 2(1 − 8k) 2(19k + 1)
= − + (1 − 3q)3/2 ,
3 27 27
with equality for

(16 3 − 19)(1 − 3q)3/2 = (19 3 − 4)(9q − 2 − 27r). (*)

Therefore, it suffices to show that

(1 − 8k)q 2(1 − 8k) 2(19k + 1)


2k(1 − 4q + 2q2 ) + q(1 − 2q) ≥ − + (1 − 3q)3/2 ,
3 27 27
160 Vasile Cîrtoaje

which can be written as


kA + B ≥ 0,
where  
A = 19 1 − (1 − 3q)3/2 − 18q(4 − 3q),

B = 1 − (1 − 3q)3/2 + 9q(1 − 3q).


Since B ≥ 0, it suffices to show that A ≥ 0. This is true if
 
18 1 − (1 − 3q)3/2 − 18q(4 − 3q) ≥ 0,

which is equivalent to
 
3 1 − (1 − 3q)3/2 − 3q(4 − 3q) ≥ 0.

Using the substitution 


t= 1 − 3q, 0 ≤ t ≤ 1,
the inequality becomes

3(1 − t 3 ) − (1 − t 2 )(3 + t 2 ) ≥ 0,

which is equivalent to
t 2 (1 − t)(2 − t) ≥ 0.
This inequality is true, with equality for t = 1, hence for q = 0. From (*), we get

−(10 − 3)
r= .
97
Thus, the original inequality is an equality when a, b, c satisfy

(a − b)(b − c)(c − a) ≥ 0

and are proportional to the roots of the equation


97w3 − 97w2 + 10 − 3 = 0.

Second Solution. We will find a stronger inequality of the form


a4 + b4 + c 4 + (1 + 2 3 )(a3 b + b3 c + c 3 a) ≥ q[kp2 + m(p2 − 3q)],

where k > 0, m > 0. Since


     2
q[kp2 + m(p2 − 3q)] = (k + m) ab a2 + (2k − m) ab
    
= (2k − m) a2 b2 + (5k − m)a bc a + (k + m) a3 b + a b3 ,
pqr Method 161

the inequality can be written as


    
a4 + A a2 b2 + Ba bc a+C a3 b + D a b3 ≥ 0,

where

A = −2k + m, B = −5k + m, C = −k − m, D = −k − m + 1 + 2 3.

According to the statement in Remark 1 from P 2.18, the inequality holds for all
real a, b, c if
1 + A+ B + C + D = 0
and
3(1 + A) = C 2 + C D + D2 .

2(1 + 3) 7(1 + 3)
These conditions are satisfied for k = and m = . Thus, the
9 9
proof is completed.

P 2.29. If a, b, c are real numbers such that a b + bc + ca ≥ 0, then


a4 + b4 + c 4 + 2 2 (a3 b + b3 c + c 3 a) ≥ a b3 + bc 3 + ca3 .

(Vasile C., 2009)


Solution. For m = 2 2, we write the inequality as follows:


 
2(a4 + b4 + c 4 ) + (m − 1) a b(a2 + b2 ) + (m + 1) a b(a2 − b2 ) ≥ 0,

2(p4 − 4p2 q + 2q2 + 4pr) + (m − 1)[q(p2 − 2q) − pr] ≥ (m + 1)p(a − b)(b − c)(c − a).
Since the statement remains unchanged by replacing a, b, c with −a, −b, −c, re-
spectively, we may assume p ≥ 0. For p = 0, the inequality is true. For p > 0, it
suffices to show that

2(p4 − 4p2 q + 2q2 ) + (m − 1)q(p2 − 2q) ≥ (m − 9)r + (m + 1)p|(a − b)(b − c)(c − a)|.

Due to homogeneity, we may set

1
p = 1, 0≤q≤ .
3
Thus, we need to show that

2(1 − 4q + 2q2 ) + (m − 1)q(1 − 2q) ≥ (m − 9)r + (m + 1)|(a − b)(b − c)(c − a)|,


162 Vasile Cîrtoaje

which is equivalent to

2(1 − 4q + 2q2 ) + (m − 1)q(1 − 2q) m − 9


≥ r + |(a − b)(b − c)(c − a)|,
m+1 m+1
m−9
Applying Corollary 1 for β = , we have
27(m + 1)
m−9
r + |(a − b)(b − c)(c − a)| ≤
m+1
(m − 9)q 2(m − 9) 2(18 + m)
≤ − + (1 − 3q)3/2 ,
3(m + 1) 27(m + 1) 27(m + 1)
with equality for

2(m − 9)(1 − 3q)3/2 = (18 + m)(2 − 9q + 27r). (*)

Therefore, it suffices to show that


(m − 9)q 2(m − 9) 2(18 + m)
2(1 − 4q + 2q2 ) + (m − 1)q(1 − 2q) ≥ − + (1 − 3q)3/2 ,
3 27 27
which can be written as
mA + 2B ≥ 0,
where
2q(1 − q) 2  
A= + 1 − (1 − 3q)3/2 ,
3 27
2 
B = −3q(1 − q) + 1 − (1 − 3q)3/2 .
3
Since A ≥ 0, it suffices to show that B ≥ 0. Using the substitution

t = 1 − 3q, 0 ≤ t ≤ 1,

we have
−(1 − t 2 )(2 + t 2 ) 2 t 2 (1 − t)2
B= + (1 − t 3 ) = ≥ 0.
3 3 3
The inequality mA + 2B ≥ 0 is an equality for q = 0. From (*), we get

2−9
r= .
7
Thus, the original inequality is an equality when a, b, c satisfy

(a − b)(b − c)(c − a) ≥ 0

and are proportional to the roots of the equation


7w3 − 7w2 + 9 − 2 = 0.
pqr Method 163

Second Solution. We will find a stronger inequality of the form


a4 + b4 + c 4 + 2 2 (a3 b + b3 c + c 3 a) ≥ a b3 + bc 3 + ca3 + q[kp2 + m(p2 − 3q)],

where k > 0, m > 0. Since


     2
q[kp2 + m(p2 − 3q)] = (k + m) ab a2 + (2k − m) ab
    
= (2k − m) a2 b2 + (5k − m)a bc a + (k + m) a3 b + a b3 ,
the inequality can be written as
    
a4 + A a2 b2 + Ba bc a+C a3 b + D a b3 ≥ 0,

where

A = −2k + m, B = −5k + m, C = −k − m + 2 2, D = −k − m − 1.

According to the statement in Remark 1 from P 2.18, the inequality holds for all
real a, b, c if
1 + A+ B + C + D = 0
and
3(1 + A) = C 2 + C D + D2 .

2 2 7 2
These conditions are satisfied for k = and m = . Thus, the proof is
9 9
completed.

P 2.30. If a, b, c are real numbers such that a b + bc + ca ≥ 0, then

(a + b + c)(a3 + b3 + c 3 ) + 5(a3 b + b3 c + c 3 a) ≥ 0.

(Vasile C., 2008)

First Solution. Since the statement remains unchanged by replacing a, b, c with


−a, −b, −c, respectively, we may assume p ≥ 0. Since p = 0 and q ≥ 0 involve
a = b = c = 0, consider next p > 0. Due to homogeneity, we may set

p2 1
p = 1, 0≤q≤ = .
3 3
The desired inequality becomes as follows:
 
2(a + b + c)(a3 + b3 + c 3 ) + 5 a b(a2 + b2 ) + 5 a b(a2 − b2 ) ≥ 0,

2p(3r + p3 − 3pq) + 5q(p2 − 2q) − 5pr ≥ 5p(a − b)(b − c)(c − a),


164 Vasile Cîrtoaje

2(3r + 1 − 3q) + 5q(1 − 2q) − 5r ≥ 5(a − b)(b − c)(c − a),


2 − q − 10q2 ≥ −r + 5(a − b)(b − c)(c − a).
Thus, we need to show that

2 − q − 10q2 −1
≥ r + |(a − b)(b − c)(c − a)|.
5 5
−1
Applying Corollary 1 for β = , we have
135
−1 −9q 2 52
r + |(a − b)(b − c)(c − a)| ≤ + + (1 − 3q)3/2 ,
5 135 135 135
with equality for
(1 − 3q)3/2 = 13(9q − 2 − 27r). (*)
Therefore, it suffices to show that

2 − q − 10q2 −9q 2 52
≥ + + (1 − 3q)3/2 ,
5 135 135 135
which can be rewritten as

26 − 9q − 135q2 ≥ 26(1 − 3q)3/2 .

Using the substitution 


t= 1 − 3q, 0 ≤ t ≤ 1,
the inequality becomes

26 − 3(1 − t 2 ) − 15(1 − t 2 )2 ≥ 26t 3 ,

8 + 33t 2 − 26t 3 − 15t 4 ≥ 0,


(1 − t)(8 + 8t + 41t 2 + 15t 3 ) ≥ 0.
The last inequality is true, with equality for t = 1. Notice that t = 1 involves q = 0,
and (*) gives r = −1/13. Thus, the original inequality is an equality when a, b, c
satisfy
(a − b)(b − c)(c − a) ≥ 0
and are proportional to the roots of the equation

13w3 − 13w2 + 1 = 0.

Second Solution. We will find a stronger inequality of the form

(a + b + c)(a3 + b3 + c 3 ) + 5(a3 b + b3 c + c 3 a) ≥ q[kp2 + m(p2 − 3q)],


pqr Method 165

where k > 0, m > 0. Since


     2
q[kp2 + m(p2 − 3q)] = (k + m) ab a2 + (2k − m) ab
    
= (2k − m) a2 b2 + (5k − m)a bc a + (k + m) a3 b + a b3 ,
the inequality can be written as
    
a4 + A a2 b2 + Ba bc a+C a3 b + D a b3 ≥ 0,

where

A = −2k + m, B = −5k + m, C = −k − m + 6, D = −k − m + 1.

According to the statement in Remark 1 from P 2.18, the inequality holds for all
real a, b, c if
1 + A+ B + C + D = 0
and
3(1 + A) = C 2 + C D + D2 .
8 28
These conditions are satisfied for k = and m = . Thus, the proof is completed.
9 9

P 2.31. If a, b, c are real numbers such that




1+ 21 + 8 7
k(a b + bc + ca) = a + b + c ,
2 2 2
k≥ ≈ 3.7468,
2
then
a3 b + b3 c + c 3 a ≥ 0.
(Vasile C., 2012)

Solution. Write the inequality as follows:


 
a b(a2 + b2 ) + a b(a2 − b2 ) ≥ 0,

q(p2 − 2q) − pr − p(a − b)(b − c)(c − a) ≥ 0.


Since the statement remains unchanged by replacing a, b, c with −a, −b, −c, re-
spectively, we may assume p ≥ 0. For p = 0, we have

2(a2 + b2 + c 2 ) = 2k(a b + bc + ca) = −k(a2 + b2 + c 2 ),

which implies a2 + b2 + c 2 = 0, a = b = c = 0. For p > 0, it suffices to show that

q(p2 − 2q) ≥ pr + p|(a − b)(b − c)(c − a)|.


166 Vasile Cîrtoaje

Due to homogeneity, we may set p = 1, which implies

1
q= .
k+2
Thus, we need to show that

q(1 − 2q) ≥ r + |(a − b)(b − c)(c − a)|.

1
Applying Corollary 1 for β = , we have
27

q 2 4 7
r + |(a − b)(b − c)(c − a)| ≤ − + (1 − 3q)3/2 ,
3 27 27
with equality for

(1 − 3q)3/2 = 7(2 − 9q + 27r). (*)


Therefore, it suffices to show that

q 2 4 7
q(1 − 2q) ≥ − + (1 − 3q)3/2 ,
3 27 27
which can be rewritten as

1 + 9q − 27q2 ≥ 2 7 (1 − 3q)3/2 . (**)

Using the substitution



k − 1
t= 1 − 3q = ,
k+2
the inequality becomes

1 + 3(1 − t 2 ) − 3(1 − t 2 )2 ≥ 2 7 t 3 ,

1 + 3t 2 − 2 7 t 3 − 3t 4 ≥ 0,



1 + t + (2 + 7)t 2 1 − t + (2 − 7)t 2 ≥ 0.
We only need to show that

1 − t + (2 − 7)t 2 ≥ 0,

which is equivalent to


k − 1
k − 1
1 − ( 7 − 2) ≥ ,
k+2 k+2



(3 − 7)k + 7 ≥ (k − 1)(k + 2).
pqr Method 167

By squaring, the inequality becomes



(2 7 − 5)k2 − (2 7 − 5)k − 3 ≤ 0,

k2 − k − 2 7 − 5 ≤ 0.


1 + 21 + 8 7
The last inequality is true for k ≥ , with equality for
2


1 + 21 + 8 7
k= .
2
From (*) and (**), we get the equality condition

1 + 9q − 27q2 = 14(2 − 9q + 27r),

which is equivalent to

5 1 5 + k − k2
14r = −1 + 5q − q2 = −1 + − = .
k + 2 (k + 2)2 (k + 2)2

The original inequality is an equality when a = b = c = 0. For




1 + 21 + 8 7
k=
2

(which satisfies k2 − k − 2 7 − 5 = 0), the equality condition above becomes

5 + k − k2 −1
r= =
.
14(k + 2)2 7(k + 2)2

Thus, the equality holds also when a, b, c satisfy

(a − b)(b − c)(c − a) > 0

and are proportional to the roots of the equation

1 1
w3 − w2 + w+
= 0.
k+2 7(k + 2)2

P 2.32. If a, b, c are nonnegative real numbers, then

3(a4 + b4 + c 4 ) + 4(a3 b + b3 c + c 3 a) ≥ 7(a b3 + bc 3 + ca3 ).

(Vasile C., 2009)


168 Vasile Cîrtoaje

Solution. It suffices to show that there exist α, β, γ ∈  such that the sharper in-
equality f (a, b, c) ≥ 0 holds for any real a, b, c and f (1, 1, 1) = 0, where
   
f (a, b, c) = 3 a4 + 4 a3 b − 7 a b3 − bc(αa + β b + γc)2 ,

f (1, 1, 1) = −3(α + β + γ)2 .


Since
  
bc(αa + β b + γc)2 =2βγ b2 c 2 + α(α + 2β + 2γ)a bc a
 
+ β2 a 3 b + γ2 a b3 ,
we have
    
f (a, b, c) = A0 a4 + A a2 b2 + Ba bc a+C a3 b + D a b3 ,
where
A0 = 3, A = −2βγ, B = −α(α + 2β + 2γ), C = 4 − β 2, D = −7 − γ2 .
Choosing
γ = −1,
we have
A0 = 3, A = 2β, B = −α(α + 2β − 2), C = 4 − β 2, D = −8.
For α + β + γ = 0, we have A0 + A + B + C + D = 0. According to the statement in
Remark 2 from P 2.18, if A0 > 0, A0 + A + B + C + D = 0 and
3A0 (A0 + A) ≥ C 2 + C D + D2 ,
then the inequality f (a, b, c) ≥ 0 holds for all real a, b, c. Since
3A0 (A0 + A) − (C 2 + C D + D2 ) = f (β), f (β) = 18β − 21 − β 4 ,
and  
3 15
f = > 0,
2 16
we choose
3
β=
2
and
−1
α = −β − γ = .
2
As a consequence, the inequality
   1
3 a4 + 4 a3 b ≥ 7 a b3 + bc(−a + 3b − 2c)2
4
holds for any real a, b, c. Thus, the proof is completed. The original inequality is
an equality for a = b = c.
pqr Method 169

P 2.33. If a, b, c are nonnegative real numbers, then

16(a4 + b4 + c 4 ) + 52(a3 b + b3 c + c 3 a) ≥ 47(a b3 + bc 3 + ca3 ).

(Vasile C., 2009)

Solution. It suffices to show that there exist α, β, γ ∈  such that the sharper in-
equality f (a, b, c) ≥ 0 holds for any real a, b, c and f (1, 1, 1) = 0, where
   
f (a, b, c) = 16 a4 + 52 a3 b − 47 a b3 − bc(αa + β b + γc)2 ,

f (1, 1, 1) = 63 − 3(α + β + γ)2 .


Since
  
bc(αa + β b + γc)2 =2βγ b2 c 2 + α(α + 2β + 2γ)a bc a
 
+ β2 a 3 b + γ2 a b3 ,

we have
    
f (a, b, c) = A0 a4 + A a2 b2 + Ba bc a+C a3 b + D a b3 ,

where

A0 = 16, A = −2βγ, B = −α(α + 2β + 2γ), C = 52 − β 2 , D = −47 − γ2 .

The original inequality is an equality for a = 0, b = 1 and c = 2. Since

f (0, 1, 2) = −2(β + 2γ)2 ,

we choose
β = −2γ.
We have

A0 = 16, A = 4γ2 , B = −α(α − 2γ), C = 52 − 4γ2 , D = −47 − γ2 .

According to the statement in Remark 2 from P 2.18, if A0 > 0, A0 +A+B+C + D = 0


and
3A0 (A0 + A) = C 2 + C D + D2 ,
then the inequality f (a, b, c) ≥ 0 holds for all real a, b, c. Since

3A0 (A0 + A) − (C 2 + C D + D2 ) = −21(γ − 3)2 (γ + 3)2 ,

we choose
γ=3
170 Vasile Cîrtoaje

and
β = −2γ = −6.
In addition, for

α= 21 − β − γ = 21 + 3
we have 3(A0 + A + B + C + D) = f (1, 1, 1) = 0. As a consequence, the inequality
    
2
16 a4 + 52 a3 b ≥ 47 a b3 + bc ( 21 + 3)a − 6b + 3c

holds for any real a, b, c. The original inequality is an equality for a = 0 and b = c/2
(or any cyclic permutation).

P 2.34. If a, b, c are nonnegative real numbers, then

a4 + b4 + c 4 + 5(a3 b + b3 c + c 3 a) ≥ 6(a2 b2 + b2 c 2 + c 2 a2 ).

(Vasile C., 2009)

Solution. It suffices to show that there exist α, β, γ ∈  such that the sharper in-
equality f (a, b, c) ≥ 0 holds for any real a, b, c and f (1, 1, 1) = 0, where
   
f (a, b, c) = a4 + 5 a3 b − 6 a2 b2 − bc(αa + β b + γc)2 ,

f (1, 1, 1) = −3(α + β + γ)2 .


Since
  
bc(αa + β b + γc)2 =2βγ b2 c 2 + α(α + 2β + 2γ)a bc a
 
+ β2 a 3 b + γ2 a b3 ,

we have
    
f (a, b, c) = a4 + A a2 b2 + Ba bc a+C a3 b + D a b3 ,

where

A = −6 − 2βγ, B = −α(α + 2β + 2γ), C = 5 − β 2, D = −γ2 .

Choosing
3
γ=− ,
2
we have
9
A = −6 + 3β, B = −α(α + 2β − 3), C = 5 − β 2, D=− .
4
pqr Method 171

According to the statement in Remark 1 from P 2.18, if 1 + A + B + C + D = 0 and

3(1 + A) ≥ C 2 + C D + D2 ,

then the inequality f (a, b, c) ≥ 0 holds for all real a, b, c. Since


141
3(1 + A) − (C 2 + C D + D2 ) = f (β), 4 f (β) = 36β − − 9β 2 − 4(5 − β 2 )2 ,
4
and  
9 11
f = > 0,
4 64
we choose
9
β= .
4
In addition, for
−3
α = −β − γ =
4
we have 3(1 + A + B + C + D) = f (1, 1, 1) = 0. As a consequence, the inequality
   9 
a4 + 5 a3 b ≥ 6 a2 b2 + bc(−a + 3b − 2c)2
16
holds for any real a, b, c. Thus, the proof is completed. The original inequality is
an equality for a = b = c.

P 2.35. If a, b, c are nonnegative real numbers such that a + b + c = 4, then


473
a3 b + b3 c + c 3 a + a bc ≤ 27.
64
(Vasile C., 2009)

Solution. Write the inequality in the homogeneous form g(a, b, c) ≥ 0, where

g(a, b, c) = 27(a + b + c)4 − 256(a3 b + b3 c + c 3 a) − 473a bc(a + b + c).

It suffices to show that there exist α, β, γ ∈  such that the sharper inequality
f (a, b, c) ≥ 0 holds for any real a, b, c, where

f (a, b, c) = g(a, b, c) − bc(αa + β b + γc)2 ,

f (1, 1, 1) = −3(α + β + γ)2 .


Since
    
(a + b + c)4 = a4 + 6 a2 b2 + 12a bc a+4 a3 b + 4 a b3
172 Vasile Cîrtoaje

and
  
bc(αa + β b + γc)2 =2βγ b2 c 2 + α(α + 2β + 2γ)a bc a
 
+ β2 a 3 b + γ2 a b3 ,

we have
    
f (a, b, c) = A0 a4 + A a2 b2 + Ba bc a+C a3 b + D a b3 ,

where
A0 = 27, A = 162 − 2βγ, B = −149 − α(α + 2β + 2γ),
C = −148 − β , 2
D = 108 − γ2 .
Since
g(1, 1, 1) = g(0, 3, 1) = 0,
we need to have also
f (1, 1, 1) = f (0, 3, 1) = 0.
From
f (1, 1, 1) = −3(α + β + γ)2
and
f (0, 3, 1) = −3(3β + γ)2 ,
we get
γ = −3β, α = −β − γ = 2β,
therefore

A0 = 27, A = 162+ 6β 2 , B = −149+ 4β 2 , C = −148− β 2 , D = 108− 9β 2 .

We see that A0 + A + B + C + D = 0. According to the statement in Remark 2 from


P 2.18, if A0 > 0 and
3A0 (A0 + A) = C 2 + C D + D2 ,
then the inequality f (a, b, c) ≥ 0 holds for all real a, b, c. Since

3A0 (A0 + A) − (C 2 + C D + D2 ) = −91(β 2 − 5)2 ,

we choose

β= 5, α = 2β = 2 5, γ = −α − β = −3 5.
As a consequence, the inequality

27(a + b + c)4 − 256(a3 b + b3 c + c 3 a) − 473a bc(a + b + c) ≥ 5 bc(2a + b − 3c)2

holds for any real a, b, c. The original inequality is an equality for a = b = c = 4/3,
and also for a = 0, b = 3 and c = 1 (or any cyclic permutation).
pqr Method 173

P 2.36. If a, b, c are positive real numbers such that a + b + c = 3, then

a2 b2 c 2
+ + + 5(a b + bc + ca) ≥ 18.
b c a
(Michael Rozenberg and Vasile C., 2009)

Solution. Write the inequality in the homogeneous forms:

a2 b2 c 2 15(a b + bc + ca)
+ + + ≥ 6(a + b + c),
b c a a+b+c
  a2 b2    a2 b2  30(a b + bc + ca)
+ + − + ≥ 12(a + b + c),
b a b a a+b+c
(p2 − 2q)q − pr p(a − b)(b − c)(c − a) 30q
+ + ≥ 12p,
r r p
 
30q
(p2 − 2q)q ≥ 13p − r − p(a − b)(b − c)(c − a).
p
It suffices to show that
 
30q
(p2 − 2q)q ≥ 13p − r + p|(a − b)(b − c)(c − a)|.
p

Due to homogeneity, we may set p = 1. From p2 ≥ 3q, we get


1
0≤q≤ .
3
Thus, we need to show that

(1 − 2q)q ≥ (13 − 30q)r + |(a − b)(b − c)(c − a)|. (*)

1 1 13 − 30q
Case 1: ≤ q ≤ . Applying Corollary 1 for β = , we have
9 3 27
(13 − 30q)q 2(13 − 30q) 2A
(13 − 30q)r + |(a − b)(b − c)(c − a)| ≤ − + (1 − 3q)3/2 ,
3 27 27
where
A= 27 + (13 − 30q)2 .
Therefore, it suffices to show that
(13 − 30q)q 2(13 − 30q) 2A
(1 − 2q)q ≥ − + (1 − 3q)3/2 ,
3 27 27
which is equivalent to
  
(1 − 3q) 13 − 36q − A 1 − 3q ≥ 0.
174 Vasile Cîrtoaje

This is true if
(13 − 36q)2 ≥ A2 (1 − 3q),
which can be written as

27(100q3 − 72q2 + 16q − 1) ≥ 0,

3(10q − 3)2 (9q − 1) + 3q(3 − 8q) ≥ 0.


Clearly, the last inequality is true.
1
Case 2: 0 < q ≤ . Since
9

|(a − b)(b − c)(c − a)| = −27r 2 − 2(2p3 − 9pq)r + p2 q2 − 4q3

= −27r 2 − 2(2 − 9q)r + q2 − 4q3 ,

the inequality (*) becomes



(1 − 2q)q − (13 − 30q)r ≥ −27r 2 − 2(2 − 9q)r + q2 − 4q3 .

From the known inequality q2 ≥ 3pr, we get r ≤ q2 /3, therefore


1
(1 − 2q)q − (13 − 30q)r ≥ (1 − 2q)q − (13 − 30q)q2
3
1
= q(1 − 3q)(3 − 10q) > 0.
3
Thus, we only need to show that

[(1 − 2q)q − (13 − 30q)r]2 ≥ −27r 2 − 2(2 − 9q)r + q2 − 4q3 ,

which is equivalent to

[27 + (13 − 30q)2 ]r 2 + 2[2 − 9q − q(1 − 2q)(13 − 30q)]r + 4q4 ≥ 0.

It suffices to show that

2 − 9q ≥ q(1 − 2q)(13 − 30q).

We can get this inequality by multiplying the inequalities

10(2 − 9q) > 13 − 30q

and
1 > 10q(1 − 2q).
The last inequality is true because

1 − 10q(1 − 2q) > 1 − 10q + 9q2 = (1 − q)(1 − 9q) ≥ 0.


pqr Method 175

The original inequality is an equality for a = b = c = 1.

Remark. The following generalization holds:


• Let a, b, c be positive real numbers such that a + b + c = 3. If k ≤ k0 , where
k0 ≈ 6.1708 is the largest positive root of the equation

x 4 − 11x 3 + 72x 2 − 304x + 269 = 0,

then
a2 b2 c 2
+ + + (k − 1)(a b + bc + ca) ≥ 3k.
b c a

P 2.37. If a, b, c are positive real numbers, then

a b c 5(a2 + b2 + c 2 − a b − bc − ca)
+ + −3≥ .
b c a a2 + b2 + c 2 + a b + bc + ca
(Vasile C., 2009)

Solution. Write the inequality as follows:


a b
 
a b

10(a2 + b2 + c 2 − a b − bc − ca)
+ + − −6≥ .
b a b a a2 + b2 + c 2 + a b + bc + ca

pq − 3r (a − b)(b − c)(c − a) 10(p2 − 3q)


−6+ ≥ ,
r r p2 − q
19p2 − 39q
pq + (a − b)(b − c)(c − a) ≥ r.
p2 − q
It suffices to show that

19p2 − 39q
pq ≥ r + |(a − b)(b − c)(c − a)|.
p2 − q

Due to homogeneity, we may set p = 1. From p2 ≥ 3q, we get

1
0≤q≤ .
3
Thus, we need to show that

19 − 39q
q≥ r + |(a − b)(b − c)(c − a)|. (*)
1−q
176 Vasile Cîrtoaje

1 1 19 − 39q
Case 1: ≤ q ≤ . Applying Corollary 1 for β = , we have
15 3 27(1 − q)

19 − 39q (19 − 39q)q 2(19 − 39q) 2A


r+|(a−b)(b−c)(c−a)| ≤ − + (1−3q)3/2 ,
1−q 3(1 − q) 27(1 − q) 27(1 − q)
where
A= 27(1 − q)2 + (19 − 39q)2 .
Therefore, it suffices to show that
(19 − 39q)q 2(19 − 39q) 2A
q≥ − + (1 − 3q)3/2 ,
3(1 − q) 27(1 − q) 27
which is equivalent to
  
(1 − 3q) 19 − 54q − A 1 − 3q ≥ 0.

This is true if
(19 − 54q)2 ≥ A2 (1 − 3q),
which can be written as

172q3 − 120q2 + 24q − 1 ≥ 0,

(3q − 1)2 (15q − 1) + q(37q2 − 21q + 3) ≥ 0.


The last inequality is true because
 
21 2 3
37q2 − 21q + 3 = 37 q − + > 0.
74 148

1
Case 2: 0 < q ≤ . Since
15

|(a − b)(b − c)(c − a)| = −27r 2 − 2(2p3 − 9pq)r + p2 q2 − 4q3

= −27r 2 − 2(2 − 9q)r + q2 − 4q3 ,

the inequality (*) becomes


19 − 39q
q− r ≥ −27r 2 − 2(2 − 9q)r + q2 − 4q3 .
1−q

From the known inequality q2 ≥ 3pr, we get r ≤ q2 /3, therefore

19 − 39q (19 − 39q)q2


q− r ≥q−
1−q 3(1 − q)
q(1 − 3q)(3 − 13q)
= > 0.
3(1 − q)
pqr Method 177

Thus, we only need to show that


 
19 − 39q 2
q− r ≥ −27r 2 − 2(2 − 9q)r + q2 − 4q3 ,
1−q
which is equivalent to the obvious inequality

Ar 2 + 4Br + 4q3 ≥ 0,

where  2
19 − 39q
A = 27 + > 0,
1−q
(1 − 15q) + 24q2
B= > 0.
1−q
The original inequality is an equality for a = b = c.

P 2.38. If a, b, c are positive real numbers, then

a b c 16(a2 + b2 + c 2 − a b − bc − ca)
+ + −3≥ 2 .
b c a a + b2 + c 2 + 6(a b + bc + ca)
(Vasile C., 2012)
Solution. Write the inequality as follows:
a b a b 32(a2 + b2 + c 2 − a b − bc − ca)
+ + − −6≥ 2 .
b a b a a + b2 + c 2 + 6(a b + bc + ca)

pq − 3r (a − b)(b − c)(c − a) 32(p2 − 3q)


−6+ ≥ ,
r r p2 + 4q
41p2 − 60q
pq + (a − b)(b − c)(c − a) ≥ r.
p2 + 4q
It suffices to show that
41p2 − 60q
pq ≥ r + |(a − b)(b − c)(c − a)|.
p2 + 4q

Due to homogeneity, we may set p = 1. From p2 ≥ 3q, we get


1
0≤q≤ .
3
Thus, we need to show that
41 − 60q
q≥ r + |(a − b)(b − c)(c − a)|. (*)
1 + 4q
178 Vasile Cîrtoaje

1 1 41 − 60q
Case 1: ≤ q ≤ . Applying Corollary 1 for β = , we have
28 3 27(1 + 4q)

41 − 60q (41 − 60q)q 2(41 − 60q) 2A


r+|(a−b)(b−c)(c−a)| ≤ − + (1−3q)3/2 ,
1 + 4q 3(1 + 4q) 27(1 + 4q) 27(1 + 4q)
where
A= 27(1 + 4q)2 + (41 − 60q)2 ,
with equality for

2(41 − 60q)(1 − 3q)3/2 = A(2 − 9q + 27r). (**)

Therefore, it suffices to show that


(41 − 60q)q 2(41 − 60q) 2A
q≥ − + (1 − 3q)3/2 ,
3(1 + 4q) 27(1 + 4q) 27(1 + 4q)
which is equivalent to
  
(1 − 3q) 41 − 108q − A 1 − 3q ≥ 0.

This is true if
(41 − 108q)2 ≥ A2 (1 − 3q),
which can be written as

448q3 − 240q2 + 36q − 1 ≥ 0,

(4q − 1)2 (28q − 1) ≥ 0.


The last inequality is true, with equality for q = 1/4. From (**), we get r = 1/56.
1
Case 2: 0 < q ≤ . Since
28

|(a − b)(b − c)(c − a)| = −27r 2 − 2(2p3 − 9pq)r + p2 q2 − 4q3

= −27r 2 − 2(2 − 9q)r + q2 − 4q3 ,

the inequality (*) becomes


41 − 60q
q− r ≥ −27r 2 − 2(2 − 9q)r + q2 − 4q3 .
1 + 4q

From the known inequality q2 ≥ 3pr, we get r ≤ q2 /3, therefore

41 − 60q (41 − 60q)q2


q− r ≥q−
1 + 4q 3(1 + 4q)
q(1 − 3q)(3 − 20q)
= > 0.
3(1 + 4q)
pqr Method 179

Thus, we only need to show that


 2
41 − 60q
q− r ≥ −27r 2 − 2(2 − 9q)r + q2 − 4q3 ,
1 + 4q

which is equivalent to the obvious inequality

Ar 2 + 4Br + 4q3 ≥ 0,

where  2
41 − 60q
A = 27 + > 0,
1+q
(1 − 21q) + 12q2
B= > 0.
1 + 4q
The original inequality is an equality for a = b = c, and also when a, b, c satisfy

(a − b)(b − c)(c − a) < 0

and are proportional to the roots of the equation

56w3 − 56w2 + 14w − 1 = 0.

The last equality conditions are equivalent to

a b c

π =

=


7 − tan 7 − tan 7 − tan
7 7 7
(or any cyclic permutation).
Remark. This inequality is stronger than the inequality

a b c 14(a2 + b2 + c 2 )
+ + +2≥ , (A)
b c a (a + b + c)2

which is stronger than the inequality

a b c 7(a b + bc + ca) 17
+ + + ≥ , (B)
b c a a2 + b2 + c 2 2
because
16(a2 + b2 + c 2 − a b − bc − ca) 14(a2 + b2 + c 2 )
≥ −2
a2 + b2 + c 2 + 6(a b + bc + ca) (a + b + c)2
and
14(a2 + b2 + c 2 ) 17 7(a b + bc + ca)
−2≥ − ,
(a + b + c)2 2 a2 + b2 + c 2
180 Vasile Cîrtoaje

respectively. Notice that the last inequalities are equivalent to

(a2 + b2 + c 2 − 2a b − 2bc − 2ca)2 ≥ 0.

The inequalities (A) and (B) are equalities for


a b c

π =

=


7 − tan 7 − tan 7 − tan
7 7 7
(or any cyclic permutation).

P 2.39. If a, b, c are real numbers such that a b + bc + ca ≥ 0, then

(a2 + b2 + c 2 )(a3 + b3 + c 3 ) + 5(a4 b + b4 c + c 4 a) ≥ 0.

(Vasile C., 2008)


Solution. Since the statement remains unchanged by replacing a, b, c with −a, −b, −c,
respectively, we may assume p ≥ 0. Since p = 0 and q ≥ 0 involve a = b = c = 0,
consider next p > 0. Due to homogeneity, we may set

p2 1
p = 1, 0≤q≤ = .
3 3
The desired inequality becomes as follows:
     
2 a2 a3 + 5 a b(a3 + b3 ) + 5 a b(a3 − b3 ) ≥ 0,
         
2 a2 a3 + 5 ab a3 − 5a bc a+5 a b(a3 − b3 ) ≥ 0,
      
2 a2 + 5 ab a3 − 5a bc a2 + 5 a b(a3 − b3 ) ≥ 0,

(2p2 + q)(3r + p3 − 3pq) − 5(p2 − 2q)r − 5(p2 − q)(a − b)(b − c)(c − a) ≥ 0,


(2 + q)(3r + 1 − 3q) − 5(12 − 2q)r − 5(1 − q)(a − b)(b − c)(c − a) ≥ 0,
2 − 5q − 3q2 ≥ −(1 + 13q)r + 5(1 − q)(a − b)(b − c)(c − a).
Thus, we need to show that
2 − 5q − 3q2 −(1 + 13q)r
≥ + |(a − b)(b − c)(c − a)|.
5(1 − q) 5(1 − q)
−(1 + 13q)
Applying Corollary 1 for β = , we have
135(1 − q)
−(1 + 13q)r
+ |(a − b)(b − c)(c − a)| ≤
5(1 − q)
pqr Method 181

−9(1 + 13q)q 2(1 + 13q) 2A


≤ + + (1 − 3q)3/2 ,
135(1 − q) 135(1 − q) 135(1 − q)
where
A= 675(1 − q)2 + (1 + 13q)2 ,
with equality for

−2(1 + 13q)(1 − 3q)3/2 = A(2 − 9q + 27r). (*)

Therefore, it suffices to show that

2 − 5q − 3q2 −9(1 + 13q)q 2(1 + 13q) 2A


≥ + + (1 − 3q)3/2 ,
5(1 − q) 135(1 − q) 135(1 − q) 135(1 − q)
which can be rewritten as

26 − 75q + 18q2 ≥ A(1 − 3q)3/2 .

Since
A= 262 − 4q(331 − 211q) ≤ 26
and 
1 − 3q ≤ 1,
it suffices to show that

26 − 75q + 18q2 ≥ 26(1 − 3q),

which is equivalent to
3q(1 + 6q) ≥ 0.
For q = 0, (*) gives r = −1/13. Thus, the original inequality is an equality when
a, b, c satisfy
(a − b)(b − c)(c − a) ≥ 0
and are proportional to the roots of the equation

13w3 − 13w2 + 1 = 0.

P 2.40. If a, b, c are real numbers such that

a + b + c = 3, a b + bc + ca ≥ 0,

then

a3 b + b3 c + c 3 a + 18 3 ≥ a b3 + bc 3 + ca3 .
(Vasile C., 2008)
182 Vasile Cîrtoaje

Solution. From 0 ≤ 3q ≤ p2 , we get

0 ≤ q ≤ 3.

Write the inequality as


18 3 ≥ (a + b + c)(a − b)(b − c)(c − a),


6 3 ≥ (a − b)(b − c)(c − a).


It suffices to show that

6 3 ≥ |(a − b)(b − c)(c − a)|.

Since


4(p2 − 3q)3 − (2p3 − 9pq + 27r)2
|(a − b)(b − c)(c − a)| =
27


(p2 − 3q)3 
≤2 = 2(3 − q) 3 − q,
27

with equality for 27r = 9pq − 2p3 , i.e. for

r = q − 2,

we only need to show that




6 3 ≥ 2(3 − q) 3 − q.

Since 0 ≤ 3 − q ≤ 3, the inequality is clearly true, with equality for q = 0. From


r = q − 2 = −2, it follows that the original inequality is an equality when a, b, c are
the roots of the equation
w3 − 3w2 + 2 = 0
and (a − b)(b − c)(c − a) ≥ 0. Equivalently, the original inequality is an equality for

a = −1, b = 2 − 2, c =2+ 2

(or any cyclic permutation)

P 2.41. If a, b, c are real numbers such that a2 + b2 + c 2 = 3, then


81 2
a3 b + b3 c + c 3 a + ≥ a b3 + bc 3 + ca3 .
32
(Vasile C., 2008)
pqr Method 183

Solution. Write the inequality in the homogeneous form:


9 2 2
(a + b2 + c 2 )2 ≥ (a + b + c)(a − b)(b − c)(c − a). (*)
32
Since the statement remains unchanged by replacing a, b, c with −a, −b, −c, re-
spectively, we may assume p ≥ 0. Thus, it suffices to show that

9 2 2
(a + b2 + c 2 )2 ≥ (a + b + c)|(a − b)(b − c)(c − a)|,
32
which can be written as

9 2 2
(p − 2q)2 ≥ p|(a − b)(b − c)(c − a)|,
32
Since


4(p2 − 3q)3 − (2p3 − 9pq + 27r)2
|(a − b)(b − c)(c − a)| =
27


(p2 − 3q)3
≤2 ,
27

with equality for 27r = 9pq − 2p3 , we only need to show that


9 2 2
(p2 − 3q)3
(p − 2q) ≥ 2p
2
.
32 27
This inequality is equivalent to

37 (p2 − 2q)4 ≥ 211 p2 (p2 − 3q)3 ,


 2 4  2 3
3p − 6q 2p − 6q
≥ p2 .
4 3
Using the substitution
2p2 − 6q
x= , x ≥ 0,
3
the inequality becomes
 4
p2 + 3x
≥ p2 x 3 .
4
This is the AM-GM inequality applied to four nonnegative numbers, with equality
for p2 = x. Thus, the homogeneous inequality (*) is an equality for

(a + b + c)(a − b)(b − c)(c − a) ≥ 0,


7
6q = −p2 , 27r = 9pq − 2p3 = − p3 .
2
184 Vasile Cîrtoaje

Since p2 − 2q = 3, we get the equality conditions

3 −3 −7
p= , q= , r= ,
2 8 16
and
−3 −3 7
p= , q= , r= .
2 8 16
In the first case, a, b, c are the roots of the equation

16w3 − 24w2 − 6w + 7 = 0,

which is equivalent to
(2w − 1)(8w2 − 8w − 7) = 0.
In the second case, a, b, c are the roots of the equation

16w3 + 24w2 − 6w − 7 = 0,

which is equivalent to
(2w + 1)(8w2 + 8w − 7) = 0.
Thus,the original inequality is an equality for

1 2+3 2 2−3 2
a= , b= , c=
2 4 4
(or any cyclic permutation), and for

−1 −2 − 3 2 −2 + 3 2
a= , b= , c=
2 4 4
(or any cyclic permutation)
Chapter 3

Highest Coefficient Cancellation


Method for Symmetric Homogeneous
Inequalities in Real Variables

3.1 Theoretical Basis


The Highest Coefficient Cancellation Method (HCC-Method) is especially appli-
cable to symmetric homogeneous polynomial inequalities of six and eight degree.
The main results in this section are based on the following Lemma (see P 2.53 in
Volume 1):
Lemma. If x, y, z are real numbers such that

x + y + z = p, x y + yz + z x = q,

where p and q are given real numbers satisfying p2 ≥ 3q, then the product r = x yz
is minimum and maximum when two of x, y, z are equal.

3.1.1. Inequalities of degree six


A symmetric and homogeneous polynomial of degree six can be written in the
form
   
f6 (x, y, z) = A1 x 6 + A2 x y(x 4 + y 4 ) + A3 x 2 y 2 (x 2 + y 2 ) + A4 x3 y3
 
+ A5 x yz x 3 + A6 x yz x y(x + y) + 3A7 x 2 y 2 z 2 ,

where A1 , · · · , A7 are real coefficients. In terms of

p = x + y + z, q = x y + yz + z x, r = x yz,

it can be rewritten as

f6 (x, y, z) = Ar 2 + g1 (p, q)r + g2 (p, q),

185
186 Vasile Cîrtoaje

where A is the highest coefficient of f6 (x, y, z), and g1 (p, q) and g2 (p, q) are polyno-
mial functions of the form

g1 (p, q) = Bp3 + C pq, g2 (p, q) = Dp6 + E p4 q + F p2 q2 + Gq3 ,

where B, C, D, E, F, G are real coefficients.


The highest coefficients of the polynomials
   
x 6, x y(x 4 + y 4 ), x 2 y 2 (x 2 + y 2 ), x 3 y 3,
 
x yz x 3 , x yz x y(x + y)

are, respectively,
3, −3, −3, 3, 3, −3.
Therefore, the highest coefficient of f6 (x, y, z) is

A = 3(A1 − A2 − A3 + A4 + A5 − A6 + A7 ).

The polynomial

P1 (x, y, z) = (A1 x 2 + A2 yz)(B1 x 2 + B2 yz)(C1 x 2 + C2 yz)

has the highest coefficient

A = 3(A1 + A2 )(B1 + B2 )(C1 + C2 ) = P1 (1, 1, 1). (3.1)

Indeed, since
 
P1 (x, y, z) =A1 B1 C1 x 6 + A2 B2 C2 x3 y3

+ (B1 C1 A2 + C1 A1 B2 + A1 B1 C2 )x yz x3
+ 3(A1 B2 C2 + B1 C2 A2 + C1 A2 B2 )x y z ,2 2 2

we have

A =3A1 B1 C1 + 3A2 B2 C2 + 3(B1 C1 A2 + C1 A1 B2 + A1 B1 C2 )


+ 3(A1 B2 C2 + B1 C2 A2 + C1 A2 B2 )
=3(A1 + A2 )(B1 + B2 )(C1 + C2 ).

Similarly, we can show that the polynomial



P2 (x, y, z) = (A1 x 2 + A2 yz)(B1 y 2 + B2 z x)(C1 z 2 + C2 x y)

has the highest coefficient

A = 3(A1 + A2 )(B1 + B2 )(C1 + C2 ) = P2 (1, 1, 1), (3.2)


Highest Coefficient Cancellation Method for Real Variables 187

and the polynomial

P3 (x, y, z) = (A1 x 2 + A2 yz)(A1 y 2 + A2 z x)(A1 z 2 + A2 x y)

has the highest coefficient

A = (A1 + A2 )3 = P3 (1, 1, 1). (3.3)

With regard to

P4 (x, y, z) = (x − y)2 ( y − z)2 (z − x)2 ,


from

P4 (x, y, z) = (p2 − 2q − z 2 − 2x y)(p2 − 2q − x 2 − 2 yz)(p2 − 2q − y 2 − 2z x),

it follows that P4 has the same highest coefficient as

P3 (x, y, z) = (−z 2 − 2x y)(−x 2 − 2 yz)(− y 2 − 2z x),

that is, according to (3.3),

A = P3 (1, 1, 1) = (−1 − 2)3 = −27.

∗∗∗

Based on Lemma above, Theorem 1 bellow gives for A ≤ 0 the necessary and
sufficient conditions to have f6 (x, y, z) ≥ 0 for all real numbers x, y, z which satisfy

k1 p2 + k2 q ≥ 0, (3.4)

where k1 and k2 are given real numbers (see P 2.75 and Remark 3 in Volume 1,
page 112).
Theorem 1 (Vasile Cirtoaje, 2008). Let f6 (x, y, z) be a symmetric homogeneous
polynomial of degree six which has the highest coefficient A ≤ 0. The inequality
f6 (x, y, z) ≥ 0 holds for all real numbers x, y, z satisfying (3.4) if and only if

f6 (x, 1, 1) ≥ 0
for all real x satisfying k1 (x + 2)2 + k2 (2x + 1) ≥ 0.
For k1 = k2 = 0, we get
Corollary 1. Let f6 (x, y, z) be a symmetric homogeneous polynomial of degree six
which has the highest coefficient A ≤ 0. The inequality f6 (x, y, z) ≥ 0 holds for all
real numbers x, y, z if and only if

f6 (x, 1, 1) ≥ 0
for all real x.
188 Vasile Cîrtoaje

∗∗∗

Further, consider the inequality

f6 (x, y, z) ≥ 0,

where x, y, z are real numbers and f6 (x, y, z) is a symmetric homogeneous poly-


nomial of degree six with the highest coefficient A > 0. The highest coefficient
cancellation method for proving such an inequality uses the above Theorem 1 and
the following three ideas:
1) finding a nonnegative symmetric homogeneous function f¯6 (x, y, z) of the form
 2
q2
f¯6 (x, y, z) = r + A1 pq + A2 p3 + A3 , (3.5)
p

where A1 , A2 , A3 are real numbers chosen such that

f6 (x, y, z) ≥ A f¯6 (x, y, z) ≥ 0

for all real numbers x, y, z;


2) seeing that the difference f6 (x, y, z) − A f¯6 (x, y, z) has the highest coefficient
equal to zero, therefore the inequality

f6 (x, y, z) ≥ A f¯6 (x, y, z)

holds if and only if it holds for y = z = 1 (Theorem 1);


3) choosing a suitable real number

ξ ∈ (−∞, 0) ∪ (3, ∞)

and treating successively the cases p2 < ξq and p2 ≥ ξq.

• In this chapter, we consider that the function f¯6 (x, y, z) depends on only two
parameters α and β. Let us define the following nonnegative functions:

4(x − 1)4 (x − α)2 (x − β)2


fα,β (x) = , (3.6)
9(4 − α − β − 2αβ)2 (x + 2)2

gα,β (x) = (x − α)2 ḡα,β (x) (3.7)


 2 2
αx + α(α + 6)x − 8 β(x + 2)(2αx + x + α − 4)
= (x − α)2 + ,
(α + 2)3 (α + 2)2

 2
hα,β (x) = x + α(x + 2)(2x + 1) + β(x + 2)3 , (3.8)
Highest Coefficient Cancellation Method for Real Variables 189

where α and β are real numbers. Note that the parameters α and β of the function
fα,β may be also conjugate complex numbers.

• The function fα,β (x) has been derived by setting y = z = 1 in the associated
function  2
b1 2 c1 q2
f¯6 (x, y, z) = r − pq + p3 + · (3.9)
a1 a1 a1 p
with

a1 = 3(4 − α − β − 2αβ), b1 = 10 + α + β, c1 = 2(2 + α)(2 + β),

which satisfies

f¯6 (1, 1, 1) = 0, f¯6 (α, 1, 1) = 0, f¯6 (β, 1, 1) = 0;

therefore,
 2
b1 2 c1 (2x + 1)2
fα,β (x) = f¯6 (x, 1, 1) = x − (x + 2)(2x + 1) + (x + 2)3 + · .
a1 a1 a1 x +2

Setting
β = −2, β = −1, β = 0, β = 1, β → ∞
in (3.6), we get in succession:

4(x − 1)4 (x − α)2


fα,−2 (x) = ,
81(2 + α)2

4(x + 1)2 (x − 1)4 (x − α)2


fα,−1 (x) = ,
9(5 + α)2 (x + 2)2
4x 2 (x − 1)4 (x − α)2
fα,0 (x) = ,
9(4 − α)2 (x + 2)2
4(x − 1)6 (x − α)2
fα,1 (x) = ,
81(1 − α)2 (x + 2)2
4(x − 1)4 (x − α)2
fα,∞ (x) = .
9(1 + 2α)2 (x + 2)2
In particular,
4(x − 1)4
f∞,−2 (x) = ,
81
4(x − 1)4 (x + 1)2
f−1,∞ (x) = ,
9(x + 2)2
4x 2 (x − 1)4
f0,∞ (x) = ,
9(x + 2)2
190 Vasile Cîrtoaje

4(x − 1)6
f1,∞ (x) = ,
81(x + 2)2
(x − 1)4
f∞,∞ (x) = .
9(x + 2)2

Notice that the relative degree of the rational functions fα,β (x), fα,∞ (x) and f∞,∞ (x)
are six, four and two, respectively.

• The function gα,β (x) is derived by setting y = z = 1 in the associated function


 2
α + (α + 2)(2α + 1)β 3
f¯6 (x, y, z) = r + β pq − p
(α + 2)3

which satisfies
f¯6 (α, 1, 1) = 0.
Therefore,
 2
α + (α + 2)(2α + 1)β
gα,β (x) = f¯6 (x, 1, 1) = x + β(x + 2)(2x + 1) − (x + 2)3 .
(α + 2)3

Setting
α = 0, α = −1, α = 1, β =0
in (3.7), we get:
 2
β
ḡ0,β (x) = (x + 2)(x − 4) − 1 ,
4
 2
ḡ−1,β (x) = x 2 + 5x + 8 + β(x + 2)(x + 5) ,
 2
1 x +8
ḡ1,β (x) = (x − 1)2 + β(x + 2) ,
9 3
[αx 2 + α(α + 6)x − 8]2
ḡα,0 (x) = ,
(α + 2)6
In particular,
ḡ0,0 (x) = 1,
ḡ−1,0 (x) = (x 2 + 5x + 8)2 ,

(x − 1)2 (x + 8)2
ḡ1,0 (x) = .
729

• The function hα,β (x) is derived by setting y = z = 1 in the associated function


 2
f¯6 (x, y, z) = r + αpq + β p3 .
Highest Coefficient Cancellation Method for Real Variables 191

Therefore,
 2
hα,β (x) = f¯6 (x, 1, 1) = x + α(x + 2)(2x + 1) + β(x + 2)3 .

• Let
ξ ∈ (−∞, 0) ∪ (3, ∞).
For y = z = 1, the condition
p2 < ξq
involves   
x ∈ ,  = ξ − 2 − ξ2 − 3ξ , ξ − 2 + ξ2 − 3ξ , (3.10)
while the condition
p2 ≥ ξq
involves
x ∈  \ .
Using the substitution
(2 + η)2
ξ= ,
1 + 2η
we have    
4−η −1
 = η, , η ∈ (−∞, −2) ∪ ,1 ,
1 + 2η 2
   
4−η −1
= ,η , η ∈ −2, ∪ (1, ∞).
1 + 2η 2
Thus, we can check that

η → −∞ ⇒ ξ → −∞,  = (−∞, −1/2),


···
η = −5 ⇒ ξ = −1,  = (−5, −1),
η = −4 ⇒ ξ = −4/7,  = (−4, −8/7),
η = −3 ⇒ ξ = −1/5,  = (−3, −7/5),
η = −2 ⇒ ξ = 0,  = (−2, −2) = ,

and

η=1 ⇒ ξ = 3,  = (1, 1) = ,
η=2 ⇒ ξ = 16/5,  = (2/5, 2),
η=3 ⇒ ξ = 25/7,  = (1/7, 3),
η=4 ⇒ ξ = 4,  = (0, 4),
···
η→∞ ⇒ ξ → ∞,  = (−1/2, ∞).
192 Vasile Cîrtoaje

Theorem 2. Let f6 (x, y, z) be a symmetric homogeneous polynomial of degree six


with the highest coefficient A > 0. The inequality f6 (x, y, z) ≥ 0 holds for any real
numbers x, y, z if there exist five real numbers α, β, γ, δ and ξ, and

Eα,β ∈ { fα,β , gα,β , hα,β },

Fγ,δ ∈ { fγ,δ , gγ,δ , hγ,δ },


such that the following two conditions are satisfied:
(a) f6 (x, 1, 1) ≥ AEα,β (x) for x ∈ ;
(b) f6 (x, 1, 1) ≥ AFγ,δ (x) for x ∈  \ .

Proof. Let
 2
q2
E1 (x, y, z) = r + A1 pq + A2 p3 + A3
p
and  2
q2
F1 (x, y, z) = r + B1 pq + B2 p3 + B3
p
be the functions associated to Eα,β (x) and Fγ,δ (x), respectively; this means that

E1 (x, 1, 1) = Eα,β (x),

F1 (x, 1, 1) = Fγ,δ (x).


Let us denote
E2 (x, y, z) = f6 (x, y, z) − AE1 (x, y, z)
and
F2 (x, y, z) = f6 (x, y, z) − AF1 (x, y, z).
Since AE1 (x, y, z) ≥ 0 and AF1 (x, y, z) ≥ 0, the inequality f6 (x, y, z) ≥ 0 holds for
all real x, y, z if
(a) E2 (x, y, z) ≥ 0 for p2 < ξq;
(b) F2 (x, y, z) ≥ 0 for p2 ≥ ξq.
According to Theorem 1, because E2 (x, y, z) and F2 (x, y, z) has the highest coeffi-
cient zero, these conditions are satisfied if and only if
(a) E2 (x, 1, 1) ≥ 0 for (x + 2)2 < ξ(2x + 1);
(b) F2 (x, 1, 1) ≥ 0 for (x + 2)2 ≥ ξ(2x + 1).
Since these conditions are equivalent to
(a) f6 (x, 1, 1) − AEα,β (x) ≥ 0 for x ∈ ;
(b) f6 (x, 1, 1) − AFγ,δ (x) ≥ 0 for x ∈  \ ,
the proof is completed.
Highest Coefficient Cancellation Method for Real Variables 193

For ξ = 0 or ξ = 3, when  =  , we get


Corollary 2. Let f6 (x, y, z) be a symmetric homogeneous polynomial of degree six with
the highest coefficient A > 0. The inequality f6 (x, y, z) ≥ 0 holds for any real numbers
x, y, z if there exist two real numbers γ and δ such that f6 (x, 1, 1) ≥ AFγ,δ (x) for all
x ∈ .

The following Proposition 1 is useful in many applications based on Theorem 2


or Corollary 2.
Proposition 1. Let f6 (x, y, z) be a symmetric homogeneous polynomial of degree six,
and let
f (x) = f6 (x, 1, 1) − AEα,β (x),
where A is the highest coefficient of f6 (x, y, z) and

Eα,β ∈ { fα,−2 , gα,β , hα,β }.

If
f6 (0, 1, −1) = 0,
then
f (−2) = 0.
Proof. Write f6 (x, y, z) in the form

f6 (x, y, z) = Ar 2 + g1 (p, q)r + g2 (p, q).

where
g1 (p, q) = Bp3 + C pq,
g2 (p, q) = Dp6 + E p4 q + F p2 q2 + Gq3 ,
For (x, y, z) = (0, 1, −1), we have r = 0, p = 0 and q = −1, therefore

f6 (0, 1, −1) = −G.

For (x, y, z) = (−2, 1, 1), we have r = −2, p = 0 and q = −3, therefore

f6 (−2, 1, 1) = 4A − 27G = 4A + 27 f6 (0, 1, −1).

Thus, the hypothesis f6 (0, 1, −1) = 0 involves

f6 (−2, 1, 1) = 4A.

Since
fα,−2 (−2) = 4, gα,β (−2) = 4, hα,β (−2) = 4,
we get
f (−2) = f6 (−2, 1, 1) − AEα,β (−2) = 4A − 4A = 0.
194 Vasile Cîrtoaje

Remark 1. The function f¯6 (x, y, z) associated to fα,β (x) (given by (3.9)) is zero for

(x, y, z) = (1, 1, 1), (x, y, z) = (α, 1, 1), (x, y, z) = (β, 1, 1).

If β = −2, then the function associated to fα,−2 (x) has the expression
 2
α+8 2
f¯6 (x, y, z) = r − pq + p3 , (3.11)
9α + 18 9α + 18
and is zero for

(x, y, z) = (1, 1, 1), (x, y, z) = (α, 1, 1), (x, y, z) = (0, 1, −1).

In addition, if α → ∞, then
 2
1
f¯6 (x, y, z) = r − pq
9
is zero for

(x, y, z) = (1, 1, 1), (x, y, z) = (1, 0, 0), (x, y, z) = (0, 1, −1).

If β → ∞, then
 2
1 2α + 4 q2
f¯6 (x, y, z) = r + pq − ·
6α + 3 6α + 3 p

is zero for

(x, y, z) = (1, 1, 1), (x, y, z) = (α, 1, 1), (x, y, z) = (1, 0, 0).

If, in addition, α → ∞, then


 2
1 q2
f¯6 (x, y, z) = r − ·
3 p

is zero for
(x, y, z) = (1, 1, 1), (x, y, z) = (1, 0, 0).

Remark 2. If
f6 (x, 1, 1) = Afγ,−2 (x)
for all x ∈ , then there is k ≥ 0 such that the following identity holds:

f6 (x, y, z) = A f¯6 (x, y, z) + k(x − y)2 ( y − z)2 (z − x)2 ,

where, according to (3.11),


 2
γ+8 2
f¯6 (x, y, z) = r − pq + p3 .
9γ + 18 9γ + 18
Highest Coefficient Cancellation Method for Real Variables 195

In addition, if the coefficient of the product

(x − y)2 ( y − z)2 (z − x)2

is the best possible in the inequality f6 (x, y, z) ≥ 0 , then k = 0 and the following
identity holds:
 2
γ+8 2
f6 (x, y, z) = A r − pq + p3 . (3.12)
9γ + 18 9γ + 18

Remark 3. Theorem 2 is also valid for the case where the parameters α and β of
the function fα,β (x) are conjugate complex numbers. For example, if k > 0 and
 
α= −k, β =− −k,

then, according to (3.6), we have

(x − 1)4 (x 2 + k)2
f
−k,−
−k = . (3.13)
9(k − 2)2 (x + 2)2

Remark 4. Consider the inequality f6 (x, y, z) ≥ 0, where

f6 (0, 1, −1) = 0.

• If
f6 (x, 1, 1) = (x − α)2 g(x), α = −2,
where g is a polynomial function, then the condition (a) in Theorem 2 applied for

Eα,β = gα,β , −2 ∈ ,

is satisfied if and only if


ḡ(x) ≥ 0, x ∈ ,
where
ḡ(x) = g(x) − Aḡα,β (x),
 2
αx + α(α + 6)x − 8 β(x + 2)(2αx + x + α − 4)
2
ḡα,β (x) = + . (3.14)
(α + 2)3 (α + 2)2
Denote
f (x) = f6 (x, 1, 1) − Agα,β (x).
From
f (x) = (x − α)2 g(x) − A(x − α)2 ḡα,β (x) = (x − α)2 ḡ(x)
and f (−2) = 0 (see Proposition 1), it follows that

ḡ(−2) = 0.
196 Vasile Cîrtoaje

To have ḡ(x) ≥ 0 in the vicinity of x = −2 ∈ , the condition ḡ (−2) = 0 is


necessary. This condition involves

α (α + 2)2 g (−2)
β= + . (3.15)
3(α + 2) 12A

In conclusion, in the case x = −2 ∈ , the condition (a) in Theorem 2 is satisfied if


and only if
g(x) ≥ Aḡα,β (x), x ∈ ,
with β given by (3.15).

Similarly, in the case −2 ∈  \  ( =  in Corollary 2), the condition (b) in


Theorem 2 or the condition in Corollary 2 are satisfied if and only if

g(x) ≥ Aḡγ,δ (x), x ∈  \ ,

with δ given by
γ (γ + 2)2 g (−2)
δ= + (3.16)
3(γ + 2) 12A
and
 2
γx 2 + γ(γ + 6)x − 8 δ(x + 2)(2γx + x + γ − 4)
ḡγ,δ (x) = + . (3.17)
(γ + 2)3 (γ + 2)2

• The condition (a) in Theorem 2 applied for

Eα,β = hα,β , −2 ∈ ,

has the form


f (x) ≥ 0, x ∈ ,
where
f (x) = f6 (x, 1, 1) − Ahα,β (x).
According to Proposition 1, we have

f (−2) = 0.

In order to have f (x) ≥ 0 in the vicinity of x = −2 ∈ , the condition f (−2) = 0


is necessary. This condition involves

1 h (−2)
α= + , (3.18)
3 12A
where
h(x) = f6 (x, 1, 1).
Highest Coefficient Cancellation Method for Real Variables 197

In conclusion, in the case x = −2 ∈ , the condition (a) in Theorem 2 is satisfied if


and only if
f6 (x, 1, 1) ≥ Ahα,β (x), x ∈ ,
with α given by (3.18).
Similarly, in the case −2 ∈  \  ( =  in Corollary 2), the condition (b) in
Theorem 2 or the condition in Corollary 2 are satisfied if and only if

f6 (x, 1, 1) ≥ Ahγ,δ (x), x ∈  \ ,

with
1 h (−2)
γ= + (3.19)
3 12A
and  2
hγ,δ (x) = x + γ(x + 2)(2x + 1) + δ(x + 2)3 . (3.20)

Remark 5. By Theorem 1, it follows that Theorem 2 and Corollary 2 can be ex-


tended to the case where the real numbers x, y, z satisfy the condition

k1 (x + y + z)2 + k2 (x y + yz + z x) ≥ 0,

where k1 and k2 are two fixed real numbers. More precisely, all conditions con-
cerning f6 (x, 1, 1) in Theorem 2 and Corollary 2 need to be satisfied only for

k1 (x + 2)2 + k2 (2x + 1) ≥ 0.

3.1.2. Inequalities of degree eight


A symmetric and homogeneous polynomial of degree eight f8 (x, y, z) can be writ-
ten in term of

p = x + y + z, q = x y + yz + z x, r = x yz,

in the form
g(r) = A(p, q)r 2 + g3 (p, q)r + g4 (p, q),
where A(p, q), g3 (p, q) and g4 (p, q) are polynomial functions. The highest polyno-
mial A(p, q) of f8 (x, y, z) has the form

A(p, q) = μ1 p2 + μ2 q, (3.21)

where μ1 and μ2 are real constants.

∗∗∗

The following theorem is an extension of Theorem 1 to symmetric and homoge-


neous polynomial inequalities of degree eight.
198 Vasile Cîrtoaje

Theorem 3. Let f8 (x, y, z) be an eighth degree symmetric homogeneous polynomial


which has the highest polynomial A(p, q). The inequality f8 (x, y, z) ≥ 0 holds for all
real x, y, z satisfying A(p, q) ≤ 0 if and only if

f8 (x, 1, 1) ≥ 0

for all real x such that A(x + 2, 2x + 1) ≤ 0.


Proof. For fixed p and q, the inequality f8 (x, y, z) ≥ 0 can be written as g(r) ≥ 0,
where
g(r) = A(p, q)r 2 + g3 (p, q)r + g4 (p, q)
is a quadratic function. Since g(r) is concave for A(p, q) ≤ 0, it is minimum when r
is minimum or maximum, that is, when two of x, y, z are equal (see Lemma above).
Due to the homogeneity (of even degree), we may take y = z = 1 and y = z = 0. As
shown in Volume 1 (Remark 3 from page 112), the case y = z = 0 is not necessary.
Corollary 3. Let f8 (x, y, z) be an eighth degree symmetric homogeneous polynomial
having the highest polynomial A(p, q). The inequality f8 (x, y, z) ≥ 0 holds for all
real x, y, z satisfying A(p, q) ≤ 0 if f8 (x, 1, 1) ≥ 0 for all real x.

∗∗∗

The following theorem is an extension of Theorem 2 to symmetric and homoge-


neous polynomial inequalities of degree eight.
Theorem 4. Let f8 (x, y, z) be a symmetric homogeneous polynomial of degree eight
having the highest coefficient A(p, q). The inequality f8 (x, y, z) ≥ 0 holds for any
real numbers x, y, z satisfying A(p, q) ≥ 0 if there exist five real numbers α, β, γ, δ
and ξ, and
Eα,β ∈ { fα,β , gα,β , hα,β },
Fγ,δ ∈ { fγ,δ , gγ,δ , hγ,δ },
such that the following two conditions are satisfied:
(a) f8 (x, 1, 1) ≥ AEα,β (x) for x ∈  and A(x + 2, 2x + 1) ≥ 0;
(b) f8 (x, 1, 1) ≥ AFγ,δ (x) for x ∈  \  and A(x + 2, 2x + 1) ≥ 0.

For ξ = 0 or ξ = 3, when  =  , we get


Corollary 4. Let f8 (x, y, z) be a symmetric homogeneous polynomial of degree eight
having the highest coefficient A(p, q). The inequality f8 (x, y, z) ≥ 0 holds for any
real numbers x, y, z satisfying A(p, q) ≥ 0 if there exist two real numbers γ and δ
such that
f8 (x, 1, 1) ≥ A(x + 2, 2x + 1)Eγ,δ (x)
for all real x satisfying A(x + 2, 2x + 1) ≥ 0, where

Eγ,δ ∈ { fγ,δ , gγ,δ , hγ,δ }.


Highest Coefficient Cancellation Method for Real Variables 199

Remark 6. Theorem 3, Theorem 4, Corollary 3 and Corollary 4 can be extended


to the case where the real numbers x, y, z satisfy the condition

k1 (x + y + z)2 + k2 (x y + yz + z x) ≥ 0,

where k1 and k2 are two fixed real numbers. More precisely, all conditions con-
cerning f8 (x, 1, 1) need to be satisfied for

k1 (x + 2)2 + k2 (2x + 1) ≥ 0.
200 Vasile Cîrtoaje
Highest Coefficient Cancellation Method for Real Variables 201

3.2 Applications

3.1. If x, y, z are real numbers, then



(x + 2 y)(x + 2z)(2x + y)(2x + z)(x − y)(x − z) + 3(x − y)2 ( y − z)2 (z − x)2 ≥ 0.

3.2. If x, y, z are real numbers, then



(x 2 − 4 yz)2 (x − y)(x − z) + 3(x − y)2 ( y − z)2 (z − x)2 ≥ 0.

3.3. If x, y, z are real numbers, then



(x 2 + yz)(x + 2 y)(x + 2z)(x − y)(x − z) + 2(x − y)2 ( y − z)2 (z − x)2 ≥ 0.

3.4. If x, y, z are real numbers, then

2x 2 + 3 yz 2 y 2 + 3z x 2z 2 + 3x y 5
+ + ≤ .
4x 2 + y 2 + z 2 4 y 2 + z 2 + x 2 4z 2 + x 2 + y 2 2

3.5. If x, y, z are real numbers, then

9x 2 − 4 yz 9 y 2 − 4z x 9z 2 − 4x y 15
+ + ≥ .
3x 2 + 2 y 2 + 2z 2 3 y 2 + 2z 2 + 2x 2 3z 2 + 2x 2 + 2 y 2 7

3.6. Let x, y, z be real numbers, no two of which are zero. If

x 2 + y 2 + z 2 ≥ 2(x y + yz + z x),

then
x 2 − 6 yz y 2 − 6z x z 2 − 6x y
+ 2 + 2 ≥ 0.
y2 + yz + z 2 z + zx + x 2 x + x y + y2

3.7. If x, y, z be real numbers, no two of which are zero, then

8x 2 + 3 yz 8 y 2 + 3z x 8z 2 + 3x y
+ 2 + 2 ≥ 11.
y + yz + z
2 2 z + zx + x 2 x + x y + y2
202 Vasile Cîrtoaje

3.8. If x, y, z are real numbers, no two of which are zero, then

8x 2 − 5 yz 8 y 2 − 5z x 8z 2 − 5x y
+ 2 + 2 ≥ 9.
y − yz + z
2 2 z − zx + x 2 x − x y + y2

3.9. If x, y, z are real numbers, no two of which are zero, then

5x 2 + 2 yz 5 y 2 + 2z x 5z 2 + 2x y
+ 2 + 2 ≥ 3.
2 y2 + 3 yz + 2z 2 2z + 3z x + 2x 2 2x + 3x y + 2 y 2

3.10. If x, y, z are real numbers, then

(x + y)(x + z) ( y + z)( y + x) (z + x)(z + y) 4


+ + 2 ≤ .
7x 2 + y 2 + z 2 7 y 2 + z2 + x 2 7z + x 2 + y 2 3

3.11. If x, y, z are real numbers, then

6x( y + z) − yz 6 y(z + x) − z x 6z(x + y) − x y 33


+ + ≤ .
12x 2 + y 2 + z 2 12 y 2 + z 2 + x 2 12z 2 + x 2 + y 2 4

3.12. If x, y, z are real numbers, then

x( y + z) − yz y(z + x) − z x z(x + y) − x y 3
+ + ≤ .
x 2 + 3 y 2 + 3z 2 y 2 + 3z 2 + 3x 2 z 2 + 3x 2 + 3 y 2 7

3.13. If x, y, z are real numbers, then


 1
yz(2x 2 + yz)(x − y)(x − z) + (x − y)2 ( y − z)2 (z − x)2 ≥ 0.
2

3.14. If x, y, z are real numbers, then


 3
(x 2 − yz)2 (x − y)(x − z) ≥ (x − y)2 ( y − z)2 (z − x)2 .
4

3.15. If x, y, z are real numbers, then



(x 2 + 8 yz)2 (x − y)(x − z) + 15(x − y)2 ( y − z)2 (z − x)2 ≥ 0.
Highest Coefficient Cancellation Method for Real Variables 203

3.16. Let x, y, z be real numbers. If k ∈ , then


 7k2 − 20k − 20
(x 2 + yz)(x− y)(x−z)(x−k y)(x−kz)+ (x− y)2 ( y−z)2 (z−x)2 ≥ 0.
24

3.17. If x, y, z are distinct real numbers, then


yz zx xy 1
+ + + ≥ 0.
( y − z)2 (z − x)2 (x − y)2 4

3.18. Let x, y, z be distict real numbers. If k ∈ , then

(x − k y)(x − kz) ( y − kz)( y − k x) (z − k x)(z − k y) k2


+ + ≥ 2 + 2k − .
( y − z) 2 (z − x) 2 (x − y) 2 4

3.19. If x, y, z are real numbers, then


 1
(x 2 + 2 yz)(x 2 − y 2 )(x 2 − z 2 ) + (x − y)2 ( y − z)2 (z − x)2 ≥ 0.
2

3.20. Let x, y, z be real numbers. If k ∈ , then


 3(k + 2)2 (x − y)2 ( y − z)2 (z − x)2
(x − y)(x − z)(x − k y)(x − kz) ≥ .
4(x 2 + y 2 + z 2 − x y − yz − z x)

3.21. Let x, y, z be real numbers such that x y + yz + z x ≥ 0. If k ∈ , then


  
k2
(x 2 + yz)(x − y)(x − z)(x − k y)(x − kz) ≥ 1 + k − (x − y)2 ( y − z)2 (z − x)2 .
4

3.22. If x, y, z are real numbers, then



(x 2 + 2 yz)2 (x − y)(x − z) ≥ 0.

3.23. If x, y, z are real numbers, then



x 2 (x 2 + yz)(x − y)(x − z) ≥ (x − y)2 ( y − z)2 (z − x)2 .
204 Vasile Cîrtoaje

3.24. Let x, y, z be real numbers. If 0 ≤ k ≤ 27, then


  
k2
(x 2 + yz)(x − y)(x − z)(x − k y)(x − kz) ≥ 1 + k − (x − y)2 ( y − z)2 (z − x)2 .
4

3.25. If x, y, z are real numbers, then



(x 2 + yz)(x − y)(x − z)(x − 28 y)(x − 28z) + 167(x − y)2 ( y − z)2 (z − x)2 ≥ 0.

3.26. If x, y, z are real numbers, then


 1
(x 2 + yz)(x 2 − y 2 )(x 2 − z 2 ) + (x − y)2 ( y − z)2 (z − x)2 ≥ 0.
4

−3
3.27. Let x, y, z be real numbers. If −2 ≤ k ≤ , then
2
  
k2
(x 2 + yz)(x − y)(x − z)(x − k y)(x − kz) ≥ 1 + k − (x − y)2 ( y − z)2 (z − x)2 ,
4

3.28. Let x, y, z be real numbers. If −5 ≤ k ≤ −2 and

k4 − 8k3 − 7k2 − 20k − 20


δk = ,
4(k − 1)2
then

(x 2 + yz)(x − y)(x − z)(x − k y)(x − kz) + δk (x − y)2 ( y − z)2 (z − x)2 ≥ 0.

3.29. If x, y, z are real numbers, then


 15(x − y)2 ( y − z)2 (z − x)2
(x 2 + yz)(x + y)(x + z) ≥ .
32(x 2 + y 2 + z 2 )

3.30. If x, y, z are real numbers, then


 7
(x − y)(x − z)(x 2 + y 2 )(x 2 + z 2 ) ≥ (x − y)2 ( y − z)2 (z − x)2 .
4
Highest Coefficient Cancellation Method for Real Variables 205

3.31. If x, y, z are real numbers such that x y + yz + z x ≥ 0, then


 15
(x − y)(x − z)(x 2 + y 2 )(x 2 + z 2 ) ≥ (x − y)2 ( y − z)2 (z − x)2 .
4

3.32. If x, y, z are real numbers, then


 3
(x − y)(x − z)(x 2 + x y + y 2 )(x 2 + xz + z 2 ) ≥ (x − y)2 ( y − z)2 (z − x)2 .
4

3.33. If x, y, z are real numbers such that x y + yz + z x ≥ 0, then



(x − y)(x − z)(x 2 + x y + y 2 )(x 2 + xz + z 2 ) ≥ 3(x − y)2 ( y − z)2 (z − x)2 .

3.34. If x, y, z are real numbers, then

3
(x 2 + y 2 )( y 2 + z 2 )(z 2 + x 2 ) ≥ 8x 2 y 2 z 2 + (x − y)2 ( y − z)2 (z − x)2 .
8

3.35. If x, y, z are real numbers, then

15
(x 2 +2 y 2 +2z 2 )( y 2 +2z 2 +2x 2 )(z 2 +2x 2 +2 y 2 ) ≥ 125x 2 y 2 z 2 + (x− y)2 ( y−z)2 (z−x)2 .
2

3.36. If x, y, z are real numbers, then

15
(2x 2 + y 2 +z 2 )(2 y 2 +z 2 + x 2 )(2z 2 + x 2 + y 2 ) ≥ 64x 2 y 2 z 2 + (x − y)2 ( y −z)2 (z− x)2 .
4

3.37. If x, y, z are real numbers, then

8(x 2 + x y + y 2 )( y 2 + yz + z 2 )(z 2 + z x + x 2 ) ≥ 3(x 2 + y 2 )( y 2 + z 2 )(z 2 + x 2 ).

3.38. If x, y, z are nonnegative real numbers, then



(16x 2 + 3 yz)(x − y)(x − z)(x − 4 y)(x − 4z) + 52(x − y)2 ( y − z)2 (z − x)2 ≥ 0.
206 Vasile Cîrtoaje

3.39. If x, y, z are real numbers, then



x 4 (x − y)(x − z) ≥ (x − y)2 ( y − z)2 (z − x)2 .

13 − 3 17 13 + 3 17
3.40. Let x, y, z be real numbers. If ≤k≤ , then
2 2
  2 
k
(x 2 + k yz)2 (x − y)(x − z) + − 1 (x − y)2 ( y − z)2 (z − x)2 ≥ 0.
4

3.41. Let x, y, z be real numbers. If k ∈ (−∞, −4] ∪ [−1, 0], then


  2 
k
(x 2 + k yz)2 (x − y)(x − z) + − 1 (x − y)2 ( y − z)2 (z − x)2 ≥ 0.
4

3.42. Let x, y, z be real numbers. If k ≥ 0, then


  2 
k
(x 2 + k yz)2 (x − y)(x − z) + − 1 (x − y)2 ( y − z)2 (z − x)2 ≥ 0.
4

3.43. If x, y, z are nonnegative real numbers, then



x 2 (x 2 + 8 yz)(x − y)(x − z) ≥ (x − y)2 ( y − z)2 (z − x)2 .

3.44. If x, y, z are real numbers, then



(x − y)(x − z)(x − 2 y)(x − 2z)(2x − y)(2x − z) + 15(x − y)2 ( y − z)2 (z − x)2 ≥ 0.

3.45. If x, y, z are real numbers, then



(x − y)(x − z)(x − 2 y)(x − 2z)(x − 3 y)(x − 3z) ≥ 3(x − y)2 ( y − z)2 (z − x)2 .

3.46. If x, y, z are real numbers, then



(x− y)(x−z)(2x+3 y)(2x+3z)(3x+2 y)(3x+2z)+15(x− y)2 ( y−z)2 (z−x)2 ≥ 0.
Highest Coefficient Cancellation Method for Real Variables 207

3.47. If x, y, z are real numbers, then


 1
(x + y)(x + z)(x 2 − y 2 )(x 2 − z 2 ) + (x − y)2 ( y − z)2 (z − x)2 ≥ 0.
4

3.48. If x, y, z are real numbers, then



(x − y)(x − z)(x − 4 y)2 (x − 4z)2 + 39(x − y)2 ( y − z)2 (z − x)2 ≥ 0.

3.49. Let x, y, z be real numbers, and let



⎪ 20 + 12k − 4k2 − k4

⎪ , k ∈ (−∞, −2] ∪ [4, ∞)
⎨ 4(1 − k)2
αk = 1 + k, k ∈ [−2, 1] .




5 − 3k, k ∈ [1, 4]
Then,

x 2 (x − y)(x − z)(x − k y)(x − kz) ≥ αk (x − y)2 ( y − z)2 (z − x)2 .

3.50. Let x, y, z be real numbers, and let


⎧ 2
⎪ k

⎪ , k ∈ (−∞, −2] ∪ [1, ∞)

⎪ 4
⎨ 
−k(8 + 11k + 8k2 ) −1
βk = , k ∈ −2, .

⎪ 4(1 − k)2 2

⎪ 

⎩ 1, −1
k∈ ,1
4 2
Then,

yz(x − y)(x − z)(x − k y)(x − kz) + βk (x − y)2 ( y − z)2 (z − x)2 ≥ 0.

3.51. Let x, y, z be real numbers, and let



⎪ (k + 1)(5k − 3)

⎪ , k ∈ (−∞, −5] ∪ [1, ∞)

⎪ 16

(k + 1)(k − 7k − 16k − 32)
3 2
γk = , k ∈ [−5, −2] .

⎪ 4(k − 1)2



⎩ k2
, k ∈ [−2, 1]
4
Then,

(x 2 − y 2 )(x 2 − z 2 )(x − k y)(x − kz) + γk (x − y)2 ( y − z)2 (z − x)2 ≥ 0.
208 Vasile Cîrtoaje

3.52. If x, y, z are real numbers, then



yz(4x 2 + 3 yz)(x − y)(x − z) + (x − y)2 ( y − z)2 (z − x)2 ≥ 0.

3.53. If x, y, z are real numbers, then


 1
x 4 (x + 2 y)(x + 2z) + 5x 2 y 2 z 2 + (x − y)2 ( y − z)2 (z − x)2 ≥ 0.
2

3.54. If x, y, z are real numbers, no two of which are zero, then


 1 9
≥ .
2 y 2 − 3 yz + 2z 2 4(x 2 + y 2 + z 2 ) − 3(x y + yz + z x)

3.55. Let x, y, z be real numbers. If k > 1, then

k x 2 + 2 yz k y 2 + 2z x kz 2 + 2x y k−1
+ + ≥ .
k x 2 + y 2 + z 2 k y 2 + z 2 + x 2 kz 2 + x 2 + y 2 k+1

3.56. If x, y, z are real numbers such that x y + yz + z x < 0, then


1 1 1 1
+ + + ≤ 0.
3x 2 + y 2 + z 2 3 y 2 + z 2 + x 2 3z 2 + x 2 + y 2 x y + yz + z x

3.57. If x, y, z are real numbers, then

x 2 − yz y2 − z x z2 − x y
+ + 2 ≤ 1.
3x 2 + y +z
2 2 3y + z + x
2 2 2 3z + x 2 + y 2

3.58. Let x, y, z be real numbers. If k > 1, then


yz zx xy 1
+ + + ≥ 0.
k x 2 + y 2 + z 2 k y 2 + z 2 + x 2 kz 2 + x 2 + y 2 2

3.59. If x, y, z are real numbers, then


yz zx xy 1
+ + + ≥ 0.
x 2 + 4 y 2 + 4z 2 y 2 + 4z 2 + 4x 2 z 2 + 4x 2 + 4 y 2 8
Highest Coefficient Cancellation Method for Real Variables 209

3.60. If x, y, z are real numbers, then


 1 7
≤ .
x 2 + 4 y 2 + 4z 2 4(x 2 + y 2 + z 2 ) + 3(x y + yz + z x)

3.61. If x, y, z are real numbers such that x y + yz + z x ≥ 0, then


 2 45
≥ .
4x 2 + y 2 + z 2 14(x 2 + y 2 + z 2 ) + x y + yz + z x

3.62. If x, y, z are real numbers such that x y + yz + z x ≥ 0, then


 1 45
≥ .
x 2 + 4 y 2 + 4z 2 44(x 2 + y 2 + z 2 ) + x y + yz + z x

3.63. If x, y, z are real numbers, then

x(−x + 4 y + 4z) y(− y + 4z + 4x) z(−z + 4x + 4 y) 21


+ + ≤ .
y 2 + z2 z2 + x 2 x2 + y2 2

3.64. If x, y, z are real numbers, no two of which are zero, then

x 2 + 3 yz y 2 + 3z x z 2 + 3x y
+ 2 + 2 ≥ 1.
y2 − yz + z 2 z − zx + x 2 x − x y + y2

3.65. If x, y, z are real numbers, then

(4x − y − z)2 (4 y − z − x)2 (4z − x − y)2


+ 2 + 2 ≥ 12.
2 y − 3 yz + 2z
2 2 2z − 3z x + 2x 2 2x − 3x y + y 2

3.66. If x, y, z are real numbers, then

(3 y + 3z − 4x)2 (3z + 3x − 4 y)2 (3x + 3 y − 4z)2


+ + ≥ 12.
2 y 2 − 3 yz + 2z 2 2z 2 − 3z x + 2x 2 2x 2 − 3x y + y 2

3.67. Let x, y, z be real numbers. If k > −2, then

4(x 2 + k x y + y 2 )( y 2 + k yz + z 2 )(z 2 + kz x + x 2 ) ≥ (2 − k)(x − y)2 ( y − z)2 (z − x)2 .


210 Vasile Cîrtoaje

3.68. If x, y, z are real numbers, then

(x 2 + y 2 )( y 2 + z 2 )(z 2 + x 2 ) + 2(x 2 y + y 2 z + z 2 x)(x y 2 + yz 2 + z x 2 ) ≥ x 2 y 2 z 2 .

3.69. Let x, y, z be real numbers. If k ∈ (−∞, −2] ∪ (0, ∞), then

2
x 6 + y 6 + z 6 − 3x 2 y 2 z 2 + (x 2 + k yz)( y 2 + kz x)(z 2 + k x y) ≥ 0.
k

3.70. If x, y, z are real numbers, then

2(2x 2 + y 2 +z 2 )(2 y 2 +z 2 + x 2 )(2z 2 + x 2 + y 2 ) ≥ 89x 2 y 2 z 2 +9(x − y)2 ( y −z)2 (z − x)2 .

3.71. If x, y, z are real numbers such that x + y + z = 3, then

13x − 1 13 y − 1 13z − 1 3
+ 2 + ≤ .
x 2 + 23 y + 23 z 2 + 23 2

3.72. If x, y, z are real numbers, then

5(x 2 + y 2 + z 2 )3 ≥ 108x 2 y 2 z 2 + 10(x − y)2 ( y − z)2 (z − x)2 .

3.73. If x, y, z are real numbers, then

(x 2 + y 2 + z 2 )3 + 2(2x 2 + yz)(2 y 2 + z x)(2z 2 + x y) ≥ 27x 2 y 2 z 2 .

3.74. If x, y, z are real numbers, no two of which are zero, then

x 2 + 2 yz y 2 + 2z x z 2 + 2x y 3(x y + yz + z x)
+ 2 + 2 ≥ .
y2 + yz + z 2 z + zx + x 2 x + x y + y2 x 2 + y 2 + z2

3.75. If x, y, z are real numbers, no two of which are zero, then

x 2 − 2 yz y 2 − 2z x z 2 − 2x y 3(x y + yz + z x)
+ 2 + 2 + ≥ 0.
y2 − yz + z 2 z − zx + x 2 x − x y + y2 x 2 + y 2 + z2
Highest Coefficient Cancellation Method for Real Variables 211

3.76. If x, y, z are real numbers, no two of which are zero, then

x2 y2 z2 (x + y + z)2
+ 2 + 2 ≥ 2 .
y2 − yz + z 2 z − zx + x 2 x −xy+ y 2 x + y 2 + z2

3.77. If x, y, z are real numbers such that x yz = 0, then

( y + z)2 (z + x)2 (x + y)2 10(x + y + z)2


+ + ≥2+ .
x 2 y 2 z 2 3(x 2 + y 2 + z 2 )

3.78. If x, y, z are real numbers, no two of which are zero, then

32x 2 + 49 yz 32 y 2 + 49z x 32z 2 + 49x y 81(x + y + z)2


+ + ≥ .
y 2 + z2 z2 + x 2 x2 + y2 2(x 2 + y 2 + z 2 )

3.79. If x, y, z are real numbers, no two of which are zero, then

x 2 + 4 yz y 2 + 4z x z 2 + 4x y 15(x y + yz + z x)
(a) + 2 + 2 ≥ ;
y 2 + z2 z + x2 x + y2 2(x 2 + y 2 + z 2 )

2x 2 + 9 yz 2 y 2 + 9z x 2z 2 + 9x y 33(x y + yz + z x)
(b) + 2 + ≥ .
y 2 + z2 z + x2 x2 + y2 2(x 2 + y 2 + z 2 )

3.80. If x, y, z are distinct real numbers, then

x2 y2 z2 4(x y + yz + z x)
+ + ≥ .
( y − z) 2 (z − x) 2 (x − y)2 x 2 + y 2 + z2

3.81. If x, y, z are distinct real numbers, then

x2 y2 z2 (x + y + z)2
+ + ≥ 2 .
( y − z) 2 (z − x) 2 (x − y) 2 x + y 2 + z2

3.82. If x, y, z are real numbers, then

2x y 2 yz 2z x 4(x y + yz + z x)
+ + +1≥ .
x 2 + y 2 y 2 + z2 z2 + x 2 x 2 + y 2 + z2
212 Vasile Cîrtoaje
Highest Coefficient Cancellation Method for Real Variables 213

3.3 Solutions

P 3.1. If x, y, z are real numbers, then



(x + 2 y)(x + 2z)(2x + y)(2x + z)(x − y)(x − z) + 3(x − y)2 ( y − z)2 (z − x)2 ≥ 0.

(Vasile C., 2012)

Solution. Write the inequality as f6 (x, y, z) ≥ 0, where


 (
f6 (x, y, z) = (x + 2 y)(x + 2z)(2x + y)(2x + z)(x − y)(x − z) + 3 ( y − z)2 .

Since
(x + 2 y)(x + 2z) = 2q + x 2 + 2 yz,
(2x + y)(2x + z) = 2q + 4x 2 − yz,
(x − y)(x − z) = x 2 + 2 yz − q,
( y − z)2 = −x 2 − 2 yz + p2 − 2q,
f6 (x, y, z) has the same highest coefficient A as

P1 (x, y, z) + 3P3 (x, y, z),

where 
P1 (x, y, z) = (x 2 + 2 yz)(4x 2 − yz)(x 2 + 2 yz),
(
P3 (x, y, z) = (−x 2 − 2 yz).
According to (3.1) and (3.3), we get

A = P1 (1, 1, 1) + 3P3 (1, 1, 1)


= 3(1 + 2)(4 − 1)(1 + 2) + 3(−1 − 2)3 = 81 − 81 = 0.

By Corollary 1, we only need to show that f6 (x, 1, 1) ≥ 0 for x ∈ . We have

f (x, 1, 1) = (x + 2)2 (2x + 1)2 (x − 1)2 ≥ 0.

The equality holds for x = y = z, for −x/2 = y = z (or any cyclic permutation),
and for −2x = y = z (or any cyclic permutation).

P 3.2. If x, y, z are real numbers, then



(x 2 − 4 yz)2 (x − y)(x − z) + 3(x − y)2 ( y − z)2 (z − x)2 ≥ 0.

(Vasile C., 2012)


214 Vasile Cîrtoaje

Solution. Write the inequality as f6 (x, y, z) ≥ 0, where

f6 (x, y, z) = f (x, y, z) + 3(x − y)2 ( y − z)2 (z − x)2 ,



f (x, y, z) = (x 2 − 4 yz)2 (x − y)(x − z).
Since
(x − y)(x − z) = x 2 + 2 yz − q,
f (x, y, z) has the same highest coefficient A1 as

P1 (x, y, z) = (x 2 − 4 yz)2 (x 2 + 2 yz),

that is, according to (3.1),

A1 = P1 (1, 1, 1) = 81.

Since the product (x − y) ( y − z)2 (z − x)2 has the highest coefficient equal to −27,
2

f6 (x, y, z) has the highest coefficient

A = A1 + 3(−27) = 0.

By Corollary 1, we only need to show that f6 (x, 1, 1) ≥ 0 for all real x. This is true
because
f (x, 1, 1) = (x 2 − 4)2 (x − 1)2 ≥ 0.

The equality holds for x = y = z, for x + y + z = 0, and for x/2 = y = z (or any
cyclic permutation).
Observation. The inequality is equivalent to

p2 (p4 − 7p2 q + 16q2 − 12pr) ≥ 0.

P 3.3. If x, y, z are real numbers, then



(x 2 + yz)(x + 2 y)(x + 2z)(x − y)(x − z) + 2(x − y)2 ( y − z)2 (z − x)2 ≥ 0.

(Vasile C., 2012)

Solution. Write the inequality as f6 (x, y, z) ≥ 0, where

f6 (x, y, z) = f (x, y, z) + 2(x − y)2 ( y − z)2 (z − x)2 ,

f (x, y, z) = (x 2 + yz)(x + 2 y)(x + 2z)(x − y)(x − z).


Since
(x + 2 y)(x + 2z) = x 2 + 2 yz + 2q,
Highest Coefficient Cancellation Method for Real Variables 215

(x − y)(x − z) = x 2 + 2 yz − q,
f (x, y, z) has the same highest coefficient A1 as

P1 (x, y, z) = (x 2 + yz)(x 2 + 2 yz)2 ,

that is, according to (3.1),

A1 = P1 (1, 1, 1) = 54.

Since the product (x − y)2 ( y − z)2 (z − x)2 has the highest coefficient equal to −27,
f6 (x, y, z) has the highest coefficient

A = A1 + 2(−27) = 0.

According to Corollary 1, we only need to show that f6 (x, 1, 1) ≥ 0 for all real x.
Indeed,
f (x, 1, 1) = (x 2 + 1)(x − 1)2 (x + 2)2 ≥ 0.

The equality holds for x = y = z and for x + y + z = 0.


Observation. The inequality is equivalent to
 
(x + y + z)2 (x 2 + y 2 )(x − y)2 + ( y 2 + z 2 )( y − z)2 + (z 2 + x 2 )(z − x)2 ≥ 0.

P 3.4. If x, y, z are real numbers, then

2x 2 + 3 yz 2 y 2 + 3z x 2z 2 + 3x y 5
+ + 2 ≤ .
4x + y + z
2 2 2 4y + z + x
2 2 2 4z + x 2 + y 2 2

(Vasile C., 2012)

Solution. Write the inequality as f6 (x, y, z) ≥ 0, where


( 
f6 (x, y, z) = 5 (4x 2 + y 2 + z 2 ) − 2 (2x 2 + 3 yz)(4 y 2 + z 2 + x 2 )(4z 2 + x 2 + y 2 ).

Since
4x 2 + y 2 + z 2 = 3x 2 + p2 − 2q,
f6 (x, y, z) has the same highest coefficient A as f (x, y, z), where
( 
f (x, y, z) = 5 (3x 2 ) − 2 (2x 2 + 3 yz)(3 y 2 )(3z 2 )

= 135x 2 y 2 z 2 − 108x 2 y 2 z 2 − 54 y 3z3

= 27x y z − 54
2 2 2 3 3
y z ,
216 Vasile Cîrtoaje

that is
A = 27 − 162 = −135.
According to Corollary 1, we only need to prove the original inequality for

y = z = 1.

Thus, we need to show that

2x 2 + 3 2(3x + 2) 5
+ ≤ ,
2(2x 2 + 1) x2 + 5 2
which is equivalent to
(x − 1)2 (2x − 1)2 ≥ 0.

The equality holds for x = y = z and for 2x = y = z (or any cyclic permutation).

P 3.5. If x, y, z are real numbers, then

9x 2 − 4 yz 9 y 2 − 4z x 9z 2 − 4x y 15
+ + ≥ .
3x 2 + 2 y 2 + 2z 2 3 y 2 + 2z 2 + 2x 2 3z 2 + 2x 2 + 2 y 2 7
(Vasile C., 2012)

Solution. Write the inequality as f6 (x, y, z) ≥ 0, where



f6 (x, y, z) =7 (9x 2 − 4 yz)(3 y 2 + 2z 2 + 2x 2 )(3z 2 + 2x 2 + 2 y 2 )
− 15(3x 2 + 2 y 2 + 2z 2 )(3 y 2 + 2z 2 + 2x 2 )(3z 2 + 2x 2 + 2 y 2 ).

Since
3x 2 + 2 y 2 + 2z 2 = x 2 + 2(p2 − 2q),
f6 (x, y, z) has the same highest coefficient A as f (x, y, z), where
 (
f (x, y, z) = 7 (9x 2 − 4 yz)( y 2 )(z 2 ) − 15 (x 2 )

= 174x 2 y 2 z 2 − 28 y 3z3,

that is
A = 174 − 84 = 90.
Since A > 0 and

f6 (x, 1, 1) =7(9x 2 − 4)(2x 2 + 5)2 + 14(9 − 4x)(2x 2 + 5)(3x 2 + 4)


− 15(2x 2 + 5)2 (3x 2 + 4)
=4(2x 2 + 5)(x − 1)2 (3x − 4)2 ,
Highest Coefficient Cancellation Method for Real Variables 217

we apply Corollary 2 for

(x − 1)4 (3x − 4)2


Fγ,δ (x) = f4/3,−2 (x) = .
81 · 25
We have
2(x − 1)4 (3x − 4)2
Af4/3,−2 (x) = ,
45
2(x − 1)2 (3x − 4)2 f (x)
f6 (x, 1, 1) − Af4/3,−2 (x) = ,
45
where

f (x) = 90(2x 2 + 5) − (x − 1)2 > (2x 2 + 5) − (x − 1)2 = (x + 1)2 + 3 > 0.

3x
The equality holds for x = y = z, and also for = y = z (or any cyclic
4
permutation).

Observation. Similarly, we can prove the following generalization:


• Let x, y, z be real numbers. If k > 0, k = 1, then

 k(k−3) 2
k−1 x + 2 yz 3(k + 1)(k − 2)
≤ ,
k x 2 + y 2 + z2 (k − 1)(k + 2)

kx
with equality for x = y = z, and for = y = z (or any cyclic permutation).
2
For
( 
f6 (x, y, z) = m (k x 2 + y 2 + z 2 ) − (nx 2 + 2 yz)(k y 2 + z 2 + x 2 )(kz 2 + x 2 + y 2 ),

where
3(k + 1)(k − 2) k(k − 3)
m= , n= ,
(k − 1)(k + 2) k−1
we have
9(k + 1)(k − 1)(2 − k)
A=
k+2
and
2
f6 (x, 1, 1) = (x 2 + k + 1)(x − 1)2 (k x − 2)2 .
k+2
For k ∈ (0, 1) ∪ [2, ∞), we have A ≤ 0. According to Corollary 1, we only need
to prove that f6 (x, 1, 1) ≥ 0 for x ∈ , which is clearly true.
For k ∈ (1, 2), we have A > 0. According to Corollary 2, it suffices to prove that

f6 (x, 1, 1) ≥ Af2/k,−2 (x)


218 Vasile Cîrtoaje

for x ∈ . We have
(x − 1)4 (k x − 2)2
f2/k,−2 (x) = ,
81(k + 1)2
(k − 1)(2 − k)(x − 1)4 (k x − 2)2
Af2/k,−2 (x) = ,
9(k + 1)(k + 2)
(x − 1)2 (k x − 2)2 f (x)
f6 (x, 1, 1) − Af2/k,−2 (x) = ,
9(k + 1)(k + 2)
where
f (x) = 18(k + 1)(x 2 + k + 1) − (k − 1)(2 − k)(x − 1)2 .
Since
2(x 2 + k + 1) − (x − 1)2 = (x + 1)2 + 2k > 0,
we have

f (x) > 9(k + 1)(x − 1)2 − (k − 1)(2 − k)(x − 1)2 = (k2 + 6k + 11)(x − 1)2 ≥ 0.

P 3.6. Let x, y, z be real numbers, no two of which are zero. If

x 2 + y 2 + z 2 ≥ 2(x y + yz + z x),

then
x 2 − 6 yz y 2 − 6z x z 2 − 6x y
+ 2 + 2 ≥ 0.
y2+ yz + z 2 z + zx + x 2 x + x y + y2
(Vasile C., 2012)

Solution. Write the inequality as f6 (x, y, z) ≥ 0, where



f6 (x, y, z) = (x 2 − 6 yz)(x 2 + x y + y 2 )(x 2 + xz + z 2 ) ≥ 0.

Since

x 2 + x y + y 2 = p2 − 2q − z 2 + x y, x 2 + xz + z 2 = p2 − 2q − y 2 + xz,

f6 (x, y, z) has the same highest coefficient A as



P2 (x, y, z) = (x 2 − 6 yz)(−z 2 + x y)(− y 2 + z x),

that is, according to (3.2),


A = P2 (1, 1, 1) = 0.
According to Theorem 1, we only need to show that f6 (x, 1, 1) ≥ 0 for x 2 + 2 ≥
2(2x + 1), that is for
x ∈ (−∞, 0] ∪ [4, ∞).
Highest Coefficient Cancellation Method for Real Variables 219

We have

f6 (x, 1, 1) = (x 2 − 6)(x 2 + x + 1)2 + 6(1 − 6x)(x 2 + x + 1)


= (x 2 + x + 1)x(x 3 + x 2 − 5x − 42).

Case 1: x ≤ 0. We need to show that x 3 + x 2 − 5x − 42 ≤ 0. Indeed,

x 3 + x 2 − 5x − 42 = x(x + 1)2 − (x + 3)2 − 33 < 0.

Case 2: x ≥ 4. We need to show that x 3 + x 2 − 5x − 42 ≥ 0. We have

x 3 + x 2 − 5x − 42 ≥ 5x 2 − 5x − 42 ≥ 15x − 42 > 0.

The equality holds for x = 0 and y = z (or any cyclic permutation).

P 3.7. If x, y, z be real numbers, no two of which are zero, then

8x 2 + 3 yz 8 y 2 + 3z x 8z 2 + 3x y
+ 2 + 2 ≥ 11.
y + yz + z
2 2 z + zx + x 2 x + x y + y2

(Vasile C., 2012)

Solution. Write the inequality as f6 (x, y, z) ≥ 0, where


 (
f6 (x, y, z) = (8x 2 + 3 yz)(x 2 + x y + y 2 )(x 2 + xz + z 2 ) − 11 ( y 2 + yz + z 2 ).

Since
y 2 + yz + z 2 = yz − x 2 + p2 − 2q,
f6 (x, y, z) has the same highest coefficient A as

P2 (x, y, z) − 11P3 (x, y, z),

where
 (
P2 (x, y, z) = (8x 2 + 3 yz)(x y − z 2 )(xz − y 2 ), P3 (x, y, z) = ( yz − x 2 ).

Therefore,
A = P2 (1, 1, 1) − 11P3 (1, 1, 1) = 0.
According to Corollary 1, we only need to prove the original inequality for y = z =
1. Thus, we need to show that

8x 2 + 3 2(8 + 3x)
+ 2 ≥ 11,
3 x + x +1
220 Vasile Cîrtoaje

which is equivalent to

4x 4 + 4x 3 − 11x 2 − 6x + 9 ≥ 0,

(x − 1)2 (2x + 3)2 ≥ 0.


−2x
The equality holds for x = y = z, and also for = y = z (or any cyclic
3
permutation).

P 3.8. If x, y, z are real numbers, no two of which are zero, then

8x 2 − 5 yz 8 y 2 − 5z x 8z 2 − 5x y
+ + ≥ 9.
y 2 − yz + z 2 z 2 − z x + x 2 x 2 − x y + y 2
(Vasile C., 2012)

Solution. Write the inequality as f6 (x, y, z) ≥ 0, where


 (
f6 (x, y, z) = (8x 2 − 5 yz)(x 2 − x y + y 2 )(x 2 − xz + z 2 ) − 9 ( y 2 − yz + z 2 ).

From

f6 (x, y, z) = (8x 2 − 5 yz)(p2 − 2q − z 2 − x y)(p2 − 2q − y 2 − xz)
(
−9 (p2 − 2q − x 2 − yz),

it follows that f6 (x, y, z) has the same highest coefficient A as

P2 (x, y, z) + 9P3 (x, y, z),

where
 (
P2 (x, y, z) = (8x 2 − 5 yz)(z 2 + x y)( y 2 + xz), P3 (x, y, z) = (x 2 + yz),

that is, according to (3.2) and (3.3),

A = P2 (1, 1, 1) + 9P3 (1, 1, 1) = 3 · 3 · 2 · 2 + 9 · 8 = 108.

Since

f6 (x, 1, 1) = (8x 2 − 5)(x 2 − x + 1)2 + 2(8 − 5x)(x 2 − x + 1) − 9(x 2 − x + 1)2


= 2(x 2 − x + 1)(x − 1)2 (2x + 1)2 ,

we apply Corollary 2 for

4(x − 1)4 (2x + 1)2


Fγ,δ (x) = f−1/2,−2 (x) = .
729
Highest Coefficient Cancellation Method for Real Variables 221

We need to show that f6 (x, 1, 1) ≥ Af−1/2,−2 (x) for x ∈ . Indeed,

2(x − 1)2 (2x + 1)2 (19x 2 − 11x + 19)


f6 (x, 1, 1) − Af−1/2,−2 (x) = ≥ 0.
27

The equality holds for x = y = z, and for −2x = y = z (or any cyclic permuta-
tion).

Observation. Similarly, we can prove the following generalization:


• Let x, y, z be real numbers, no two of which are zero. If −2 < k ≤ 1, then

8x 2 + k(4 − k) yz 8 y 2 + k(4 − k)z x 8z 2 + k(4 − k)x y 3(8 + 4k − k2 )


+ + ≥ ,
y + k yz + z
2 2 z + kz x + x
2 2 x + kx y + y
2 2 k+2

−2x
with equality for x = y = z, and for = y = z (or any cyclic permutation).
k+2

P 3.9. If x, y, z are real numbers, no two of which are zero, then

5x 2 + 2 yz 5 y 2 + 2z x 5z 2 + 2x y
+ 2 + 2 ≥ 3.
2 y2 + 3 yz + 2z 2 2z + 3z x + 2x 2 2x + 3x y + 2 y 2

(Vasile C., 2012)

Solution. Write the inequality as f6 (x, y, z) ≥ 0, where


 (
f6 (x, y, z) = (5x 2 +2 yz)(2x 2 +3x y+2 y 2 )(2x 2 +3xz+2z 2 )−3 (2 y 2 +3 yz+2z 2 ).

From

f6 (x, y, z) = (5x 2 + 2 yz)(3x y − 2z 2 + 2p2 − 4q)(3xz − 2 y 2 + 2p2 − 4q)
(
−3 (3 yz − 2x 2 + 2p2 − 4q),

it follows that f6 (x, y, z) has the same highest coefficient A as

P2 (x, y, z) − 3P3 (x, y, z),

where
 (
P2 (x, y, z) = (5x 2 +2 yz)(3x y −2z 2 )(3xz−2 y 2 ), P3 (x, y, z) = (3 yz−2x 2 ),

that is,
A = P2 (1, 1, 1) − 3P3 (1, 1, 1) = 21 − 3 = 18.
222 Vasile Cîrtoaje

Since
f6 (x, 1, 1)
= (5x 2 + 2)(2x 2 + 3x + 2) + 14(5 + 2x) − 21(2x 2 + 3x + 2)
2x 2 + 3x + 2
= 10x 4 + 15x 3 − 28x 2 − 29x + 32
= (x − 1)2 (10x 2 + 35x + 32).

we apply Corollary 2 for

4(x − 1)4 (x − γ)2


Fγ,δ (x) = fγ,−2 (x) = .
81(2 + γ)2

We need to show that

f6 (x, 1, 1) ≥ Afγ,−2 (x), x ∈ ,

which is equivalent to f (x) ≥ 0, where

8(x − 1)2 (x − γ)2


f (x) = (2x 2 + 3x + 2)(x − 1)2 (10x 2 + 35x + 32) − .
9(2 + γ)2

Since the original inequality is an equality for (x, y, z) = (0, 1, −1), that is

f6 (0, 1, −1) = 0,

we have f (−2) = 0 for all real γ (see Proposition 1). To have f (x) ≥ 0 in the
vicinity of x = −2, the condition f (−2) = 0 is necessary. This condition involves
γ = −50/37 and

f (x) = (x + 2)2 (11591x 2 + 17474x + 9743) ≥ 0.

The equality holds for x = y = z, and for x = 0 and y + z = 0 (or any cyclic
permutation).

Observation. Similarly, we can prove the following generalization:


3
• Let x, y, z be real numbers, no two of which are zero. If ≤ k < 2, then
2
 2(4 − k)x 2 + (2 − k)(1 + 2k) yz
≥ 3(5 − 2k),
y 2 + k yz + z 2

with equality for x = y = z, and also for x = 0 and y + z = 0 (or any cyclic
permutation).
Highest Coefficient Cancellation Method for Real Variables 223

P 3.10. If x, y, z are real numbers, then

(x + y)(x + z) ( y + z)( y + x) (z + x)(z + y) 4


+ + 2 ≤ .
7x 2 + y 2 + z 2 7 y 2 + z2 + x 2 7z + x 2 + y 2 3

(Vasile C., 2009)

Solution. Write the inequality as f6 (x, y, z) ≥ 0, where


( 
f6 (x, y, z) = 4 (7x 2 + y 2 + z 2 ) − 3 (x + y)(x + z)(7 y 2 + z 2 + x 2 )(7z 2 + x 2 + y 2 ).

Since
7x 2 + y 2 + z 2 = 6x 2 + p2 − 2q, (x + y)(x + z) = x 2 + q,
f6 (x, y, z) has the same highest coefficient A as
( 
4 (6x 2 ) − 3 x 2 (6 y 2 )(6z 2 ),

that is
A = 4 · 63 − 9 · 62 = 540.
Since
f6 (x, 1, 1)
= 4(7x 2 + 2)(x 2 + 8) − 3(x + 1)[(x + 1)(x 2 + 8) + 4(7x 2 + 2)]
x2 + 8
= 25x 4 − 90x 3 + 121x 2 − 72x + 16 = (x − 1)2 (5x − 4)2 ,

we apply Corollary 2 for

(x − 1)4 (5x − 4)2


Fγ,δ (x) = f4/5,−2 (x) = .
81 · 49
We need to show that
f6 (x, 1, 1) ≥ Af4/5,−2 (x)
for x ∈ . Since
20(x − 1)4 (5x − 4)2
Af4/5,−2 (x) = ,
147
we get
(x − 1)2 (5x − 4)2 f (x)
f6 (x, 1, 1) − Af4/5,−2 (x) = ,
147
where

f (x) = 147(x 2 + 8) − 20(x − 1)2 > 40(x 2 + 8) − 20(x − 1)2 = 20[(x + 1)2 + 14] > 0.

Thus, the proof is completed. The equality holds for x = y = z, and also for
x y z
= = (or any cyclic permutation).
4 5 5
224 Vasile Cîrtoaje

P 3.11. If x, y, z are real numbers, then


6x( y + z) − yz 6 y(z + x) − z x 6z(x + y) − x y 33
+ + ≤ .
12x 2 + y 2 + z 2 12 y 2 + z 2 + x 2 12z 2 + x 2 + y 2 14
(Vasile C., 2009)
Solution. Write the inequality as f6 (x, y, z) ≥ 0, where f6 (x, y, z) is
( 
33 (12x 2 + y 2 + z 2 ) − 14 (6x y + 6xz − yz)(12 y 2 + z 2 + x 2 )(12z 2 + x 2 + y 2 ).
Since
12x 2 + y 2 + z 2 = 11x 2 + p2 − 2q, 6x y + 6xz − yz = 6q − 7 yz,
f6 (x, y, z) has the same highest coefficient A as f (x, y, z), where
( 
f (x, y, z) = 33 (11x 2 ) − 14 (−7 yz)(11 y 2 )(11z 2 )

= 33 · 113 x 2 y 2 z 2 + 98 · 112 y 3z3,
that is
A = 33 · 113 + 3 · 98 · 112 = 332 · 73.
Since
f6 (x, 1, 1)
= 33(12x 2 + 2)(x 2 + 13) − 14[(12x − 1)(x 2 + 13) + 2(5x + 6)(12x 2 + 2)]
x 2 + 13
= 44(9x 4 − 42x 3 + 73x 2 − 56x + 16) = 44(x − 1)2 (3x − 4)2 ,
we apply Corollary 2 for
(x − 1)4 (3x − 4)2
Fγ,δ (x) = f4/3,−2 (x) = .
2025
We need to show that
f6 (x, 1, 1) ≥ Af4/3,−2 (x)
for x ∈ . Since
8833(x − 1)4 (3x − 4)2
Af4/3,−2 (x) = ,
225
we get
11(x − 1)2 (3x − 4)2 f (x)
f6 (x, 1, 1) − Af4/3,−2 (x) = ,
225
where
f (x) = 900(x 2 + 13) − 803(x − 1)2
> 900(x 2 + 13) − 810(x − 1)2 = 90[(x + 9)2 + 40] > 0.
Thus, the proof is completed. The equality holds for x = y = z, and also for
x y z
= = (or any cyclic permutation).
4 3 3
Highest Coefficient Cancellation Method for Real Variables 225

P 3.12. If x, y, z are real numbers, then


x( y + z) − yz y(z + x) − z x z(x + y) − x y 3
+ + ≤ .
x 2 + 3 y 2 + 3z 2 y 2 + 3z 2 + 3x 2 z 2 + 3x 2 + 3 y 2 7
(Vasile C., 2009)
Solution. Write the inequality as f6 (x, y, z) ≥ 0, where f6 (x, y, z) is
( 
3 (x 2 + 3 y 2 + 3z 2 ) − 7 (x y + xz − yz)( y 2 + 3z 2 + 3x 2 )(z 2 + 3x 2 + 3 y 2 ).

Since

x 2 + 3 y 2 + 3z 2 = −2x 2 + 3(p2 − 2q), x y + xz − yz = q − 2 yz,

f6 (x, y, z) has the same highest coefficient A as f (x, y, z), where


( 
f (x, y, z) = 3 (−2x 2 ) − 7 (−2 yz)(−2 y 2 )(−2z 2 )

= −24x 2 y 2 z 2 + 56 y 3z3,

that is
A = −24 + 168 = 144.
Since
f6 (x, 1, 1)
= 3(x 2 + 6)(3x 2 + 4) − 7[(2x − 1)(3x 2 + 4) + 2(x 2 + 6)]
3x 2 + 4
= 9x 4 − 42x 3 + 73x 2 − 56x + 16 = (x − 1)2 (3x − 4)2 ,

we apply Corollary 2 for


(x − 1)4 (3x − 4)2
Fγ,δ (x) = f4/3,−2 (x) = .
2025
We need to show that
f6 (x, 1, 1) ≥ Af4/3,−2 (x)
for x ∈ . Since
16(x − 1)4 (3x − 4)2
Af4/3,−2 (x) = ,
225
we get
(x − 1)2 (3x − 4)2 f (x)
f6 (x, 1, 1) − Af4/3,−2 (x) = ,
225
where

f (x) = 225(3x 2 +4)−16(x−1)2 > 16(3x 2 +4)−16(x−1)2 = 16[x 2 +2+(x+1)2 ] > 0.

Thus, the proof is completed. The equality holds for x = y = z, and also for
x y z
= = (or any cyclic permutation).
4 3 3
226 Vasile Cîrtoaje

P 3.13. If x, y, z are real numbers, then


 1
yz(2x 2 + yz)(x − y)(x − z) + (x − y)2 ( y − z)2 (z − x)2 ≥ 0.
2
(Vasile C., 2012)
Solution. Write the inequality as f6 (x, y, z) ≥ 0, where

f6 (x, y, z) = f (x, y, z) + (x − y)2 ( y − z)2 (z − x)2 ,



f (x, y, z) = 2 yz(2x 2 + yz)(x − y)(x − z).
Since (x − y)(x − z) = x 2 + 2 yz − q, f (x, y, z) has the same highest coefficient A1
as 
P1 (x, y, z) = 2 yz(2x 2 + yz)(x 2 + 2 yz),
that is, according to (3.1),

A1 = P1 (1, 1, 1) = 54.

Therefore, f6 (x, y, z) has the highest coefficient

A = A1 + (−27) = 27.

Since
f6 (x, 1, 1) = f (x, 1, 1) = 2(2x 2 + 1)(x − 1)2 ,
we apply Corollary 2 for

4(x − 1)4
Fγ,δ (x) = f∞,−2 (x) = .
81
We need to show that
f6 (x, 1, 1) ≥ Af∞,−2 (x)
for x ∈ . Indeed, we get

2(x − 1)2 (2x + 1)2


f6 (x, 1, 1) − Af∞,−2 (x) = ≥ 0.
3
Thus, the proof is completed. The equality holds for x = y = z, and also for
y = z = 0 (or any cyclic permutation).

P 3.14. If x, y, z are real numbers, then


 3
(x 2 − yz)2 (x − y)(x − z) ≥ (x − y)2 ( y − z)2 (z − x)2 .
4
(Vasile C., 2012)
Highest Coefficient Cancellation Method for Real Variables 227

Solution. Write the inequality as f6 (x, y, z) ≥ 0, where


3
f6 (x, y, z) = f (x, y, z) − (x − y)2 ( y − z)2 (z − x)2 ,
4

f (x, y, z) = (x − yz)2 (x − y)(x − z).
2

Since
(x − y)(x − z) = x 2 + 2 yz − q,
f (x, y, z) has the same highest coefficient A1 as

P1 (x, y, z) = (x 2 − yz)2 (x 2 + 2 yz),

that is, according to (3.1),


A1 = P1 (1, 1, 1) = 0.
Therefore, f6 (x, y, z) has the highest coefficient
3 81
A = A1 − (−27) = .
4 4
Since
f6 (x, 1, 1) = (x 2 − 1)2 (x − 1)2 ,
we apply Corollary 2 for
4(x − 1)4 (x + 1)2
Fγ,δ (x) = f−1,−2 (x) = .
81
We need to show that
f6 (x, 1, 1) ≥ Af−1,−2 (x)
for x ∈ , which is an identity. Thus, the proof is completed. The equality holds
for
9x yz + 2(x + y + z)3 = 7(x + y + z)(x y + yz + z x).

Observation. The coefficient of the product (x − y)2 ( y − z)2 (z − x)2 is the best
possible. Setting x = 0, y = 1 and z = −1 in the inequality

(x 2 − yz)2 (x − y)(x − z) ≥ k(x − y)2 ( y − z)2 (z − x)2 ,

3
we get k ≤ . According to (3.12) from Remark 2, the identity holds:
4
1
f6 (x, y, z) = (9r − 7pq + 2p3 )2 .
4
Therefore, the original inequality is equivalent to

(9r − 7pq + 2p3 )2 ≥ 0.


228 Vasile Cîrtoaje

P 3.15. If x, y, z are real numbers, then



(x 2 + 8 yz)2 (x − y)(x − z) + 15(x − y)2 ( y − z)2 (z − x)2 ≥ 0.

(Vasile C., 2012)


Solution. Write the inequality as f6 (x, y, z) ≥ 0, where
f6 (x, y, z) = f (x, y, z) + 15(x − y)2 ( y − z)2 (z − x)2 ,

f (x, y, z) = (x 2 + 8 yz)2 (x − y)(x − z).
Since
(x − y)(x − z) = x 2 + 2 yz − q,
f (x, y, z) has the same highest coefficient A1 as

P1 (x, y, z) = (x 2 + 8 yz)2 (x 2 + 2 yz),
that is, according to (3.1),
A1 = P1 (1, 1, 1) = 729.
Therefore, f6 (x, y, z) has the highest coefficient
A = A1 + 15(−27) = 324.
Since
f6 (x, 1, 1) = (x 2 + 8)2 (x − 1)2 ,
we apply Corollary 2 for
4(x − 1)4
Fγ,δ (x) = f∞,−2 (x) = .
81
We need to show that
f6 (x, 1, 1) ≥ Af∞,−2 (x)
for x ∈ . Indeed,
f6 (x, 1, 1) − Af∞,−2 (x) = (x − 1)2 g(x),
where
g(x) = (x 2 + 8)2 − 16(x − 1)2 = (x + 2)2 (x 2 − 4x + 12) ≥ 0.
Thus, the proof is completed. The equality holds for x = y = z, and also for x = 0
and y + z = 0.
Observation. The coefficient of the product (x − y)2 ( y − z)2 (z − x)2 is the best
possible. Setting x = 0, y = 1 and z = −1 in the inequality

(x 2 + 8 yz)2 (x − y)(x − z) + k(x − y)2 ( y − z)2 (z − x)2 ≥ 0,
we get k ≥ 15.
Highest Coefficient Cancellation Method for Real Variables 229

P 3.16. Let x, y, z be real numbers. If k ∈ , then


 7k2 − 20k − 20
(x 2 + yz)(x− y)(x−z)(x−k y)(x−kz)+ (x− y)2 ( y−z)2 (z−x)2 ≥ 0.
24
(Vasile C., 2010)

Solution. Write the inequality as f6 (x, y, z) ≥ 0, where

7k2 − 20k − 20
f6 (x, y, z) = f (x, y, z) + (x − y)2 ( y − z)2 (z − x)2 ,
24

f (x, y, z) = (x 2 + yz)(x − y)(x − z)(x − k y)(x − kz).
Since
(x − y)(x − z) = x 2 + 2 yz − q,
(x − k y)(x − kz) = x 2 + (k + k2 ) yz − kq,
f (x, y, z) has the same highest coefficient A1 as

P1 (x, y, z) = (x 2 + yz)(x 2 + 2 yz)[x 2 + (k + k2 ) yz];

that is, according to (3.1),

A1 = P1 (1, 1, 1) = 18(1 + k + k2 ).

Therefore, f6 (x, y, z) has the highest coefficient

7k2 − 20k − 20 81(k + 2)2


A = A1 + (−27) = .
24 8
On the other hand

f6 (x, 1, 1) = (x 2 + 1)(x − 1)2 (x − k)2 .

For k = −2, we have A = 0. Since f6 (x, 1, 1) ≥ 0 for any real x, the conclusion
follows by Corollary 1.
For k = −2, we apply Corollary 2 for

4(x − 1)4 (x − k)2


Fγ,δ (x) = f k,−2 (x) = .
81(2 + k)2
We have
(x 2 − 1)2 (x − k)2
f6 (x, 1, 1) − Af k,−2 (x) = ≥ 0.
2
Thus, the proof is completed. The equality holds for x = y = z, and for x/k = y = z
(or any cyclic permutation) if k = 0. If k = 0, then the equality holds also for x = 0
and y = z (or any cyclic permutation).
230 Vasile Cîrtoaje

P 3.17. If x, y, z are distinct real numbers, then

yz zx xy 1
+ + + ≥ 0.
( y − z)2 (z − x)2 (x − y)2 4

Solution. Write the inequality as f6 (x, y, z) ≥ 0, where

1
f6 (x, y, z) = f (x, y, z) + (x − y)2 ( y − z)2 (z − x)2 ,
4

f (x, y, z) = yz(x − y)2 (x − z)2 .
Since
(x − y)(x − z) = x 2 + 2 yz − q,
f (x, y, z) has the same highest coefficient A1 as

P1 (x, y, z) = yz(x 2 + 2 yz)2 ;

that is, according to (3.1),

A1 = P1 (1, 1, 1) = 27.

Therefore, f6 (x, y, z) has the highest coefficient

27 81
A = A1 − = .
4 4
Since
f6 (x, 1, 1) = (x − 1)4 ,
we apply Corollary 2 for

4(x − 1)4
Fγ,δ (x) = f∞,−2 (x) = .
81
We have
f6 (x, 1, 1) − Af∞,−2 (x) = 0.
Thus, the proof is completed. The inequality is an equality for all distinct real x, y, z
which satisfy
9x yz = (x + y + z)(x y + yz + z x).

Observation. The coefficient of the product (x − y)2 ( y − z)2 (z − x)2 in the in-
equality f6 (x, y, z) ≥ 0 is the best possible. Thus, setting x = 0, y = 1 and z = −1
in the inequality

yz(x − y)2 (x − z)2 + k(x − y)2 ( y − z)2 (z − x)2 ≥ 0,
Highest Coefficient Cancellation Method for Real Variables 231

1
we get k ≥ . According to (3.12) from Remark 2, the identity holds:
4
1
f6 (x, y, z) = (9r − pq)2 .
4
Therefore, for distinct x, y, z, the original inequality is equivalent to

(9r − pq)2 ≥ 0.

P 3.18. Let x, y, z be distinct real numbers. If k ∈ , then

(x − k y)(x − kz) ( y − kz)( y − k x) (z − k x)(z − k y) k2


+ + ≥ 2 + 2k − .
( y − z) 2 (z − x) 2 (x − y) 2 4

(Vasile C., 2010)

Solution. Write the inequality as f6 (x, y, z) ≥ 0, where


 
k2
f6 (x, y, z) = f (x, y, z) − 2 + 2k − (x − y)2 ( y − z)2 (z − x)2 ,
4

f (x, y, z) = (x − y)2 (x − z)2 (x − k y)(x − kz).
Since

(x − y)(x − z) = x 2 + 2 yz − q, (x − k y)(x − kz) = x 2 + (k + k2 ) yz − kq

f (x, y, z) has the same highest coefficient A1 as



P1 (x, y, z) = (x 2 + 2 yz)2 [x 2 + (k + k2 ) yz];

that is, according to (3.1),

A1 = P1 (1, 1, 1) = 27(1 + k + k2 ).

Therefore, f6 (x, y, z) has the highest coefficient


 
k2 81(k + 2)2
A = A1 − (−27) 2 + 2k − = .
4 4
In addition,
f6 (x, 1, 1) = (x − 1)4 (x − k)2 ≥ 0.

For k = −2, we have A = 0. Since f6 (x, 1, 1) ≥ 0 for any real x, the conclusion
follows (by Corollary 1).
232 Vasile Cîrtoaje

For k = −2, we apply Corollary 2 for

4(x − 1)4 (x − k)2


Fγ,δ (x) = f k,−2 (x) = .
81(2 + k)2
Since
f6 (x, 1, 1) − Af k,−2 (x) = 0,
the proof is completed. The equality occurs for all distinct real x, y, z which satisfy

(9k + 18)x yz + 2(x + y + z)3 = (k + 8)(x + y + z)(x y + yz + z x).

Observation 1. For distinct x, y, z, the original inequality is equivalent to


 2
(9k + 18)r − (k + 8)pq + 2p3 ≥ 0.

Observation 2. For k → ∞, we get the inequality in P 3.17. For k = 0, k = 1 and


k = −1, we get respectively the inequalities:

x2 y2 z2
+ + ≥ 2,
( y − z)2 (z − x)2 (x − y)2

(x − y)(x − z) ( y − z)( y − x) (z − x)(z − y) 15


+ + ≥ ,
( y − z)2 (z − x)2 (x − y)2 4
(x + y)(x + z) ( y + z)( y + x) (z + x)(z + y) 1
+ + + ≥ 0.
( y − z)2 (z − x)2 (x − y)2 4

P 3.19. If x, y, z are real numbers, then


 1
(x 2 + 2 yz)(x 2 − y 2 )(x 2 − z 2 ) + (x − y)2 ( y − z)2 (z − x)2 ≥ 0.
2
(Vasile C., 2012)

Solution. Let 
f (x, y, z) = (x 2 + 2 yz)(x 2 − y 2 )(x 2 − z 2 )
and
1
f6 (x, y, z) = f (x, y, z) + (x − y)2 ( y − z)2 (z − x)2 .
2
Since
(x − y)(x − z) = x 2 + 2 yz − q, (x + y)(x + z) = x 2 + q,
f (x, y, z) has the same highest coefficient A1 as

P1 (x, y, z) = (x 2 + 2 yz)2 x 2 ,
Highest Coefficient Cancellation Method for Real Variables 233

that is, according to (3.1),

A1 = P1 (1, 1, 1) = 3(1 + 2)2 = 27.

Therefore f6 (x, y, z) has the highest coefficient


27 27
A = A1 − = .
2 2
Since
f6 (x, 1, 1) = (x 2 + 2)(x 2 − 1)2 ,
we apply Corollary 2 for
4(x − 1)4 (x + 1)2
Fγ,δ (x) = f−1,−2 (x) = .
81
Since
(x 2 − 1)2 (x + 2)2
f6 (x, 1, 1) − Af−1,−2 (x) = ≥ 0,
3
the proof is completed. The equality holds for x = y = z, for −x = y = z (or any
cyclic permutation), and also for x = 0 and y + z = 0 (or any cyclic permutation).
Observation. Similarly, we can prove the following generalization:
• Let x, y, z be real numbers. If k ∈ , then

(x 2 + 2 yz)(x − y)(x − z)(x − k y)(x − kz) ≥
 
k2
≥ 1+k− (x − y)2 ( y − z)2 (z − x)2 ≥ 0,
2
with equality for x = y = z, for x/k = y = z (or any cyclic permutation) if k = 0,
and for x = 0 and y + z = 0 (or any cyclic permutation). If k = 0, then the equality
holds also for x = 0 and y = z (or any cyclic permutation).
We have
f6 (x, 1, 1) = (x 2 + 2)(x − 1)2 (x − k)2 ,
27(k + 2)2
A= ,
2
(x − 1) (x − k)2 (x + 2)2
2
f6 (x, 1, 1) − Af k,−2 (x) = ≥ 0.
3

P 3.20. Let x, y, z be real numbers. If k ∈ , then


 3(k + 2)2 (x − y)2 ( y − z)2 (z − x)2
(x − y)(x − z)(x − k y)(x − kz) ≥ .
4(x 2 + y 2 + z 2 − x y − yz − z x)
(Vasile C., 2012)
234 Vasile Cîrtoaje

Solution. Write the inequality as f6 (x, y, z) ≥ 0, where



f6 (x, y, z) =4(x 2 + y 2 + z 2 − x y − yz − z x) (x − y)(x − z)(x − k y)(x − kz)
− 3(k + 2)2 (x − y)2 ( y − z)2 (z − x)2 .

Since
x 2 + y 2 + z 2 − x y − yz − z x = p2 − 3q,
f6 (x, y, z) has the highest coefficient

A = 81(k + 2)2 .

On the other hand,


f6 (x, 1, 1) = 4(x − 1)4 (x − k)2 .
For k = −2, we have A = 0. Since f6 (x, 1, 1) ≥ 0 for any real x, the conclusion
follows by Corollary 1.
For k = −2, we apply Corollary 2 for

4(x − 1)4 (x − k)2


Fγ,δ (x) = f k,−2 (x) = .
81(k + 2)2

We need to show that

f6 (x, 1, 1) ≥ Af k,−2 (x), x ∈ ,

which is an identity.
The equality holds for all real x, y, z which satisfy

9(k + 2)x yz + 2(x + y + z)3 = (k + 8)(x + y + z)(x y + yz + z x)

and are not all equal.

Observation 1. The coefficient of the product (x − y)2 ( y − z)2 (z − x)2 is the best
possible. Setting x = 0, y = 1 and z = −1 in the inequality
 (x − y)2 ( y − z)2 (z − x)2
(x − y)(x − z)(x − k y)(x − kz) ≥ αk ,
x2 + y 2 + z 2 − x y − yz − z x

3(k + 2)2
we get αk ≤ . According to (3.12) from Remark 2, the identity holds:
4
 2
f6 (x, y, z) = 9(k + 2)r − (k + 8)pq + 2p3 .

Therefore, if x, y, z are not all equal, then the original inequality is equivalent to
 2
9(k + 2)r − (k + 8)pq + 2p3 ≥ 0.
Highest Coefficient Cancellation Method for Real Variables 235

Observation 2. For k = −2, the original inequality has the form



(x − y)(x − z)(x + 2 y)(x + 2z) ≥ 0,

which is equivalent to

(x + y + z)2 (x 2 + y 2 + z 2 − x y − yz − z x) ≥ 0.

The equality holds for x = y = z, and also for x + y + z = 0.


Observation 3. Since
1 2
≥ ,
x 2 + y 2 + z 2 − x y − yz − z x 3(x 2 + y 2 + z 2 )
with equality for x + y + z = 0, the following weaker inequality holds
 (k + 2)2 (x − y)2 ( y − z)2 (z − x)2
(x − y)(x − z)(x − k y)(x − kz) ≥ ,
2(x 2 + y 2 + z 2 )
with equality for x = y = z, for x/k = y = z (or any cyclic permutation) if k = 0,
and for x = 0 and y + z = 0 (or any cyclic permutation). If k = 0, then the equality
holds also for x = 0 and y = z (or any cyclic permutation).
Observation 4. Adding the inequality from Observation 3 written in the form
 (k + 2)2
(x 2 + y 2 + z 2 )(x − y)(x − z)(x − k y)(x − kz) − (x − y)2 ( y − z)2 (z − x)2
2
and the identity

(2 yz − y 2 − z 2 )(x − y)(x − z)(x − k y)(x − kz)+
+(k2 + k + 1)(x − y)2 ( y − z)2 (z − x)2 = 0,

we get the inequality in Observation from the preceding P 3.19.


Observation 5. Substituting k − 1 for k in P 3.20, and using then the identity
 
(x − y)(x −z)[x −(k−1) y][x −(k−1)z] = (x − y)(x −z)(x −k y+z)(x −kz+ y),

we get the following statement:


• Let x, y, z be real numbers. If k ∈ , then
 3(k + 1)2 (x − y)2 ( y − z)2 (z − x)2
(x − y)(x − z)(x − k y + z)(x − kz + y) ≥ ,
4(x 2 + y 2 + z 2 − x y − yz − z x)
with equality for all real x, y, z which satisfy

9(k + 1)x yz + 2(x + y + z)3 = (k + 7)(x + y + z)(x y + yz + z x)

and are not all equal.


236 Vasile Cîrtoaje

P 3.21. Let x, y, z be real numbers such that x y + yz + z x ≥ 0. If k ∈ , then


  
k2
(x 2 + yz)(x − y)(x − z)(x − k y)(x − kz) ≥ 1 + k − (x − y)2 ( y − z)2 (z − x)2 .
4

(Vasile C., 2012)

Solution. Let

f (x, y, z) = (x 2 + yz)(x − y)(x − z)(x − k y)(x − kz)

and  
k2
f6 (x, y, z) = f (x, y, z) − 1 + k − (x − y)2 ( y − z)2 (z − x)2 .
4
Since

(x − y)(x − z) = x 2 + 2 yz − q, (x − k y)(x − kz) = x 2 + k(1 + k) yz − kq

f (x, y, z) has the same highest coefficient A1 as



P1 (x, y, z) = (x 2 + yz)(x 2 + 2 yz)[x 2 + k(1 + k) yz],

that is, according to (3.1),

A1 = P1 (1, 1, 1) = 3(1 + 1)(1 + 2)(1 + k + k2 ) = 18(1 + k + k2 ).

Therefore, f6 (x, y, z) has the highest coefficient


 
k2 45(k + 2)2
A = 18(1 + k + k2 ) − (−27) 1 + k − = .
4 4

For k = −2, we only need to show that f6 (x, 1, 1) ≥ 0 for 2x +1 ≥ 0 (see Theorem
1). Indeed,
f6 (x, 1, 1) = (x 2 + 1)(x − 1)2 (x + 2)2 ≥ 0.
Consider next k = −2. Since A > 0 and

f6 (x, 1, 1) = (x 2 + 1)(x − 1)2 (x − k)2 ,

we will apply Corollary 2 for

4(x − 1)4 (x − k)2


Fγ,δ (x) = f k,−2 (x) = .
81(2 + k)2

Thus, according to Remark 5, we need to show that

f6 (x, 1, 1) − Af k,−2 (x) ≥ 0


Highest Coefficient Cancellation Method for Real Variables 237

for 2x + 1 ≥ 0. Indeed, we have

5(x − 1)4 (x − k)2


f6 (x, 1, 1) − Af k,−2 (x) = (x 2 + 1)(x − 1)2 (x − k)2 −
9
2(x − 1)2 (x − k)2 (x + 2)(2x + 1)
= ≥ 0.
9

The equality holds for x = y = z, and for x/k = y = z (or any cyclic permuta-
tion) if k = 0 and 2k + 1 ≥ 0. If k = 0, then the equality holds also for x = 0 and
y = z (or any cyclic permutation).

P 3.22. If x, y, z are real numbers, then



(x 2 + 2 yz)2 (x − y)(x − z) ≥ 0.

(Vasile C., 2012)

Solution. Let 
f6 (x, y, z) = (x 2 + 2 yz)2 (x − y)(x − z).
Since (x − y)(x − z) = x 2 + 2 yz − q, f6 (x, y, z) has the same highest coefficient as

P1 (x, y, z) = (x 2 + 2 yz)3 ,

that is, according to (3.1),


A = P1 (1, 1, 1) = 81.
Since
f6 (x, 1, 1) = (x 2 + 2)2 (x − 1)2 ,
we apply Corollary 2. There are two methods to do this.
First method. By selecting

4(x − 1)4 (x − γ)2


Fγ,δ (x) = fγ,−2 (x) = ,
81(γ + 2)2
we need to show that there exists a real γ such that

f6 (x, 1, 1) ≥ Afγ,−2 (x)

for all real x. We have


4(x − 1)4 (x − γ)2
Afγ,−2 (x) = ,
(γ + 2)2

(x − 1)2 g1 (x)g2 (x)


f6 (x, 1, 1) − Afγ,−2 (x) = ,
(γ + 2)2
238 Vasile Cîrtoaje

where

g1 (x) = (γ + 2)(x 2 + 2) − 2(x − 1)(x − γ)


= (x + 2)(γx + 2),

g2 (x) = (γ + 2)(x 2 + 2) + 2(x − 1)(x − γ)


= (γ + 4)x 2 − 2(γ + 1)x + 4(γ + 1).

Choosing γ = 1, we get
g1 (x) = (x + 2)2 ≥ 0,

g2 (x) = 5x 2 − 4x + 8 = (2x − 1)2 + x 2 + 7 > 0.


The equality holds for x = y = z, and for x = 0 and y + z = 0 (or any cyclic
permutation).
Second method. Since

f6 (x, 1, 1) = (x 2 + 2)2 (x − 1)2

and

f6 (0, y, z) = 4 y 3 z 3 + ( y − z)( y 5 − z 5 ), f6 (0, 1, −1) = −4 + 4 = 0,

we select
Fγ,δ = g1,δ
with δ given by (3.16). We have

f6 (x, 1, 1) = (x − 1)2 g(x), g(x) = (x 2 + 2)2 ,

g (x) = 4x(x 2 + 2), g (−2) = −48,


γ (γ + 2) g (−2) −1
2
δ= + = .
3(γ + 2) 12A 3
According to Remark 4, we need to show that

g(x) ≥ Aḡ1,δ (x),

where  2
1 x +8 4
ḡ1,δ (x) = (x − 1)2 + δ(x + 2) = (x − 1)4 .
9 9 729
We have
4
g(x) − Aḡγ,δ (x) = (x 2 + 2)2 − (x − 1)4
9
1
= (x + 2)2 (5x 2 − 4x + 8) ≥ 0.
9
Highest Coefficient Cancellation Method for Real Variables 239

P 3.23. If x, y, z are real numbers, then



x 2 (x 2 + yz)(x − y)(x − z) ≥ (x − y)2 ( y − z)2 (z − x)2 .

(Vasile C., 2012)

Solution. Let 
f (x, y, z) = x 2 (x 2 + yz)(x − y)(x − z)
and
f6 (x, y, z) = f (x, y, z) − (x − y)2 ( y − z)2 (z − x)2 .
Since
(x − y)(x − z) = x 2 + 2 yz − q,
f (x, y, z) has the same highest coefficient A1 as

P1 (x, y, z) = x 2 (x 2 + yz)(x 2 + 2 yz),

that is, according to (3.1),

A1 = P1 (1, 1, 1) = 3(1 + 1)(1 + 2) = 18.

Therefore, f6 (x, y, z) has the highest coefficient

A = 18 − (−27) = 45.

Since
f6 (x, 1, 1) = x 2 (x 2 + 1)(x − 1)2 ,
x 2 (x − 1)4
f0,−2 (x) = ,
81

5x 2 (x − 1)4
f6 (x, 1, 1) − Af0,−2 (x) = x 2 (x 2 + 1)(x − 1)2 −
9
2x 2 (x − 1)2 (x + 2)(2x + 1)
= ,
9
we will apply Theorem 2 for

ξ → ∞, Eα,β = f0,−2 , Fγ,δ = fγ,−2 .

Notice that ξ → ∞ involves


  
−1 −1
= ,∞ ,  \  = −∞, .
2 2

The condition (a), namely f6 (x, 1, 1) ≥ Af0,−2 (x) for x ∈ , is satisfied.


240 Vasile Cîrtoaje

To prove the condition (b), we see that −2 ∈  \  and

f6 (0, y, z) = ( y − z)2 ( y 4 + y 3 z + yz 3 + z 4 ), f6 (0, 1, −1) = 0.

According to Proposition 1, the difference f6 (x, 1, 1) − Afγ,−2 (x) is zero for x = −2.
Therefore, we will use the Cauchy-Schwarz inequality

[(−2)2 + 1](x 2 + 1) ≥ (−2x + 1)2 ,

which is equivalent to (x + 2)2 ≥ 0, to get

f6 (x, 1, 1) ≥ F (x),

where the polynomial F (x) is a perfect square:

1 2
F (x) = x (2x − 1)2 (x − 1)2 .
5
Thus, the condition (b) in Theorem 2 is satisfied if

F (x) ≥ Afγ,−2 (x)

for x ≤ −1/2. We have

4(x − 1)4 (x − γ)2


fγ,−2 (x) = ,
81(2 + γ)2
1 2 20(x − 1)4 (x − γ)2
F (x) − Afγ,−2 (x) = x (2x − 1)2 (x − 1)2 −
5 9(2 + γ)2
(x − 1)2 g1 (x)g2 (x)
= ,
45(2 + γ)2

where

g1 (x) = 3(2 + γ)x(2x − 1) − 10(x − 1)(x − γ) = (x + 2)[γ(6x − 5) + 2x],

g2 (x) = 3(2 + γ)x(2x − 1) + 10(x − 1)(x − γ).


Choosing
−4
γ= ,
17
g1 (x) is a perfect square,

10(x + 2)2
g1 (x) = ≥ 0,
17
and
10(35x 2 − 22x − 4) 10(−8x − 4) −40(2x + 1)
g2 (x) = > = ≥ 0.
17 17 17
Highest Coefficient Cancellation Method for Real Variables 241

The equality holds for x = y = z, for x = 0 and y = z (or any cyclic permutation),
and for x = 0 and y + z = 0 (or any cyclic permutation).
Observation 1. Similarly, applying Theorem 2 for

13k − 4
ξ → ∞, Eα,β = f k,−2 , Fγ,δ = fγ,−2 , γ= ,
k + 17
we can prove the following generalization:
−7
• Let x, y, z be real numbers. If ≤ k ≤ 1, then
17
  
k2
(x 2 + yz)(x − y)(x − z)(x − k y)(x − kz) ≥ 1 + k − (x − y)2 ( y − z)2 (z − x)2 ,
4
with equality for x = y = z, for x/k = y = z (or any cyclic permutation) if k = 0,
and for x = 0 and y + z = 0 (or any cyclic permutation). If k = 0, then the equality
holds also for x = 0 and y = z (or any cyclic permutation).
We have

f6 (x, y, z) = (x 2 + yz)(x − y)(x − z)(x − k y)(x − kz)
 
k2
− 1+k− (x − y)2 ( y − z)2 (z − x)2 ,
4

45(k + 2)2
A= ,
4
  
−1 −1
= , ∞ ,  \  = −∞, ,
2 2
f (x, 1, 1) = (x 2 + 1)(x − 1)2 (x − k)2 ,
4(x − 1)4 (x − k)2 5(x − 1)4 (x − k)2
f k,−2 (x) = , Af k,−2 (x) = ,
81(2 + k)2 9
2(x − 1)2 (x − k)2 (x + 2)(2x + 1)
f6 (x, 1, 1) − Af k,−2 (x) = ,
9
1
F (x) = (2x − 1)2 (x − 1)2 (x − k)2 ,
5
4(x − 1)4 [(k + 17)x − 13k + 4]2
fγ,−2 (x) = ,
81 · 225(k + 2)2
(x − 1)4 [(k + 17)x − 13k + 4]2
Afγ,−2 (x) = ,
405
(x − 1)2 g1 (x)g2 (x)
F (x) − Afγ,−2 (x) = ,
405
g1 (x) = (1 − k)(x + 2)2 ≥ 0,
242 Vasile Cîrtoaje

g2 (x) = (k + 35)x 2 − 2(16k + 11)x + 2(11k − 2),


 
−1 9(17k + 7)
g2 (x) ≥ g2 = ≥ 0.
2 4

Observation 2. By leaving out the inequality

1
x 2 + 1 ≥ (2x − 1)2 ,
5
we can prove the inequality from Observation 1 for the extended range

−8

≤ k ≤ 9 5 − 17 ≈ 3.1246.
19
We have
(x − 1)2 g1 (x)
f6 (x, 1, 1) − Afγ,−2 (x) = ,
405

g1 (x) = 405(x 2 + 1)(x 2 − 2k x + k2 ) − (x − 1)2 [(k + 17)x − 13k + 4]2


= (x + 2)2 g2 (x),

g2 (x) = (116 − 34k − k2 )x 2 − 2(11 + 86k − 16k2 )x + 59k2 + 26k − 4,


116 − 34k − k2 ≥ 0,
 
−1 9(k + 2)(19k + 8)
g2 (x) ≥ g2 = ≥ 0.
2 4

Observation 3. Actually, the inequality from Observation 1 holds for k ∈ [−2, 28].

P 3.24. Let x, y, z be real numbers. If 0 ≤ k ≤ 27, then


  
k2
(x 2 + yz)(x − y)(x − z)(x − k y)(x − kz) ≥ 1 + k − (x − y)2 ( y − z)2 (z − x)2 .
4

(Vasile C., 2014)

Solution. Write the inequality as f6 (x, y, z) ≥ 0. As shown in the proof of P 3.21,


f6 (x, y, z has the highest coefficient

45(k + 2)2
A= .
4
Since
f (x, 1, 1) = (x 2 + 1)(x − 1)2 (x − k)2 ,
Highest Coefficient Cancellation Method for Real Variables 243

4(x − 1)4 (x − k)2


f k,−2 (x) =
81(2 + k)2
and

5(x − 1)4 (x − k)2


f6 (x, 1, 1) − Af k,−2 (x) = (x 2 + 1)(x − 1)2 (x − k)2 −
9
2(x − 1)2 (x − k)2 (x + 2)(2x + 1)
= ,
9
we will apply Theorem 2 for

ξ → ∞, Eα,β = f k,−2 , Fγ,δ = hγ,δ .

Notice that ξ → ∞ involves


  
−1 −1
= ,∞ ,  \  = −∞, .
2 2

The condition (a) in Theorem 2, namely f6 (x, 1, 1) − Af k,−2 (x) ≥ 0 for x > −1/2,
is satisfied.
To prove the condition (b), namely f6 (x, 1, 1) ≥ Ahγ,δ (x) for x ≤ −1/2, we see

−1
that −2 ∈ −∞, and
2
   k2

f6 (0, y, z) = k2 y 3 z 3 +( y −z) ( y 5 − z 5 ) − k yz( y 3 − z 3 ) − 1 + k − y 2 z 2 ( y −z)2 ,
4

f6 (0, 1, −1) = −k2 + 2(2 + 2k) − (4 + 4k − k2 ) = 0.


Therefore, according to Remark 4, we select γ given by (3.19),

1 h (−2)
γ= + ,
3 12A
where
h(x) = f6 (x, 1, 1) = (x 2 + 1)(x − 1)2 (x − k)2 .
We have

h (x) = 2x(x − 1)2 (x − k)2 + 2(x 2 + 1)(x − 1)(x − k)2 + 2(x 2 + 1)(x − 1)2 (x − k),

h (−2) = −6(k + 2)(11k + 37),


1 h (−2) −7k − 44
γ= + = ,
3 12A 45(k + 2)
 2
7k + 44
hγ,δ (x) = x − (x + 2)(2x + 1) + δ(x + 2)3 .
45(k + 2)
244 Vasile Cîrtoaje

Write the inequality f6 (x, 1, 1) ≥ Ahγ,δ (x) in the form


f (x) ≥ 0,
where
45(k + 2)2
f (x) = (x 2 + 1)(x − 1)2 (x − k)2 − hγ,δ (x).
4
We choose
2
δ=
9(k + 2)
to have f (−1/2) = 0. Next, we get
 2
7k + 44 2
hγ,δ (x) = x − (x + 2)(2x + 1) + (x + 2)3
45(k + 2) 9(k + 2)
4[5x 3 − (7k + 14)x 2 + 5(k − 1)x − 7k − 4]2
=
2025(k + 2)2
and
45 f (x) = 45(x 2 + 1)(x − 1)2 (x − k)2 − [5x 3 − (7k + 14)x 2 + 5(k − 1)x − 7k − 4]2
= (2x + 1)(x + 2)2 f1 (x),
where
f1 (x) = 10x 3 − 10(k + 2)x 2 − 2(k2 − 6k − 1)x − k2 − 14k − 4.
We need to show that f1 (x) ≤ 0 for x ≤ −1/2. Since 10x 3 ≤ −5x 2 , we have
f1 (x) ≤ −5x 2 − 10(k + 2)x 2 − 2(k2 − 6k − 1)x − k2 − 14k − 4
= −5(2k + 5)x 2 − 2(k2 − 6k − 1)x − k2 − 14k − 4.
So, we need to show that f2 (x) ≥ 0, where
f2 (x) = 5(2k + 5)x 2 + 2(k2 − 6k − 1)x + k2 + 14k + 4.
We have  2
k2 − 6k − 1 f3 (k)
f2 (x) = 5(2k + 5) x + + ,
10k + 25 5(2k + 5)
where
f3 (k) = 99 + 378k + 131k2 + 22k3 − k4
> 108k + 131k2 + 22k3 − k4
= k(27 − k)(4 + 5k + k2 ) ≥ 0.
Thus, the proof is completed.
The equality holds for x = y = z, for x/k = y = z (or any cyclic permutation) if
k = 0, and for x = 0 and y + z = 0 (or any cyclic permutation). If k = 0, then the
equality holds also for x = 0 and y = z (or any cyclic permutation).
Highest Coefficient Cancellation Method for Real Variables 245

P 3.25. If x, y, z are real numbers, then



(x 2 + yz)(x − y)(x − z)(x − 28 y)(x − 28z) + 167(x − y)2 ( y − z)2 (z − x)2 ≥ 0.

(Vasile C., 2014)

Solution. This is the inequality of the preceding P 3.24 for k = 28. Having in
view the proof of P 3.24, we only need to show that there exists a real δ such that
f (x) ≥ 0 for x ≤ −1/2, where

45(k + 2)2
f (x) = (x 2 + 1)(x − 1)2 (x − k)2 − hγ,δ (x),
4
= (x 2 + 1)(x − 1)2 (x − 28)2 − 10125hγ,δ (x),
−7k − 44 −8
γ= = ,
45(k + 2) 45
 2
hγ,δ (x) = x + γ(x + 2)(2x + 1) + δ(x + 2)3
 2
8
= x− (x + 2)(2x + 1) + δ(x + 2)3 .
45

Actually, there is a unique δ such that f (x) ≥ 0 for all x ≤ −1/2, namely
17
δ= .
2250
For this value of δ, we have

(x + 2)4 (211x 2 − 6956x − 3056)


f (x) = ,
500
where
211x 2 − 6956x − 3056 > −6956x − 3056 > 0.

The equality holds for x = y = z, for x/28 = y = z (or any cyclic permutation),
and for x = 0 and y + z = 0 (or any cyclic permutation).
Observation 1. Similarly, using
−7k − 44
γ=
45(k + 2)
and
17
δ= ,
75(k + 2)
therefore
 2
7k + 44 17
hγ,δ (x) = x − (x + 2)(2x + 1) + (x + 2)3 ,
45(k + 2) 75(k + 2)
246 Vasile Cîrtoaje

we can prove the following statement:


• Let x, y, z be real numbers. If −97/200 ≤ k ≤ 28, then
  
k2
(x 2 + yz)(x − y)(x − z)(x − k y)(x − kz) ≥ 1 + k − (x − y)2 ( y − z)2 (z − x)2 ,
4

with equality for x = y = z, for x/k = y = z (or any cyclic permutation) if k = 0,


and for x = 0 and y + z = 0 (or any cyclic permutation). If k = 0, then the equality
holds also for x = 0 and y = z (or any cyclic permutation).

P 3.26. If x, y, z are real numbers, then


 1
(x 2 + yz)(x 2 − y 2 )(x 2 − z 2 ) + (x − y)2 ( y − z)2 (z − x)2 ≥ 0.
4

(Vasile C., 2014)

Solution. Let 
f (x, y, z) = (x 2 + yz)(x 2 − y 2 )(x 2 − z 2 )
and
1
f6 (x, y, z) = f (x, y, z) + (x − y)2 ( y − z)2 (z − x)2 .
4
Since
(x − y)(x − z) = x 2 + 2 yz − q, (x + y)(x + z) = x 2 + q,
f (x, y, z) has the same highest coefficient A1 as

P1 (x, y, z) = (x 2 + yz)(x 2 + 2 yz)x 2 ,

that is, according to (3.1),

A1 = P1 (1, 1, 1) = 3(1 + 1)(1 + 2) = 18.

Therefore, f6 (x, y, z) has the highest coefficient

1 45
A = 18 + (−27) = .
4 4
Since
f6 (x, 1, 1) = (x 2 + 1)(x 2 − 1)2 ,
4(x − 1)4 (x + 1)2
f−1,−2 (x) = ,
81
Highest Coefficient Cancellation Method for Real Variables 247

5(x − 1)4 (x + 1)2


f6 (x, 1, 1) − Af−1,−2 (x) = (x 2 + 1)(x 2 − 1)2 −
9
2(x 2 − 1)2 (x + 2)(2x + 1)
= ,
9
we will apply Theorem 2 for

ξ → ∞, Eα,β = f−1,−2 , Fγ,δ = g−1,δ .

Notice that ξ → ∞ involves


  
−1 −1
= ,∞ ,  \  = −∞, .
2 2

The condition (a) in Theorem 2, namely f6 (x, 1, 1)−Af−1,−2 (x) ≥ 0 for x > −1/2,
is satisfied.
To provethe condition (b), namely f6 (x, 1, 1) ≥ Ag−1,δ (x) for x ≤ −1/2, we see
−1
that −2 ∈ −∞, and
2
1 2 2
f6 (0, y, z) = y 3 z 3 + ( y 2 − z 2 )( y 4 − z 4 ) + y z ( y − z)2 ,
4
f6 (0, 1, −1) = −1 + 0 + 1 = 0.
According to Proposition 1, the difference f6 (x, 1, 1) − Ag−1,δ (x) is zero for x = −2.
Therefore, we will use the inequality

1
x 2 + 1 ≥ (2x − 1)2 ,
5
which is an equality for x = −2, to get

f6 (x, 1, 1) ≥ F (x),

where the polynomial F (x) is a perfect square:

1
F (x) = (2x − 1)2 (x 2 − 1)2 .
5
Thus, the condition (b) in Theorem 2 is satisfied if

F (x) ≥ Ag−1,δ (x)

for x ≤ −1/2. We have

g−1,δ (x) = (x + 1)2 [x 2 + 5x + 8 + δ(x + 2)(x + 5)]2 ,

F (x) − Ag−1,δ (x) = (x + 1)2 g(x),


248 Vasile Cîrtoaje

where
(2x − 1)2 (x − 1)2 45 2
g(x) = − [x + 5x + 8 + δ(x + 2)(x + 5)]2
5 4
g1 (x)g2 (x)
= ,
20

g1 (x) = 2(2x − 1)(x − 1) − 15[x 2 + 5x + 8 + δ(x + 2)(x + 5)]


= −(x + 2)[(15δ + 11)x + 75δ + 59],

g2 (x) = 2(2x − 1)(x − 1) + 15[x 2 + 5x + 8 + δ(x + 2)(x + 5)].


Choosing
−37
δ= ,
45
g1 (x) is a perfect square,

4(x + 2)2
g1 (x) = ≥ 0,
3
and
4(5x 2 − 13x − 1)
g2 (x) = > 0.
3
The equality holds for x = y = z, for −x = y = z (or any cyclic permutation),
and for x = 0 and y + z = 0 (or any cyclic permutation).
Observation 1. Similarly, applying Theorem 2 for

−7k − 44
ξ → ∞, Eα,β = f k,−2 , Fγ,δ = g k,δ , δ= ,
45(k + 2)
we can prove the following generalization:
−5
• Let x, y, z be real numbers. If ≤ k ≤ 1, then
3
  
k2
(x 2 + yz)(x − y)(x − z)(x − k y)(x − kz) ≥ 1 + k − (x − y)2 ( y − z)2 (z − x)2 ,
4
with equality for x = y = z, for x/k = y = z (or any cyclic permutation) if k = 0,
and for x = 0 and y + z = 0 (or any cyclic permutation). If k = 0, then the equality
holds also for x = 0 and y = z (or any cyclic permutation).
We have

f6 (x, y, z) = (x 2 + yz)(x − y)(x − z)(x − k y)(x − kz)
 
k2
− 1+k− (x − y)2 ( y − z)2 (z − x)2 ,
4
Highest Coefficient Cancellation Method for Real Variables 249

45(k + 2)2
A= ,
4
  
−1 −1
= , ∞ ,  \  = −∞, ,
2 2
f (x, 1, 1) = (x 2 + 1)(x − 1)2 (x − k)2 ,
4(x − 1)4 (x − k)2 5(x − 1)4 (x − k)2
f k,−2 (x) = , Af k,−2 (x) = ,
81(2 + k)2 9
2(x − 1)2 (x − k)2 (x + 2)(2x + 1)
f6 (x, 1, 1) − Af k,−2 (x) = ,
9
1
F (x) = (2x − 1)2 (x − 1)2 (x − k)2 ,
5
 2 2
k x + k(k + 6)x − 8 δ(x + 2)(2k x + x + k − 4)
g k,δ (x) = (x − k)2 + ,
(k + 2)3 (k + 2)2
(x − k)2 g 2 (x)
Ag k,δ (x) = ,
180(k + 2)4

g(x) = 45k x 2 + 45k(k + 6)x − 360 − (7k + 44)(x + 2)(2k x + x + k − 4)


= −2(k + 2)[(7k + 11)x 2 − (5k + 22)x + 7k + 2],

(x − k)2 [(7k + 11)x 2 − (5k + 22)x + 7k + 2]2


Ag k,δ (x) = ,
45(k + 2)2
(x − k)2 g1 (x)g2 (x)
F (x) − Ag k,δ (x) = ,
45(k + 2)2
g1 (x) = (1 − k)(x + 2)2 ≥ 0,
g2 (x) = (13k + 23)x 2 − 2(7k + 20)x + 10k + 8,
 
−1 27(3k + 5)
g2 (x) ≥ g2 = ≥ 0.
2 4

Observation 2. By leaving out the simplifying inequality

1
x 2 + 1 ≥ (2x − 1)2 ,
5
we can prove the inequality from Observation 1 for the extended range

−17 13 + 9 5
≤k≤ ≈ 8.281.
10 4
We have
(x − k)2 g1 (x)
f6 (x, 1, 1) − Ag k,γ (x) = ,
45(k + 2)2
250 Vasile Cîrtoaje

g1 (x) = 45(k + 2)2 (x 2 + 1)(x − 1)2 − [(7k + 11)x 2 − (5k + 22)x + 7k + 2]2
= (x + 2)2 g2 (x),

g2 (x) = (59 + 26k − 4k2 )x 2 − 2(8 + k)(7 + 2k)x + 44 + 38k − k2 ,


59 + 26k − 4k2 ≥ 0,
 
−1 27(10k + 17)
g2 (x) ≥ g2 = ≥ 0.
2 4

−3
P 3.27. Let x, y, z be real numbers. If −2 ≤ k ≤ , then
2
  
k2
(x 2 + yz)(x − y)(x − z)(x − k y)(x − kz) ≥ 1 + k − (x − y)2 ( y − z)2 (z − x)2 ,
4
(Vasile C., 2014)
Solution. Let

f6 (x, y, z) = (x 2 + yz)(x − y)(x − z)(x − k y)(x − kz)
 
k2
− 1+k− (x − y)2 ( y − z)2 (z − x)2 .
4
As shown in the proof of P 3.21, f6 (x, y, z) has the highest coefficient

45(k + 2)2
A= .
4
In addition,
f6 (x, 1, 1) = (x 2 + 1)(x − 1)2 (x − k)2 .
For k = −2, we have A = 0. Since f6 (x, 1, 1) ≥ 0 for all real x, the conclusion
follows from Corollary 1. Consider further that
−3
−2 < k ≤ ,
2
and apply Theorem 2 for
−7k − 44
Eα,β = g k,β , β= , Fγ,δ = f k,∞ ,
45(k + 2)
4(k + 2)2
and η = 2k + 2 ∈ (−2, −1], which involves ξ = and
4k + 5
  
2 − 2k 2 − 2k
= , 2k + 2 ,  \  = −∞, ∪ [2k + 2, ∞).
5 + 4k 5 + 4k
Highest Coefficient Cancellation Method for Real Variables 251

Condition (a). We need to show that


−7k − 44
f (x, 1, 1) ≥ Ag k,β (x), β=
45(k + 2)

for x ∈ . Since
1
f (x, 1, 1) ≥ F (x) = (2x − 1)2 (x − 1)2 (x − k)2 ,
5
it suffices to show that
F (x) ≥ Ag k,β (x).
As shown at Observation 1 of the preceding P 3.26, we have

(x − k)2 g1 (x)g2 (x)


F (x) − Ag k,β (x) = ,
45(k + 2)2

g1 (x) = (1 − k)(x + 2)2 ≥ 0,


g2 (x) = (13k + 23)x 2 − 2(7k + 20)x + 10k + 8.
We claim that
g2 (x) ≥ g2 (2k + 2) ≥ 0.
Since

g2 (x) − g2 (2k + 2) = (x − 2k − 2)g3 (x), g3 (x) = (13k + 23)x + 26k2 + 58k + 6,

we need to show that g3 (x) ≤ 0. Since

g3 (x) = 13(k + 2)(2k + 3) + g4 (x) ≤ g4 (x), g4 (x) = (13k + 23)x − 3(11k + 24),

it suffices to show that g4 (x) ≤ 0. Because

−x −3(11k + 24)x
−1 ≤ , −3(11k + 24) ≤ ,
2k + 2 2k + 2
we get

3(11k + 24)x 13(k + 2)(2k − 1)x


g4 (x) ≤ (13k + 23)x − = ≤ 0.
2k + 2 2(k + 1)
Also, we have
g2 (2k + 2) = 2(k + 2)(26k2 + 32k + 5),
26k + 32k + 5 = 13k(2k + 3) + 7(−k) + 5 > 0,
2

therefore g2 (2k + 2) ≥ 0.
Condition (b). We need to show that

f (x, 1, 1) ≥ Af k,∞ (x)


252 Vasile Cîrtoaje

for 
2 − 2k
x ∈  \  = −∞, ∪ [2k + 2, ∞).
5 + 4k
Since
1
f (x, 1, 1) ≥ F (x) = (x − 1)4 (x − k)2 ,
2
it suffices to show that
F (x) ≥ Af k,∞ (x),
where
4(x − 1)4 (x − k)2
f k,∞ (x) = .
9(1 + 2k)2 (x + 2)2
We have
(x − 1)4 (x − k)2 g1 (x)
F (x) − Af k,∞ (x) = ,
2(1 + 2k)2 (x + 2)2
g1 (x) = (1 + 2k)2 (x + 2)2 − 10(k + 2)2 .
It suffices to show that g2 (x) ≥ 0, where

g2 (x) = (1 + 2k)2 (x + 2)2 − 16(k + 2)2 .


2 − 2k
Case 1: x ≤ . Since
5 + 4k
2 − 2k 6(k + 2)
x +2≤ +2= < 0,
5 + 4k 5 + 4k
the inequality g2 (x) ≥ 0 holds if

(1 + 2k)(x + 2) ≥ 4(k + 2).

Indeed,
6(k + 2)
(1 + 2k)(x + 2) − 4(k + 2) ≥ (1 + 2k) − 4(k + 2)
5 + 4k
−2(k + 2)(2k + 7)
= > 0.
5 + 4k
Case 2: x ≥ 2k + 2. Since
x + 2 ≥ 2(k + 2) > 0,
the inequality g2 (x) ≥ 0 holds if

−(1 + 2k)(x + 2) ≥ 4(k + 2).

Indeed,

−(1 + 2k)(x + 2) − 4(k + 2) ≥ −2(1 + 2k)(k + 2) − 4(k + 2)


= −2(k + 2)(2k + 3) ≥ 0.
Highest Coefficient Cancellation Method for Real Variables 253

The equality holds for x = y = z, for x/k = y = z (or any cyclic permutation) if
k = 0, and for x = 0 and y + z = 0 (or any cyclic permutation). If k = 0, then the
equality holds also for x = 0 and y = z (or any cyclic permutation).
Observation 1. From P 3.23-3.27 and the observations attached to them, the fol-
lowing generalization follows:
• Let x, y, z be real numbers. If −2 ≤ k ≤ 28, then
  
k2
(x 2 + yz)(x − y)(x − z)(x − k y)(x − kz) ≥ 1 + k − (x − y)2 ( y − z)2 (z − x)2 ,
4
with equality for x = y = z, for x/k = y = z (or any cyclic permutation) if k = 0,
and for x = 0 and y + z = 0 (or any cyclic permutation). If k = 0, then the equality
holds also for x = 0 and y = z (or any cyclic permutation).
Note that the coefficient of the product (x − y)2 ( y −z)2 (z− x)2 is the best possible.
Setting x = k, y = 1 + t and z = 1 − t, the inequality

(x 2 + yz)(x − y)(x − z)(x − k y)(x − kz) ≥ δk (x − y)2 ( y − z)2 (z − x)2

turns into
A(k, δk )t 6 + B(k, δk )t 4 + C(k, δk )t 2 ≥ 0,
where
A(k, δk ) = −4δk + 4 + 4k − k2 .
From the necessary condition A(k, δk ) ≥ 0, we get

k2
δk ≤ 1 + k − .
4

Observation 2. Substituting k − 1 for k, and using then the identity



(x 2 + yz)(x − y)(x − z)[x − (k − 1) y][x − (k − 1)z] =

= (x 2 + yz)(x − y)(x − z)(x − k y + z)(x − kz + y) + 2k(x − y)2 ( y − z)2 (z − x)2 ,
the statement from Observation 1 becomes as follows:
• Let x, y, z be real numbers. If −1 ≤ k ≤ 29, then
 (k + 1)2
(x 2 + yz)(x− y)(x−z)(x−k y+z)(x−kz+ y)+ (x− y)2 ( y−z)2 (z−x)2 ≥ 0,
4
with equality for x = y = z, for x/(k − 1) = y = z (or any cyclic permutation) if
k = 1, and for x = 0 and y + z = 0 (or any cyclic permutation). If k = 1, then the
equality holds also for x = 0 and y = z (or any cyclic permutation).
254 Vasile Cîrtoaje

P 3.28. Let x, y, z be real numbers. If −5 ≤ k ≤ −2 and

k4 − 8k3 − 7k2 − 20k − 20


δk = ,
4(k − 1)2

then

(x 2 + yz)(x − y)(x − z)(x − k y)(x − kz) + δk (x − y)2 ( y − z)2 (z − x)2 ≥ 0.

(Vasile C., 2013)

Solution. Denote

f (x, y, z) = (x 2 + yz)(x − y)(x − z)(x − k y)(x − kz),

and write the inequality as f6 (x, y, z) ≥ 0, where

f6 (x, y, z) = f (x, y, z) + δk (x − y)2 ( y − z)2 (z − x)2 .

As shown at P 3.21, f (x, y, z) has the highest coefficient A1 = 18(k2 + k + 1). As a


consequence, f6 (x, y, z) has the highest coefficient

9(k + 2)2 (5k2 − 4k + 17)


A = A1 − 27δk = .
4(k − 1)2

In addition,
f6 (x, 1, 1) = (x 2 + 1)(x − 1)2 (x − k)2 .
For k = −2, we have A = 0. Since f6 (x, 1, 1) ≥ 0 for any real x, the conclusion
follows from Corollary 1.
Consider further that −5 ≤ k < −2. Since

4(x − 1)4 (x − k)2


f k,−2 (x) =
81(k + 2)2

and

(x − 1)2 (x − k)2 [(k − 4)x + (2k + 1)][(2k + 1)x + k − 4]


f6 (x, 1, 1) − Af k,−2 (x) = ,
9(k − 1)2

(k + 2)2
we apply Theorem 2 for η = k, which involves ξ = ,
2k + 1
   
4−k 4−k
 = k, ,  \  = (−∞, k] ∪ ,∞ ,
2k + 1 2k + 1

and for
Eα,β = f k,−2 , Eγ,δ = f k,∞ .
Highest Coefficient Cancellation Method for Real Variables 255

The condition (a), namely f6 (x, 1, 1) ≥ Af k,−2 (x) for x ∈ , is satisfied since
4−k 3(k − 1)(k + 5)
(k − 4)x + (2k + 1) > (k − 4) · + (2k + 1) = ≥ 0,
2k + 1 2k + 1
4−k
(2k + 1)x + k − 4 > (2k + 1) · + k − 4 = 0.
2k + 1
The condition (b) is satisfied if f6 (x, 1, 1) ≥ Af k,∞ (x) for x ∈  \ , where

4(x − 1)4 (x − k)2


f k,∞ (x) = .
9(2k + 1)2 (x + 2)2
We have
(x − 1)2 (x − k)2 f (x)
f6 (x, 1, 1) − Af k,∞ (x) = ,
(k − 1)2 (2k + 1)2 (x + 2)2
where

f (x) = (k − 1)2 (2k + 1)2 (x 2 + 1)(x + 2)2 − (k + 2)2 (5k2 − 4k + 17)(x − 1)2 .

Thus, we need to show that f (x) ≥ 0 for


 
4−k
x ∈ (−∞, k] ∪ ,∞ , −5 ≤ k < −2.
2k + 1
The inequality f (x) ≥ 0 is equivalent to
 
4−k
h(x) ≤ h ,
2k + 1
where
(x − 1)2
h(x) = .
(x 2 + 1)(x + 2)2
From
−2(x − 1)(x − 3)(x 2 + x + 1)
h (x) = ,
(x 2 + 1)2 (x + 2)3
it follows that h is increasing on (−∞, −2) ∪ [1, 3] and decreasing on (−2, 1] ∪
[3, ∞).
Case 1: x ≤ k. Since h is increasing on [−5, −2) and k ∈ [−5, −2), we have
h(x) ≤ h(k). Thus, it suffices to show that
 
4−k
h(k) ≤ h .
2k + 1
We have
 
4−k (k − 1)2 (k − 1)2 (2k + 1)2
h(k) − = 2 −
2k + 1 (k + 1)(k + 2)2 (k + 2)2 (5k2 − 4k + 17)
4(1 − k)(k2 + 2)
= 2 ≤ 0.
(k + 1)(k + 2)(5k2 − 4k + 17)
256 Vasile Cîrtoaje

4−k
Case 2: x ≥ . Since
2k + 1
4−k
∈ (−2, −1],
2k + 1

4−k
h is decreasing on , 1 ∪ [3, ∞) and increasing on [1, 3]. Therefore, we
 2k+ 1  
4−k 4−k
have h(x) ≤ h if and only if h(3) ≤ h . On the other hand, since
2k + 1 2k + 1
4−k
h is decreasing on , −1 , we have
2k + 1
 
4−k
h ≥ h(−1).
2k + 1
Therefore, it suffices to show that h(3) ≤ h(−1). Since
2
h(−1) = 2, h(3) = ,
125
the conclusion follows.
The equality holds for x = y = z, and for x/k = y = z (or any cyclic permutation).
Observation 1. The coefficient δk of the product (x − y)2 ( y − z)2 (z − x)2 in the
original inequality is the best possible. Setting x = k, y = 1 + t and z = 1 − t, the
original inequality turns into

A(k, δk )t 6 + B(k, δk )t 4 + C(k, δk )t 2 ≥ 0,

where

C(k, δk ) = (k − 1)2 [4(k − 1)2 δk − k4 + 8k3 + 7k2 + 20k + 20].

From the necessary conditions C(k, δk ) ≥ 0, we get

k4 − 8k3 − 7k2 − 20k − 20


δk ≥ .
4(k − 1)2

Observation 2. Substituting k − 1 for k and using then the identity



(x 2 + yz)(x − y)(x − z)[x − (k − 1) y][x − (k − 1)z] =

= (x 2 + yz)(x − y)(x − z)(x − k y + z)(x − kz + y) + 2k(x − y)2 ( y − z)2 (z − x)2 ,
we get the following statement:
• Let x, y, z be real numbers, and let

(k + 1)2 (k2 − 6k + 2)
δk∗ = .
4(k − 2)2
Highest Coefficient Cancellation Method for Real Variables 257

If −4 ≤ k ≤ −1, then

(x 2 + yz)(x − y)(x − z)(x − k y + z)(x − kz + y) + δk∗ (x − y)2 ( y − z)2 (z − x)2 ≥ 0,

with equality for x = y = z, and for x/(k − 1) = y = z (or any cyclic permutation).

P 3.29. If x, y, z are real numbers, then


 15(x − y)2 ( y − z)2 (z − x)2
(x 2 + yz)(x + y)(x + z) ≥ .
32(x 2 + y 2 + z 2 )
(Vasile C., 2013)

Solution. Write the inequality as f6 (x, y, z) ≥ 0, where


 15
f6 (x, y, z) = (x 2 + y 2 + z 2 ) (x 2 + yz)(x + y)(x + z) − (x − y)2 ( y − z)2 (z − x)2 .
32
The function f6 (x, y, z) has the highest coefficient
15 405
A=− (−27) = .
32 32
Since
f6 (x, 1, 1) = (x 2 + 2)(x + 1)2 (x 2 + 5),
we apply Corollary 2 for

Fγ,δ (x) = g−1,δ (x) = (x + 1)2 [x 2 + 5x + 8 + δ(x + 2)(x + 5)]2 .

So, we only need to show that there exists a real number δ such that

f6 (x, 1, 1) − Ag−1,δ (x) ≥ 0

for all real x. We have


(x + 1)2 f (x)
f6 (x, 1, 1) − Ag−1,δ (x) = ,
32
where

f (x) = 32(x 2 + 2)(x 2 + 5) − 405[x 2 + 5x + 8 + δ(x + 2)(x + 5)]2 .

Since
f (−5) = 0,
the condition f (−5) = 0 is necessary to have f (x) ≥ 0 in the vicinity of x = −5.
This condition involves
−59
δ=
81
258 Vasile Cîrtoaje

and
4g(x)
f (x) = ,
81
where

g(x) = 648(x 2 + 2)(x 2 + 5) − 5(11x 2 − 4x + 29)2


= 43x 4 + 440x 3 + 1266x 2 + 1160x + 2275
= (x + 5)2 (43x 2 + 10x + 91) ≥ 0.

The equality holds for −x = y = z (or any cyclic permutation).

P 3.30. If x, y, z are real numbers, then


 7
(x − y)(x − z)(x 2 + y 2 )(x 2 + z 2 ) ≥ (x − y)2 ( y − z)2 (z − x)2 .
4
(Vasile C., 2012)
Solution. Let

f (x, y, z) = 4 (x − y)(x − z)(x 2 + y 2 )(x 2 + z 2 )

and
f6 (x, y, z) = f (x, y, z) − 7(x − y)2 ( y − z)2 (z − x)2 .
Since (x − y)(x − z) = x 2 + 2 yz − q and

x 2 + y 2 = p2 − 2q − z 2 , x 2 + z 2 = p2 − 2q − y 2 ,

f (x, y, z) has the same highest coefficient A1 as



P2 (x, y, z) = 4 (x 2 + 2 yz)(−z 2 )(− y 2 ),

that is, according to (3.2),

A1 = P2 (1, 1, 1) = 36.

Therefore, f6 (x, y, z) has the highest coefficient

A = 36 − 7(−27) = 225.

Since
f6 (x, 1, 1) = (x − 1)2 (x 2 + 1)2 ,
we will apply Corollary 2 for

4(x − 1)4 (x − γ)2


Fγ,δ (x) = fγ,−2 (x) = .
81(2 + γ)2
Highest Coefficient Cancellation Method for Real Variables 259

Thus, we need to show that there exists a real γ such that

f6 (x, 1, 1) − Afγ,−2 (x) ≥ 0

for x ∈ . We have
(x − 1)2 g(x)
f6 (x, 1, 1) − Afγ,−2 (x) = ,
9(2 + γ)2
where

g(x) = 9(γ + 2)2 (x 2 + 1)2 − 25(x − 1)2 (x − γ)2


= (x + 2)[(3γ + 1)x + 3 − γ][(3γ + 11)x 2 − 5(γ + 1)x + 8γ + 6].

Choosing γ = 1/7, we get

100(x + 2)2 (8x 2 − 4x + 5)


g(x) = ≥ 0.
49

The equality holds for x = y = z, and also for x = 0 and y + z = 0 (or any cyclic
permutation).

P 3.31. If x, y, z are real numbers such that x y + yz + z x ≥ 0, then


 15
(x − y)(x − z)(x 2 + y 2 )(x 2 + z 2 ) ≥ (x − y)2 ( y − z)2 (z − x)2 .
4
(Vasile C., 2012)

Solution. Let

f (x, y, z) = 4 (x − y)(x − z)(x 2 + y 2 )(x 2 + z 2 )

and
f6 (x, y, z) = f (x, y, z) − 15(x − y)2 ( y − z)2 (z − x)2 .
As shown at the preceding P 3.30, f (x, y, z) has the highest coefficient A1 = 36,
therefore f6 (x, y, z) has the highest coefficient

A = A1 − 15(−27) = 441.

Since
f6 (x, 1, 1) = 4(x − 1)2 (x 2 + 1)2 ,
we will apply Corollary 2 for

4(x − 1)4 (x − γ)2


Fγ,δ (x) = fγ,−2 (x) = .
81(2 + γ)2
260 Vasile Cîrtoaje

Having in view Remark 5, we need to show that there exists a real γ such that

f6 (x, 1, 1) − Afγ,−2 (x) ≥ 0

for 2x + 1 ≥ 0. We have

4(x − 1)2 g(x)


f6 (x, 1, 1) − Afγ,−2 (x) = ,
9(2 + γ)2

where
g(x) = 9(2 + γ)2 (x 2 + 1)2 − 49(x − 1)2 (x − γ)2 .
Choosing γ = 1/3, which involves g(−1/2) = 0, we get

49  
g(x) = 9(x 2 + 1)2 − (x − 1)2 (3x − 1)2
9
196(2x + 1)(3x 2 − 2x + 2)
= ≥ 0.
9

The equality holds for x = y = z, and also when x, y, z are proportional to the
roots of the equation
7t 3 − 21t 2 + 18 = 0.

Observation. The last equality condition follows from the necessary condition
f¯6 (x, y, z) = 0, where, according to (3.11),
 2  
γ+8 2 25 2 3 2
f¯6 (x, y, z) = r − pq + p3 = r − pq + p .
9γ + 18 9γ + 18 63 21

Moreover, if p = 3 and q = 0, then the condition f¯6 (x, y, z) = 0 involves r = −18/7.

P 3.32. If x, y, z are real numbers, then


 3
(x − y)(x − z)(x 2 + x y + y 2 )(x 2 + xz + z 2 ) ≥ (x − y)2 ( y − z)2 (z − x)2 .
4
(Vasile C., 2012)

Solution. Let

f (x, y, z) = (x − y)(x − z)(x 2 + x y + y 2 )(x 2 + xz + z 2 )

and
3
f6 (x, y, z) = f (x, y, z) − (x − y)2 ( y − z)2 (z − x)2 .
4
Highest Coefficient Cancellation Method for Real Variables 261

Since (x − y)(x − z) = x 2 + 2 yz − q and

x 2 + x y + y 2 = p2 − 2q − z 2 + x y, x 2 + xz + z 2 = p2 − 2q − y 2 + xz,

f (x, y, z) has the same highest coefficient A1 as



P2 (x, y, z) = (x 2 + 2 yz)(z 2 − x y)( y 2 − xz),

that is, according to (3.2),


A1 = P2 (1, 1, 1) = 0.
Therefore, f6 (x, y, z) has the highest coefficient
3 81
A = − (−27) = .
4 4
Since
f6 (x, 1, 1) = (x − 1)2 (x 2 + x + 1)2 ,
we will apply Corollary 2 for

4(x − 1)4 (x − γ)2


Fγ,δ (x) = fγ,−2 (x) = .
81(2 + γ)2

Thus, we need to show that there exists a real γ such that

f6 (x, 1, 1) − Afγ,−2 (x) ≥ 0

for x ∈ . We have
(x − 1)2 g(x)
f6 (x, 1, 1) − Afγ,−2 (x) = ,
(2 + γ)2

where

g(x) = (γ + 2)2 (x 2 + x + 1)2 − (x − 1)2 (x − γ)2


= (x + 2)[(γ + 1)x + 1][(γ + 3)x 2 + x + 2γ + 2].

Choosing γ = −1/2, we get

(x + 2)2 (5x 2 + 2x + 2)
g(x) = ≥ 0.
4

The equality holds for x = y = z, and also for x = 0 and y + z = 0 (or any cyclic
permutation).
Observation 1. Similarly, applying Corollary 2 for

1 − 4k
Fγ,δ = fγ,−2 , γ= ,
7−k
262 Vasile Cîrtoaje

we can prove the following generalization:



8
• Let x, y, z be real numbers. If k ∈ −2, , then
5
  
7
(x − y)(x − z)(x 2 + k x y + y 2 )(x 2 + k xz + z 2 ) ≥ − k (x − y)2 ( y − z)2 (z − x)2 ,
4
with equality for x = y = z, and also for x = 0 and y + z = 0 (or any cyclic
permutation).
For

f6 (x, y, z) = (x − y)(x − z)(x 2 + k x y + y 2 )(x 2 + k xz + z 2 )
 
7
− − k (x − y)2 ( y − z)2 (z − x)2 ,
4
we have
9
A = (2k − 5)2 ,
4
f6 (x, 1, 1) = (x − 1)2 (x 2 + k x + 1)2 ,
(k + 2)(x − 1)2 (x + 2)2 h(x)
f6 (x, 1, 1) − Afγ,−2 (x) = ,
81
where

h(x) = (16 − k)x 2 + 2(7k − 4)x + 10 − 4k


 
7k − 4 2 9(2 + k)(8 − 5k)
= (16 − k) x + + ≥ 0.
16 − k 16 − k

Observation 2. Actually, the inequality from Observation 1 holds for k ∈ [−2, 5/2].
For k = 5/2 and k = 2, we get the inequalities in P 3.1 and P 3.47, respectively.

P 3.33. If x, y, z are real numbers such that x y + yz + z x ≥ 0, then



(x − y)(x − z)(x 2 + x y + y 2 )(x 2 + xz + z 2 ) ≥ 3(x − y)2 ( y − z)2 (z − x)2 .

(Vasile C., 2012)

Solution. Let

f (x, y, z) = (x − y)(x − z)(x 2 + x y + y 2 )(x 2 + xz + z 2 )

and
f6 (x, y, z) = f (x, y, z) − 3(x − y)2 ( y − z)2 (z − x)2 .
Highest Coefficient Cancellation Method for Real Variables 263

As shown at the preceding P 3.32, f (x, y, z) has the highest coefficient A1 = 0.


Therefore, f6 (x, y, z) has the highest coefficient

A = 0 − 3(−27) = 81.

On the other hand,


f6 (x, 1, 1) = (x − 1)2 (x 2 + x + 1)2 .
We will apply Corollary 2 for

(x − 1)4 x 2
Fγ,δ (x) = f0,−2 (x) = .
81
Having in view Remark 5, we need to show that

f6 (x, 1, 1) − Af0,−2 (x) ≥ 0

for 2x + 1 ≥ 0. Indeed, we have

f6 (x, 1, 1) − Af0,−2 (x) = (x − 1)2 (x 2 + x + 1)2 − (x − 1)4 x 2


= (x − 1)2 (2x + 1)(2x 2 + 1) ≥ 0.

The equality holds for x = y = z, and also when x, y, z are proportional to the
roots of the equation
t 3 − 3t 2 + 3 = 0.

Observation 1. The last equality condition follows from the necessary condition
f¯6 (x, y, z) = 0, where
 2  2
γ+8 2 4 1
f¯6 (x, y, z) = r − pq + p3 = r − pq + p3
9γ + 18 9γ + 18 9 9
(see (3.11) from Remark 1). Moreover, if p = 3 and q = 0, then the condition
f¯6 (x, y, z) = 0 involves r = −3.
Observation 2. Similarly, applying Corollary 2 for
1−k
Fγ,δ = fγ,−2 , γ= ,
3
we can prove the following generalization:
• Let x, y, z be real numbers such that x y + yz + z x ≥ 0. If k ∈ [−2, 5/2], then
 (3 − k)(5 + k)
(x− y)(x−z)(x 2 +k x y+ y 2 )(x 2 +k xz+z 2 ) ≥ (x− y)2 ( y−z)2 (z−x)2 ,
4
with equality for x = y = z, and also when x, y, z are proportional to the roots of the
equation
(7 − k)t 3 − 3(7 − k)t 2 + 18 = 0.
264 Vasile Cîrtoaje

For

f6 (x, y, z) = (x − y)(x − z)(x 2 + k x y + y 2 )(x 2 + k xz + z 2 )
(3 − k)(5 + k)
− (x − y)2 ( y − z)2 (z − x)2 ,
4
we have
9
A = (k − 7)2 ,
4
f6 (x, 1, 1) = (x − 1)2 (x 2 + k x + 1)2 ,
and, for 2x + 1 ≥ 0,
1
f6 (x, 1, 1) − Afγ,−2 (x) = (k + 2)(x − 1)2 (2x + 1)[6x 2 + 4(k − 1)x + 4 − k] ≥ 0.
9

P 3.34. If x, y, z are real numbers, then

3
(x 2 + y 2 )( y 2 + z 2 )(z 2 + x 2 ) ≥ 8x 2 y 2 z 2 + (x − y)2 ( y − z)2 (z − x)2 .
8
(Vasile C., 2011)

Solution. Write the inequality as f6 (x, y, z) ≥ 0, where


( 3
f6 (x, y, z) = ( y 2 + z 2 ) − 8x 2 y 2 z 2 − (x − y)2 ( y − z)2 (z − x)2 .
8
Since ( (
( y 2 + z2) = (−x 2 + p2 − 2q),
the polynomial f6 (x, y, z) has the same highest coefficient as

3
−x 2 y 2 z 2 − 8x 2 y 2 z 2 − (x − y)2 ( y − z)2 (z − x)2 ,
8
that is
81 9
A = −9 + = .
8 8
Since
f6 (x, 1, 1) = 2(x 2 + 1)2 − 8x 2 = 2(x 2 − 1)2 ,
4(x − 1)4 (x + 1)2
f−1,∞ (x) = ,
9(x + 2)2
3(x 2 − 1)2 (x + 1)(x + 5)
f6 (x, 1, 1) − Af−1,∞ (x) = ,
2(x + 2)2
Highest Coefficient Cancellation Method for Real Variables 265

we apply Theorem 2 for


η = −5, ξ = −1,
 = (−5, −1),  \  = (−∞, −5] ∪ [−1, ∞),
and for
Eα,β = f−1,−2 , Fγ,δ = f−1,∞ .

The condition (b), namely f6 (x, 1, 1) − Af−1,∞ (x) ≥ 0 for x ∈ (−∞, −5] ∪
[−1, ∞), is satisfied.
The condition (a) is satisfied if f6 (x, 1, 1) − Af−1,−2 (x) ≥ 0 for x ∈ (−5, −1). We
have
4(x − 1)4 (x + 1)2
f−1,−2 (x) = ,
81
(x 2 − 1)2 (x + 5)(7 − x)
f6 (x, 1, 1) − Af−1,−2 (x) = ≥ 0.
18
The equality holds for x = y = z, and for −x = y = z (or any cyclic permuta-
tion).

P 3.35. If x, y, z are real numbers, then


15
(x 2 +2 y 2 +2z 2 )( y 2 +2z 2 +2x 2 )(z 2 +2x 2 +2 y 2 ) ≥ 125x 2 y 2 z 2 + (x− y)2 ( y−z)2 (z−x)2 .
2
(Vasile C., 2011)

Solution. Write the inequality as f6 (x, y, z) ≥ 0, where


( 15
f6 (x, y, z) = (x 2 + 2 y 2 + 2z 2 ) − 125x 2 y 2 z 2 − (x − y)2 ( y − z)2 (z − x)2 .
2
Since ( (
(x 2 + 2 y 2 + 2z 2 ) = (−x 2 + 2p2 − 4q),
the polynomial f6 (x, y, z) has the same highest coefficient as
15
−x 2 y 2 z 2 − 125x 2 y 2 z 2 − (x − y)2 ( y − z)2 (z − x)2 ,
2
that is
15 · 27 153
A = −126 + = .
2 2
Since
f6 (x, 1, 1) = (x 2 + 4)(2x 2 + 3)2 − 125x 2 = 4(x 2 − 1)2 (x 2 + 9),
4(x − 1)4 (x + 1)2
f−1,−2 (x) = ,
81
266 Vasile Cîrtoaje

2(x 2 − 1)2 (x + 5)(x + 29)


f6 (x, 1, 1) − Af−1,−2 (x) = ,
9
we apply Theorem 2 for
η = −5, ξ = −1,
 = (−5, −1),  \  = (−∞, −5] ∪ [−1, ∞),
and for
Eα,β = f−1,−2 , Fγ,δ = f−1,−1 .

The condition (a), namely f6 (x, 1, 1) − Af−1,−2 (x) ≥ 0 for x ∈ (−5, −1), is satis-
fied.
The condition (b) is satisfied if f6 (x, 1, 1) − Af−1,−1 (x) ≥ 0 for x ∈ (−∞, −5] ∪
[−1, ∞). We have
(x − 1)4 (x + 1)4
f−1,−1 (x) = ,
36(x + 2)2
(x 2 − 1)2 g(x)
f6 (x, 1, 1) − Af−1,−1 (x) = ,
8(x + 2)2
where

g(x) = 32(x 2 + 9)(x + 2)2 − 17(x 2 − 1)2


= 15x 4 + 128x 3 + 450x 2 + 1152x + 1135
= (x + 5)g1 (x), g1 (x) = 15x 3 + 53x 2 + 185x + 227.

We need to show that g1 (x) ≤ 0 for x ≤ −5, and g1 (x) ≥ 0 for x ≥ −1. Indeed, if
x ≤ −5, then

g1 (x) < 5(3x 3 + 11x 2 + 37x + 54) = 5(x + 2)(3x 2 + 5x + 27) < 0,

and if x ≥ −1, then

g1 (x) > 15x 2 (x + 1) + 185(x + 1) ≥ 0.

The equality holds for x = y = z, and for −x = y = z (or any cyclic permuta-
tion).

P 3.36. If x, y, z are real numbers, then

15
(2x 2 + y 2 +z 2 )(2 y 2 +z 2 + x 2 )(2z 2 + x 2 + y 2 ) ≥ 64x 2 y 2 z 2 + (x − y)2 ( y −z)2 (z− x)2 .
4
(Vasile C., 2011)
Highest Coefficient Cancellation Method for Real Variables 267

Solution. Write the inequality as f6 (x, y, z) ≥ 0, where


( 15
f6 (x, y, z) = (2x 2 + y 2 + z 2 ) − 64x 2 y 2 z 2 − (x − y)2 ( y − z)2 (z − x)2 .
4
Since ( (
(2x 2 + y 2 + z 2 ) = (x 2 + p2 − 2q),
the polynomial f6 (x, y, z) has the same highest coefficient as
15
x 2 y 2 z 2 − 64x 2 y 2 z 2 − (x − y)2 ( y − z)2 (z − x)2 ,
4
that is
15 · 27 153
A = −63 + = .
4 4
Since
f6 (x, 1, 1) = 2(x 2 + 1)(x 2 + 3)2 − 64x 2 = 2(x 2 − 1)2 (x 2 + 9),
4(x − 1)4 (x + 1)2
f−1,−2 (x) = ,
81
(x − 1) (x + 5)(x + 29)
2 2
f6 (x, 1, 1) − Af−1,−2 (x) = ,
9
we apply Theorem 2 for
η = −5, ξ = −1,
 = (−5, −1),  \  = (−∞, −5] ∪ [−1, ∞),
and for
Eα,β = f−1,−2 , Fγ,δ = f−1,−1 .

The condition (a), namely f6 (x, 1, 1) − Af−1,−2 (x) ≥ 0 for x ∈ (−5, −1), is satis-
fied.
The condition (b) is satisfied if f6 (x, 1, 1) − Af−1,−1 (x) ≥ 0 for x ∈ (−∞, −5] ∪
[−1, ∞). We have
(x − 1)4 (x + 1)4
f−1,−1 (x) = ,
36(x + 2)2
(x 2 − 1)2 (x + 5)g1 (x)
f6 (x, 1, 1) − Af−1,−1 (x) = ≥ 0,
16(x + 2)2
where
g1 (x) = 15x 3 + 53x 2 + 185x + 227.
As shown at the preceding P 3.35, we have (x + 5)g1 (x) ≥ 0.
The equality holds for x = y = z, and for −x = y = z (or any cyclic permuta-
tion).
Observation 1. The inequalities in P 3.34, P 3.35 and P 3.36 are particular cases
of the following statement:
268 Vasile Cîrtoaje

• Let x, y, z be real numbers. If k ≥ 0, then


( 3(k + 2)(2k + 1)
(k x 2 + y 2 + z 2 ) ≥ (k + 2)3 x 2 y 2 z 2 + (x − y)2 ( y − z)2 (z − x)2 ,
16
with equality for x = y = z, and for −x = y = z (or any cyclic permutation).

We have
9(2k2 + 29k + 2)
A= ,
16
f6 (x, 1, 1) = (x 2 − 1)2 [k x 2 + 2(k + 1)2 ],
(x 2 − 1)2 (x + 5)g(x)
f6 (x, 1, 1) − Af−1,−2 (x) = ,
36
g(x) = (−2k2 + 7k − 2)x + 14k2 + 23k + 14.

Observation 2. The coefficient of the product (x − y)2 ( y − z)2 (z − x)2 in the


inequality from Observation 1 is the best possible. Setting x = 1, y = 1 + t and
z = 1 − t, the inequality
(
(k x 2 + y 2 + z 2 ) ≥ (k + 2)3 x 2 y 2 z 2 + δk (x − y)2 ( y − z)2 (z − x)2

turns into
A(k, δk )t 6 + B(k, δk )t 4 + 4C(k, δk )t 2 ≥ 0,
where
C(k, δk ) = 3(k + 2)(2k + 1) − 16δk .
From the necessary condition C(k, δk ) ≥ 0, we get

3(k + 2)(2k + 1)
δk ≤ .
16

P 3.37. If x, y, z are real numbers, then

8(x 2 + x y + y 2 )( y 2 + yz + z 2 )(z 2 + z x + x 2 ) ≥ 3(x 2 + y 2 )( y 2 + z 2 )(z 2 + x 2 ).

(Vasile C., 2013)

Solution. Write the inequality as f6 (x, y, z) ≥ 0, where


( (
f6 (x, y, z) = 8 ( y 2 + yz + z 2 ) − 3 ( y 2 + z 2 ).

Since ( (
f6 (x, y, z) = 8 (p2 − 2q + yz − x 2 ) − 3 (p2 − 2q − x 2 ),
Highest Coefficient Cancellation Method for Real Variables 269

f6 (x, y, z) has the same highest coefficient as


( (
f (x, y, z) = 8 ( yz − x 2 ) − 3 (−x 2 ),

that is, according to (3.3),


A = f (1, 1, 1) = 3.
Since

f6 (x, 1, 1) = 24(x 2 + x + 1)2 − 6(x 2 + 1)2 = 6(x + 1)2 (3x 2 + 2x + 3),

we apply Corollary 2 for

Fγ,δ (x) = g−1,δ (x) = (x + 1)2 [x 2 + 5x + 8 + δ(x + 2)(x + 5)]2 .

So, we only need to show that there exists a real number δ such that

f6 (x, 1, 1) − Ag−1,δ (x) ≥ 0

for all real x. We have

f6 (x, 1, 1) − Ag−1,δ (x) = 3(x + 1)2 g(x),

where
g(x) = 2(3x 2 + 2x + 3) − [x 2 + 5x + 8 + δ(x + 2)(x + 5]2 .
Choosing δ = −1, we get

h(x) = 2(3x 2 + 2x + 3) − 4(x + 1)2 = 2(x − 1)2 ≥ 0.

The equality holds for −x = y = z (or any cyclic permutation).

P 3.38. If x, y, z are nonnegative real numbers, then



(16x 2 + 3 yz)(x − y)(x − z)(x − 4 y)(x − 4z) + 52(x − y)2 ( y − z)2 (z − x)2 ≥ 0.

(Vasile C., 2013)

Solution. Write the inequality as f6 (x, y, z) ≥ 0, where



f6 (x, y, z) = (16x 2 +3 yz)(x− y)(x−z)(x−4 y)(x−4z)+52(x− y)2 ( y−z)2 (z−x)2 .

Since

(x − y)(x − z) = x 2 + 2 yz − q, (x − 4 y)(x − 4z) = x 2 + 20 yz − 4q,


270 Vasile Cîrtoaje

f6 (x, y, z) has the same highest coefficient A as



(16x 2 + 3 yz)(x 2 + 2 yz)(x 2 + 20 yz) + 52(x − y)2 ( y − z)2 (z − x)2 ,

that is, according to (3.1),

A = 3(16 + 3)(1 + 2)(1 + 20) − 27 · 52 = 3 · 729.

On the other hand,

f6 (x, 1, 1) = (16x 2 + 3)(x − 1)2 (x − 4)2 ,

4(x − 1)4 (x − 4)2 (x − β)2


f4,β = ,
729β 2 (x + 2)2
(x − 1)2 (x − 4)2 f (x)
f6 (x, 1, 1) − Af4,β (x) = ,
β 2 (x + 2)2
f (x) = β 2 (16x 2 + 3)(x + 2)2 − 12(x − 1)2 (x − β)2 .
Since f (0) = 0, we chose β = −2/3 to have f (0) = 0 and

4x 2 (100 + 82x − 11x 2 ) 4x 2 (48 + 82x − 11x 2 ) 4x 2 (8 − x)(6 + 11x)


f (x) = ≥ = .
9 9 9
Thus, we apply Theorem 2 for η = 8, which involves ξ = 100/17,
  
−4 −4
= ,8 ,  \  = −∞, ∪ [8, ∞),
17 17
and for
Eα,β = f4,−2/3 , Fγ,δ = f2,−2 .
The condition (a), namely f6 (x, 1, 1) ≥ Af4,−2/3 (x) for x ∈ (−4/17, 8), is satisfied
because f (x) ≥ 0.
The condition (b), namely f6 (x, 1, 1) ≥ Af2,−2 (x) for x ∈ (−∞, −4/17]∪[8, ∞),
where
(x − 1)4 (x − 2)2
f2,−2 (x) = ,
324
is equivalent to (x − 1)2 f (x) ≥ 0, where
27
f (x) = (16x 2 + 3)(x − 4)2 − (x − 1)2 (x − 2)2 .
4
To show that f (x) ≥ 0, we use the Cauchy-Schwarz inequality

(4 + 3)(16x 2 + 3) ≥ (−8x + 3)2 .

Thus, it suffices to show that


27
(8x − 3)2 (x − 4)2 ≥ (x − 1)2 (x − 2)2 .
28
Highest Coefficient Cancellation Method for Real Variables 271

This inequality is true if

(8x − 3)(x − 4) ≥ (x − 1)(x − 2).

Indeed,

(8x − 3)(x − 4) − (x − 1)(x − 2) = 7x 2 − 32x + 10 > 6x 2 − 32x + 10


= 2(x − 5)(3x − 1) > 0.

The equality holds for x = y = z, and for x/4 = y = z (or any cyclic permutation).
Observation. The coefficient of the product (x − y)2 ( y − z)2 (z − x)2 is the best
possible. Setting x = 4, y = 1 + t and z = 1 − t, the inequality

(16x 2 + 3 yz)(x − y)(x − z)(x − 4 y)(x − 4z) + α(x − y)2 ( y − z)2 (z − x)2 ≥ 0

turns into
At 6 + B t 4 + C t 2 ≥ 0,
where C = 324(α − 52). The necessary condition C ≥ 0 involves α ≥ 52.

P 3.39. If x, y, z are real numbers, then



x 4 (x − y)(x − z) ≥ (x − y)2 ( y − z)2 (z − x)2 .

(Vasile C., 2009)

Solution. Write the inequality as f6 (x, y, z) ≥ 0, where



f6 (x, y, z) = x 4 (x − y)(x − z) − (x − y)2 ( y − z)2 (z − x)2 .

Since (x − y)(x − z) = x 2 + 2 yz − q, f6 (x, y, z) has the same highest coefficient A


as 
x 4 (x 2 + 2 yz) − (x − y)2 ( y − z)2 (z − x)2 ,
that is, according to (3.1),

A = 3(1 + 2) + 27 = 36.

Because
f6 (x, 1, 1) = x 4 (x − 1)2 ,
x 4 (x − 1)4
f0,0 (x) = ,
36(x + 2)2
3x 4 (x − 1)2 (2x + 1)
f6 (x, 1, 1) − Af0,0 (x) = ,
(x + 2)2
272 Vasile Cîrtoaje

we apply Theorem 2 for

Eα,β = f0,0 , Fγ,δ = fγ,−2 ,

and for ξ → ∞, which involves


  
−1 −1
= ,∞ ,  \  = −∞, .
2 2

The condition (a), namely f6 (x, 1, 1) ≥ Af0,0 (x) for x > −1/2, is satisfied.
The condition (b) is satisfied if there is a real number γ such that

f6 (x, 1, 1) ≥ Afγ,−2 (x)

for x ≤ −1/2. We have


4(x − 1)4 (x − γ)2
fγ,−2 (x) = ,
81(2 + γ)2

(x − 1)2 g1 (x)g2 (x)


f6 (x, 1, 1) − Afγ,−2 (x) = ,
9(2 + γ)2
where

g1 (x) = 3(2 + γ)x 2 − 4(x − 1)(x − γ)


= (x + 2)[(3γ + 2)x − 2γ],

g2 (x) = 3(2 + γ)x 2 + 4(x − 1)(x − γ)


= (3γ + 10)x 2 − 4(γ + 1)x + 4γ.
−1
Choosing γ = , we have
2
1
g1 (x) = (x + 2)2 ≥ 0,
2
1
g2 (x) = (17x 2 − 4x − 4) > 0.
2
The equality holds for x = y = z, for x = 0 and y = z (or any cyclic permutation),
and for x = 0 and y + z = 0 (or any cyclic permutation).

13 − 3 17 13 + 3 17
P 3.40. Let x, y, z be real numbers. If ≤k≤ , then
2 2
  2 
k
(x 2 + k yz)2 (x − y)(x − z) + − 1 (x − y)2 ( y − z)2 (z − x)2 ≥ 0.
4
(Vasile C., 2012)
Highest Coefficient Cancellation Method for Real Variables 273

Solution. Write the inequality as f6 (x, y, z) ≥ 0, where


  2 
k
f6 (x, y, z) = (x 2 + k yz)2 (x − y)(x − z) + − 1 (x − y)2 ( y − z)2 (z − x)2 .
4
Since (x − y)(x − z) = x 2 + 2 yz − q, f6 (x, y, z) has the same highest coefficient A
as  2 
 k
(x 2 + k yz)2 (x 2 + 2 yz) + − 1 (x − y)2 ( y − z)2 (z − x)2 ,
4
that is, according to (3.1),
 2 
k 9
A = 9(1 + k)2 + − 1 (−27) = (k + 4)2 .
4 4
Since
f6 (x, 1, 1) = (x 2 + 1)2 (x − 1)2 ,
we apply Corollary 2 for
 2
1 x +8
Fγ,δ (x) = g1,δ (x) = (x − 1)4 + δ(x + 2) .
9 9
Thus, we only need to show that there exists a real number δ such that

f6 (x, 1, 1) ≥ Ag1,δ (x)

for all x ∈ . We have


(x − 1)2 g1 (x)g2 (x)
f6 (x, 1, 1) − Ag1,δ (x) = ,
324
where
g1 (x) = 18(x 2 + k) + (k + 4)(x − 1)[x + 8 + 3δ(x + 2)],
g2 (x) = 18(x 2 + k) − (k + 4)(x − 1)[x + 8 + 3δ(x + 2)].
Since
g1 (x) = (x + 2)[(k + 22)x + 5k − 16 + 3(k + 4)δ(x − 1)],
we choose
k − 20
δ=
3(k + 4)
to get
g1 (x) = (x + 2)[(k + 22)x + 5k − 16 + (k − 20)(x − 1)]
= 2(k + 1)(x + 2)2 ≥ 0,

g2 (x) = 18(x 2 + k) − (x − 1)[(k + 4)(x + 8) + (k − 20)(x + 2)]


= (17 − k)x 2 − 4(k + 1)x + 2(7k − 2)
 
2k + 2 2 18(k2 − 13k + 4)
= (17 − k) x − − ≥ 0.
17 − k 17 − k
274 Vasile Cîrtoaje

Thus, the proof is completed.


The equality holds for x = y = z, and for x = 0 and y + z = 0 (or any cyclic
permutation).
Observation. We may give a similar solution by applying Corollary 2 for

Fγ,δ = fγ,−2 .

Since
4(x − 1)4 (x − γ)2
fγ,−2 (x) = ,
81(2 + γ)2
we have
(x − 1)2 f1 (x) f2 (x)
f6 (x, 1, 1) − Afγ,−2 (x) = ,
9(2 + γ)2
where

f1 (x) = 3(2 + γ)(x 2 + k) − (k + 4)(x − 1)(x − γ)


= (x + 2)[(3γ + 2 − k)x + (k − 2)γ + 3k],

f2 (x) = 3(2 + γ)(x 2 + k) + (k + 4)(x − 1)(x − γ).


By choosing
5k − 4
γ= , k = 8,
8−k
we get
(k + 1)(k + 4)(x + 2)2
f1 (x) = ,
8−k
(k + 4)[(17 − k)x 2 − 4(k + 1)x + 2(7k − 2)]
f2 (x) = ,
8−k
therefore

(k + 1)(k + 4)2 (x + 2)2 [(17 − k)x 2 − 4(k + 1)x + 2(7k − 2)]


f1 (x) f2 (x) = ≥ 0.
(8 − k)2

For k = 8, by choosing γ → ∞ (see P 3.15), we have

4(x − 1)4
f∞,−2 (x) = ,
81

f6 (x, 1, 1) − Af∞,−2 (x) = (x + 2)2 (x 2 − 4x + 12) ≥ 0.


Highest Coefficient Cancellation Method for Real Variables 275

P 3.41. Let x, y, z be real numbers. If k ∈ (−∞, −4] ∪ [−1, 0], then


  2 
k
(x 2 + k yz)2 (x − y)(x − z) + − 1 (x − y)2 ( y − z)2 (z − x)2 ≥ 0.
4
(Vasile C., 2012)

Solution. Denote 
m= −k, m ≥ 0.
Write the inequality as f6 (x, y, z) ≥ 0, where
  
k2
f6 (x, y, z) = (x 2 + k yz)2 (x − y)(x − z) + − 1 (x − y)2 ( y − z)2 (z − x)2 .
4
As shown at the preceding P 3.40, f6 (x, y, z) has the highest coefficient
9
A = (k + 4)2 .
4
Also,
f6 (x, 1, 1) = (x 2 + k)2 (x − 1)2 .

Case 1: k ≤ −4. The case k = −4 is treated in P 3.2. Further, consider k < −4.
Since
4(x − 1)4 (x − m)2 (x + m)2 (x 2 + k)2 (x − 1)2
f−m,m (x) = = ,
9(4 + 2m ) (x + 2)
2 2 2 9(2 − k)2 (x + 2)2
3(x 2 + k)2 (x − 1)2 [(8 − k)x + 4 − 5k][(−k)x + 4 − k]
f6 (x, 1, 1) − Af−m,m (x) = ,
4(2 − k)2 (x + 2)2
we apply Theorem 2 and Remark 3 for

Eα,γ = g1,β , Fγ,δ = f−m,m ,

5k − 4 (4 − k)2
and η = < −2, which involves ξ = and
8−k k(8 − k)
    
5k − 4 4 − k 5k − 4 4−k
= , ,  \  = −∞, ∪ ,∞ .
8−k k 8−k k

The condition (b), namely f6 (x, 1, 1) − Af−m,m (x) ≥ 0 for x ∈  \ , is satisfied.


The condition (a) is satisfied if f6 (x, 1, 1) − Ag1,β (x) ≥ 0 for x ∈ . By choosing

k − 20
β= ,
3(k + 4)
we have (see the preceding P 3.40)

(k + 1)(x − 1)2 (x + 2)2 g2 (x)


f6 (x, 1, 1) − Ag1,β (x) = ,
162
276 Vasile Cîrtoaje

where
g2 (x) = (17 − k)x 2 − 4(1 + k)x + 2(7k − 2).
Since k +1 < 0, we
 must showthat g2 (x) ≤ 0 for x ∈ . Since 17 − k > 0, this is
5k − 4 4−k
true if g2 ≤ 0 and g2 ≤ 0. We have
8−k k
 
5k − 4 9(k + 1)(k + 4)2
g2 = < 0,
8−k (8 − k)2
 
4−k (k + 4)(17k2 − 59k + 68)
g2 = < 0.
k k2

Case 2: k ∈ [−1, 0]. Since the case k = −1 is treated in P 3.14, and the case
k = 0 in P 3.39, consider further k ∈ (−1, 0). The proof is similar to the one of the
case 1. We set  
5k − 4 −1
η= ∈ −1, ,
8−k 2
we involves
    
4 − k 5k − 4 4−k 5k − 4
= , ,  \  = −∞, ∪ ,∞ .
k 8−k k 8−k

The condition (b) is clearly satisfied. Since k + 1 > 0, the condition (a) is satisfied
if g2 (x) ≥ 0 for  
4 − k 5k − 4
x ∈= , .
k 8−k
Since g2 is decreasing, we have
 
5k − 4 9(k + 1)(k + 4)2
g2 (x) ≥ g3 = > 0.
8−k (8 − k)2

Thus, the proof is completed.



for x = y = z, for x/ −k = y = z (or any cyclic permutation)
The equality holds
if k = 0, for −x/ −k = y = z (or any cyclic permutation) if k = 0, and for x = 0
and y + z = 0 (or any cyclic permutation). If k = 0, then the equality holds also
for x = 0 and y = z (or any cyclic permutation).

P 3.42. Let x, y, z be real numbers. If k ≥ 0, then


  
k2
(x 2 + k yz)2 (x − y)(x − z) + − 1 (x − y)2 ( y − z)2 (z − x)2 ≥ 0.
4
(Vasile C., 2012)
Highest Coefficient Cancellation Method for Real Variables 277

Solution. Write the inequality as f6 (x, y, z) ≥ 0, where f6 (x, y, z) has the highest
coefficient (see P 3.40)
9
A = (k + 4)2 .
4
We have
f6 (x, 1, 1) = (x 2 + k)2 (x − 1)2 ,
and
(x 2 + k)2 (x − 1)2
f
−k,−
−k (x) = ,
9(2 − k)2 (x + 2)2
3(x 2 + k)2 (x − 1)2 [(8 − k)x + 4 − 5k][(−k)x + 4 − k]
f6 (x, 1, 1)−Af
−k,−
−k (x) = .
4(2 − k)2 (x + 2)2
Case 1: 0 ≤ k ≤ 2. The cases k = 2 and k = 0 are treated in P 3.22 and P 3.39,
respectively. Consider further 0 < k < 2. We choose
 
5k − 4 −1
η= ∈ ,1 ,
8−k 2
which involves
    
5k − 4 4 − k 5k − 4 4−k
= , ,  \  = −∞, ∪ ,∞ .
8−k k 8−k k

According to Theorem 2 and Remark 3, it suffices to prove that


(a) f6 (x, 1, 1) − Af
−k,−
−k (x) ≥ 0 for x ∈ ;
(b) f6 (x, 1, 1) − Ag1,δ (x) ≥ 0 for x ∈  \ .
The condition (a) is satisfied. With regard to the condition (b), by choosing

k − 20
δ= ,
3(k + 4)

we have (see P 3.40)

2(k + 1)(x − 1)2 (x + 2)2 g2 (x)


f6 (x, 1, 1) − Ag1,δ (x) = ,
162
where
g2 (x) = (17 − k)x 2 − 4(1 + k)x + 2(7k − 2).
Since k + 1 > 0, we have to show that g2 (x) ≥ 0 for
  
5k − 4 4−k
x ∈  \  = −∞, ∪ ,∞ .
8−k k

If

13 − 3 17 13 + 3 17
≤k≤ ,
2 2
278 Vasile Cîrtoaje

then the inequality g2 (x) ≥ 0 holds for all x ∈ . So, we only need to show that
g2 (x) ≥ 0 for

13 − 3 17
0<k< ≈ 0.315.
2
  
5k − 4 4−k
Since g2 is decreasing on −∞, and increasing on , ∞ , we only
  8−k   k
5k − 4 4−k
need to show that g2 ≥ 0 and g2 ≥ 0. Indeed, we have
8−k k
 
5k − 4 9(k + 1)(k + 4)2
g2 = >0
8−k (8 − k)2

and  
4−k (k + 4)(17k2 − 59k + 68)
g2 = > 0.
k k2
Thus, the proof is completed.
Case 2: 2 < k ≤ 8. The inequality is treated in P 3.40.
Case 3: k > 8. We choose
5k − 4
η= ∈ (−∞, −5),
8−k
which involves
    
5k − 4 4 − k 5k − 4 4−k
= , ,  \  = −∞, ∪ ,∞ .
8−k k 8−k k

According to Theorem 2 and Remark 3, it suffices to prove that


(a) f6 (x, 1, 1) − Ag1,δ (x) ≥ 0 for x ∈ ;
(b) f6 (x, 1, 1) − Af−m,m (x) ≥ 0 for x ∈  \ .
The condition (b) is satisfied because

[(8 − k)x + 4 − 5k][(−k)x + 4 − k] ≥ 0.

With regard to the condition (a), by choosing

k − 20
δ= ,
3(k + 4)

we have (see P 3.40)

2(k + 1)(x − 1)2 (x + 2)2 g2 (x)


f6 (x, 1, 1) − Ag1,δ (x) = ,
162
where
g2 (x) = (17 − k)x 2 − 4(1 + k)x + 2(7k − 2).
Highest Coefficient Cancellation Method for Real Variables 279

We have to show that g2 (x) ≥ 0 for x ∈ . For 8 < k ≤ 17, g2 is decreasing on ,


therefore  
4−k (k + 4)(17k2 − 59k + 68)
g2 (x) ≥ g2 = > 0.
k k2
 
5k − 4
For k > 17, since 17 − k < 0, it suffices to show that g2 ≥ 0 and
  8−k
4−k
g2 ≥ 0. Indeed, we have
k
 
5k − 4 9(k + 1)(k + 4)2
g2 = >0
8−k (8 − k)2
and  
4−k (k + 4)(17k2 − 59k + 68)
g2 = > 0.
k k2
Thus, the proof is completed.
For k > 0, the equality holds for x = y = z, and for x = 0 and y + z = 0 (or any
cyclic permutation).
Observation. From P 3.40, P 3.41 and P 3.42, we get the following generalization:

• Let x, y, z be real numbers. If k ∈ (−∞, −4] ∪ [−1, ∞), then


  2 
k
(x 2 + k yz)2 (x − y)(x − z) + − 1 (x − y)2 ( y − z)2 (z − x)2 ≥ 0.
4

The inequalities in P 3.2, P 3.14, P 3.22 and P 3.39 are particular cases of this
general statement.
The coefficient of the product (x − y)2 ( y −z)2 (z − x)2 is the best possible. Setting
x = 1, y = 1 + t and z = 1 − t, the inequality

(x 2 + k yz)2 (x − y)(x − z) + αk (x − y)2 ( y − z)2 (z − x)2 ≥ 0.

turns into
At 6 + B t 4 + C t 2 ≥ 0,
k2
where A = 4αk + 4 − k2 . The necessary condition A ≥ 0 involves αk ≥ − 1.
4

P 3.43. If x, y, z are real numbers, then



x 2 (x 2 + 8 yz)(x − y)(x − z) ≥ (x − y)2 ( y − z)2 (z − x)2 .

(Vasile C., 2013)


280 Vasile Cîrtoaje

Solution. Write the inequality as f6 (x, y, z) ≥ 0, where



f6 (x, y, z) = x 2 (x 2 + 8 yz)(x − y)(x − z) − (x − y)2 ( y − z)2 (z − x)2 .

Since (x − y)(x − z) = x 2 + 2 yz − q, f6 (x, y, z) has the same highest coefficient A


as 
x 2 (x 2 + 8 yz)(x 2 + 2 yz) − (x − y)2 ( y − z)2 (z − x)2 ,
that is, according to (3.1),

A = 3(1 + 8)(1 + 2) + 27 = 108.

On the other hand,


f6 (x, 1, 1) = x 2 (x 2 + 8)(x − 1)2 .
Since
x 2 (x − 1)4
f0,−2 (x) =
81
and
4x 2 (x − 1)4
f6 (x, 1, 1) − Af0,−2 (x) = x 2 (x 2 + 8)(x − 1)2 −
3
x 2 (x − 1)2 (x + 2)(10 − x)
= ,
3
we apply Theorem 2 for

Eα,γ = f0,−2 , Fγ,δ = hγ,δ ,

and for η = 10, which implies ξ = 48/7 and


  
−2 −2
= , 10 ,  \  = −∞, ∪ [10, ∞).
7 7
 
−2
Condition (a): x ∈  = , 10 . The condition f6 (x, 1, 1) ≥ Af0,−2 (x) is satis-
7
fied. 
−2
Condition (b): x ∈  \  = −∞, ∪ [10, ∞). We need to show that there
7
are γ and δ such that f6 (x, 1, 1) ≥ Ahγ,δ . Since

f6 (0, y, z) = ( y − z)( y 5 − z 5 ) − y 2 z 2 ( y − z)2 ,

f6 (0, 1, −1) = 2 · 2 − 4 = 0,
according to (3.19), we will choose

1 h (−2)
γ= + ,
3 12A
Highest Coefficient Cancellation Method for Real Variables 281

where
h(x) = f6 (x, 1, 1) = x 2 (x 2 + 8)(x − 1)2 .
We get
h (−2) −2 −1
h (−2) = −864, = , γ= ,
12A 3 3
therefore  2
1
h−1/3,δ = x − (x + 2)(2x + 1) + δ(x + 2)3 ,
3
f6 (x, 1, 1) − Ah−1/3,δ (x) = x 2 (x 2 + 8)(x − 1)2 − 12[−2x 2 − 2x − 2 + 3δ(x + 2)3 ]2 .

The inequality f6 (x, 1, 1) − Ah−1/3,δ (x) ≥ 0 holds in the vicinity of −2 only for
δ = 5/54. Setting this value for δ, we get
 2
5
f6 (x, 1, 1) − Ah−1/3,δ (x) = x 2 (x 2 + 8)(x − 1)2 − 12 −2x 2 − 2x − 2 + (x + 2)3
18
1
= x 2 (x 2 + 8)(x − 1)2 − (5x 3 − 6x 2 + 24x + 4)2
27
1
= (2x 6 + 6x 5 − 33x 4 − 184x 3 − 312x 2 − 192x − 16)
27
1
= (x + 2)4 (2x 2 − 10x − 1) ≥ 0.
27

The equality holds for x = y = z, for x = 0 and y = z (or any cyclic permutation),
and for x = 0 and y + z = 0 (or any cyclic permutation).

Observation 1. From the inequalities in P 3.39 and P 3.43, we get the following
inequality:

x 2 (x 2 + k yz)(x − y)(x − z) ≥ (x − y)2 ( y − z)2 (z − x)2 , 0 ≤ k ≤ 8.

Indeed, since the left hand side is linear in k, it suffices to prove this inequality only
for k = 0 (see P 3.39) and k = 8 (see P 3.43).

Observation 2. Notice that the coefficient of the product (x − y)2 ( y − z)2 (z − x)2
in the inequality from Observation 1 is the best possible. Indeed, setting x = 0,
y = 1 + t and z = 1 − t, the inequality

x 2 (x 2 + k yz)(x − y)(x − z) ≥ α(x − y)2 ( y − z)2 (z − x)2

turns into
At 4 + B t 2 + C ≥ 0,
where A = 1 − α. The necessary condition A ≥ 0 involves α ≤ 1.
282 Vasile Cîrtoaje

P 3.44. If x, y, z are real numbers, then



(x − y)(x − z)(x − 2 y)(x − 2z)(2x − y)(2x − z) + 15(x − y)2 ( y − z)2 (z − x)2 ≥ 0.

(Vasile C., 2012)

Solution. Let

f (x, y, z) = (x − y)(x − z)(x − 2 y)(x − 2z)(2x − y)(2x − z)

and
f6 (x, y, z) = f (x, y, z) + 15(x − y)2 ( y − z)2 (z − x)2 .
Using the identities

(x − y)(x − z) = x 2 + 2 yz − q,
(x − 2 y)(x − 2z) = x 2 + 6 yz − 2q,
(2x − y)(2x − z) = 4x 2 + 3 yz − 2q,

it follows that f (x, y, z) has the same highest coefficient A1 as



P1 (x, y, z) = (x 2 + 2 yz)(x 2 + 6 yz)(4x 2 + 3 yz),

that is, according to (3.1),

A1 = P1 (1, 1, 1) = 3(1 + 2)(1 + 6)(4 + 3) = 441.

Therefore, f6 (x, y, z) has the highest coefficient

A = 441 + 15(−27) = 36.

Since
f6 (x, 1, 1) = (x − 1)2 (x − 2)2 (2x − 1)2 ,
4(x − 1)4 (x − 2)2 (2x − 1)2
f2,1/2 (x) = ,
9(x + 2)2
3(x − 1)2 (x − 2)2 (2x − 1)2 (5x − 2)(2 − x)
f6 (x, 1, 1) − Af2,1/2 (x) = ,
(x + 2)2
apply Theorem 2 for
Eα,γ = f2,1/2 , Fγ,δ = f2,−2 ,
and for η = 2, which implies ξ = 16/5 and
  
2 2
= ,2 ,  \  = −∞, ∪ [2, ∞).
5 5
 
2
The condition (a), namely f6 (x, 1, 1) ≥ Af2,1/2 (x) for x ∈ , 2 , is satisfied.
5
Highest Coefficient Cancellation Method for Real Variables 283

2
The condition (b) is satisfied if f6 (x, 1, 1) ≥ Af2,−2 (x) for x ∈ −∞, ∪[2, ∞).
5
From
(x − 1)4 (x − 2)2
f2,−2 (x) = ,
324
it follows that this inequality is equivalent to

(x − 1)2 (x − 2)2 (5x − 2)(7x − 4) ≥ 0,

which is true.
The equality holds for x = y = z, for x/2 = y = z (or any cyclic permutation),
and for 2x = y = z (or any cyclic permutation).
Observation. The coefficient of the product (x − y)2 ( y − z)2 (z − x)2 is the best
possible. Setting x = y + z in

(x − y)(x − z)(x − 2 y)(x − 2z)(2x − y)(2x − z) + α(x − y)2 ( y − z)2 (z − x)2 ≥ 0,

we get
( y − z)2 [(α − 15) y 2 z 2 + 4( y 2 + z 2 )( y − z)2 ] ≥ 0,
which holds for all real numbers y and z only if α ≥ 15.

P 3.45. If x, y, z are real numbers, then



(x − y)(x − z)(x − 2 y)(x − 2z)(x − 3 y)(x − 3z) ≥ 3(x − y)2 ( y − z)2 (z − x)2 .

(Vasile C., 2012)

Solution. Let

f (x, y, z) = (x − y)(x − z)(x − 2 y)(x − 2z)(x − 3 y)(x − 3z),

and
f6 (x, y, z) = f (x, y, z) − 3(x − y)2 ( y − z)2 (z − x)2 .
From

(x − y)(x − z) = x 2 + 2 yz − q,
(x − 2 y)(x − 2z) = x 2 + 6 yz − 2q,
(x − 3 y)(x − 3z) = x 2 + 12 yz − 3q,

it follows that f (x, y, z) has the same highest coefficient A1 as



P1 (x, y, z) = (x 2 + 2 yz)(x 2 + 6 yz)(x 2 + 12 yz),
284 Vasile Cîrtoaje

that is, according to (3.1),

A1 = P1 (1, 1, 1) = 3(1 + 2)(1 + 6)(1 + 12) = 819.

Thus, f6 (x, y, z) has the highest coefficient

A = 819 − 3(−27) = 900.

Since
f6 (x, 1, 1) = (x − 1)2 (x − 2)2 (x − 3)2 ,
4(x − 1)4 (x − 2)2 (x − 3)2
f2,3 (x) = ,
1521(x + 2)2
3(x − 1)2 (x − 2)2 (x − 3)2 (2 + 11x)(46 − 7x)
f6 (x, 1, 1) − Af2,3 (x) = ,
169(x + 2)2
apply Theorem 2 for
Eα,γ = f2,3 , Fγ,δ = fγ,−2 ,
and for η = 46/7, which implies ξ = 400/77 and
    
−2 46 −2 46
= , ,  \  = −∞, ∪ ,∞ .
11 7 11 7
 
−2 46
The condition (a), namely f6 (x, 1, 1) ≥ Af2,3 (x) for x ∈ , , is satisfied.
11 7
The condition (b) is satisfied if there exists a real γ such that f6 (x, 1, 1) ≥ Afγ,−2 (x)
  
−2 46
for x ∈ −∞, ∪ , ∞ . We have
11 7

4(x − 14 (x − γ)2
fγ,−2 (x) = ,
81(2 + γ)2

(x − 1)2 g(x)
f6 (x, 1, 1) − Afγ,−2 (x) = ,
9(2 + γ)2
where

g(x) = 9(2 + γ)2 (x − 2)2 (x − 3)2 − 400(x − 1)2 (x − γ)2 = g1 (x)g2 (x),

g1 (x) = 3(2 + γ)(x − 2)(x − 3) − 20(x − 1)(x − γ),


g2 (x) = 3(2 + γ)(x − 2)(x − 3) + 20(x − 1)(x − γ).
Since
g1 (x) = (x + 2)[(3γ − 14)x + 18 − γ],
we choose
46
γ=
7
Highest Coefficient Cancellation Method for Real Variables 285

to get
40
g1 (x) = (x + 2)2 ≥ 0,
7
40
g2 (x) = (8x 2 − 49x + 50) > 0.
7
The equality holds for x = y = z, for x/2 = y = z (or any cyclic permutation),
for x/3 = y = z (or any cyclic permutation), and for x = 0 and y + z = 0 (or any
cyclic permutation).
Observation. The coefficient of the product (x − y)2 ( y − z)2 (z − x)2 is the best
possible. Setting x = 0, y = 1 and z = −1 in the inequality

(x − y)(x − z)(x − 2 y)(x − 2z)(x − 3 y)(x − 3z) ≥ α(x − y)2 ( y − z)2 (z − x)2

involves α ≤ 3.

P 3.46. If x, y, z are real numbers, then



(x− y)(x−z)(2x+3 y)(2x+3z)(3x+2 y)(3x+2z)+15(x− y)2 ( y−z)2 (z−x)2 ≥ 0.

(Vasile C., 2012)


Solution. Let

f (x, y, z) = (x − y)(x − z)(2x + 3 y)(2x + 3z)(3x + 2 y)(3x + 2z)

and
f6 (x, y, z) = f (x, y, z) + 15(x − y)2 ( y − z)2 (z − x)2 .
From

(x − y)(x − z) = x 2 + 2 yz − q,
(2x + 3 y)(2x + 3z) = 4x 2 + 3 yz + 6q,
(3x + 2 y)(3x + 2z) = 9x 2 − 2 yz + 6q,

it follows that f (x, y, z) has the same highest coefficient A1 as



P1 (x, y, z) = (x 2 + 2 yz)(4x 2 + 3 yz)(9x 2 − 2 yz),

that is, according to (3.1),

A1 = P1 (1, 1, 1) = 3(1 + 2)(4 + 3)(9 − 2) = 441.

Thus, f6 (x, y, z) has the highest coefficient

A = 441 + 15(−27) = 36.


286 Vasile Cîrtoaje

Since
f6 (x, 1, 1) = (x − 1)2 (2x + 3)2 (3x + 2)2 ,
4(x − 1)4 (2x + 3)2 (3x + 2)2
f−3/2,−2/3 (x) = ,
9 · 625(x + 2)2
3(x − 1)2 (2x + 3)2 (3x + 2)2 (7x + 18)(29x + 46)
f6 (x, 1, 1) − Af−3/2,−2/3 (x) = ,
625(x + 2)2
apply Theorem 2 for η = −46/29, which implies ξ = −16/203 and
    
−18 −46 −18 −46
= , ,  \  = −∞, ∪ ,∞ ,
7 29 7 29
and for
Eα,γ = fα,−2 , Fγ,δ = f−3/2,−2/3 .

The condition
 (a)is satisfied if there exists a real α such that f6 (x, 1, 1) ≥ Afα,−2 (x)
−18 −46
for x ∈ , . We have
7 29
4(x − 14 (x − α)2
fα,−2 (x) = ,
81(2 + α)2

(x − 1)2 g(x)
f6 (x, 1, 1) − Afα,−2 (x) = ,
9(2 + α)2
where
g(x) = 9(2 + α)2 (2x + 3)2 (3x + 2)2 − 16(x − 1)2 (x − α)2 .
It is easy to check that g(−2) = 0. Choosing α = −46/29, the inequality g(x) ≥ 0
is equivalent to
(x + 2)2 (83x 2 + 134x + 8) ≥ 0,
which is true because 83x 2 + 134x + 8 > 0 for x ≤ −46/29.

−18
The condition (b), namely f6 (x, 1, 1) ≥ Af−3/2,−2/3 (x) for x ∈ −∞, ∪
  7
−46
, ∞ , is satisfied.
29
The equality holds for x = y = z, for −3x/2 = y = z (or any cyclic permutation),
for −2x/3 = y = z (or any cyclic permutation), and for x = 0 and y + z = 0 (or
any cyclic permutation).
Observation. The coefficient of the product (x − y)2 ( y − z)2 (z − x)2 is the best
possible. Setting x = 0, y = 1 and z = −1 in

(x − y)(x −z)(2x +3 y)(2x +3z)(3x +2 y)(3x +2z)+α(x − y)2 ( y −z)2 (z− x)2 ≥ 0,

yields α ≥ 15.
Highest Coefficient Cancellation Method for Real Variables 287

P 3.47. If x, y, z are real numbers, then


 1
(x + y)(x + z)(x 2 − y 2 )(x 2 − z 2 ) + (x − y)2 ( y − z)2 (z − x)2 ≥ 0.
4
(Vasile C., 2012)

Solution. Let

f (x, y, z) = (x + y)2 (x + z)2 (x − y)(x − z)

and
1
f6 (x, y, z) = f (x, y, z) + (x − y)2 ( y − z)2 (z − x)2 .
4
Since
(x + y)(x + z) = x 2 + q, (x − y)(x − z) = x 2 + 2 yz − q,
f (x, y, z) has the same highest coefficient A1 as

P1 (x, y, z) = x 4 (x 2 + 2 yz),

that is, according to (3.1),


A1 = P1 (1, 1, 1) = 9.
Therefore, f6 (x, y, z) has the highest coefficient

1 9
A = 9 + (−27) = .
4 4
On the other hand, since

f6 (x, 1, 1) = (x + 1)4 (x − 1)2 ,

(x + 1)4 (x − 1)4
f−1,−1 (x) = ,
36(x + 2)2
(x + 1)4 (x − 1)4
Af−1,−1 (x) = ,
16(x + 2)2
3(x + 1)4 (x − 1)2 (x + 3)(5x + 7)
f6 (x, 1, 1) − Af−1,−1 (x) = ,
16(x + 2)2
we will apply Theorem 2 for η = −3, which involves ξ = −1/55,
   
−7 −7
 = −3, ,  \  = (−∞, −3] ∪ ,∞ ,
5 5

and for
Eα,β = fα,−2 , Fγ,δ = f−1,−1 .
288 Vasile Cîrtoaje
 
−7
Condition (a): x ∈  = −3, . We need to show that there exists a real α
5
such that
f6 (x, 1, 1) ≥ Afα,−2 (x).
Since
4(x − 1)4 (x − α)2
fα,−2 (x) = ,
81(α + 2)2
(x − 1)4 (x − α)2
Afα,−2 (x) = ,
9(α + 2)2
we have
g1 (x)g2 (x)
f6 (x, 1, 1) − Afα,−2 (x) = ,
9(α + 2)2
where

g1 (x) = 3(α + 2)(x + 1)2 − (x − 1)(x − α) = (x + 2)[(3α + 5)x + α + 3],

g2 (x) = 3(α + 2)(x + 1)2 + (x − 1)(x − α).


Since −2 ∈ , we choose
−7
α=
5
to get
4
g1 (x) = (x + 2)2 ≥ 0
5
and
2
g2 (x) = (7x 2 + 10x + 1).
5
−7
For x ≤ , we have g2 (x) > 0 because
5
7 (5x + 7)(35x + 1)
7x 2 + 10x + 1 > 7x 2 + 10x + = ≥ 0.
25 25
 
−7
Condition (b): x ∈  \  = (−∞, −3] ∪ , ∞ . As shown above, we have
5
3(x + 1)4 (x − 1)2 (x + 3)(5x + 7)
f6 (x, 1, 1) − Af−1,−1 (x) = ≥ 0.
16(x + 2)2

The equality holds for x = y = z, for −x = y = z (or any cyclic permutation),


and for x = 0 and y + z = 0 (or any cyclic permutation).
Observation 1. Similarly, we can prove the following generalization:
• Let x, y, z be real numbers. If
1
αk = (k2 − 2k − 2)(k2 + 2k + 2), −2 ≤ k ≤ 0,
4
Highest Coefficient Cancellation Method for Real Variables 289

then

(x − y)(x − z)(x − k y)2 (x − kz)2 + αk (x − y)2 ( y − z)2 (z − x)2 ≥ 0,

with equality for x = y = z, for x/k = y = z (or any cyclic permutation) if k = 0,


and for x = 0 and y + z = 0 (or any cyclic permutation). If k = 0, then the equality
holds also for x = 0 and y = z (or any cyclic permutation).
For k = 0, we get the inequality from P 3.39. Further, consider that

−2 ≤ k < 0.

We have
9
A = (k + 2)4 ,
4
f6 (x, y, z) = (x − 1)2 (x − k)2 ,
(x − 1)4 (x − k)4
f k,k (x) = ,
9(k − 1)2 (k + 2)2
(x − 1)2 (x − k)4
f6 (x, 1, 1) − Af k,k (x) = h(x),
4(k − 1)2 (x + 2)2

h(x) = 4(k − 1)2 (x + 2)2 − (k + 2)2 (x − 1)2


= (−k x + 2 − k)[(4 − k)x + 2 − 5k].

Therefore, we apply Theorem 2 for

2−k (k + 2)2
η= ≤ −2, ξ= ,
k k(4 − k)
    
2 − k 5k − 2 2−k 5k − 2
= , ,  \  = −∞, ∪ ,∞ ,
k 4−k k 4−k
and for
Eα,β = fα,−2 , Fγ,δ = f k,k .
  
2−k 5k − 2
Condition (b): x ∈  \  = −∞, ∪ , ∞ . Since h(x) ≥ 0, we
k 4−k
have f6 (x, 1, 1) − Af k,k (x) ≥ 0.
 
2 − k 5k − 2
Condition (a): x ∈  = , . By choosing
k 4−k
5k − 2
α= ,
4−k
we get
(x − 1)2
f6 (x, 1, 1) − Afα,−2 (x) = g1 (x)g2 (x),
9
290 Vasile Cîrtoaje

g1 (x) = 3(α + 2)(x − k)2 − (k + 2)2 (x − 1)(x − α)


= (x + 2)[(3α − k2 − 4k + 2)x + (k2 − 2k − 2)α + 3k2 ]
(1 − k)2 (2 + k)
= (x + 2)2 ≥ 0,
4−k
k+2
g2 (x) = 3(α + 2)(x − k)2 + (k + 2)2 (x − 1)(x − α) = g3 (x),
4−k
g3 (x) = (17 + 2k − k2 )x 2 − 4(1 + 7k + k2 )x − 2(2 − 4k − 7k2 ).
Since  
5k − 2
g3 (x) ≥ g3 >0
4−k
for x ∈  and −2 ≤ k < 0, the condition (a) is satisfied, too.
Observation 2. Similarly, we can prove that the inequality from Observation 1 is
also valid for k ∈ (−∞, −2) ∪ (4, ∞). To prove this, we choose
 
2−k −1 (k + 2)2
η= ∈ −2, , ξ= ,
k 2 k(4 − k)
    
5k − 2 2 − k 5k − 2 2−k
= , ,  \  = −∞, ∪ ,∞ .
4−k k 4−k k
If x ∈ , we have
 
5k − 2
g3 (x) ≥ g3 > 0, k ∈ (−∞, −2),
4−k
 
2−k
g3 (x) ≥ g3 > 0, k ∈ (4, 14].
k
In addition, we have g3 (x) > 0 for k ∈ [6, ∞) because
   
5k − 2 2−k
g3 > 0, g3 > 0.
4−k k

P 3.48. If x, y, z are real numbers, then



(x − y)(x − z)(x − 4 y)2 (x − 4z)2 + 39(x − y)2 ( y − z)2 (z − x)2 ≥ 0.

(Vasile C., 2015)


Highest Coefficient Cancellation Method for Real Variables 291

Solution. Let

f (x, y, z) = (x − y)(x − z)(x − 4 y)2 (x − 4z)2

and
f6 (x, y, z) = f (x, y, z) − 39(x − y)2 ( y − z)2 (z − x)2 .
Since

(x − y)(x − z) = x 2 + 2 yz − q, (x − 4 y)(x − 4z) = x 2 + 20 yz − 4q,

f (x, y, z) has the same highest coefficient A1 as



P1 (x, y, z) = (x 2 + 2 yz)(x 2 + 20 yz)2 ,

that is, according to (3.1),

A1 = P1 (1, 1, 1) = 3969

Therefore, f6 (x, y, z) has the highest coefficient

A = 3969 + 39(−27) = 2916.

On the other hand, since

f6 (x, 1, 1) = (x − 1)2 (x − 4)4 ,

(x − 1)4 (x − 4)4
f4,4 (x) = ,
2916(x + 2)2
(x − 1)4 (x − 4)4
Af4,4 (x) = ,
(x + 2)2
3(x − 1)2 (x − 4)4 (2x + 1)
f6 (x, 1, 1) − Af4,4 (x) = ,
(x + 2)2
we will apply Theorem 2 for η → ∞, which involves ξ → ∞,
  
−1 −1
= ,∞ ,  \  = −∞, ,
2 2
and for
Eα,β = f4,4 Fγ,δ = f∞,−2 .
 
−1
Condition (a): x ∈  = , ∞ . Because
2
f6 (x, 1, 1) − Af4,4 (x) ≥ 0,

this condition is satisfied.


292 Vasile Cîrtoaje

−1
Condition (b): x ∈  \  = −∞, . We have
2

4(x − 1)4
f∞,−2 (x) = ,
81
Af∞,−2 (x) = 144(x − 1)4 ,
f5 (x, 1, 1) − Af∞,−2 (x) = (x − 1)2 (x + 2)2 (x 2 − 20x + 29) ≥ 0.

The equality holds for x = y = z, for x/4 = y = z (or any cyclic permutation),
and for x = 0 and y + z = 0 (or any cyclic permutation).
Observation 1. Similarly, we can prove the following generalization:
• Let x, y, z be real numbers. If
1
αk = (k2 − 2k − 2)(k2 + 2k + 2), 1 ≤ k ≤ 4,
4
then

(x − y)(x − z)(x − k y)2 (x − kz)2 + αk (x − y)2 ( y − z)2 (z − x)2 ≥ 0,

with equality for x = y = z, for x/k = y = z (or any cyclic permutation) if k = 0,


and for x = 0 and y + z = 0 (or any cyclic permutation).
The cases k = 1 and k = 4 are treated in P 3.18 and P 3.48, respectively. Further,
assume that
1 < k < 4.
We have
9
A = (k + 2)4 ,
4
f6 (x, y, z) = (x − 1)2 (x − k)2 ,
(x − 1)4 (x − k)4
f k,k (x) = ,
9(k − 1)2 (k + 2)2
(x − 1)2 (x − k)4
f6 (x, 1, 1) − Af k,k (x) = (−k x + 2 − k)[(4 − k)x + 2 − 5k].
4(k − 1)2 (x + 2)2
Therefore, we apply Theorem 2 for
 
2−k −1 (k + 2)2
η= ∈ ,1 , ξ= ,
k 2 k(4 − k)
    
2 − k 5k − 2 2−k 5k − 2
= , ,  \  = −∞, ∪ ,∞ ,
k 4−k k 4−k
and for
Eα,β = f k,k , Fγ,δ = fγ,−2 .
Highest Coefficient Cancellation Method for Real Variables 293
 
2 − k 5k − 2
Condition (a): x ∈  = , . Since f6 (x, 1, 1) − Af k,k (x) ≥ 0, this
k 4−k
condition is satisfied.
  
2−k 5k − 2
Condition (b): x ∈  \  = −∞, ∪ , ∞ . By choosing
k 4−k
5k − 2
α= ,
4−k
we get
(x − 1)2
f6 (x, 1, 1) − Afα,−2 (x) = g1 (x)g2 (x),
9

g1 (x) = (x + 2)[(3α − k2 − 4k + 2)x + (k2 − 2k − 2)α + 3k2 ]


(1 − k)2 (2 + k)
= (x + 2)2 ≥ 0,
4−k
k+2
g2 (x) = 3(α + 2)(x − k)2 + (k + 2)2 (x − 1)(x − α) = g3 (x),
4−k
g3 (x) = (17 + 2k − k2 )x 2 − 4(1 + 7k + k2 )x − 2(2 − 4k − 7k2 ).
We have g3 (x) > 0 for x ∈  \  and 1 < k < 4, because

2 − k 2(1 + 7k + k2 ) 5k − 2
< <
k 17 + 2k − k2 4−k
and    
2−k 5k − 2
g3 > 0, g3 > 0.
k 4−k

Observation 2. The inequality from Observation 1 is also valid for

k ∈ (0, 1).

The proof is similar to the one from Observation 1, but we choose

2−k (k + 2)2
η= ∈ (1, ∞), ξ= ,
k k(4 − k)
    
5k − 2 2 − k 5k − 2 2−k
= , ,  \  = −∞, ∪ ,∞ .
4−k k 4−k k
We have g3 (x) > 0 for x ∈  \  and 0 < k < 1, because

5k − 2 2(1 + 7k + k2 ) 2 − k
< <
4−k 17 + 2k − k2 k
and    
5k − 2 2−k
g3 > 0, g3 > 0.
4−k k
294 Vasile Cîrtoaje

Observation 3. From P 3.47, P 3.48 and the observations attached to them, the
following generalization follows:

• Let x, y, z be real numbers. If

1
αk = (k2 − 2k − 2)(k2 + 2k + 2), k ∈ ,
4
then

(x − y)(x − z)(x − k y)2 (x − kz)2 + αk (x − y)2 ( y − z)2 (z − x)2 ≥ 0,

with equality for x = y = z, for x/k = y = z (or any cyclic permutation) if k = 0,


and for x = 0 and y + z = 0 (or any cyclic permutation). If k = 0, then the equality
holds also for x = 0 and y = z (or any cyclic permutation).

P 3.49. Let x, y, z be real numbers, and let



⎪ 20 + 12k − 4k2 − k4

⎪ , k ∈ (−∞, −2] ∪ [4, ∞)
⎨ 4(1 − k)2
αk = 1 + k, k ∈ [−2, 1] .




5 − 3k, k ∈ [1, 4]

Then,

x 2 (x − y)(x − z)(x − k y)(x − kz) ≥ αk (x − y)2 ( y − z)2 (z − x)2 .

(Vasile C., 2009)

Solution. Denote

f (x, y, z) = x 2 (x − y)(x − z)(x − k y)(x − kz),

and write the inequality as f6 (x, y, z) ≥ 0, where

f6 (x, y, z) = f (x, y, z) − αk (x − y)2 ( y − z)2 (z − x)2 ,

Since

(x − y)(x − z) = x 2 + 2 yz − q, (x − k y)(x − kz) = x 2 + (k + k2 ) yz − kq,

f (x, y, z) has the same highest coefficient A1 as



P1 (x, y, z) = x 2 (x 2 + 2 yz)[x 2 + (k + k2 ) yz],
Highest Coefficient Cancellation Method for Real Variables 295

that is, according to (3.1),

A1 = P1 (1, 1, 1) = 3(1 + 2)(1 + k + k2 ) = 9(k2 + k + 1).

Therefore, since the product (x − y)2 ( y − z)2 (z − x)2 has the highest coefficient
equal to −27, f6 (x, y, z) has the highest coefficient

⎪ 9(2 + k)2 (4 − k)2

⎪ , k ∈ (−∞, −2] ∪ [4, ∞)
⎨ 4(1 − k)2
A = A1 + 27αk = 9(2 + k)2 , k ∈ [−2, 1] .




9(4 − k)2 , k ∈ [1, 4]

In addition, we have

f6 (x, 1, 1) = x 2 (x − 1)2 (x − k)2 ,

4x 2 (x − 1)4 (x − k)2 4(x − 1)4 (x − k)2


f k,0 (x) = , f k,−2 (x) = .
9(k − 4)2 (x + 2)2 81(k + 2)2
We will consider the following cases:

k ∈ {−2, 4}, k = 0,

k ∈ (−∞, −2), k ∈ (4, ∞), k ∈ (−2, 0), k ∈ (0, 1], k ∈ [1, 4).

Case 1: k ∈ {−2, 4}. Since A = 0 and f6 (x, 1, 1) ≥ 0 for any real x, the conclusion
follows by Corollary 1.

Case 2: k = 0. This case is treated in P 3.39.

Case 3: k ∈ (−∞, −2). Since

9(2 + k)2 (4 − k)2


A= ,
4(1 − k)2

3x 2 (x − 1)2 (x − k)3 [−(2k + 1)x + 4 − k]


f6 (x, 1, 1) − Af k,0 (x) = ,
(1 − k)2 (x + 2)2
we apply Theorem 2 for
 
4−k −1 (k + 2)2
η= ∈ −2, , ξ= ,
2k + 1 2 2k + 1
   
4−k 4−k
 = k, ,  \  = (−∞, k] ∪ ,∞ ,
2k + 1 2k + 1
and for
Eα,β = f k,−2 , Fγ,δ = f k,0 .
296 Vasile Cîrtoaje
 
4−k
The condition (b), namely f6 (x, 1, 1) ≥ Af k,0 (x) for x ∈ (−∞, k]∪ ,∞ ,
2k + 1
is satisfied.
 
4−k
The condition (a) is satisfied if f6 (x, 1, 1) ≥ Af k,−2 (x) for x ∈ k, . From
2k + 1

(x − 1)2 (x − k)2 [−(2k + 1)x + 4 − k][(7 − 4k)x + k − 4]


f6 (x, 1, 1) − Af k,−2 (x) = ,
9(1 − k)2
  
4−k 4−k
it follows that f6 (x, 1, 1) − Af k,−2 (x) ≥ 0 for x ∈ −∞, ∪ ,∞ ,
  2k + 1 7 − 4k
4−k
therefore for x ∈ k, .
2k + 1
Case 4: k ∈ (4, ∞). Since

9(2 + k)2 (4 − k)2


A= ,
4(1 − k)2

3x 2 (x − 1)2 (x − k)3 [4 − k − (2k + 1)x]


f6 (x, 1, 1) − Af k,0 (x) = ,
(1 − k)2 (x + 2)2
we apply Theorem 2 for
(k + 2)2
η = k, ξ= ,
2k + 1
  
4−k 4−k
= , k ,  \  = −∞, ∪ [k, ∞),
2k + 1 2k + 1
and for
Eα,β = f k,0 , Fγ,δ = f k,−2 .
 
4−k
The condition (a), namely f6 (x, 1, 1) ≥ Af k,0 (x) for x ∈ , k , is satisfied.
2k + 1

4−k
The condition (b) is satisfied if f6 (x, 1, 1) ≥ Af k,−2 (x) for x ∈ −∞, ∪
2k + 1
[k, ∞). From

(x − 1)2 (x − k)2 [(2k + 1)x + k − 4][(4k − 7)x + 4 − k]


f6 (x, 1, 1) − Af k,−2 (x) = ,
9(1 − k)2
  
4−k k−4
it follows that f6 (x, 1, 1) − Af k,−2 (x) ≥ 0 for x ∈ −∞, ∪ ,∞ ,
 2k + 1 4k − 7
4−k
therefore for x ∈ −∞, ∪ [k, ∞).
2k + 1
Case 5: k ∈ (−2, 0). Since
A = 9(2 + k)2 ,
Highest Coefficient Cancellation Method for Real Variables 297

3x 2 (x − 1)2 (x − k)2 (−k x + 4)[(k + 8)x + 4 − 4k]


f6 (x, 1, 1) − Af k,0 (x) = ,
(4 − k)2 (x + 2)2
we apply Theorem 2 for
 
4k − 4 −1 4(k + 2)2
η= ∈ −2, , ξ= ,
k+8 2 k(k + 8)
    
4 4k − 4 4 4k − 4
= , ,  \  = −∞, ∪ ,∞ ,
k k+8 k k+8
and for
Eα,β = fα,−2 , Fγ,δ = f k,0 .
  
4 4k − 4
The condition (b), namely f6 (x, 1, 1) ≥ Af k,0 (x) for x ∈ −∞, ∪ ,∞ ,
k k+8
is satisfied.
 
4 4k − 4
The condition (a), namely f6 (x, 1, 1) ≥ Afα,−2 (x) for x ∈  = , , is true
k k+8
if g1 (x)g2 (x) ≥ 0, where

g1 (x) = 3(2 + α)x(x − k) − 2(2 + k)(x − 1)(x − α),

g2 (x) = 3(2 + α)x(x − k) + 2(2 + k)(x − 1)(x − α).


Since
g1 (x) = (x + 2)[(3α + 2 − 2k)x − (2 + k)α],
we choose
4(k − 1)
α=
k+8
to get
2(1 − k)(k + 2)
g1 (x) = (x + 2)2 ≥ 0,
k+8
2(k + 2)
g2 (x) = g3 (x), g3 (x) = (k + 17)x 2 − 2(7k + 2)x + 4k − 4.
k+8
Since g3 is strictly decreasing on , we have
 
4k − 4 36(1 − k)(k + 2)2
g3 (x) ≥ g3 = > 0.
k+8 (k + 8)2

Case 6: k ∈ (0, 1]. Since


A = 9(2 + k)2 ,

3x 2 (x − 1)2 (x − k)2 (−k x + 4)[(k + 8)x + 4 − 4k]


f6 (x, 1, 1) − Af k,0 (x) = ,
(4 − k)2 (x + 2)2
298 Vasile Cîrtoaje

we apply Theorem 2 for

4 4(k + 2)2
η= ∈ [4, ∞), ξ= ,
k k(k + 8)
    
4k − 4 4 4k − 4 4
= , ,  \  = −∞, ∪ ,∞ ,
k+8 k k+8 k
and for
Eα,β = f k,0 , Fγ,δ = fγ,−2 .

4k − 4 4
The condition (a), namely f6 (x, 1, 1) ≥ Af k,0 (x) for x ∈ , , is satisfied.
k+8 k

4k − 4
The condition (b) is satisfied if f6 (x, 1, 1) ≥ Afγ,−2 (x) for x ∈ −∞, ∪
  k+8
4
, ∞ . This is true if g1 (x)g2 (x) ≥ 0, where
k

g1 (x) = 3(2 + α)x(x − k) − 2(2 + k)(x − 1)(x − α),

g2 (x) = 3(2 + α)x(x − k) + 2(2 + k)(x − 1)(x − α).


Since
g1 (x) = (x + 2)[(3α + 2 − 2k)x − (2 + k)α],
we choose
4(k − 1)
α=
k+8
to get
2(1 − k)(k + 2)
g1 (x) = (x + 2)2 ≥ 0,
k+8
2(k + 2)
g2 (x) = g3 (x), g3 (x) = (k + 17)x 2 − 2(7k + 2)x + 4k − 4.
k+8
  
4k − 4 4
Since g3 is strictly decreasing on −∞, and strictly increasing on ,∞ ,
k+8 k
we get
 
4k − 4 36(1 − k)(k + 2) 2
g3 (x) ≥ g3 = ≥0
k+8 (k + 8)2
4k − 4
for x ≤ , and
k+8
 
4 4(k3 − 15k2 + 68)
g3 (x) ≥ g3 = >0
k k2
4
for x ≥ .
k
Highest Coefficient Cancellation Method for Real Variables 299

Case 7: k ∈ [1, 4). Since


A = 9(4 − k)2 ,

3x 3 (x − 1)2 (x − k)2 (4 − x)
f6 (x, 1, 1) − Af k,0 (x) = ,
(x + 2)2
we apply Theorem 2 for
η = 4, ξ = 4,
 = (0, 4),  \  = (−∞, 0] ∪ [4, ∞),
and for
Eα,β = f k,0 , Fγ,δ = f0,−2 .

The condition (a), namely f6 (x, 1, 1) ≥ Af k,0 (x) for x ∈ (0, 4), is satisfied.

The condition (b) is satisfied if f6 (x, 1, 1) ≥ Af0,−2 (x) for x ∈ (−∞, 0] ∪ [4, ∞).
We have

(k − 1)x 2 (x − 1)2 (x − 4)[(7 − k)x − 2k − 4]


f6 (x, 1, 1) − Af0,−2 (x) = ≥ 0,
9
since
(x − 4)[(7 − k)x − 2k − 4] ≥ 0
for x ≤ 0, and

(7 − k)x − 2k − 4 ≥ 4(7 − k) − 2k − 4 = 6(4 − k) > 0

for x ≥ 4.
The proof is completed. The equality hold for x = y = z, for x = 0 and y = z
(or any cyclic permutation), for x/k = y = z (or any cyclic permutation) if k = 0,
and for x = 0 and y + z = 0 (or any cyclic permutation) if k ∈ [−2, 1].

Observation 1. The coefficient αk of the product (x − y)2 ( y −z)2 (z − x)2 is the best
possible. Setting x = k, y = 1 + t and z = 1 − t, the inequality in P 3.49 becomes

A(k, αk )t 6 + B(k, αk )t 4 + C(k, αk )t 2 ≥ 0,

where

A(k, αk ) = 4(1 + k − αk ),
C(k, αk ) = (1 − k)2 [20 + 12k − 4k2 − k4 − 4(1 − k)2 αk ].

From the necessary conditions A(k, αk ) ≥ 0 and C(k, αk ) ≥ 0, we get

20 + 12k − 4k2 − k4
αk ≤ 1 + k, αk ≤ ,
4(1 − k)2
300 Vasile Cîrtoaje

respectively. In addition, setting x = 0, the inequality in P 3.49 becomes

( y − z)2 [ y 4 + z 4 − (k − 1) yz( y 2 + z 2 ) + (1 − k − αk ) y 2 z 2 ] ≥ 0.

For y = z = 1, from the necessary inequality

y 4 + z 4 − (k − 1) yz( y 2 + z 2 ) + (1 − k − αk ) y 2 z 2 ≥ 0,

we get
αk ≤ 5 − 3k.

Observation 2. There are some relevant particular cases of the inequality in P 3.49.
• For k = −2, the inequality turns into

x 2 (x − y)(x − z)(x + 2 y)(x + 2z) + (x − y)2 ( y − z)2 (z − x)2 ≥ 0,

which is equivalent to
"  #
(x + y + z)2 x 4 + y 4 + z 4 + x yz(x + y + z) − x y(x 2 + y 2 ) ≥ 0.

The equality occurs for x = y = z, for x + y + z = 0, and for x = 0 and y = z (or


any cyclic permutation).
• For k = 1, the inequality turns into

x 2 (x − y)2 (x − z)2 ≥ 2(x − y)2 ( y − z)2 (z − x)2 ,

which is equivalent to
"  #2
x 3 + y 3 + z 3 + 3x yz − x y(x + y) ≥ 0.
!
The equality occurs for x 3 + y 3 + z 3 + 3x yz = x y(x + y).
• For k = 4, the inequality turns into

x 2 (x − y)(x − z)(x − 4 y)(x − 4z) + 7(x − y)2 ( y − z)2 (z − x)2 ≥ 0,

which is equivalent to

(x 2 + y 2 + z 2 − x y − yz − z x)(x 2 + y 2 + z 2 − 2x y − 2 yz − 2z x)2 ≥ 0.


The equality occurs for x = y = z, and for x= y+ z (or any cyclic permuta-
tion).
Observation 3. Substituting k − 1 for k and using then the identity

x 2 (x − y)(x − z)[x − (k − 1) y][x − (k − 1)z] =
Highest Coefficient Cancellation Method for Real Variables 301

= x 2 (x − y)(x − z)(x − k y + z)(x − kz + y) + k(x − y)2 ( y − z)2 (z − x)2 ,
we get the following statement:
• Let x, y, z be real numbers, and let

⎪ (3 − k)(1 + k)3

⎪ , k ∈ (−∞, −1] ∪ [5, ∞)
⎨ 4(2 − k)2
α∗k = 0, k ∈ [−1, 2] .




4(2 − k), k ∈ [2, 5]

Then,

x 2 (x − y)(x − z)(x − k y + z)(x − kz + y) ≥ α∗k (x − y)2 ( y − z)2 (z − x)2 ,

with equality for x = y = z, for x = 0 and y = z (or any cyclic permutation), for
x/(k − 1) = y = z (or any cyclic permutation) if k = 1, and for x = 0 and y + z = 0
(or any cyclic permutation) if k ∈ [−1, 2].

P 3.50. Let x, y, z be real numbers, and let


⎧ 2
⎪ k

⎪ , k ∈ (−∞, −2] ∪ [1, ∞)

⎪ 4
⎨ 
−k(8 + 11k + 8k2 ) −1
βk = , k ∈ −2, .

⎪ 4(1 − k) 2 2

⎪ 

⎩ 1, −1
k∈ ,1
4 2
Then,

yz(x − y)(x − z)(x − k y)(x − kz) + βk (x − y)2 ( y − z)2 (z − x)2 ≥ 0.

(Vasile C., 2009)


Solution. Denote

f (x, y, z) = yz(x − y)(x − z)(x − k y)(x − kz),

and write the inequality as f6 (x, y, z) ≥ 0, where

f6 (x, y, z) = f (x, y, z) + βk (x − y)2 ( y − z)2 (z − x)2 .

Since (x − y)(x − z) = x 2 + 2 yz − q and (x − k y)(x − kz) = x 2 + (k + k2 ) yz − kq,


f (x, y, z) has the same highest coefficient A1 as

P1 (x, y, z) = yz(x 2 + 2 yz)[x 2 + (k + k2 ) yz],
302 Vasile Cîrtoaje

that is, according to (3.1),

A1 = P1 (1, 1, 1) = 3(1 + 2)(1 + k + k2 ) = 9(k2 + k + 1).

Therefore, f6 (x, y, z) has the highest coefficient




⎪ 9(k + 2)2

⎪ , k ∈ (−∞, −2] ∪ [1, ∞)

⎪ 4
⎨ 
9(k + 2) (2k + 1)
2 2
−1
A = A1 − 27βk = , k ∈ −2, .

⎪ 4(k − 1) 2 2

⎪ 

⎪ 9(2k + 1)2 −1
⎩ , k∈ ,1
4 2
Also, we have
f6 (x, 1, 1) = (x − 1)2 (x − k)2 ,
4(x − 1)4 (x − k)2 4(x − 1)4 (x − k)2
f k,−2 (x) = , f k,∞ (x) = .
81(k + 2)2 9(1 + 2k)2 (x + 2)2
We will consider the following cases:
) *
−1
k ∈ −2, , k = 1,
2
   
−1 −1
k ∈ −2, , k∈ ,1 , k ∈ (−∞, −2) ∪ (1, ∞).
2 2
) *
−1
Case 1: k ∈ −2, . Since A = 0 and f6 (x, 1, 1) ≥ 0 for any real x, the conclusion
2
follows by Corollary 1.
4(x − 1)4
Case 2: k = 1. Since A = 81/4, f∞,−2 = and
81
f6 (x, 1, 1) − Af∞,−2 (x) = (x − 1)4 − (x − 1)4 = 0,

the inequality follows from Corollary 1. Notice that this case is treated in P 3.17.
 
−1
Case 3: k ∈ −2, . Since
2
9(k + 2)2 (2k + 1)2 (k + 2)2 (x − 1)4 (x − k)2
A= , Af k,∞ (x) = ,
4(k − 1)2 (k − 1)2 (x + 2)2

3(x − 1)2 (x − k)3 [−(2k + 1)x + 4 − k]


f6 (x, 1, 1) − Af k,∞ (x) = ,
(k − 1)2 (x + 2)2

we apply Theorem 2 for


(k + 2)2
η = k, ξ= ,
2k + 1
Highest Coefficient Cancellation Method for Real Variables 303
  
4−k 4−k
= ,k ,  \  = −∞, ∪ [k, ∞),
2k + 1 2k + 1
and for
Eα,β = f k,−2 , Fγ,δ = f k,∞ .

4−k
The condition (b), namely f6 (x, 1, 1) − Af k,∞ (x) ≥ 0 for x ∈ −∞, ∪
2k + 1
[k, ∞), is satisfied.
 
4−k
The condition (a) is satisfied if f6 (x, 1, 1) ≥ Af k,−2 (x) for x ∈ , k . In-
2k + 1
deed, we have
(2k + 1)(x − 1)4 (x − k)2
Af k,−2 (x) =
9(k − 1)2
and

f6 (x, 1, 1) − Af k,−2 (x) =


(x − 1)2 (x − k)2 [4 − k − (2k + 1)x][2 − 5k + (2k + 1)x]
= ≥ 0,
9(k − 1)2
since
4 − k − (2k + 1)x > 0,
2 − 5k + (2k + 1)x > 2 − 5k + (2k + 1)k = 2(k − 1)2 > 0.
 
−1
Case 4: k ∈ , 1 . Since
2
9(2k + 1)2 (x − 1)4 (x − k)2
A= , Af k,∞ (x) = ,
4 (x + 2)2
3(x − 1)2 (x − k)2 (2x + 1)
f6 (x, 1, 1) − Af k,∞ (x) = ,
(x + 2)2
we apply Theorem 2 for
η → ∞, ξ → ∞,
  
−1 −1
= ,∞ ,  \  = −∞, ,
2 2
and for
Eα,β = f k,∞ , Fγ,δ = f∞,−2 .

−1
The condition (a), namely f6 (x, 1, 1) − Af k,∞ (x) ≥ 0 for x > , is satisfied.
2
−1
The condition (b) is satisfied if f6 (x, 1, 1) ≥ Af∞,−2 (x) for x ≤ . Indeed, we
2
have
(2k + 1)2 (x − 1)4
Af∞,−2 (x) = ,
9
304 Vasile Cîrtoaje

(1 − k)(x − 1)2 (2x + 1)[2(k + 2)x − 5k − 1]


f6 (x, 1, 1) − Af∞,−2 (x) = ≥ 0,
9
since 1 − k > 0, 2x + 1 ≤ 0 and

2(k + 2)x − 5k − 1 ≤ −(k + 2) − 5k − 1 = −3(2k + 1) < 0.


 
−1
Case 5: k ∈ (−∞, −2) ∪ (1, ∞). This case reduces to the case k ∈ , 1 by
2
1 1 1 1
substituting , , , for x, y, z, k, respectively.
x y z k
The proof is completed. The equality hold for x = y = z, for y = z = 0 (or any
cyclic permutation), for x/k = y = z (or any cyclic permutation) if k = 0, and for
x = 0 and y +z = 0 (or any cyclic permutation) if k ∈ (−∞, −2]∪[1, ∞). If k = 0,
then the equality holds also for x = 0 and y = z (or any cyclic permutation).
Observation 1. The coefficient βk of the product (x − y)2 ( y −z)2 (z − x)2 is the best
possible. Setting x = k, y = 1 + t and z = 1 − t, the inequality in P 3.50 becomes

A(k, βk )t 6 + B(k, βk )t 4 + C(k, βk )t 2 ≥ 0,

where
A(k, βk ) = 4βk − k2 ,
C(k, βk ) = (1 − k)2 [4(1 − k)2 βk + k(8 + 11k + 8k2 )].
From the necessary conditions A(k, βk ) ≥ 0 and C(k, βk ) ≥ 0, we get

k2 −k(8 + 11k + 8k2 )


βk ≥ , βk ≥ ,
4 4(1 − k)2
respectively. In addition, for x → ∞, the inequality in P 3.50 becomes

yz + βk ( y − z)2 ≥ 0.
1
Setting y = 1 and z = −1, we get βk ≥ .
4
Observation 2. There are some relevant particular cases of the inequality in P 3.50.
• For k = −2, the inequality turns into

yz(x − y)(x − z)(x + 2 y)(x + 2z) + (x − y)2 ( y − z)2 (z − x)2 ≥ 0,

which is equivalent to
 
(x + y + z)2 x 2 ( y − z)2 + y 2 (z − x)2 + z 2 (x − y)2 ≥ 0.

The equality holds for x = y = z, for x + y + z = 0, and for y = z = 0 (or any cyclic
permutation).
Highest Coefficient Cancellation Method for Real Variables 305

• For k = −1/2, the inequality turns into



yz(x − y)(x − z)(2x + y)(2x + z) + (x − y)2 ( y − z)2 (z − x)2 ≥ 0,

which is equivalent to

(x y + yz + z x)2 (x 2 + y 2 + z 2 − x y − yz − z x) ≥ 0.

The equality holds for x = y = z, and for x y + yz + z x = 0.


• For k = 1, the inequality turns into
 1
yz(x − y)2 (x − z)2 + (x − y)2 ( y − z)2 (z − x)2 ≥ 0,
4
which is equivalent to
 2
x( y − z)2 + y(z − x)2 + z(x − y)2 ≥ 0.

The equality holds for x( y − z)2 + y(z − x)2 + z(x − y)2 = 0.

Observation 3. Adding the inequality in P 3.50 multiplied by 2 and the identity



( y 2 +z 2 −2 yz)(x − y)(x −z)(x −k y)(x −kz) = (k2 +k+1)(x − y)2 ( y −z)2 (z− x)2 ,

we get the following statement:


• Let x, y, z be real numbers, and let
⎧ 2
⎪ k

⎪ + k + 1, k ∈ (−∞, −2] ∪ [1, ∞)

⎪ 2
⎨ 
2k + 6k + 11k + 6k + 2
4 3 2
−1
Ak = , k ∈ −2, .

⎪ 2(k − 1) 2 2

⎪ 

⎩ k2 + k + 1 , −1
k∈ ,1
2 2
Then,

( y 2 + z 2 )(x − y)(x − z)(x − k y)(x − kz) ≥ Ak (x − y)2 ( y − z)2 (z − x)2 ,

with equality for x = y = z, for y = z = 0 (or any cyclic permutation), for x/k =
y = z (or any cyclic permutation) if k = 0, and for x = 0 and y + z = 0 (or any cyclic
permutation) if k ∈ (−∞, −2] ∪ [1, ∞). If k = 0, then the equality holds also for
x = 0 and y = z (or any cyclic permutation).

Observation 4. Adding the inequality in P 3.50 multiplied by 4 and the identity



( y 2 +z 2 −2 yz)(x − y)(x −z)(x −k y)(x −kz) = (k2 +k+1)(x − y)2 ( y −z)2 (z− x)2 ,
306 Vasile Cîrtoaje

we get the following statement:


• Let x, y, z be real numbers, and let

⎪ k + 1, k ∈ (−∞, −2] ∪ [1, ∞)



⎨ k4 + 7k3 + 11k2 + 7k + 1 
−1
Bk = , k ∈ −2, .
⎪ (k − 1) 2 2

⎪ 

⎩ k2 + k, −1
k∈ ,1
2
Then,

( y + z)2 (x − y)(x − z)(x − k y)(x − kz) ≥ Bk (x − y)2 ( y − z)2 (z − x)2 ,

with equality for x = y = z, for y = z = 0 (or any cyclic permutation), for x/k =
y = z (or any cyclic permutation) if k = 0, and for x = 0 and y + z = 0 (or any cyclic
permutation) if k ∈ (−∞, −2] ∪ [1, ∞). If k = 0, then the equality holds also for
x = 0 and y = z (or any cyclic permutation).

Observation 5. Substituting k − 1 for k and using then the identity



yz(x − y)(x − z)[x − (k − 1) y][x − (k − 1)z] =

= yz(x − y)(x − z)(x − k y + z)(x − kz + y) + k(x − y)2 ( y − z)2 (z − x)2 ,
we get the following statement:
• Let x, y, z be real numbers, and let

⎪ (1 + k)2

⎪ , k ∈ (−∞, −1] ∪ [2, ∞)

⎪ 4
⎨ 
(5 − 4k)(1 + k)2 1
βk∗ = , k ∈ −1, .

⎪ 4(2 − k) 2 2

⎪ 

⎩ 1 + 4k , 1
k∈ ,2
4 2
Then,

yz(x − y)(x − z)(x − k y + z)(x − kz + y) + βk∗ (x − y)2 ( y − z)2 (z − x)2 ≥ 0,

with equality for x = y = z, for y = z = 0 (or any cyclic permutation), for x/(k−1) =
y = z (or any cyclic permutation) if k = 1, and for x = 0 and y + z = 0 (or any cyclic
permutation) if k ∈ (−∞, −1] ∪ [2, ∞). If k = 1, then the equality holds also for
x = 0 and y = z (or any cyclic permutation).
Highest Coefficient Cancellation Method for Real Variables 307

P 3.51. Let x, y, z be real numbers, and let



⎪ (k + 1)(5k − 3)

⎪ , k ∈ (−∞, −5] ∪ [1, ∞)

⎪ 16

(k + 1)(k − 7k − 16k − 32)
3 2
γk = , k ∈ [−5, −2] .

⎪ 4(k − 1)2


⎪ 2
⎩ k , k ∈ [−2, 1]
4
Then,

(x 2 − y 2 )(x 2 − z 2 )(x − k y)(x − kz) + γk (x − y)2 ( y − z)2 (z − x)2 ≥ 0.

(Vasile C., 2009)

Solution. Denote

f (x, y, z) = (x 2 − y 2 )(x 2 − z 2 )(x − k y)(x − kz),

and write the desired inequality as f6 (x, y, z) ≥ 0, where

f6 (x, y, z) = f (x, y, z) + γk (x − y)2 ( y − z)2 (z − x)2 .

Since
(x + y)(x + z) = x 2 + q, (x − y)(x − z) = x 2 + 2 yz − q,
(x − k y)(x − kz) = x 2 + (k + k2 ) yz − kq,
f (x, y, z) has the same highest coefficient A1 as

P1 (x, y, z) = x 2 (x 2 + 2 yz)[x 2 + (k + k2 ) yz],

that is, according to (3.1),

A1 = P1 (1, 1, 1) = 9(k2 + k + 1).

Therefore, f6 (x, y, z) has the highest coefficient



⎪ 9(k + 5)2

⎪ , k ∈ (−∞, −5] ∪ [1, ∞)

⎪ 16

9(k + 2)2 (k + 5)2
A = A1 − 27γk = , k ∈ [−5, 2] .

⎪ 4(k − 1)2



⎩ 9(k + 2) ,
2
k ∈ [−2, 1]
4
In addition, we have
f6 (x, 1, 1) = (x 2 − 1)2 (x − k)2 ,
308 Vasile Cîrtoaje

4(x − 1)4 (x + 1)2 (x − k)2


f−1,k (x) = ,
9(k + 5)2 (x + 2)2
4(x − 1)4 (x + 1)2 4(x − 1)4 (x − k)2
f−1,−2 (x) = , f k,−2 (x) = .
81 81(k + 2)2

We will consider the following cases:

k ∈ {−5, −2}, k ∈ (−∞, −5) ∪ [1, ∞), k ∈ (−5, −2), k ∈ (−2, 1).

Case 1: k ∈ {−5, −2}. Since A = 0 and f6 (x, 1, 1) ≥ 0 for any real x, the conclusion
follows by Corollary 1.

Case 2: k ∈ (−∞, −5) ∪ [1, ∞). Since

9(k + 5)2 (x − 1)4 (x + 1)2 (x − k)2


A= , Af−1,k (x) = ,
16 4(x + 2)2

3(x 2 − 1)2 (x − k)2 (x + 5)(x + 1)


f6 (x, 1, 1) − Af−1,k (x) = ,
4(x + 2)2
we apply Theorem 2 for
η = −1, ξ = −1,

 = (−5, −1),  \  = (−∞, −5] ∪ [−1, ∞),


and for
Eα,β = f−1,−2 , Fγ,δ = f−1,k .

The condition (b), namely f6 (x, 1, 1)−Af−1,k (x) ≥ 0 for x ∈ (−∞, −5]∪[−1, ∞),
is satisfied.

The condition (a) is satisfied if f6 (x, 1, 1) ≥ Af−1,−2 (x) for x ∈ (−5, −1). We
have
(k + 5)2 (x − 1)4 (x + 1)2
Af−1,−2 (x) = ,
36
(k − 1)(x + 5)(x 2 − 1)2 [7k + 5 − (k + 11)x]
f6 (x, 1, 1) − Af−1,−2 (x) = .
36
Thus, it suffices to show that 7k + 5 − (k + 11)x ≤ 0 for k ∈ (−∞, −5), and 7k +
5 − (k + 11)x ≥ 0 for k ∈ [1, ∞). Indeed, for k ∈ (−∞, −5), we have

7k + 5 − (k + 11)x = k(7 − x) + 5 − 11x < (−5)(7 − x) + 5 − 11x = −6(x + 5) < 0,

and for k ∈ [1, ∞), we have

7k + 5 − (k + 11)x > 7k + 5 − (k + 11)(−1) = 8(k + 2) > 0.


Highest Coefficient Cancellation Method for Real Variables 309

Case 3: k ∈ (−5, −2). Since

9(k + 2)2 (k + 5)2 (k + 2)2 (x − 1)4 (x + 1)2 (x − k)2


A= , Af−1,k (x) = ,
4(k − 1)2 (k − 1)2 (x + 2)2

3(x 2 − 1)2 (x − k)3 [−(2k + 1)x + 4 − k]


f6 (x, 1, 1) − Af−1,k (x) = ,
(k − 1)2 (x + 2)2
we apply Theorem 2 for

4−k (k + 2)2
η= , ξ= ,
2k + 1 2k + 1
   
4−k 4−k
 = k, ,  \  = (−∞, k] ∪ ,∞ ,
2k + 1 2k + 1
and for
Eα,β = f k,−2 , Fγ,δ = f−1,k .
 
4−k
The condition (b), namely f6 (x, 1, 1)−Af−1,k (x) ≥ 0 for x ∈ (−∞, k]∪ ,∞ ,
2k + 1
is satisfied.
 
4−k
The condition (a) is satisfied if f6 (x, 1, 1) ≥ Af k,−2 (x) for x ∈ k, . We
2k + 1
have
(k + 5)2 (x − 1)4 (x − k)2
Af k,−2 (x) =
9(k − 1)2
and

f6 (x, 1, 1) − Af k,−2 (x) =


4(x − 1)2 (x − k)2 [2k + 1 + (k − 4)x][k − 4 + (2k + 1)x]
= ≥0
9(k − 1)2

because
4−k 3(k − 1)(k + 5)
2k + 1 + (k − 4)x > 2k + 1 + (k − 4) · = > 0,
2k + 1 2k + 1
4−k
k − 4 + (2k + 1)x > k − 4 + (2k + 1) · = 0.
2k + 1
Case 4: k ∈ (−2, 1). Since

9(k + 2)2 (k + 2)2 (x − 1)4 (x + 1)2 (x − k)2


A= , Af−1,k (x) = ,
4 (k + 5)2 (x + 2)2

(x − 1)2 (x + 1)2 (x − k)2 h(x)


f6 (x, 1, 1) − Af−1,k (x) = ,
(k + 5)2 (x + 2)2
310 Vasile Cîrtoaje

where

h(x) = (k + 5)2 (x + 2)2 − (k + 2)2 (x − 1)2


= 3(x + k + 4)[(2k + 7)x + k + 8],

we apply Theorem 2 for

−(k + 2)2
η = −k − 4, ξ= ,
2k + 7
   
−k − 8 −k − 8
 = −k − 4, ,  \  = (−∞, −k − 4] ∪ ,∞ .
2k + 7 2k + 7
and for
Eα,β = fα,−2 , Fγ,δ = f−1,k .

 (b), namely f6 (x, 1, 1) − Af−1,k (x) ≥ 0 for x ∈ (−∞, −k − 4] ∪


 The condition
−k − 8
, ∞ , is satisfied.
2k + 7
The condition
 (a) is satisfied
 if there is a real α such that f6 (x, 1, 1) ≥ Afα,−2 (x)
−k − 8
for x ∈ −k − 4, . Since
2k + 7

(k + 2)2 (x − 1)4 (x − α)2


Afα,−2 =
9(2 + α)2

the inequality is equivalent to

(x − 1)2 g1 (x)g2 (x) ≥ 0,


where

g1 (x) = 3(α + 2)(x + 1)(x − k) − (k + 2)(x − 1)(x − α)


= (x + 2)[(3α − k + 4)x − (2k + 1)α − 3k],

g2 (x) = 3(α + 2)(x + 1)(x − k) + (k + 2)(x − 1)(x − α).


 
−k − 8
Since −2 ∈ −k − 4, , we choose
2k + 7

−k − 8
α=
2k + 7
to get

2(1 − k)(k + 2)(x + 2)2 2(k + 2)g3 (x)


g1 (x) = ≥ 0, g2 (x = ,
2k + 7 2k + 7
Highest Coefficient Cancellation Method for Real Variables 311

where
g3 (x) = (k + 8)x 2 + 5(1 − k)x − 5k − 4.
Since
−k − 8
g3 (x) = 2(k + 8)x + 5(1 − k) < 2(k + 8) ·
+ 5(1 − k)
2k + 7
−3(4k + 19k + 31) −3(2k + 19k + 30)
2 2
= <
2k + 7 2k + 7
−3(k + 2)(2k + 15)
= < 0,
2k + 7
g3 is strictly decreasing on , therefore
 
−k − 8 9(1 − k)(k + 2)2
g3 (x) > g3 = > 0.
2k + 7 (2k + 7)2

The proof is completed. The equality holds for x = y = z, for x/k = y = z (or
any cyclic permutation) if k = 0, for −x = y = z (or any cyclic permutation), and
for x = 0 and y + z = 0 (or any cyclic permutation) if k ∈ [−2, 1]. If k = 0, then
the equality holds also for x = 0 and y = z (or any cyclic permutation).

Observation 1. The coefficient γk of the product (x − y)2 ( y − z)2 (z − x)2 is the


best possible. Setting x = k, y = 1 + t and z = 1 − t, the inequality in P 3.51 turns
into
A(k, γk )t 6 + B(k, γk )t 4 + C(k, γk )t 2 ≥ 0,
where
A(k, γk ) = 4γk − k2 ,
C(k, γk ) = (k − 1) [4(k − 1)2 γk − (k + 1)(k3 − 7k2 − 16k − 32)].
2

From the necessary conditions A(k, γk ) ≥ 0 and C(k, γk ) ≥ 0, we get

k2 (k + 1)(k3 − 7k2 − 16k − 32)


γk ≥ , γk ≥ .
4 4(k − 1)2

Also, for x = −1, y = 1 + t and z = 1 − t, the inequality in P 3.51 becomes

A(k, γk )t 6 + B(k, γk )t 4 + C(k, γk )t 2 ≥ 0,


where

A(k, γk ) = 4γk − k2 , C(k, γk ) = 4[16γk − (k + 1)(5k − 3)].

From the necessary conditions A(k, γk ) ≥ 0 and C(k, γk ) ≥ 0, we get

k2 (k + 1)(5k − 3)
γk ≥ , γk ≥ .
4 16
312 Vasile Cîrtoaje

Observation 2. There are some relevant particular cases of the inequality in P 3.51.
• For k = 1, the inequality turns into
 1
(x + y)(x + z)(x − y)2 (x − z)2 + (x − y)2 ( y − z)2 (z − x)2 ≥ 0,
4
which is equivalent to
 2 2
x (2x − y − z) + y 2 (2 y − z − x) + z 2 (2z − x − y) ≥ 0.

The equality holds for x 2 (2x − y − z) + y 2 (2 y − z − x) + z 2 (2z − x − y) = 0.


• For k = −2, the inequality turns into

(x 2 − y 2 )(x 2 − z 2 )(x + 2 y)(x + 2z) + (x − y)2 ( y − z)2 (z − x)2 ≥ 0,

which is equivalent to

(x + y + z)2 (x 4 + y 4 + z 4 − x 2 y 2 − y 2 z 2 − z 2 x 2 ) ≥ 0.

The equality holds for x = y = z, for x + y + z = 0, and for −x = y = z (or any


cyclic permutation).
• For k = −5, the inequality turns into

(x 2 − y 2 )(x 2 − z 2 )(x + 5 y)(x + 5z) + 7(x − y)2 ( y − z)2 (z − x)2 ≥ 0,

which is equivalent to

(x 2 + y 2 + z 2 + 3x y + 3 yz + 3z x)2 (x 2 + y 2 + z 2 − x y − yz − z x) ≥ 0.

The equality holds for x = y = z, and for x 2 + y 2 + z 2 + 3x y + 3 yz + 3z x = 0.


Observation 3. Substituting k − 1 for k and using then the identity

(x 2 − y 2 )(x 2 − z 2 )[x − (k − 1) y][x − (k − 1)z] =

= (x 2 − y 2 )(x 2 − z 2 )(x − k y + z)(x − kz + y) + k(x − y)2 ( y − z)2 (z − x)2 ,

we get the following statement:


• Let x, y, z be real numbers, and let

⎪ k(5k + 8)

⎪ , k ∈ (−∞, −4] ∪ [2, ∞)

⎪ 16

k(k + 1) (k − 8)
2
γ∗k = , k ∈ [−4, −1] .

⎪ 4(k − 2)2

⎪ (k + 1)2

⎩ , k ∈ [−1, 2]
4
Highest Coefficient Cancellation Method for Real Variables 313

Then,

(x 2 − y 2 )(x 2 − z 2 )(x − k y + z)(x − kz + y) + γ∗k (x − y)2 ( y − z)2 (z − x)2 ≥ 0,

with equality for x = y = z, for x/(k − 1) = y = z (or any cyclic permutation) if


k = 1, for −x = y = z (or any cyclic permutation), and for x = 0 and y + z = 0 (or
any cyclic permutation) if k ∈ [−1, 2]. If k = 1, then the equality holds also for x = 0
and y = z (or any cyclic permutation).

P 3.52. If x, y, z are real numbers, then



yz(4x 2 + 3 yz)(x − y)(x − z) + (x − y)2 ( y − z)2 (z − x)2 ≥ 0.

(Vasile C., 2014)

Solution. We write the inequality as f6 (x, y, z) ≥ 0, where



f6 (x, y, z) = yz(4x 2 + 3 yz)(x − y)(x − z) + (x − y)2 ( y − z)2 (z − x)2 .

Since (x − y)(x − z) = x 2 + 2 yz − q, f6 (x, y, z) has the same highest coefficient A


as 
yz(4x 2 + 3 yz)(x 2 + 2 yz) + (x − y)2 ( y − z)2 (z − x)2 ,
that is, according to (3.1),
A = 63 − 27 = 36.
Since
f6 (x, 1, 1) = (4x 2 + 3)(x − 1)2 ,
4(x − 1)4
f∞,−2 (x) = ,
81
(x − 1)2 (10x + 11)(2x + 1)
f6 (x, 1, 1) − Af∞,−2 (x) = ,
9
we apply Theorem 2 for ξ → ∞, which involves
  
−1 −1
= ,∞ ,  \  = −∞, ,
2 2
and for
Eα,β = f∞,−2 , Eγ,δ = h0,δ ,

The condition (a), namely f6 (x, 1, 1) − Af∞,−2 (x) ≥ 0 for x > −1/2, is satisfied.
The condition (b) is satisfied if f6 (x, 1, 1) ≥ Ah0,δ (x) for x ≤ −1/2. We have

Ah0,δ (x) = 36[x + δ(x + 2)3 ]2 ,


314 Vasile Cîrtoaje

f6 (x, 1, 1) − Ah0,δ (x) = f (x),


f (x) = (4x + 3)(x − 1)2 − 36[x + δ(x + 2)(2x + 1)]2 .
2

Since f (−1/2) = 0, a necessary condition to have f (x) ≥ 0 in a vicinity of x = −1/2


is f (−1/2) = 0. This implies δ = −5/36, when

36 f (x) = 36(4x 2 + 3)(x − 1)2 − (10x 2 − 11x + 10)2


= 44x 4 − 68x 3 − 69x 2 + 4x + 8
= (2x + 1)2 (11x 2 − 28x + 8) ≥ 0.

The equality holds for x = y = z, and for y = z = 0 (or any cyclic permutation).
Observation 1. Similarly, we can prove the following generalization:
• Let x, y, z be real numbers. If


1 13 + 3 33
k∈ , k1 , k1 = ≈ 0.9448,
2 32
then
 1
yz(x 2 + k yz)(x − y)(x − z) + (x − y)2 ( y − z)2 (z − x)2 ≥ 0,
4
with equality for x = y = z, and also for y = z = 0 (or any cyclic permutation).
For
 1
f6 (x, y, z) = yz(x 2 + k yz)(x − y)(x − z) + (x − y)2 ( y − z)2 (z − x)2 ,
4
we have
9
A = (4k + 1),
4
f6 (x, 1, 1) = (x 2 + k)(x − 1)2 ,

Condition (a). For x > −1/2, we have


(x − 1)2 [2(2 − k)x + 5k − 1](2x + 1)
f6 (x, 1, 1) − Af∞,−2 (x) = ≥ 0.
9
Condition (b). For x ≤ −1/2, choosing
1 − 8k
δ= ,
9(1 + 4k)
we get
9(1 + 4k) f6 (x, 1, 1) − [(1 − 8k)x 2 + (7 − 2k)x + 1 − 8k]2
f6 (x, 1, 1) − Ah0,δ (x) =
9(1 + 4k)
(2x + 1)2 g(x)
= ,
9(1 + 4k)
Highest Coefficient Cancellation Method for Real Variables 315

where
g(x) = (2 + 13k − 16k2 )x 2 − 2(5 + k − 4k2 )x − 1 + 25k − 28k2
= (2 + 13k − 16k2 )(x 2 + 2) − (5 + k − 4k2 )(2x + 1) ≥ 0.

Observation 2. Actually, the following more general statement holds:


• Let x, y, z be real numbers. If

13 + 3 33
k ∈ [0, k1 ] , k1 = ≈ 0.9448,
32
then
 1
yz(x 2 + k yz)(x − y)(x − z) + (x − y)2 ( y − z)2 (z − x)2 ≥ 0,
4
with equality for x = y = z, and also for y = z = 0 (or any cyclic permutation).
Since the left hand side of the inequality is linear in k, the inequality holds for
k ∈ [0, k1 ] if and only if it holds for k = 0 and k = k1 . These cases are treated in P
3.50 and Observation 1, respectively.
Observation 3. Replacing x, y, z with 1/x, 1/ y, 1/z, respectively, the statement
from Observation 2 becomes as follows:
• Let x, y, z be real numbers. If

13 + 3 33
k ∈ [0, k1 ] , k1 = ≈ 0.9448,
32
then
 1
yz(k x 2 + yz)(x − y)(x − z) + (x − y)2 ( y − z)2 (z − x)2 ≥ 0,
4
with equality for x = y = z, and also for y = z = 0 (or any cyclic permutation).

P 3.53. If x, y, z are real numbers, then


 1
x 4 (x + 2 y)(x + 2z) + 5x 2 y 2 z 2 + (x − y)2 ( y − z)2 (z − x)2 ≥ 0.
2
(Vasile C., 2011)

Solution. Write the inequality as f6 (x, y, z) ≥ 0, where


 1
f6 (x, y, z) = x 4 (x + 2 y)(x + 2z) + 5x 2 y 2 z 2 + (x − y)2 ( y − z)2 (z − x)2 .
2
Since
(x + 2 y)(x + 2z) = x 2 + 2 yz + 2q,
316 Vasile Cîrtoaje

f6 (x, y, z) has the same highest coefficient as


 1
x 4 (x 2 + 2 yz) + 5x 2 y 2 z 2 + (x − y)2 ( y − z)2 (z − x)2 ,
2
that is, according to (3.1),
27 1
A=9+5− = .
2 2
We have
1 2 2
y z ( y − z)2 ,
f6 (0, y, z) = y 6 + z 6 + 2 yz( y 4 + z 4 ) +
2
f6 (0, 1, −1) = 2 − 4 + 2 = 0,

f6 (x, 1, 1) = x 4 (x + 2)2 + 6(2x + 1) + 5x 2


= x 6 + 4x 5 + 4x 4 + 5x 2 + 12x + 6
= (x + 1)2 g(x),

where
g(x) = x 4 + 2x 3 − x 2 + 6.
Since
f6 (−1, 1, 1) = 0, f6 (0, 1, −1) = 0,
we apply Corollary 2 for Fγ,δ = g−1,δ , where δ is given by (3.16):

γ (γ + 2)2 g (−2) −1 g (−2)


δ= + = + .
3(γ + 2) 12A 3 6

According to Remark 4, it suffices to show that

g(x) ≥ Aḡ−1,δ (x)

for x ∈ , where

ḡ−1,δ (x) = [x 2 + 5x + 8 + δ(x + 2)(x + 5)]2 .

We have
g (x) = 4x 3 + 6x 2 − 2x, g (−2) = −4, δ = −1,
ḡ−1,−1 (x) = [x + 5x + 8 − (x + 2)(x + 5)] = 4(x + 1)2 ,
2 2

g(x) − Aḡ−1,−1 (x) = x 4 + 2x 3 − 3x 2 − 4x + 4 = (x + 2)2 (x − 1)2 ≥ 0.

The equality holds for −x = y = z (or any cyclic permutation), and for x = 0
and y + z = 0 (or any cyclic permutation).
Highest Coefficient Cancellation Method for Real Variables 317

P 3.54. If x, y, z are real numbers, no two of which are zero, then


 1 9
≥ .
2 y 2 − 3 yz + 2z 2 4(x 2 + y 2 + z 2 ) − 3(x y + yz + z x)

(Vasile C., 2009)

Solution. Write the inequality as f6 (x, y, z) ≥ 0, where


 (
f6 (x, y, z) = P(x, y, z) (2x 2 −3x y+2 y 2 )(2x 2 −3xz+2z 2 )−9 (2 y 2 −3 yz+2z 2 ),

P(x, y, z) = 4(x 2 + y 2 + z 2 ) − 3(x y + yz + z x).


Since
2 y 2 − 3 yz + 2z 2 = −2x 2 − 3 yz + 2(p2 − 2q),
f6 (x, y, z) has the same highest coefficient A as
(
P3 (x, y, z) = −9 (−2x 2 − 3 yz),

that is
A = P3 (1, 1, 1) = −9(−2 − 3)3 = 1125.
Since
P(x, 1, 1) = 4x 2 − 6x + 5,

f6 (x, 1, 1)
= (4x 2 − 6x + 5)[(2x 2 − 3x + 2) + 2] − 9(2x 2 − 3x + 2)
2x 2 − 3x + 2
= 2(4x 4 − 12x 3 + 13x 2 − 6x + 1) = 2(x − 1)2 (2x − 1)2 ,

we apply Corollary 2 for

4(x − 1)4 (2x − 1)2


Fγ,δ (x) = f1/2,−2 (x) = .
2025
We need to show that
f6 (x, 1, 1) ≥ Af1/2,−2 (x)
for x ∈ . We have
20(x − 1)4 (2x − 1)2
Af1/2,−2 (x) = ,
9
2(x − 1)2 (2x − 1)2 f (x)
f6 (x, 1, 1) − Af1/2,−2 (x) = ,
9
where
f (x) = 9(2x 2 − 3x + 2) − 10(x − 1)2 = 8x 2 − 7x + 8 > 0.

The equality holds for x = y = z, and for 2x = y = z (or any cyclic permutation).
Observation. Similarly, we can prove the following generalization:
318 Vasile Cîrtoaje

• Let x, y, z be real numbers, no two of which are zero. If −2 ≤ k ≤ 2, then


 1 9
≥ ,
y 2 + k yz + z 2 2(x 2 + y 2 + z 2 ) + k(x y + yz + z x)
with equality for x = y = z, and for −x/(k + 1) = y = z (or any cyclic permutation)
if k = −1. If k = −1, then the equality holds also for x = 0 and y = z (or any cyclic
permutation).
For
 (
f6 (x, y, z) = P(x, y, z) (x 2 + k x y + y 2 )(x 2 + k xz + z 2 ) − 9 ( y 2 + k yz + z 2 ),

P(x, y, z) = 2(x 2 + y 2 + z 2 ) + k(x y + yz + z x),


we have
A = −9(k − 1)3 ,
P(x, 1, 1) = 2x 2 + 2k x + k + 4,
f6 (x, 1, 1)
=P(x, 1, 1)[x 2 + k x + 1 + 2(k + 2)] − 9(k + 2)(x 2 + k x + 1)
x2 + kx + 1
= 2x 4 + 4k x 3 + (2k2 − 4k − 4)x 2 − 4k(k + 1)x + 2(k + 1)2
= 2(x − 1)2 (x + k + 1)2 .

Case 1: 1 ≤ k ≤ 2. Since A ≤ 0, it suffices to show that f6 (x, 1, 1) ≥ 0 for x ∈ 


(see Corollary 1), which is true.

Case 2: −2 ≤ k < 1. We apply Corollary 2 for


4(x − 1)4 (x + k + 1)2
Fγ,δ (x) = f−k−1,−2 (x) = .
81(1 − k)2
We need to show that
f6 (x, 1, 1) ≥ Af−k−1,−2 (x)
for x ∈ . We have
4(1 − k)(x − 1)4 (x + k + 1)2
Af−k−1,−2 (x) = ,
9
2(x − 1)2 (x − k − 1)2 f (x)
f6 (x, 1, 1) − Af−k−1,−2 (x) = ,
9
where
f (x) = 9(x 2 + k x + 1) − 2(1 − k)(x − 1)2
= (2k + 7)x 2 + (5k + 4)x + 2k + 7
 
5k + 4 2 9(k + 2)(10 − k)
= (2k + 7) x + + ≥ 0.
4k + 14 4(2k + 7)
Highest Coefficient Cancellation Method for Real Variables 319

P 3.55. Let x, y, z be real numbers. If k > 1, then


k x 2 + 2 yz k y 2 + 2z x kz 2 + 2x y k−1
+ + ≥ .
k x 2 + y 2 + z 2 k y 2 + z 2 + x 2 kz 2 + x 2 + y 2 k+1
(Vasile C., 2011)
Solution. We write the inequality as f6 (x, y, z) ≥ 0, where

f6 (x, y, z) =(k + 1) (k x 2 + 2 yz)(k y 2 + z 2 + x 2 )(kz 2 + x 2 + y 2 )
(
− (k − 1) (k x 2 + y 2 + z 2 ).

From

f6 (x, y, z) =(k + 1) (k x 2 + 2 yz)[(k − 1) y 2 + p2 − 2q][(k − 1)z 2 + p2 − 2q]
(
− (k − 1) [(k − 1)x 2 + p2 − 2q],

it follows that f6 (x, y, z) has the same highest coefficient A as



f (x, y, z) =(k + 1)(k − 1)2 y 2 z 2 (k x 2 + 2 yz) − (k − 1)4 x 2 y 2 z 2
 
=(k + 1)(k − 1)2 3kr 2 + 2 y 3 z 3 − (k − 1)4 r 2 ,

that is

A = (k + 1)(k − 1)2 (3k + 6) − (k − 1)4 = (k − 1)2 (k + 5)(2k + 1) > 0.

We have

f6 (0, y, z) =(k + 1)[2 yz(k y 2 + z 2 )(kz 2 + y 2 ) + k( y 2 + z 2 )(2k y 2 z 2 + y 4 + z 4 ]


− (k − 1)( y 2 + z 2 )(k y 2 + z 2 )(kz 2 + y 2 ),

f6 (0, 1, −1) = (k + 1)[−2(k + 1)2 + 4k(k + 1)] − 2(k − 1)(k + 1)2 = 0,

f6 (x, 1, 1) =(k x 2 + 2)(x 2 + k + 1)[(k + 1)(x 2 + k + 1) + 2(k + 1)(2x + k)


− (k − 1)(x 2 + k + 1)] = (x + k + 1)2 g(x),

where
g(x) = 2(k x 2 + 2)(x 2 + k + 1).
Since
f6 (−k − 1, 1, 1) = 0, f6 (0, 1, −1) = 0,
we apply Corollary 2 for Fγ,δ = g−k−1,δ , where δ is given by (3.16):

γ (γ + 2)2 g (−2)
δ= +
3(γ + 2) 12A
k+1 g (−2)
= + .
3(k − 1) 12(k + 5)(2k + 1)
320 Vasile Cîrtoaje

According to Remark 4, it suffices to show that

g(x) ≥ Aḡ−k−1,δ (x)

for x ∈ , where
 2
γx 2 + γ(γ + 6)x − 8 δ(x + 2)(2γx + x + γ − 4)
ḡ−k−1,δ (x) = +
(γ + 2)3 (γ + 2)2

(k + 1)x 2 − (k + 1)(k − 5)x + 8 δ(x + 2)(2k x + x + k + 5)
= − .
(k − 1)3 (k − 1)2

We have

g (x) = 4x(2k x 2 + k2 + k + 2), g (−2) = −8(k2 + 9k + 2),

−k2 + 10k + 3
δ= ,
(k − 1)(k + 5)(2k + 1)
4[(2k + 1)x 2 − (k2 + k − 2)x + k + 5]2
ḡ−k−1,δ (x) = ,
(k − 1)2 (k + 5)2 (2k + 1)2
4[(2k + 1)x 2 − (k2 + k − 2)x + k + 5]2
Aḡ−k−1,δ (x) = ,
(k + 5)(2k + 1)
2g1 (x)
g(x) − Aḡ−k−1,δ (x) = ,
(k + 5)(2k + 1)

g1 (x) = (k + 5)(2k + 1)(k x 2 + 2)(x 2 + k + 1) − 2[(2k + 1)x 2 − (k2 + k − 2)x + k + 5]2


= (k − 1)(k + 2)(x + 2)2 [(2k + 1)x 2 + k + 5] ≥ 0.

The equality holds for −x/(k + 1) = y = z (or any cyclic permutation), and for
x = 0 and y + z = 0 (or any cyclic permutation).

P 3.56. If x, y, z are real numbers such that x y + yz + z x < 0, then

1 1 1 1
+ + + ≤ 0.
3x 2 + y 2 + z 2 3 y 2 + z 2 + x 2 3z 2 + x 2 + y 2 x y + yz + z x

(Vasile C., 2011)

Solution. Since x y + yz + z x < 0, we may write the inequality as f6 (x, y, z) ≥ 0,


where
 (
f6 (x, y, z) = (x y + yz + z x) (3 y 2 + z 2 + x 2 )(3z 2 + x 2 + y 2 ) + (3x 2 + y 2 + z 2 ).
Highest Coefficient Cancellation Method for Real Variables 321

Since ( (
(3x 2 + y 2 + z 2 ) =
(2x 2 + p2 − 2q),
+
f6 (x, y, z) has the same highest coefficient as (2x 2 ), that is

A = 8.
We have

f6 (0, y, z) = yz[(3 y 2 + z 2 )(3z 2 + y 2 ) + 4( y 2 + z 2 )2 ] + ( y 2 + z 2 )(3 y 2 + z 2 )(3z 2 + y 2 ),

f6 (0, 1, −1) = −(16 + 16) + 32 = 0,

f6 (x, 1, 1) = (2x + 1)[(x 2 + 4)2 + 2(x 2 + 4)(3x 2 + 2)] + (x 2 + 4)2 (3x 2 + 2)


= (x 2 + 4)(3x 4 + 14x 3 + 21x 2 + 16x + 16).

Since
f6 (0, 1, −1) = 0,
we apply Corollary 2 for Fγ,δ = hγ,δ , where γ is given by (3.19):

1 h (−2) 1 h (−2)
γ= + = + ,
3 12A 3 96
with

h(x) = (x 2 + 4)(3x 4 + 14x 3 + 21x 2 + 16x + 16)


= 3x 6 + 14x 5 + 33x 4 + 72x 3 + 100x 2 + 64x + 64.

We have
h (x) = 2(9x 5 + 35x 4 + 66x 3 + 108x 2 + 100x + 32),
1 1 1
h (−2) = 16, γ= + = ,
3 6 2
1
h1/2,δ (x) = [x + (x + 2)(2x + 1) + δ(x + 2)3 ]2 .
2
Choosing
−1
δ= ,
4
we get
 2
1 1
h1/2,−1/4 (x) = x + (x + 2)(2x + 1) − (x + 2)3
2 4
1 3
= (x + 2x 2 − 2x + 4)2 ,
16
1
f6 (x, 1, 1) − Ah1/2,−1/4 (x) = (x + 2)2 (5x 4 + 4x 3 + 30x 2 + 8x + 28).
2
322 Vasile Cîrtoaje

For x ≥ 0, the condition f6 (x, 1, 1) − Ah1/2,−1/4 (x) ≥ 0 is clearly true. It is also true
for x < 0, since

5x 4 + 4x 3 + 30x 2 + 8x + 28 > 5(x 4 + x 3 + 6x 2 + 2x + 5)


= 5x 2 (x 2 + x + 1) + 5(5x 2 + 2x + 5)] > 0.

The equality holds for x = 0 and y + z = 0 (or any cyclic permutation).

P 3.57. If x, y, z are real numbers, then

x 2 − yz y2 − z x z2 − x y
+ + 2 ≤ 1.
3x 2 + y +z
2 2 3y + z + x
2 2 2 3z + x 2 + y 2
(Vasile C., 2011)
Solution. Write the inequality as f6 (x, y, z) ≥ 0, where
( 
f6 (x, y, z) = (3x 2 + y 2 + z 2 ) − (x 2 − yz)(3 y 2 + z 2 + x 2 )(3z 2 + x 2 + y 2 ).

From
( 
f6 (x, y, z) = (2x 2 + p2 − 2q) − (x 2 − yz)(2 y 2 + p2 − 2q)(2z 2 + p2 − 2q),

it follows that f6 (x, y, z) has the same highest coefficient as


 
g(x, y, z) = 8x 2 y 2 z 2 − 4 y 2 z 2 (x 2 − yz) = −4x 2 y 2 z 2 + 4 y 3z3.

Therefore, f6 (x, y, z) has the highest coefficient

A = −4 + 12 = 8.

We have

f6 (0, y, z) = ( y 2 + z 2 + yz)(k y 2 + z 2 )(kz 2 + y 2 ) − ( y 2 + z 2 )( y 4 + z 4 + 6 y 2 z 2 ),

f6 (0, 1, −1) = 16 − 16 = 0,

f6 (x, 1, 1) = (3x 2 + 2)(x 2 + 4)2 − (x 2 − 1)(x 2 + 4)2 − 2(1 − x)(3x 2 + 2)(x 2 + 4)


= (x 2 + 4)(2x 4 + 6x 3 + 5x 2 + 4x + 8).

Since
f6 (0, 1, −1) = 0,
we apply Corollary 2 for Fγ,δ = hγ,δ , where γ is given by (3.19):

1 h (−2) 1 h (−2)
γ= + = + ,
3 12A 3 96
Highest Coefficient Cancellation Method for Real Variables 323

with

h(x) = (x 2 + 4)(2x 4 + 6x 3 + 5x 2 + 4x + 8)
= 2x 6 + 6x 5 + 13x 4 + 28x 3 + 28x 2 + 16x + 32.
We have
h (x) = 2(6x 5 + 15x 4 + 26x 3 + 42x 2 + 28x + 8),
1 5 −1
h (−2) = −80, γ= − = ,
3 6 2
1
h−1/2,δ (x) = [x − (x + 2)(2x + 1) + δ(x + 2)3 ]2 .
2
Choosing
δ = 0,
we get
 2
1
h−1/2,0 (x) = x − (x + 2)(2x + 1)
2
1
= (2x 2 + 3x + 2)2 ,
4

f6 (x, 1, 1) − Ah−1/2,0 (x) = (x + 2)2 (2x 4 − 2x 3 + 5x 2 − 8x + 6)


≥ (x + 2)2 (x 4 − 2x 3 + 5x 2 − 8x + 4)
= (x + 2)2 (x − 1)2 (x 2 + 4) ≥ 0.

The equality holds for x = 0 and y + z = 0 (or any cyclic permutation).

P 3.58. Let x, y, z be real numbers. If k > 1, then


yz zx xy 1
+ + + ≥ 0.
k x 2 + y 2 + z 2 k y 2 + z 2 + x 2 kz 2 + x 2 + y 2 2
(Vasile C., 2011)

Solution. Write the inequality as f6 (x, y, z) ≥ 0, where


 (
f6 (x, y, z) = 2 yz(k y 2 + z 2 + x 2 )(kz 2 + x 2 + y 2 ) + (k x 2 + y 2 + z 2 ).

From
 (
f6 (x, y, z) = 2 yz[(k−1) y 2 +p2 −2q][(k−1)z 2 +p2 −2q]+ [(k−1)x 2 +p2 −2q],

it follows that f6 (x, y, z) has the same highest coefficient as



g(x, y, z) = 2(k − 1)2 y 3 z 3 + (k − 1)3 x 2 y 2 z 2 ,
324 Vasile Cîrtoaje

that is
A = 6(k − 1)2 + (k − 1)3 = (k − 1)2 (k + 5) ≥ 0.
We have

f6 (0, y, z) = 2 yz(k y 2 + z 2 )(kz 2 + y 2 ) + ( y 2 + z 2 )(k y 2 + z 2 )(kz 2 + y 2 ),

f6 (0, 1, −1) = −2(k + 1)2 + 2(k + 1)2 = 0,

f6 (x, 1, 1) = 2(x 2 + k + 1)2 + 4x(x 2 + k + 1)(k x 2 + 2) + (k x 2 + 2)(x 2 + k + 1)2


= (x 2 + k + 1)[k x 4 + 4k x 3 + (k2 + k + 4)x 2 + 8x + 4k + 4].

Since
f6 (0, 1, −1) = 0,
we apply Corollary 2 for Fγ,δ = hγ,0 , where γ is given by (3.19):

1 h (−2) 1 h (−2)
γ= + = + ,
3 12A 3 12(k − 1)2 (k + 5)
with

h(x) =(x 2 + k + 1)[k x 4 + 4k x 3 + (k2 + k + 4)x 2 + 8x + 4k + 4]


=k x 6 + 4k x 5 + 2(k2 + k + 2)(x 4 + 2x 3 ) + (k + 1)(k2 + k + 8)x 2
+ 8(k + 1)x + 4(k + 1)2 .

We have

h (x) = 2[3k x 5 + 10k x 4 + 2(k2 + k + 2)(2x 3 + 3x 2 ) + (k + 1)(k2 + k + 8)x + 4k + 4],

h (−2) = −4(k − 1)(k2 + 7k − 14),


1 k2 + 7k − 14 3−k
γ= − = ,
3 3(k − 1)(k + 5) (k − 1)(k + 5)
 2
k−3
hγ,0 (x) = x − (x + 2)(2x + 1) ,
(k − 1)(k + 5)
1  2
Ahγ,0 (x) = (6 − 2k)x 2 + (k2 − k + 10)x + 6 − 2k ,
k+5
1
f6 (x, 1, 1) − Ahγ2,0 (x) = (x + 2)2 g(x),
k+5
where

g(x) = k(k + 5)x 4 + 2(k3 + 2k2 + 9k − 8)x 2 − 8(k2 − k + 2)x + k3 + 6k2 + 17k − 4.

Since
k3 + 2k2 + 9k − 8 ≥ 3k2 + 9k − 8 > 2(k2 − k + 2)
Highest Coefficient Cancellation Method for Real Variables 325

and
k3 + 6k2 + 17k − 4 ≥ 7k2 + 17k − 4 > 4(k2 − k + 2),
we have

g(x) > 4(k2 − k + 2)x 2 − 8(k2 − k + 2)x + 4(k2 − k + 2)


= 4(k2 − k + 2)(x − 1)2 ≥ 0.

The equality holds for x = 0 and y + z = 0 (or any cyclic permutation).

P 3.59. If x, y, z are real numbers, then


yz zx xy 1
+ + + ≥ 0.
x 2 + 4 y 2 + 4z 2 y 2 + 4z 2 + 4x 2 z 2 + 4x 2 + 4 y 2 8

(Vasile C., 2011)

Solution. This inequality is the inequality from the preceding P 3.58 for
1
k= ,
4
but the proof of P 3.58 given in the case k > 1 and using Corollary 2 for δ = 0 fails.
To prove this inequality, we will apply Corollary 2 for Fγ,δ = hγ,δ , where

1 2
δ= = ,
2(k + 5) 21
3−k −44
γ= = .
(k − 1)(k + 5) 63
We have
 (
f6 (x, y, z) = 2 yz(k y 2 + z 2 + x 2 )(kz 2 + x 2 + y 2 ) + (k x 2 + y 2 + z 2 ),

A = (k − 1)2 (k + 5),

f6 (x, 1, 1) =(x 2 + k + 1)[k x 4 + 4k x 3 + (k2 + k + 4)x 2 + 8x + 4k + 4]


=k x 6 + 4k x 5 + 2(k2 + k + 2)(x 4 + 2x 3 ) + (k + 1)(k2 + k + 8)x 2
+ 8(k + 1)x + 4(k + 1)2 ,
 2
k−3 1
hγ,δ (x) = x − (x + 2)(2x + 1) + (x + 2)3 ,
(k − 1)(k + 5) 2(k + 5)
1
f6 (x, 1, 1) − Ahγ,δ (x) = (x + 2)2 (A1 x 4 + B1 x 3 + C1 x 2 + D1 x + E1 ),
4(k + 5)
326 Vasile Cîrtoaje

where

A1 = 3k2 + 22k − 1, B1 = 16(1 − k), C1 = 4k(k + 1)(k + 3),

D1 = 8(k + 1)(1 − 4k − k2 ), E1 = 4(k + 1)2 (k + 4).


For k = 1/4, the inequality f6 (x, 1, 1) ≥ Ahγ,δ (x) reduces to (x + 2)2 g(x) ≥ 0,
where
g(x) = 75x 4 + 192x 3 + 65x 2 − 10x + 425.
Clearly, g(x) > 0 for x ≥ 0. Also, for x < 0, we have

g(x) > 64x 4 + 192x 3 + 64x 2 + 256 = 64(x + 2)2 (x 2 − x + 1) ≥ 0.

The equality holds for x = 0 and y + z = 0 (or any cyclic permutation).

P 3.60. If x, y, z are real numbers, then


 1 7
≤ .
x 2 + 4 y 2 + 4z 2 4(x 2 + y 2 + z 2 ) + 3(x y + yz + z x)

(Vasile C., 2011)

Solution. Write the inequality as f6 (x, y, z) ≥ 0, where


( 
f6 (x, y, z) = 7 (x 2 + 4 y 2 + 4z 2 ) − P ( y 2 + 4z 2 + 4x 2 )(z 2 + 4x 2 + 4 y 2 ),

P = 4(x 2 + y 2 + z 2 ) + 3(x y + yz + z x).

+coefficient A of f6 (x, y, z) is equal with the highest coefficient of the


The highest
product 7 (x 2 + 4 y 2 + 4z 2 ). Since

x 2 + 4 y 2 + 4z 2 = −3x 2 + 4(p2 − 2q),

we have
A = 7(−3)3 = −189.
By Corollary 1, we only need to prove the original inequality for y = z = 1. Thus,
we need to show that
1 2 7
+ ≤ ,
x 2 + 9 4x 2 + 5 4x 2 + 6x + 11
which is equivalent to
(x − 1)2 (2x − 7)2 ≥ 0.
The equality holds for x = y = z, and also for 2x = 7 y = 7z (or any cyclic
permutation).
Highest Coefficient Cancellation Method for Real Variables 327

Observation. For x y + yz + z x > 0, using the Cauchy-Schwartz inequality

4 3 (4 + 3)2
+ ≥ ,
x2 + y +z
2 2 x y + yz + z x 4(x + y + z 2 ) + 3(x y + yz + z x)
2 2

we get the following inequality


 7 4 3
≤ 2 + ,
x 2 + 4 y 2 + 4z 2 x + y 2 + z 2 x y + yz + z x

with equality for x = y = z.

P 3.61. If x, y, z are real numbers such that x y + yz + z x ≥ 0, then


 2 45
≥ .
4x 2 + y 2 + z 2 14(x 2 + y 2 + z 2 ) + x y + yz + z x

(Vasile C., 2011)

Solution. Write the inequality as f6 (x, y, z) ≥ 0, where


 (
f6 (x, y, z) = 2P (4 y 2 + z 2 + x 2 )(4z 2 + x 2 + y 2 ) − 45 (4x 2 + y 2 + z 2 ),

P = 14(x 2 + y 2 + z 2 ) + x y + yz + z x.
The highest coefficient
+ A of f6 (x, y, z) is equal with the highest coefficient of the
product −45 (4x 2 + y 2 + z 2 ). Since

4x 2 + y 2 + z 2 = 3x 2 + 4(p2 − 2q),

we have
A = −45(3)3 < 0.
By Theorem 1, we only need to prove the original inequality for y = z = 1 and
2x + 1 ≥ 0. Thus, we need to prove that

1 4 45
+ ≥ ,
2x 2 + 1 x 2 + 5 14x 2 + 2x + 29
which is equivalent to
(x − 1)2 (2x + 1)(x + 2) ≥ 0.
The equality holds for x = y = z, and also for −2x = y = z (or any cyclic
permutation).

Observation. Similarly, we can prove the following generalization:


328 Vasile Cîrtoaje

• Let x, y, z be real numbers such that x y + yz + z x ≥ 0. If k > 1, then


 1 27(2k + 7)
≥ ,
k x 2 + y 2 + z2 (k + 8)(4k + 5)(x 2 + y 2 + z 2 ) + 2(k − 1)2 (x y + yz + z x)

with equality for x = y = z, and also for −2x = y = z (or any cyclic permutation).

For
 (
f6 (x, y, z) = P(x, y, z) (k y 2 +z 2 +x 2 )(kz 2 +x 2 + y 2 )−27(2k+7) (k x 2 + y 2 +z 2 ),

P(x, y, z) = (k + 8)(4k + 5)(x 2 + y 2 + z 2 ) + 2(k − 1)2 (x y + yz + z x),


we have
A = −27(2k + 7)(k − 1)3 < 0,
P(x, 1, 1) = (k + 8)(4k + 5)x 2 + 4(k − 1)2 x + 2(5k2 + 35k + 41),

f6 (x, 1, 1)
=P(x, 1, 1)[x 2 + k + 1 + 2(k x 2 + 2)] − 27(2k + 7)(x 2 + k + 1)(k x 2 + 2)
x2 + k + 1
= 2(k − 1)2 (x − 1)2 (2x + 1)[2(k + 5)x + 5k + 16] ≥ 0

for 2x + 1 ≥ 0.

P 3.62. If x, y, z are real numbers such that x y + yz + z x ≥ 0, then


 1 45
≥ .
x 2 + 4 y 2 + 4z 2 44(x 2 + y 2 + z 2 ) + x y + yz + z x

(Vasile C., 2011)

Solution. Write the inequality as f6 (x, y, z) ≥ 0, where


 (
f6 (x, y, z) = P ( y 2 + 4z 2 + 4x 2 )(z 2 + 4x 2 + 4 y 2 ) − 45 (x 2 + 4 y 2 + 4z 2 ),

P = 44(x 2 + y 2 + z 2 ) + x y + yz + z x.
The highest coefficient
+ A of f6 (x, y, z) is equal with the highest coefficient of the
product −45 (x 2 + 4 y 2 + 4z 2 ). Since

x 2 + 4 y 2 + 4z 2 = −3x 2 + 4(p2 − 2q),

we have
A = −45(−3)3 = 1215.
Highest Coefficient Cancellation Method for Real Variables 329

Since
f6 (x, y, z)
= (44x 2 + 2x + 89)[4x 2 + 5 + 2(x 2 + 8)] − 45(4x 2 + 5)(x 2 + 8)
4x 2 + 5
= 3(44x 2 + 2x + 89)(2x 2 + 7) − 45(4x 2 + 5)(x 2 + 8)
= 3(x − 1)2 (2x + 1)(14x + 23),

we apply Corollary 2 for

4(x − 1)4 (2x + 1)2


Fγ,δ (x) = f−1/2,−2 (x) = .
729
Thus, according to Remark 2, it suffices to show that

f6 (x, 1, 1) ≥ Af−1/2,−2 (x)

for 2x + 1 ≥ 0. We have
20(x − 1)4 (2x + 1)2
Af−1/2,−2 (x) = ,
3
(x − 1)2 (2x + 1) f (x)
f6 (x, 1, 1) − Af−1/2,−2 (x) = ,
3
where
f (x) = 9(4x 2 + 5)(14x + 23) − 20(x − 1)2 (2x + 1).
Since 4x + 5 > (x − 1)2 , we have
2

f (x) > 9(x − 1)2 (14x + 23) − 20(x − 1)2 (2x + 1) = (x − 1)2 (86x + 187) ≥ 0.

The equality holds for x = y = z, and also for −2x = y = z (or any cyclic
permutation).
Observation 1. Similarly, we can prove the following generalization:
• Let x, y, z be real numbers such that x y + yz + z x ≥ 0. If 0 ≤ k < 1, then
 1 27(2k + 7)
≥ .
k x 2 + y 2 + z2 (k + 8)(5k + 5)(x 2 + y 2 + z 2 ) + 2(k − 1)2 (x y + yz + z x)

with equality for x = y = z, and also for −2x = y = z (or any cyclic permutation).
For
 (
f6 (x, y, z) = P (k y 2 + z 2 + x 2 )(kz 2 + x 2 + y 2 ) − 27(2k + 7) (k x 2 + y 2 + z 2 ),

P = (k + 8)(5k + 5)(x 2 + y 2 + z 2 ) + 2(k − 1)2 (x y + yz + z x),


we have
A = −27(2k + 7)(k − 1)3 > 0,
330 Vasile Cîrtoaje

4(2k + 7)(1 − k)3 (x − 1)4 (2x + 1)2


Af−1/2,−2 (x) = ,
27
f6 (x, 1, 1) = 2(k − 1)2 (x 2 + k + 1)(x − 1)2 (2x + 1)[2(k + 5)x + 5k + 16],
2(1 − k)2 (x − 1)2 (2x + 1) f (x)
f6 (x, 1, 1) − Af−1/2,−2 (x) = ,
27
where

f (x) = 27(x 2 + k + 1)[2(k + 5)x + 5k + 16] − 2(2k + 7)(1 − k)(x − 1)2 (2x + 1).

Since

2(x 2 + k + 1) − (1 − k)(x − 1)2 ≥ 2(x 2 + 1) − (x − 1)2 = (x + 1)2 > 0,

it suffices to show that

27[2(k + 5)x + 5k + 16] ≥ 4(2k + 7)(2x + 1)

for 2x + 1 ≥ 0. This is true if

2[2(k + 5)x + 5k + 16] ≥ (2k + 7)(2x + 1),

which is equivalent to
6x + 8k + 25 ≥ 0.

Observation 2. Having in view Observation 1 above and Observation from the


preceding P 3.61, it follows that the concerned inequality holds for all k ≥ 0.
For k = 0, the following inequality holds under the condition x y + yz + z x ≥ 0:
2 2 2 189
+ + ≥ .
x 2 + y 2 y 2 + z2 z2 + x 2 20(x 2 + y 2 + z 2 ) + x y + yz + z x

P 3.63. If x, y, z are real numbers, then

x(−x + 4 y + 4z) y(− y + 4z + 4x) z(−z + 4x + 4 y) 21


+ + ≤ .
y 2 + z2 z2 + x 2 x2 + y2 2

(Vasile C., 2011)

Solution. Write the inequality as f6 (x, y, z) ≥ 0, where


( 
f6 (x, y, z) = 21 ( y 2 + z2) − 2 x(−x + 4 y + 4z)(x 2 + z 2 )(x 2 + y 2 ).

Since
y 2 + z 2 = −x 2 + p2 − 2q, −x + 4 y + 4z = −5x + 4p,
Highest Coefficient Cancellation Method for Real Variables 331

f6 (x, y, z) has the same highest coefficient A as



21(−x 2 )(− y 2 )(−z 2 ) − 2 x(−5x)(− y 2 )(−z 2 ),

that is
A = 9.
Since

f6 (x, 1, 1) = 42(x 2 + 1)2 − 2x(−x + 8)(x 2 + 1)2 − 8(4x + 3)(x 2 + 1),

f6 (x, 1, 1)
= 21(x 2 + 1) + x(x − 8)(x 2 + 1) − 4(4x + 3)
2(x 2 + 1)
= x 4 − 8x 3 + 22x 2 − 24x + 9 = (x − 1)2 (x − 3)2 ,

f6 (x, 1, 1) = 2(x 2 + 1)(x − 1)2 (x − 3)2 ,


we apply Corollary 2 for

4(x − 1)4 (x − 3)2


Fγ,δ (x) = f3,−2 (x) = .
2025

Thus, we need to show that f6 (x, 1, 1) ≥ Af3,−2 (x) for x ∈ . We have

4(x − 1)4 (x − 3)2


Af3,−2 (x) = ,
225

2(x − 1)2 (x − 3)2 f (x)


f6 (x, 1, 1) − Af3,−2 (x) = ,
225
where

f (x) = 225(x 2 + 1) − 2(x − 1)2 > 4(x 2 + 1) − 2(x − 1)2 = 2(x + 1)2 ≥ 0.

The equality holds for x = y = z, and also for x/3 = y = z (or any cyclic
permutation).

P 3.64. If x, y, z are real numbers, no two of which are zero, then

x 2 + 3 yz y 2 + 3z x z 2 + 3x y
+ 2 + 2 ≥ 1.
y2 − yz + z 2 z − zx + x 2 x − x y + y2

(Vasile C., 2011)


332 Vasile Cîrtoaje

Solution. Write the inequality as f6 (x, y, z) ≥ 0, where


 (
f6 (x, y, z) = (x 2 + 3 yz)(x 2 − x y + y 2 )(x 2 − xz + z 2 ) − ( y 2 − yz + z 2 ).

Since
y 2 − yz + z 2 = −x 2 − yz + p2 − 2q,
f6 (x, y, z) has the same highest coefficient A as

P2 (x, y, z) − P3 (x, y, z),

where
 (
P2 (x, y, z) = (x 2 + 3 yz)(−z 2 − x y)(− y 2 − xz), P3 (x, y, z) = (−x 2 − yz).

According to (3.2) and (3.3), f6 (x, y, z) has the highest coefficient

A = P2 (1, 1, 1) − P3 (1, 1, 1) = 48 − (−8) = 56.

On the other hand,

f6 (x, 1, 1) = (x 2 + 3)(x 2 − x + 1)2 + 2(3x + 1)(x 2 − x + 1) − (x 2 − x + 1)2


= (x 2 − x + 1)(x 4 − x 3 + 3x 2 + 4x + 4).

Since the original inequality is an equality for (x, y, z) = (0, 1, −1), that means

f6 (0, 1, −1) = 0,

we apply Corollary 2 for Fγ,δ = hγ,δ , where γ is given by (3.19):

1 h (−2)
γ= + ,
3 12A

h(x) = f6 (x, 1, 1) = (x 2 − x + 1)(x 4 − x 3 + 3x 2 + 4x + 4)


We have
1 h (−2) −25
h (−2) = −524, γ= + = .
3 12A 56
We need to show that there is a real δ such that f6 (x, 1, 1) ≥ Ahγ,δ (x) for x ∈ .
1
Choosing δ = , we have
8
 2
25 1
hγ,δ (x) = x − (x + 2)(2x + 1) + (x + 2)3
56 8
1
= 2 (7x − 8x + 15x + 6) ,
3 2 2
56
Highest Coefficient Cancellation Method for Real Variables 333

1
f6 (x, 1, 1) − Ahγ,δ (x) = (7x 6 + 6x 4 + 156x 3 + 39x 2 − 180x + 188)
56
1
= (x + 2)2 (7x 4 − 28x 3 + 90x 2 − 92x + 47)
56
1
≥ (x + 2)2 (7x 4 − 28x 3 + 74x 2 − 92x + 46)
56
1
= (x + 2)2 [7x 2 (x − 2)2 + 46(x − 1)2 ] ≥ 0.
56

The equality holds for x = 0 and y + z = 0 (or any cyclic permutation).

P 3.65. If x, y, z are real numbers, then

(4x − y − z)2 (4 y − z − x)2 (4z − x − y)2


+ + ≥ 12.
2 y 2 − 3 yz + 2z 2 2z 2 − 3z x + 2x 2 2x 2 − 3x y + y 2

(Vasile C., 2011)

Solution. Write the inequality as f6 (x, y, z) ≥ 0, where


 (
f6 = (4x − y − z)2 (2z 2 − 3z x + 2x 2 )(2x 2 − 3x y + y 2 ) − 12 (2 y 2 − 3 yz + 2z 2 ).

Since

4x − y − z = 5x − p, 2 y 2 − 3 yz + 2z 2 = −2x 2 − 3 yz + 2(p2 − 2q),

f6 (x, y, z) has the same highest coefficient A as

25P2 (x, y, z) − 12P3 (x, y, z),

where
 (
P2 (x, y, z = x 2 (−2 y 2 − 3z x)(−2z 2 − 3x y), P3 (x, y, z) = (−2x 2 − 3 yz),

that is

A = 25P2 (1, 1, 1) − 12P3 (1, 1, 1) = 25 · 75 − 12(−125) = 3375.

Since
f6 (x, 1, 1)
= 2(2x − 1)2 (2x 2 − 3x + 2) + (x − 3)2 − 6(2x 2 − 3x + 2)
2(2x 2 − 3x + 2)
= 16x 4 − 40x 3 + 33x 2 − 10x + 1 = (x − 1)2 (4x − 1)2 ,

f6 (x, 1, 1) = 2(2x 2 − 3x + 2)(x − 1)2 (4x − 1)2 ,


334 Vasile Cîrtoaje

we apply Corollary 2 for

4(x − 1)4 (4x − 1)2


Fγ,δ (x) = f1/4,−2 (x) = .
81 · 81
Thus, we need to show that f6 (x, 1, 1) ≥ Af1/4,−2 (x) for x ∈ . We have

500(x − 1)4 (4x − 1)2


Af1/4,−2 (x) = .
243
2(x − 1)2 (4x − 1)2 f (x)
f6 (x, 1, 1) − Af1/4,−2 (x) = ,
243
where

f (x) = 243(2x 2 − 3x + 2) − 250(x − 1)2


> 150(2x 2 − 3x + 2) − 250(x − 1)2 = 50(x 2 + x + 1) > 0.

The equality holds for x = y = z, and also for 4x = y = z (or any cyclic permu-
tation).
Observation. Similarly, we can prove the following generalization:


2− 4 + 2k
• Let x, y, z be real numbers. If −2 < k ≤ 1 − 5 and m = , then
4
 (m y + mz − x)2 3(2m − 1)2
≥ .
y 2 + k yz + z 2 k+2
x = y = z, and also for x/α = y = z (or any cyclic permutation),
with equality for

−k − 2k + 4
where α = .
2

P 3.66. If x, y, z are real numbers, then

(3 y + 3z − 4x)2 (3z + 3x − 4 y)2 (3x + 3 y − 4z)2


+ + ≥ 12.
2 y 2 − 3 yz + 2z 2 2z 2 − 3z x + 2x 2 2x 2 − 3x y + y 2
(Vasile C., 2011)

Solution. Write the inequality as f6 (x, y, z) ≥ 0, where


 (
f6 = (3 y +3z −4x)2 (2z 2 −3z x +2x 2 )(2x 2 −3x y + y 2 )−12 (2 y 2 −3 yz +2z 2 ).

Since

3 y + 3z − 4x = −7x + 3p, 2 y 2 − 3 yz + 2z 2 = −2x 2 − 3 yz + 2(p2 − 2q),


Highest Coefficient Cancellation Method for Real Variables 335

f6 (x, y, z) has the same highest coefficient A as


49P2 (x, y, z) − 12P3 (x, y, z),
where
 (
P2 (x, y, z = x 2 (−2z 2 − 3x y)(−2 y 2 − 3z x), P3 (x, y, z) = (−2x 2 − 3 yz),
that is
A = 49P2 (1, 1, 1) − 12P3 (1, 1, 1) = 49 · 75 − 12(−125) = 5175.
Since
f6 (x, 1, 1)
= 2(2x − 3)2 (2x 2 − 3x + 2) + (3x − 1)2 − 6(2x 2 − 3x + 2)
2(2x 2 − 3x + 2)
= 16x 4 − 72x 3 + 121x 2 − 90x + 25 = (x − 1)2 (4x − 5)2 ,
f6 (x, 1, 1) = 2(2x 2 − 3x + 2)(x − 1)2 (4x − 5)2 ,
we apply Corollary 2 for
4(x − 1)4 (4x − 5)2
Fγ,δ (x) = f5/4,−2 (x) = .
81 · 169
Thus, we need to show that f6 (x, 1, 1) ≥ Af5/4,−2 (x) for x ∈ . We have
2300(x − 1)4 (4x − 1)2
Af5/4,−2 (x) = .
1521
(x − 1)2 (4x − 1)2 f (x)
f6 (x, 1, 1) − Af5/4,−2 (x) = ,
1521
where
f (x) = 3042(2x 2 − 3x + 2) − 2300(x − 1)2
> 1500(2x 2 − 3x + 2) − 2500(x − 1)2 = 500(x 2 + x + 1) > 0.
4x
The equality holds for x = y = z, and also for = y = z (or any cyclic
5
permutation).
Observation. Similarly, we can prove the following generalization:

2 + 4 + 2k
• Let x, y, z be real numbers. If −2 < k ≤ 2 and m = , then
4
 (m y + mz − x)2 3(2m − 1)2
≥ .
y + k yz + z
2 2 k+2


for x = y = z, and also for x/α = y = z (or any cyclic permutation),
with equality
2k + 4 − k
where α = .
2
336 Vasile Cîrtoaje

P 3.67. Let x, y, z be real numbers. If k > −2, then

4(x 2 + k x y + y 2 )( y 2 + k yz + z 2 )(z 2 + kz x + x 2 ) ≥ (2 − k)(x − y)2 ( y − z)2 (z − x)2 .

(Vasile C., 2011)

Solution. Write the inequality as f6 (x, y, z) ≥ 0, where


(
f6 (x, y, z) = 4 (x 2 + k x y + y 2 ) − (2 − k)(x − y)2 ( y − z)2 (z − x)2 .

From ( (
(x 2 + k x y + y 2 ) = (p2 − 2q + k x y − z 2 ),
it follows that f6 (x, y, z) has the same highest coefficient as
(
4 (k x y − z 2 ) − (2 − k)(x − y)2 ( y − z)2 (z − x)2 ,

that is, according to (3.3),

A = 4(k − 1)3 + 27(2 − k) = (k + 2)(2k − 5)2 .

For k = 5/2, we have A = 0. Then, by Corollary 1, it suffices to show that


f6 (x, 1, 1) ≥ 0 for any real x. Indeed,

f6 (x, 1, 1) = 4(k + 2)(x 2 + k x + 1)2 > 0.


5
Further, consider k > −2, k = . Since f6 (x, 1, 1) is a polynomial function of
2
degree four and

f6 (0, y, z) = 4 y 2 z 2 ( y 2 + k yz + z 2 ) − (2 − k) y 2 z 2 ( y − z)2 ,

f6 (0, 1, −1) = 4(2 − k) − 4(2 − k) = 0,


we apply Corollary 2 for Fγ,δ = hγ,0 , where γ is given by (3.19):

1 h (−2)
γ= + ,
3 12A
with
h(x) = f6 (x, 1, 1) = 4(k + 2)(x 2 + k x + 1)2 .
We only need to show that f6 (x, 1, 1) ≥ Ahγ,0 (x) for x ∈ . We have

h (x) = 8(k + 2)(2x + k)(x 2 + k x + 1),


1 2k − 8 1
h (−2) = −8(k + 2)(k − 4)(2k − 5), γ= − = ,
3 3(2k − 5) 2k − 5
 2
1 4(x 2 + k x + 1)2
hγ,0 (x) = x + (x + 2)(2x + 1) = ,
2k − 5 (2k − 5)2
Highest Coefficient Cancellation Method for Real Variables 337

therefore
f6 (x, 1, 1) = Ahγ,0 (x).

Actually, the inequality is equivalent to

(k + 2)[x y(x + y) + yz( y + z) + z x(z + x) + 2(k − 1)x yz]2 ≥ 0.

Therefore, the equality holds for

x y(x + y) + yz( y + z) + z x(z + x) + 2(k − 1)x yz = 0.

P 3.68. If x, y, z are real numbers, then

(x 2 + y 2 )( y 2 + z 2 )(z 2 + x 2 ) + 2(x 2 y + y 2 z + z 2 x)(x y 2 + yz 2 + z x 2 ) ≥ x 2 y 2 z 2 .

(Vasile C., 2011)

Solution. Write the inequality as f6 (x, y, z) ≥ 0. From


(  
f6 (x, y, z) = (p2 − 2q − x 2 ) + 2(3x 2 y 2 z 2 + x 3 y 3 + x yz x 3) − x 2 y 2z2,

it follows that f6 (x, y, z) has the highest coefficient

A = −1 + 2(3 + 3 + 3) − 1 = 16.

Since f6 (x, 1, 1) is a polynomial function of degree four and

f6 (0, y, z) = y 2 z 2 ( y 2 + z 2 ) + 2 y 3 z 3 , f6 (0, 1, −1) = 2 − 2 = 0,

we apply Corollary 2 for Fγ,δ = hγ,0 , where γ is given by (3.19):

1 h (−2)
γ= + ,
3 12A
with
h(x) = f6 (x, 1, 1) = 4x 4 + 4x 3 + 9x 2 + 4x + 4 = (2x 2 + x + 2)2 .
We only need to show that f6 (x, 1, 1) ≥ Ahγ,0 (x) for x ∈ . We have

h (x) = 2(4x + 1)(2x 2 + x + 2), h (−2) = −112,


1 7 −1
γ= − = ,
3 12 4
 2
1 1
hγ,0 (x) = x − (x + 2)(2x + 1) = (2x 2 + x + 2)2 ,
4 16
338 Vasile Cîrtoaje

therefore
f6 (x, 1, 1) = Ahγ,0 (x).

Actually, the inequality is equivalent to

[x y(x + y) + yz( y + z) + z x(z + x) − x yz]2 ≥ 0.

Thus, the equality holds for

x y(x + y) + yz( y + z) + z x(z + x) = x yz.

P 3.69. Let x, y, z be real numbers. If k ∈ (−∞, −2] ∪ (0, ∞), then


2
x 6 + y 6 + z 6 − 3x 2 y 2 z 2 + (x 2 + k yz)( y 2 + kz x)(z 2 + k x y) ≥ 0.
k
(Vasile C., 2011)

Solution. Denote the left side of the inequality by f6 (x, y, z). According to (3.3),
the polynomial f6 (x, y, z) has the highest coefficient

2 2(k + 1)3
A = 3 − 3 + (1 + k)3 = > 0.
k k
Since
f6 (0, y, z) = y 6 + z 6 + 2 y 3 z 3 , f6 (0, 1, −1) = 1 + 1 − 2 = 0,
we apply Corollary 2 for Fγ,δ = hγ,δ , where γ is given by (3.19):

1 h (−2)
γ= + ,
3 12A
with
2
h(x) = f6 (x, 1, 1) = x 6 − 3x 2 + 2 + (x 2 + k)(k x + 1)2 .
k
We have
4
h (x) = 6x 5 − 6x +x(k x + 1)2 + 4(x 2 + k)(k x + 1),
k
−4(2k3 + 15k2 + 33k + 2)
h (−2) = ,
k
1 2k3 + 15k2 + 33k + 2 −3k(k + 3)
γ= − = ,
3 6(k + 1)3 2(k + 1)3
Thus, the condition f6 (x, 1, 1) ≥ Ahγ,δ (x) is equivalent to

2
x 6 − 3x 2 + 2 + (x 2 + k)(k x + 1)2 ≥
k
Highest Coefficient Cancellation Method for Real Variables 339

 2
2(k + 1)3 3k(k + 3)
≥ x− (x + 2)(2x + 1) + δ(x + 2)3 .
k 2(k + 1)3
k(k + 3)
Setting x = 1, this inequality becomes an equality for δ = . Choosing this
2(k + 1)3
δ, the inequality turns into

k(k − 1)2 (k + 2)(x + 2)2 (x − 1)4 ≥ 0,

which is true for x ∈ .


The equality holds for x = 0 and y + z = 0 (or any cyclic permutation).
Observation. For k = −2 and k = 1, the following identities hold:

x 6 + y 6 + z 6 − 3x 2 y 2 z 2 − (x 2 − 2 yz)( y 2 − 2z x)(z 2 − 2x y) = (x 3 + y 3 + z 3 − 2x yz)2 ,

x 6 + y 6 + z 6 − 3x 2 y 2 z 2 + 2(x 2 + yz)( y 2 + z x)(z 2 + x y) = (x 3 + y 3 + z 3 + x yz)2 .

P 3.70. If x, y, z are real numbers, then

2(2x 2 + y 2 +z 2 )(2 y 2 +z 2 + x 2 )(2z 2 + x 2 + y 2 ) ≥ 89x 2 y 2 z 2 +9(x − y)2 ( y −z)2 (z − x)2 .

(Vasile C., 2011)

Solution. Write the inequality as f6 (x, y, z) ≥ 0. Since


( (
(2x 2 + y 2 + z 2 ) = (x 2 + p2 − 2q),

the polynomial f6 (x, y, z) has the same highest coefficient as

2x 2 y 2 z 2 − 89x 2 y 2 z 2 − 9(x − y)2 ( y − z)2 (z − x)2 ,

that is
A = 2 − 89 + 9 · 27 = 156.
Since
f6 (0, y, z) = 2( y 2 + z 2 )(2 y 2 + z 2 )(2z 2 + y 2 ) − 9 y 2 z 2 ( y − z)2 ,
f6 (0, 1, −1) = 36 − 36 = 0,
we apply Corollary 2 for Fγ,δ = hγ,δ , where γ is given by (3.19):

1 h (−2)
γ= + ,
3 12A
with
h(x) = f6 (x, 1, 1) = 4(x 2 + 1)(x 2 + 3)2 − 89x 2 .
340 Vasile Cîrtoaje

We have
h (x) = 8x(x 2 + 3)3 + 16x(x 2 + 1)(x 2 + 3) − 178x,
1 43 −77
h (−2) = −1548, γ=
− = ,
3 52 156
Thus, the condition f6 (x, 1, 1) ≥ Ahγ,δ (x) is equivalent to
 2
77
4(x 2 + 1)(x 2 + 3)2 − 89x 2 ≥ 156 x − (x + 2)(2x + 1) + δ(x + 2)3 .
156
17
Setting x = 1, we can check that this inequality becomes an equality for δ = .
156
17
Choosing δ = , the inequality becomes
156

(x + 2)2 (x − 1)2 (335x 2 − 1098x + 1323) ≥ 0,

which is true for x ∈ .


The equality holds for x = 0 and y + z = 0 (or any cyclic permutation).
Observation. The following related statement holds, too.
• Let x, y, z be real numbers. If k ≥ 0, then

2(k x 2 + y 2 + z 2 )(k y 2 + z 2 + x 2 )(kz 2 + x 2 + y 2 ) ≥ (k + 1)2 (x − y)2 ( y − z)2 (z − x)2 ,

with equality for x = 0 and y + z = 0 (or any cyclic permutation).

P 3.71. If x, y, z are real numbers such that x + y + z = 3, then

13x − 1 13 y − 1 13z − 1 3
+ 2 + ≤ .
x 2 + 23 y + 23 z 2 + 23 2

(Vasile C., 2011)

Solution. Write the inequality in the homogeneous form f6 (x, y, z) ≥ 0, where


( 
f6 (x, y, z) = 3 (9x 2 + 23p2 ) − 2p (39x − p)(9 y 2 + 23p2 )(9z 2 + 23p2 ).

Clearly, the polynomial f6 (x, y, z) has the highest coefficient

A = 3 · 729.

For p = 0, we have f6 (x, y, z) = 3 · 729 x 2 y 2 z 2 , therefore

f6 (0, −1, 1) = 0.
Highest Coefficient Cancellation Method for Real Variables 341

Also, we have

f6 (x, 1, 1) = 12(23x 2 + 92x + 101)(x − 1)2 (7x + 11)2 .

4(x − 1)4 (7x + 11)2


f−11/7,−2 (x) = ,
729
Af−11/7,−2 (x) = 12(x − 1)4 (7x + 11)2 ,
f6 (x, 1, 1) − Af−11/7,−2 (x) = 24(x − 1)2 (7x + 11)2 (x + 2)(11x + 25).
As a consequence, we will apply Theorem 2 for
−25 −3
η= , ξ= ,
11 143
    
−25 −23 −25 −23
= , ,  \  = −∞, ∪ ,∞ ,
11 13 11 13
and for
Eα,β = hα,β , Eγ,δ = f−11/7,−2 ,
where α is given by (3.18):
1 h (−2)
α= + ,
3 12A
with
h(x) = f6 (x, 1, 1) = 12(23x 2 + 92x + 101)(x − 1)2 (7x + 11)2 .

−25
The condition (b), namely f6 (x, 1, 1) ≥ Af−11/7,−2 (x), is satisfied for x ∈ −∞, ∪
   11
−25 −23
[−2, ∞), hence for x ∈  \  = −∞, ∪ ,∞ .
11 13
The condition (a) is satisfied if there is a real β such that f6 (x, 1, 1) ≥ Afα,β (x)
 
−25 −23
for x ∈  = , , where
11 13
1 h (−2)
α= + .
3 12A
From
h (−2) = −729 · 64,
we get
1 −16 −13
α= ++ = ,
3 9 9
 2
13
hα,β (x) = x − (x + 2)(2x + 1) + β(x + 2)3 .
9
We choose
28
β= ,
9
342 Vasile Cîrtoaje

to have hα,β (−11/7) = 0. Therefore,


 2
13 28
hα,β (x) = x − (x + 2)(2x + 1) + (x + 2)3
9 9
4
= (7x + 11)2 (2x 2 + 7x + 9)2 ,
81
Ahα,β (x) = 108(7x + 11)2 (2x 2 + 7x + 9)2 ,
f6 (x, 1, 1) − Ahα,β (x) = −12(7x + 11)2 (x + 2)2 (13x 2 + 154x + 167)
and

13x 2 + 154x + 167 < 14(2x 2 + 11x + 12) = 14(x + 4)(2x + 3) < 0.

The original inequality is an equality for x = y = z = 1, and also for x = −11


and y = z = 7 (or any cyclic permutation).

P 3.72. If x, y, z are real numbers, then

5(x 2 + y 2 + z 2 )3 ≥ 108x 2 y 2 z 2 + 10(x − y)2 ( y − z)2 (z − x)2 .

(Vo Quoc Ba Can and Vasile Cirtoaje, 2011)

Solution. Write the inequality as f6 (x, y, z) ≥ 0, where

f6 (x, y, z) = 5(x 2 + y 2 + z 2 )3 − 108x 2 y 2 z 2 − 10(x − y)2 ( y − z)2 (z − x)2 .

The polynomial f6 (x, y, z) has the highest coefficient

A = −108 − 10(−27) = 162.

We have

f6 (0, y, z) = 5( y 2 + z 2 )3 − 10 y 2 z 2 ( y − z)2 , f6 (0, 1, −1) = 40 − 40 = 0,

h(x) = f6 (x, 1, 1) = 5x + 30x − 48x 2 + 40,


6 4

1 h (−2) −5
h (−2) = −1728, + = ,
3 12A 9
 2
5
h−5/9,β (x) = x − (x + 2)(2x + 1) + β(x + 2)3 ,
9
 2
Ah−5/9,β (x) = 2 9x − 5(x + 2)(2x + 1) + 9β(x + 2)3 ,
f6 (x, 1, 1) − Ah−5/9,β (x) = (x + 2)3 Hβ (x),
Highest Coefficient Cancellation Method for Real Variables 343

where

Hβ (x) = 5(x 3 − 6x 2 − 10x − 4) + 72β(5x 2 + 8x + 5) + 162β 2 (x + 2)3 .

First Solution. We will apply Theorem 2 for ξ → −∞, which involves


   
−1 −1
 = −∞, , \= ,∞ ,
2 2

and for
Eα,β = h−5/9,β , Eγ,δ = h−5/9,δ .

The condition (a), namely f6 (x, 1, 1) ≥ Ah−5/9,β (x) for x < −1/2, can be satisfied
10
only if Hβ (−2) = 0. This yields β = and
81
5
f6 (x, 1, 1) − Ah−5/9,10/81 (x) = (x + 2)4 (41x 2 − 88x + 38) ≥ 0.
81

The condition (b) is satisfied if there is a real δ such that f6 (x, 1, 1) ≥ Ah−5/9,δ (x)
4
for x ≥ −1/2, that is Hδ (x) ≥ 0 for x ≥ −1/2. Choosing δ = , the condition
27
Hδ (x) ≥ 0 is equivalent to

13x 3 + 18x 2 − 66x + 44 ≥ 0,

which is true because

13x 3 + 18x 2 − 66x + 44 = 13(x − 1)2 (x + 1) + (31x 2 − 53x + 31) > 0.

The equality holds for x = 0 and y + z = 0 (or any cyclic permutation).

Second Solution. Apply Theorem 2 for

2 16
η= , ξ= ,
5 5
  
2 2
= , 2 ,  \  = −∞, ∪ [2, ∞),
5 5
and for
Eα,β = h−5/9,4/27 , Eγ,δ = h−5/9,10/81 .

The condition (a) is satisfied if f6 (x, 1, 1) ≥ Ah−5/9,4/27 (x) for 2/5 < x < 2. As
shown in the first proof, this inequality can be written as

(x + 2)3 (13x 3 + 18x 2 − 66x + 44) ≥ 0,


344 Vasile Cîrtoaje

which is true since

13x 3 + 18x 2 − 66x + 44 = 13(x − 1)2 (x + 1) + (31x 2 − 53x + 31) > 0.

The condition (b) is satisfied if f6 (x, 1, 1) ≥ Ah−5/9,10/81 for x ∈ (−∞, 2/5] ∪


[2, ∞). As shown in the first proof, this inequality is equivalent to

(x + 2)4 (41x 2 − 88x + 38) ≥ 0,

which is true because

41x 2 − 88x + 38 > 36x 2 − 88x + 32 = 4(9x − 4)(x − 2) ≥ 0.

Observation. The coefficient of the product (x − y)2 ( y − z)2 (z − x)2 in P 3.72 has
the best possible value. Indeed, setting x = 0, y = 1, z = −1 in

5(x 2 + y 2 + z 2 )3 ≥ αx 2 y 2 z 2 + β(x − y)2 ( y − z)2 (z − x)2 ,

we get β ≤ 10. In addition, for β = 10, the best value of the coefficient α of the
product x 2 y 2 z 2 is 108. Setting x = 5t, y = 2t + 1, z = 2t − 1, the inequality

5(x 2 + y 2 + z 2 )3 ≥ αx 2 y 2 z 2 + 10(x − y)2 ( y − z)2 (z − x)2

becomes
A(α)t 6 + B(α)t 4 + C(α)t 2 ≥ 0,
where C(α) = 108 − α. The necessary condition C(α) ≥ 0 involves α ≤ 108.

P 3.73. If x, y, z are real numbers, then

(x 2 + y 2 + z 2 )3 + 2(2x 2 + yz)(2 y 2 + z x)(2z 2 + x y) ≥ 27x 2 y 2 z 2 .

(Vasile C., 2011)

Solution. Write the inequality as f6 (x, y, z) ≥ 0, where

f6 (x, y, z) = (x 2 + y 2 + z 2 )3 − 27x 2 y 2 z 2 + 2(2x 2 + yz)(2 y 2 + z x)(2z 2 + x y).

According to (3.3), f6 (x, y, z) has the highest coefficient

A = −27 + 2(2 + 1)3 = 27.

We have

f6 (0, y, z) = ( y 2 + z 2 )3 + 8 y 3 z 3 , f6 (0, 1, −1) = 8 − 8 = 0,


Highest Coefficient Cancellation Method for Real Variables 345

h(x) = f6 (x, 1, 1) = (x 2 + 2)3 + 2(2x 2 + 1)(x + 2)2 − 27x 2


= x 6 + 10x 4 + 16x 3 + 3x 2 + 8x + 16,
1 h (−2) −2
h (−2) = −324, + = ,
3 12A 3
 2
2
h−2/3,β (x) = x − (x + 2)(2x + 1) + β(x + 2)3 ,
3
 2
Ah−2/3,β (x) = 2 9x − 5(x + 2)(2x + 1) + 9β(x + 2)3 ,

f6 (x, 1, 1) − Ah−5/9,β (x) = (x + 2)3 Hβ (x),


where

Hβ (x) = x 3 − 6x 2 − 14x − 4 + 18β(4x 2 + 7x + 4) − 27β 2 (x + 2)3 .

We will apply Theorem 2 for ξ → −∞, which involves


   
−1 −1
 = −∞, , \= ,∞ ,
2 2

and for
Eα,β = h−2/3,β , Eγ,δ = h−2/3,δ .

The condition (a), namely f6 (x, 1, 1) ≥ Ah−2/3,β (x) for x < −1/2, can be satisfied
2
only if Hβ (−2) = 0. This yields β = and
27
1
f6 (x, 1, 1) − Ah−2/3,2/27 (x) = (x + 2)4 (23x 2 − 88x + 2) ≥ 0
27

for x < −1/2.

The condition (b) is satisfied if there is a real δ such that f6 (x, 1, 1) ≥ Ah−2/3,δ (x)
1
for x ≥ −1/2, that is Hδ (x) ≥ 0 for x ≥ −1/2. Choosing δ = , we get
6

4Hγ (x) = x 3 + 6x 2 − 8x + 8
≥ 2x 2 − 8x + 8 = 2(x − 2)2 ≥ 0.

The equality holds for x = 0 and y + z = 0 (or any cyclic permutation).


346 Vasile Cîrtoaje

P 3.74. If x, y, z are real numbers, no two of which are zero, then

x 2 + 2 yz y 2 + 2z x z 2 + 2x y 3(x y + yz + z x)
+ 2 + 2 ≥ .
y2 + yz + z 2 z + zx + x 2 x + x y + y2 x 2 + y 2 + z2

(Vasile C., 2014)

Solution. Write the inequality as f8 (x, y, z) ≥ 0, where



f8 (x, y, z) =(x 2 + y 2 + z 2 ) (x 2 + 2 yz)(z 2 + z x + x 2 )(x 2 + x y + y 2 )
(
− 3(x y + yz + z x) ( y 2 + yz + z 2 ).

Since
y 2 + yz + z 2 = yz − x 2 + p2 − 2q,
f8 (x, y, z) has the same highest polynomial as
 (
(x 2 + y 2 + z 2 ) (x 2 + 2 yz)(z x − y 2 )(x y − z 2 ) − 3(x y + yz + z x) ( yz − x 2 ),

that is, according to (3.2) and (3.3),

A(p, q) = (x 2 + y 2 + z 2 ) · 0 − 3(x y + yz + z x) · 0 = 0.

By Theorem 3, it suffices to show that f8 (x, 1, 1) ≥ 0 for all real x. We have

f8 (x, 1, 1)
= (x 2 + 2)[(x 2 + 2)(x 2 + x + 1) + 6(2x + 1)] − 9(2x + 1)(x 2 + x + 1)
x2 + x + 1
= x 6 + x 5 + 5x 4 − 2x 3 − 13x 2 + x + 7
= (x 2 − 1)2 (x 2 + x + 7) ≥ 0.

The equality holds for x = y = z, and for −x = y = z (or any cyclic permuta-
tion).

P 3.75. If x, y, z are real numbers, no two of which are zero, then

x 2 − 2 yz y 2 − 2z x z 2 − 2x y 3(x y + yz + z x)
+ + + ≥ 0.
y 2 − yz + z 2 z 2 − z x + x 2 x 2 − x y + y 2 x 2 + y 2 + z2

(Vasile C., 2014)

Solution. Write the inequality as f8 (x, y, z) ≥ 0, where



f8 (x, y, z) =(x 2 + y 2 + z 2 ) (x 2 − 2 yz)(z 2 − z x + x 2 )(x 2 − x y + y 2 )
(
+ 3(x y + yz + z x) ( y 2 − yz + z 2 ).
Highest Coefficient Cancellation Method for Real Variables 347

Since
y 2 − yz + z 2 = − yz − x 2 + p2 − 2q,
f8 (x, y, z) has the same highest polynomial as
 (
(x 2 + y 2 + z 2 ) (x 2 − 2 yz)(z x + y 2 )(x y + z 2 ) − 3(x y + yz + z x) ( yz + x 2 ),

that is, according to (3.2) and (3.3),

A(p, q) = (x 2 + y 2 + z 2 )(−12) − 3(x y + yz + z x) · 8 = −12p2 ≤ 0.

By Theorem 3, it suffices to show that f8 (x, 1, 1) ≥ 0 for all real x. We have


f8 (x, 1, 1)
= (x 2 + 2)[(x 2 − 2)(x 2 − x + 1) + 2(1 − 2x)] + 3(2x + 1)(x 2 − x + 1)
x2 − x + 1
= x 6 − x 5 + x 4 + 2x 3 − 5x 2 − x + 3
= (x 2 − 1)2 (x 2 − x + 3) ≥ 0.

The equality holds for x = y = z, and for −x = y = z (or any cyclic permuta-
tion).

P 3.76. If x, y, z are real numbers, no two of which are zero, then

x2 y2 z2 (x + y + z)2
+ + ≥ .
y 2 − yz + z 2 z 2 − z x + x 2 x 2 − x y + y 2 x 2 + y 2 + z2
(Vasile C., 2014)
Solution. Write the inequality as f8 (x, y, z) ≥ 0, where

f8 (x, y, z) =(x 2 + y 2 + z 2 ) x 2 (z 2 − z x + x 2 )(x 2 − x y + y 2 )
(
− (x + y + z)2 ( y 2 − yz + z 2 ).

Since
y 2 − yz + z 2 = − yz − x 2 + p2 − 2q,
f8 (x, y, z) has the same highest polynomial as
 (
(x 2 + y 2 + z 2 ) x 2 (z x + y 2 )(x y + z 2 ) + (x + y + z)2 ( yz + x 2 ),

that is, according to (3.2) and (3.3),

A(p, q) = (x 2 + y 2 + z 2 ) · 12 + (x + y + z)2 · 8 = 20p2 − 24q = 12p2 + 8(p2 − 3q) ≥ 0.

We have

A(x + 2, 2x + 1) = 20(x + 2)2 − 24(2x + 1) = 4(5x 2 + 8x + 14),


348 Vasile Cîrtoaje

f8 (x, 1, 1)
= (x 2 + 2)[x 2 (x 2 − x + 1) + 2] − (x + 2)2 (x 2 − x + 1)
x2 − x + 1
= x 6 − x 5 + 2x 4 − 5x 3 + 3x 2
= x 2 (x − 1)2 (x 2 + x + 3) ≥ 0.

Since A(p, q) ≥ 0 for all real x, y, z, f8 (1, 1, 1) = 0 and f8 (0, 1, 1) = 0, we apply


Corollary 4 for
x 2 (x − 1)4
E0,−2 (x) = f0,−2 (x) = .
81
We have
4x 2 (x − 1)4 (5x 2 + 8x + 14)
A(x + 2, 2x + 1) f0,−2 (x) = ,
81
x 2 (x − 1)2 g(x)
f8 (x, y, z) − A(x + 2, 2x + 1) f0,−2 (x) = ,
81
where

g(x) = 81(x 2 − x + 1)(x 2 + x + 3) − 4(x − 1)2 (5x 2 + 8x + 14)


= 61x 4 + 8x 3 + 231x 2 − 82x + 187
= 57x 4 + 4x 2 (x + 1)2 + 227x 2 − 82x + 187 > 0.

The equality holds for x = y = z, and for x = 0 and y = z (or any cyclic
permutation).

P 3.77. If x, y, z are real numbers such that x yz = 0, then

( y + z)2 (z + x)2 (x + y)2 10(x + y + z)2


+ + ≥2+ .
x 2 y 2 z 2 3(x 2 + y 2 + z 2 )

(Vasile C., 2014)

Solution. Write the inequality as f8 (x, y, z) ≥ 0, where


" #
f8 (x, y, z) = 3(x 2 + y 2 + z 2 ) y 2 z 2 ( y + z)2 − 2x 2 y 2 z 2 − 10x 2 y 2 z 2 (x + y + z)2 .

Since
  
y 2 z 2 ( y + z)2 = y 2 z 2 (p − x)2 = p2 y 2 z 2 − 2pqr + 3r 2 ,

f8 (x, y, z) has the highest polynomial

A(p, q) = 3(x 2 + y 2 + z 2 )(3 − 2) − 10(x + y + z)2 = 3(p2 − 2q) − 10p2 = −7p2 − 6q.
Highest Coefficient Cancellation Method for Real Variables 349

We have

f8 (x, 1, 1) = 3(x 2 + 2)[4 + 2x 2 (x + 1)2 − 2x 2 ] − 10x 2 (x + 2)2


= 2(3x 6 + 6x 5 + x 4 − 8x 3 − 14x 2 + 12)
= 2(x − 1)2 (3x 4 + 12x 3 + 22x 2 + 24x + 12)
= 2(x − 1)2 [3(x + 1)4 + (2x + 3)2 ] ≥ 0,

A(x + 2, 2x + 1) = −7(x + 2)2 − 6(2x + 1) = −7x 2 − 40x − 34.

Case 1: −7p2 − 6q ≤ 0. Apply Theorem 3. Since f8 (x, 1, 1) ≥ 0 for all real x, the
conclusion follows.

Case 2: −7p2 − 6q ≥ 0. Apply Corollary 4 for

Eα,β = hα,β .

Thus, we need to show that there exist two real numbers α and β such that

f8 (x, 1, 1) ≥ A(x + 2, 2x + 1)hα,β (x)

for x ∈ . Write the required inequality as h(x) ≥ 0, where


 2
h(x) = f8 (x, 1, 1) + (7x 2 + 40x + 34) x + α(x + 2)(2x + 1) + β(x + 2)3 ,

Since
h(−2) = 72 − 18(−2)2 = 0,
the condition h (−2) = 0 is necessary to have h(x) ≥ 0 in the vicinity of −2. This
condition involves α = −1. For this α, we can check that h (−2) = 0. Thus, to
have h(x) ≥ 0 in the vicinity of −2, the condition h (−2) = 0 is necessary. This
condition involves β = 2/3. For these values of α and β, we need to show that
 2
2
f8 (x, 1, 1) + (7x 2 + 40x + 34) x − (x + 2)(2x + 1) + (x + 2)3 ≥0
3

for x ∈ . This inequality is equivalent to

(x + 2)4 (14x 4 + 52x 3 + 117x 2 + 154x + 113) ≥ 0,

which is true because

14x 4 +52x 3 +117x 2 +154x +113 = (x 2 +1)2 +13x 2 (x +2)2 +7(9x 2 +22x +16) > 0.

The equality holds for x = y = z.


350 Vasile Cîrtoaje

P 3.78. If x, y, z are real numbers, no two of which are zero, then

32x 2 + 49 yz 32 y 2 + 49z x 32z 2 + 49x y 81(x + y + z)2


+ + ≥ .
y +z
2 2 z +x
2 2 x +y
2 2 2(x 2 + y 2 + z 2 )

(Vasile C., 2014)

Solution. Consider the more general inequality


 x 2 + k yz (1 + k)(x + y + z)2
≥ , k > −1,
y +z
2 2 2(x 2 + y 2 + z 2 )

which can be written as f8 (x, y, z) ≥ 0, where



f8 (x, y, z) =2(x 2 + y 2 + z 2 ) (x 2 + k yz)(z 2 + x 2 )(x 2 + y 2 )
(
− (1 + k)(x + y + z)2 ( y 2 + z 2 ).

Since y 2 + z 2 = −x 2 + p2 − 2q, f8 (x, y, z) has the same highest polynomial as



2(p2 − 2q) (x 2 + k yz) y 2 z 2 − (1 + k)p2 (−x 2 y 2 z 2 ),

that is

A(p, q) = 2(p2 − 2q)(3 + 3k) + (1 + k)p2 = (1 + k)(7p2 − 12q).

We have
A(p, q) = (1 + k)[3p2 + 4(p2 − 3q)] ≥ 0,
A(x + 2, 2x + 1) = (1 + k)[7(x + 2)2 − 12(2x + 1)] = (1 + k)(7x 2 + 4x + 16),
f8 (x, 1, 1)
= 2(x 2 + 2)[(x 2 + k)(x 2 + 1) + 4(k x + 1)] − 2(1 + k)(x + 2)2 (x 2 + 1),
x2 + 1
f8 (x, 1, 1) = 2(x 2 + 1)(x − 1)2 (x 4 + 2x 3 + 5x 2 + 4x + 4 − 2k).

49
For k = , we get
32
81
A(x + 2, 2x + 1) = (7x 2 + 4x + 16),
32
1
f8 (x, 1, 1) = (x 2 + 1)(x − 1)2 (2x + 1)2 (4x 2 + 4x + 15).
8
Since A(p, q) ≥ 0 for all real x, y, z, f8 (1, 1, 1) = 0 and f8 (−1/2, 1, 1) = 0, we apply
Corollary 4 for

4
E−1/2,−2 (x) = f−1/2,−2 (x) = (x − 1)4 (2x + 1)2 .
729
Highest Coefficient Cancellation Method for Real Variables 351

We only need to show that

f8 (x, 1, 1) − A(x + 2, 2x + 1) f−1/2,−2 (x) ≥ 0

for x ∈ . Indeed, we have


9
A(x + 2, 2x + 1) f−1/2,−2 (x) = (x − 1)4 (2x + 1)2 (7x 2 + 4x + 16),
8
1
f8 (x, 1, 1) − A(x + 2, 2x + 1) f−1/2,−2 (x) = (x − 1)2 (2x + 1)2 g(x),
72
where

g(x) = 29x 4 + 46x 3 + 156x 2 + 64x + 119


= (23x 2 + 32)(x + 1)2 + 6x 4 + 101x 2 + 87 > 0.

The equality holds for x = y = z, and also for −2x = y = z (or any cyclic
permutation).

P 3.79. If x, y, z are real numbers, no two of which are zero, then

x 2 + 4 yz y 2 + 4z x z 2 + 4x y 15(x y + yz + z x)
(a) + 2 + 2 ≥ ;
y 2 + z2 z + x2 x + y2 2(x 2 + y 2 + z 2 )

2x 2 + 9 yz 2 y 2 + 9z x 2z 2 + 9x y 33(x y + yz + z x)
(b) + 2 + ≥ .
y 2 + z2 z + x2 x2 + y2 2(x 2 + y 2 + z 2 )
(Vasile C., 2014)

Solution. Consider the more general inequality


 x 2 + k yz 3(1 + k)(x y + yz + z x)
≥ , k > −1,
y 2 + z2 2(x 2 + y 2 + z 2 )
which can be written as f8 (x, y, z) ≥ 0, where

f8 (x, y, z) =2(x 2 + y 2 + z 2 ) (x 2 + k yz)(z 2 + x 2 )(x 2 + y 2 )
(
− 3(1 + k)(x y + yz + z x) ( y 2 + z 2 ).

Since y 2 + z 2 = −x 2 + p2 − 2q, f8 (x, y, z) has the same highest polynomial as



2(p2 − 2q) (x 2 + k yz) y 2 z 2 − 3(1 + k)q(−x 2 y 2 z 2 ),

that is

A(p, q) = 2(p2 − 2q)(3 + 3k) + 3(1 + k)q = 3(1 + k)(2p2 − 3q).


352 Vasile Cîrtoaje

We have
A(p, q) = 3(1 + k)[p2 + (p2 − 3q)] ≥ 0,
A(x + 2, 2x + 1) = 3(1 + k)[2(x + 2)2 − 3(2x + 1)] = 3(1 + k)(2x 2 + 2x + 5),
f8 (x, 1, 1)
= 2(x 2 + 2)[(x 2 + k)(x 2 + 1) + 4(k x + 1)] − 6(1 + k)(2x + 1)(x 2 + 1),
x2 + 1
f8 (x, 1, 1) = 2(x 2 + 1)(x − 1)2 [x 4 + 2x 3 + (k + 6)x 2 + 4x + 5 − k].
Since A(p, q) ≥ 0 for all real x, y, z, we apply Corollary 4 for
1 2
E0,−2 (x) = f0,−2 (x) = x (x − 1)4 .
81
Thus, we need to show that

f8 (x, 1, 1) − A(x + 2, 2x + 1) f0,−2 (x) ≥ 0

for x ∈ . Since
(1 + k)(2x 2 + 2x + 5)x 2 (x − 1)4
A(x + 2, 2x + 1) f0,−2 (x) = ,
27
the inequality f8 (x, 1, 1) − A(x + 2, 2x + 1) f0,−2 (x) ≥ 0 is true if

54(x 2 + 1)[x 4 + 2x 3 + (k + 6)x 2 + 4x + 5 − k] ≥ (1 + k)x 2 (2x 2 + 2x + 5)(x − 1)2 .

Since 2(x 2 + 1) ≥ (x − 1)2 , it suffices to show that

27[x 4 + 2x 3 + (k + 6)x 2 + 4x + 5 − k] ≥ (1 + k)x 2 (2x 2 + 2x + 5),

which is equivalent to g(x) ≥ 0, where

g(x) = (25 − 2k)x 4 + (52 − 2k)x 3 + (22k + 157)x 2 + 108x + 27(5 − k).

(a) For k = 4, we have

g(x) = 17x 4 + 44x 3 + 245x 2 + 108x + 27


   
9 359
= x 4 + (4x + 1)2 x 2 + x + 2 + 194x 2 + x + 25 > 0.
4 4
The equality holds for x = y = z.
(b) For k = 9/2, we have

2g(x) = 32x 4 + 86x 3 + 512x 2 + 216x + 27


   
35 5 1533 49
= (4x + 1)2 2x 2 + x+ + 435x 2 + x+ > 0.
8 2 8 2
The equality holds for x = y = z.
Highest Coefficient Cancellation Method for Real Variables 353

P 3.80. If x, y, z are distinct real numbers, then

x2 y2 z2 4(x y + yz + z x)
+ + ≥ .
( y − z) 2 (z − x) 2 (x − y)2 x 2 + y 2 + z2

(Vasile C., 2014)

Solution. Write the inequality as f8 (x, y, z) ≥ 0, where



f8 (x, y, z) = (p2 − 2q) x 2 (x − y)2 (x − z)2 − 4q(x − y)2 ( y − z)2 (z − x)2 .

Since
(x − y)(x − z) = x 2 + 2 yz − q,
f8 (x, y, z) has the same highest polynomial as

(p2 − 2q) x 2 (x 2 + 2 yz)2 − 4q(x − y)2 ( y − z)2 (z − x)2 ,

that is, according to (3.1),

A(p, q) = (p2 − 2q) · 27 − 4q(−27) = 27(p2 + 2q).

We have
f8 (x, 1, 1) = (x 2 + 2)x 2 (x − 1)4 ≥ 0.

Case 1: p2 + 2q ≤ 0. Since f8 (x, 1, 1) ≥ 0 for all real x, the conclusion follows


from Theorem 3.

Case 2: p2 + 2q ≥ 0. Apply Corollary 4 for

4x 2 (x − 1)4 (x − γ)2
Eγ,δ (x) = fγ,0 (x) = .
9(4 − γ)2 (x + 2)2

Thus, we need to show that there exist a real number α such that

f8 (x, 1, 1) ≥ A(x + 2, 2x + 1) fγ,0 (x)

for x ∈ . Since
A(x + 2, 2x + 1) = 27(x 2 + 8x + 6),
12x 2 (x − 1)4 (x 2 + 8x + 6)(x − γ)2
A(x + 2, 2x + 1) fγ,0 (x) = ,
(4 − γ)2 (x + 2)2
x 2 (x − 1)4 h(x)
f8 (x, 1, 1) − A(x + 2, 2x + 1) fγ,0 (x) = ,
(4 − γ)2 (x + 2)2
h(x) = (4 − γ)2 (x + 2)2 (x 2 + 2) − 12(x 2 + 8x + 6)(x − γ)2 ,
354 Vasile Cîrtoaje

we need to show that h(x) ≥ 0 for x ∈ . Since h(4) = 0, the condition h (4) = 0
is necessary to have h(x) ≥ 0 in the vicinity of 4. This implies α = −2/13 and

2(37x 4 − 216x 3 + 12x 2 + 800x + 960)


h(x) =
169
2(x − 4)2 (37x 2 + 80x + 60)
= ≥ 0.
169

The equality holds for x = 0 and y = z (or any cyclic permutation).

P 3.81. If x, y, z are distinct real numbers, then

x2 y2 z2 (x + y + z)2
+ + ≥ 2 .
( y − z) 2 (z − x) 2 (x − y) 2 x + y 2 + z2

(Vasile C., 2014)

Solution. By the Cauchy-Schwarz inequality, we have


 x2 (x + y + z)2
≥ ! .
( y − z)2 ( y − z)2

Thus, it suffices to show that


1 1
! ≥ 2 ,
( y − z)2 x + y 2 + z2

which is equivalent to

x 2 + y 2 + z 2 ≤ 2(x y + yz + z x),

p2 ≤ 4q.
Consider next that
p2 ≥ 4q,
and write the inequality as f8 (x, y, z) ≥ 0, where

f8 (x, y, z) = (x 2 + y 2 +z 2 ) x 2 (x − y)2 (x −z)2 −(x + y +z)2 (x − y)2 ( y −z)2 (z− x)2 .

Since
(x − y)(x − z) = x 2 + 2 yz − q,
f8 (x, y, z) has the same highest polynomial as

(x 2 + y 2 + z 2 ) x 2 (x 2 + 2 yz)2 − (x + y + z)2 (x − y)2 ( y − z)2 (z − x)2 ,
Highest Coefficient Cancellation Method for Real Variables 355

that is, according to (3.1),

A(p, q) = (x 2 + y 2 + z 2 ) · 27 − (x + y + z)2 (−27) = 54(p2 − q).

Since
A(p, q) = 18[2p2 + (p2 − 3q)] ≥ 0,
A(x + 2, 2x + 1) = 54(x 2 + 2x + 3),
f8 (x, 1, 1) = (x 2 + 2)x 2 (x − 1)4 ,
we apply Corollary 4 for p2 ≥ 4q (see Remark 6) and

1 2
Eγ,δ (x) = f0,−2 (x) = x (x − 1)4 .
81
Thus, we need to show that

f8 (x, 1, 1) ≥ A(x + 2, 2x + 1) f0,−2 (x)

for (x + 2)2 ≥ 4(2x + 1), that is for

x ∈ (−∞, 0] ∪ [4, ∞).

Since
2(x 2 + 2x + 3)x 2 (x − 1)4
A(x + 2, 2x + 1) f0,−2 (x) = ,
3
we have
x 2 (x − 1)4 x(x − 4)
f8 (x, 1, 1) − A(x + 2, 2x + 1) f0,−2 (x) = ≥ 0.
3

The equality holds for x = 0 and y = z (or any cyclic permutation).

P 3.82. If x, y, z are real numbers, then

2x y 2 yz 2z x 4(x y + yz + z x)
+ + +1≥ .
x 2 + y 2 y 2 + z2 z2 + x 2 x 2 + y 2 + z2

(Vasile C., 2014)

Solution. Write the inequality as f8 (x, y, z) ≥ 0, where



f8 (x, y, z) = 2(p2 −2q) yz(x 2 + y 2 )(x 2 +z 2 )+(p2 −6q)(x 2 + y 2 )( y 2 +z 2 )(z 2 + x 2 ).

Since
y 2 + z 2 = −x 2 + p2 − 2q,
356 Vasile Cîrtoaje

the polynomial of degree eight f8 (x, y, z) has the same highest polynomial A(p, q)
as 
2(p2 − 2q) yz(−z 2 )(− y 2 ) + (p2 − 6q)(−z 2 )(−x 2 )(− y 2 ),
that is

A(p, q) = 6(p2 − 2q) − (p2 − 6q) = 5p2 − 6q = 2(p2 − 3q) + 3p2 > 0,

A(x + 2, 2x + 1) = 5x 2 + 8x + 14 > 0.
Since
 
f8 (x, 1, 1) = 2(x 2 + 2) (x 2 + 1)2 + 4x(x 2 + 1) + 2(x 2 − 8x − 2)(x 2 + 1)2
= 4x 2 (x 2 + 1)(x − 1)2 ,

4x 2 (x − 1)4 4x 2 (x − 1)4 (5x 2 + 8x + 14)


f0,∞ (x) = , A(x + 2, 2x + 1) f0,∞ (x) = ,
9(x + 2)2 9(x + 2)2
4x 2 (x − 1)2 f (x)
f8 (x, 1, 1) − A(x + 2, 2x + 1) f0,∞ (x) = ,
9(x + 2)2
where

f (x) = 9(x 2 + 1)(x + 2)2 − (x − 1)2 (5x 2 + 8x + 14)


= (2x + 1)(x 3 + 9x 2 + 6x + 11),

and
4(x − 1)4 4(x − 1)4 (5x 2 + 8x + 14)
f∞,−2 (x) = , A(x + 2, 2x + 1) f∞,−2 (x) = ,
81 81
4(x − 1)2 g(x)
f8 (x, 1, 1) − A(x + 2, 2x + 1) f∞,−2 (x) = ,
81
where

g(x) = 81x 2 (x 2 + 1) − (x − 1)2 (5x 2 + 8x + 14)


= (2x + 1)(19x 3 − 9x 2 + 24x − 7),

we apply Theorem 4 for


η → ∞, ξ → ∞,
  
−1 −1
= ,∞ ,  \  = −∞, ,
2 2
and for
Eα,β = f0,∞ , Fγ,δ = f∞,−2 .

−1
The condition (a), namely f8 (x, 1, 1) − A(x + 2, 2x + 1) f0,∞ (x) ≥ 0 for x > ,
2
is satisfied because

f (x) = (2x + 1)[x 2 (x + 9) + (6x + 11)] > 0.


Highest Coefficient Cancellation Method for Real Variables 357

−1
The condition (b), namely f8 (x, 1, 1) ≥ A(x + 2, 2x + 1) f∞,−2 (x) for x ≤ , is
2
satisfied because

g(x) = (2x + 1)[19x 3 + (−9x 2 ) + 24x − 7] ≥ 0.

The equality holds for x = y = z, and also for x = 0 and y = z (or any cyclic
permutation).
358 Vasile Cîrtoaje
Chapter 4

Highest Coefficient Cancellation


Method for Symmetric Homogeneous
Inequalities in Nonnegative Variables

4.1 Theoretical Basis


The Highest Coefficient Cancellation Method (HCC-Method) is especially appli-
cable to symmetric homogeneous polynomial inequalities of six and eight degree.
The main results in this section are based on the following Lemma (see P 3.57 in
Volume 1):
Lemma. If x ≤ y ≤ z are nonnegative real numbers such that
x + y + z = p, x y + yz + z x = q,
where p and q are given nonnegative real numbers satisfying p2 ≥ 3q, then the product
r = x yz is maximum when x = y, and is minimum when y = z (for p2 ≤ 4q) or
x = 0 (for p2 ≥ 4q).

4.1.1. Inequalities of degree six


A symmetric and homogeneous polynomial of degree six can be written in the
form
   
f6 (x, y, z) = A1 x 6 + A2 x y(x 4 + y 4 ) + A3 x 2 y 2 (x 2 + y 2 ) + A4 x3 y3
 
+ A5 x yz x 3 + A6 x yz x y(x + y) + 3A7 x 2 y 2 z 2 ,

where A1 , · · · , A7 are real coefficients. In terms of


p = x + y + z, q = x y + yz + z x, r = x yz,
it can be rewritten as
f6 (x, y, z) = Ar 2 + g1 (p, q)r + g2 (p, q),

359
360 Vasile Cîrtoaje

where A is the highest coefficient of f6 (x, y, z) (see section Theoretical Basis from
Chapter 3), and g1 (p, q) and g2 (p, q) are polynomial functions of the form

g1 (p, q) = Bp3 + C pq, g2 (p, q) = Dp6 + E p4 q + F p2 q2 + Gq3 ,

where B, C, D, E, F, G are real coefficients.


The highest coefficients of the polynomials
   
x 6, x y(x 4 + y 4 ), x 2 y 2 (x 2 + y 2 ), x 3 y 3,
 
x yz x 3 , x yz x y(x + y)

are, respectively,
3, −3, −3, 3, 3, −3.

As shown in Chapter 3, the polynomials



P1 (x, y, z) = (A1 x 2 + A2 yz)(B1 x 2 + B2 yz)(C1 x 2 + C2 yz),

P2 (x, y, z) = (A1 x 2 + A2 yz)(B1 y 2 + B2 z x)(C1 z 2 + C2 x y),

P3 (x, y, z) = (A1 x 2 + A2 yz)(A1 y 2 + A2 z x)(A1 z 2 + A2 x y)


and
P4 (x, y, z) = (x − y)2 ( y − z)2 (z − x)2

has the highest coefficients P1 (1, 1, 1), P2 (1, 1, 1), P3 (1, 1, 1) and −27, respectively.
Based on Lemma above, Theorem 1 bellow gives for A ≤ 0 the necessary and
sufficient conditions to have f6 (x, y, z) ≥ 0 for all nonnegative real numbers x, y, z.

Theorem 1. Let f6 (x, y, z) be a sixth degree symmetric homogeneous polynomial


having the highest coefficient A ≤ 0. The inequality f6 (x, y, z) ≥ 0 holds for all
nonnegative real numbers x, y, z if and only if f6 (x, 1, 1) ≥ 0 and f6 (0, y, z) ≥ 0
for all nonnegative real numbers x, y, z.

Theorem 1 can be extended in the following form (see P 3.76 in Volume 1, page
173):
Theorem 1 . Let f6 (x, y, z) be a sixth degree symmetric homogeneous polynomial
having the highest coefficient A ≤ 0.
(a) The inequality f6 (x, y, z) ≥ 0 holds for all nonnegative real numbers x, y, z
satisfying p2 ≤ 4q if and only if f6 (x, 1, 1) ≥ 0 for x ∈ [0, 4] ;
(b) The inequality f6 (x, y, z) ≥ 0 holds for all nonnegative real numbers x, y, z
satisfying p2 ≥ 4q if and only if f6 (x, 1, 1) ≥ 0 for x ≥ 4 , and f6 (0, y, z) ≥ 0 for
y, z ≥ 0.
Highest Coefficient Cancellation Method for Nonnegative Variables 361

∗∗∗

Further, consider the inequality

f6 (x, y, z) ≥ 0,

where x, y, z are nonnegative real numbers and f6 (x, y, z) is a symmetric homo-


geneous polynomial of degree six with the highest coefficient A > 0. The highest
coefficient cancellation method for proving such an inequality uses Theorem 1 and
the following three ideas:
1) finding a nonnegative symmetric homogeneous function f¯6 (x, y, z) of the
form  2
q2
f¯6 (x, y, z) = r + A1 pq + A2 p3 + A3 , (4.1)
p
where A1 , A2 , A3 are real numbers chosen such that

f6 (x, y, z) ≥ A f¯6 (x, y, z) ≥ 0

for all nonnegative real numbers x, y, z;


2) seeing that the the difference f6 (x, y, z) − A f¯6 (x, y, z) has the highest coef-
ficient equal to zero, therefore the inequality

f6 (x, y, z) ≥ A f¯6 (x, y, z)

holds for all nonnegative real numbers x, y, z if and only if it holds for y = z = 1
and for x = 0 (see Theorem 1);
3) treating successively the cases p2 < 4q and p2 ≥ 4q.

∗∗∗

Let us define the following nonnegative functions:

4(x − 1)4 (x − α)2 (x − β)2


fα,β (x) = , (4.2)
9(4 − α − β − 2αβ)2 (x + 2)2

[2( y + z)4 − (10 + α + β) yz( y + z)2 + 2(2 + α)(2 + β) y 2 z 2 ]2


fˆα,β ( y, z) = , (4.3)
9(4 − α − β − 2αβ)2 ( y + z)2

 2
(2x + 1)2
gα,β (x) = x + α(x + 2)(2x + 1) + β , (4.4)
x +2
y 2z2  2
ĝα,β ( y, z) = α( y + z)2 + β yz , (4.5)
( y + z)2
362 Vasile Cîrtoaje

 2
hα,β (x) = x + α(x + 2)(2x + 1) + β(x + 2)3 , (4.6)
 2
ĥα,β ( y, z) = ( y + z)2 α yz + β( y + z)2 . (4.7)

The functions fα,β (x) and fˆα,β ( y, z) have been derived by setting respectively
y = z = 1 and x = 0 in the associated function
 2
b1 2 c1 q2
f¯6 (x, y, z) = r − pq + p3 + · (4.8)
a1 a1 a1 p

with

a1 = 3(4 − α − β − 2αβ), b1 = 10 + α + β, c1 = 2(2 + α)(2 + β),

which satisfies f¯6 (1, 1, 1) = 0, f¯6 (α, 1, 1) = 0, f¯6 (β, 1, 1) = 0:


 2
b1 2 c1 (2x + 1)2
fα,β (x) = f¯6 (x, 1, 1) = x − (x + 2)(2x + 1) + (x + 2)3 + · ,
a1 a1 a1 x +2
 
2 2 2
ˆ ¯ b1 2 c1 y z
fα,β ( y, z) = f6 (0, y, z) = − yz( y + z) + ( y + z) +
3
· .
a1 a1 a1 y + z

The functions gα,β (x) and ĝα,β ( y, z) have been derived by setting respectively
y = z = 1 and x = 0 in the associated function
 2
q2
f¯6 (x, y, z) = r + αpq + β .
p

The functions hα,β (x) and ĥα,β ( y, z) have been derived by setting respectively
y = z = 1 and x = 0 in the associated function
 2
f¯6 (x, y, z) = r + αpq + β p3 .

∗∗∗

With regard to the functions fα,β (x) and fˆα,β (x), we get the following expressions
for β = −2, β = −1, β = 0, β = 1 and β → ∞:

4(x − 1)4 (x − α)2


fα,−2 (x) = ,
81(2 + α)2
4(x + 1)2 (x − 1)4 (x − α)2
fα,−1 (x) = ,
9(5 + α)2 (x + 2)2
4x 2 (x − 1)4 (x − α)2
fα,0 (x) = ,
9(4 − α)2 (x + 2)2
Highest Coefficient Cancellation Method for Nonnegative Variables 363

4(x − 1)6 (x − α)2


fα,1 (x) = ,
81(1 − α)2 (x + 2)2
4(x − 1)4 (x − α)2
fα,∞ (x) =
9(1 + 2α)2 (x + 2)2
and

( y + z)2 [2( y + z)2 − (8 + α) yz]2


fˆα,−2 ( y, z) = ,
81(2 + α)2
[2( y + z)4 − (9 + α) yz( y + z)2 + 2(2 + α) y 2 z 2 ]2
fˆα,−1 ( y, z) = ,
9(5 + α)2 ( y + z)2
( y − z)4 [2( y + z)2 − (2 + α) yz]2
fˆα,0 ( y, z) = ,
9(4 − α)2 ( y + z)2
[2( y + z)4 − (11 + α) yz( y + z)2 + 6(2 + α) y 2 z 2 ]2
fˆα,1 ( y, z) = ,
81(1 − α)2 ( y + z)2
y 2 z 2 [( y + z)2 − 2(2 + α) yz]2
fˆα,∞ ( y, z) = .
9(1 + 2α)2 ( y + z)2

In particular,

4(x − 1)4 y 2 z 2 ( y + z)2


f∞,−2 (x) = , fˆ∞,−2 ( y, z) = ,
81 81

4(x − 1)4 (x + 1)2 y 2 z 2 ( y 2 + z 2 )2


f−1,∞ (x) = , fˆ−1,∞ ( y, z) = ,
9(x + 2)2 9( y + z)2
4x 2 (x − 1)4 y 2 z 2 ( y − z)4
f0,∞ (x) = , fˆ0,∞ ( y, z) = ,
9(x + 2)2 9( y + z)2
4(x − 1)6 y 2 z 2 ( y 2 + z 2 − 4 yz)2
f1,∞ (x) = , fˆ1,∞ ( y, z) = ,
81(x + 2)2 81( y + z)2
(x − 1)4 y 4z4
f∞,∞ (x) = , fˆ∞,∞ ( y, z) = .
9(x + 2)2 9( y + z)2

With regard to the functions gα,β (x) and ĝα,β (x), we get the following particular
expressions:

gα,0 (x) = [x + α(x + 2)(2x + 1)]2 , ĝα,0 ( y, z) = α2 y 2 z 2 ( y + z)2 ,


 2
(2x + 1)2 β 2 y 4z4
g0,β (x) = x + β , ĝ0,β ( y, z) = ,
x +2 ( y + z)2
g0,0 (x) = x 2 , ĝ0,0 ( y, z) = 0.
364 Vasile Cîrtoaje

With regard to the functions hα,β (x) and ĥα,β (x), we have

hα,0 (x) = [x + α(x + 2)(2x + 1)]2 , ĥα,0 ( y, z) = α2 y 2 z 2 ( y + z)2 ,


 2
h0,β (x) = x + β(x + 2)3 , ĥ0,β ( y, z) = β 2 ( y + z)6 ,
h0,0 (x) = x 2 , ĥ0,0 ( y, z) = 0.

Notice that the relative degree of the rational functions fα,β (x), fα,∞ (x) and
f∞,∞ (x) are six, four and two, respectively. Also, gα,β (x) and g0,0 (x) have the rel-
ative degree equal to four and two, respectively, while hα,β (x), hα,0 (x) and h0,0 (x)
have the relative degree equal to six, four and two, respectively.

∗∗∗

The following theorem is useful to prove symmetric homogeneous polynomial


inequalities of sixth degree in nonnegative real variables x, y, z and having the
highest coefficient A ≥ 0.

Theorem 2. Let f6 (x, y, z) be a symmetric homogeneous polynomial of degree six


having the highest coefficient A ≥ 0. The inequality f6 (x, y, z) ≥ 0 holds for all
x, y, z ≥ 0 if there exist four real numbers α, β, γ and δ, and

Eα,β ∈ { fα,β , gα,β , hα,β },

Fγ,δ ∈ { fγ,δ , gγ,δ , hγ,δ },


such that the following three conditions are satisfied:
(a) f6 (x, 1, 1) ≥ AEα,β (x) for 0 ≤ x ≤ 4 ;
(b) f6 (x, 1, 1) ≥ AFγ,δ (x) for x ≥ 4 ;
(c) f6 (0, y, z) ≥ AF̂γ,δ ( y, z) for y, z ≥ 0.

Proof. Let
 2
q2
E1 (x, y, z) = r + A1 pq + A2 p3 + A3
p
and  2
q2
F1 (x, y, z) = r + B1 pq + B2 p3 + B3
p
be the functions associated to Eα,β (x) and Fγ,δ (x), respectively; this means that

E1 (x, 1, 1) = Eα,β (x),

F1 (x, 1, 1) = Fγ,δ (x).


Let us denote
E2 (x, y, z) = f6 (x, y, z) − AE1 (x, y, z)
Highest Coefficient Cancellation Method for Nonnegative Variables 365

and
F2 (x, y, z) = f6 (x, y, z) − AF1 (x, y, z).
Since AE1 (x, y, z) ≥ 0 and AF1 (x, y, z) ≥ 0, the inequality f6 (x, y, z) ≥ 0 holds for
all nonnegative real x, y, z if
(a) E2 (x, y, z) ≥ 0 for p2 ≤ 4q;
(b) F2 (x, y, z) ≥ 0 for p2 ≥ 4q.
According to Theorem 1 , since E2 (x, y, z) and F2 (x, y, z) has the highest coefficient
zero, these conditions are satisfied if and only if
(a) E2 (x, 1, 1) ≥ 0 for (x + 2)2 ≤ 4(2x + 1);
(b) F2 (x, 1, 1) ≥ 0 for (x +2)2 ≥ 4(2x +1), and F2 (0, y, z) ≥ 0 for ( y +z)2 ≥ 4 yz.
Since these conditions are equivalent to the condition (a), (b) and (c) in Theorem
2, the proof is completed.

For Fγ,δ = g0,0 , when F̂γ,δ = 0, we get

Corollary 1. Let f6 (x, y, z) be a symmetric homogeneous polynomial of degree six


having the highest coefficient A ≥ 0. The inequality f6 (x, y, z) ≥ 0 holds for all
x, y, z ≥ 0 if there exist two real numbers α and β, and

Eα,β ∈ { fα,β , gα,β , hα,β },

such that the following three conditions are satisfied:


(a) f6 (x, 1, 1) ≥ AEα,β (x) for 0 ≤ x ≤ 4 ;
(b) f6 (x, 1, 1) ≥ Ax 2 for x ≥ 4 ;
(c) f6 (0, y, z) ≥ 0 for y, z ≥ 0 .

Remark 1. The function f¯6 (x, y, z) associated to fα,β (x) (given by (4.8)) is zero for

(x, y, z) = (1, 1, 1), (x, y, z) = (α, 1, 1), (x, y, z) = (β, 1, 1),

and also for x = 0 and



y z α+β +2± (α + β + 10)2 − 16(α + 2)(β + 2)
+ = .
z y 4

If β = −2, then the function associated to fα,−2 (x) has the expression
 2
α+8 2
f¯6 (x, y, z) = r − pq + p3 , (4.9)
9α + 18 9α + 18

and is zero for


(x, y, z) = (1, 1, 1), (x, y, z) = (α, 1, 1).
366 Vasile Cîrtoaje

In addition, if α → ∞, then
 2
1
f¯6 (x, y, z) = r − pq
9
is zero for
(x, y, z) = (1, 1, 1), (x, y, z) = (1, 0, 0).

If β → ∞, then
 2
1 2α + 4 q2
f¯6 (x, y, z) = r + pq − ·
6α + 3 6α + 3 p

is zero for

(x, y, z) = (1, 1, 1), (x, y, z) = (α, 1, 1), (x, y, z) = (1, 0, 0).

If, in addition, α → ∞, then


 2
1 q2
f¯6 (x, y, z) = r − ·
3 p

is zero for
(x, y, z) = (1, 1, 1), (x, y, z) = (1, 0, 0).

Remark 2. If
f6 (x, 1, 1) = Afγ,−2 (x)
for all x ∈ , then there is k ≥ 0 such that the following identity holds:

f6 (x, y, z) = A f¯6 (x, y, z) + k(x − y)2 ( y − z)2 (z − x)2 ,

where, according to (4.9),


 2
γ+8 2
f¯6 (x, y, z) = r − pq + p3 .
9γ + 18 9γ + 18

In addition, if the coefficient of the product

(x − y)2 ( y − z)2 (z − x)2

in the inequality f6 (x, y, z) ≥ 0 is the best possible, then

k = 0,

and the following identity holds:


 2
γ+8 2
f6 (x, y, z) = A r − pq + p3 . (4.10)
9γ + 18 9γ + 18
Highest Coefficient Cancellation Method for Nonnegative Variables 367

Remark 3. Theorem 2 is also valid for the case where the parameters α and β of
the function fα,β (x) are conjugate complex numbers. For example, if k > 0 and
 
α = −k, β = − −k,

then, according to (4.2), we have

(x − 1)4 (x 2 + k)2
f
−k,−
−k = . (4.11)
9(k − 2)2 (x + 2)2

∗∗∗

The following theorem is also useful to prove some inequalities f6 (x, y, z) ≥ 0 ,


where f6 (x, y, z) is a symmetric homogeneous polynomial of degree six having the
highest coefficient A ≥ 0 and satisfying f6 (1, 1, 1) = 0 and/or f6 (0, 1, 1) = 0.

Theorem 3. Let f6 (x, y, z) be a symmetric homogeneous polynomial of degree six


having the highest coefficient A ≥ 0. The inequality f6 (x, y, z) ≥ 0 holds for all
x, y, z ≥ 0 if
(a) f6 (x, 1, 1) ≥ 0 for 0 ≤ x ≤ 1 ;
4Ax(x − 1)3
(b) f6 (x, 1, 1) ≥ for x ≥ 1 ;
27
(c) f6 (0, y, z) ≥ 0 for y, z ≥ 0 .

Proof. From

−27r 2 + 2(9pq − 2p3 )r + p2 q2 − 4q3 = (a − b)2 (b − c)2 (c − a)2 ≥ 0,

we get 
9pq − 2p3 − 2(p2 − 3q) p2 − 3q
r≥ .
27
Define the nonnegative function
  
9pq − 2p3 − 2(p2 − 3q) p2 − 3q
E(x, y, z) = r r − ,
27

which has the highest coefficient equal to 1. We have

E(0, y, z) = 0,

⎨ 0, 0≤ x ≤1
E(x, 1, 1) = .
⎩ 4x(x − 1) , x ≥ 1
3

27
Let us denote
E1 (x, y, z) = f6 (x, y, z) − AE(x, y, z).
368 Vasile Cîrtoaje

Since AE(x, y, z) ≥ 0, the inequality f6 (x, y, z) ≥ 0 holds for all nonnegative real
x, y, z if E1 (x, y, z) ≥ 0 for x, y, z ≥ 0. According to Theorem 1, because E1 (x, y, z)
has the highest coefficient zero, the inequality E1 (x, y, z) ≥ 0 holds for all x, y, z ≥ 0
if and only if E1 (x, 1, 1) ≥ 0 and E1 (0, y, z) ≥ 0 for all nonnegative numbers x, y, z.
These conditions are equivalent to the condition (a), (b) and (c) in Theorem 3.
Thus, the proof is completed.
Remark 4. Theorem 2 and Corollary 1 are also valid by replacing the condition (a)
with the following two conditions:
(a1 ) f6 (x, 1, 1) ≥ 0 for 0 ≤ x ≤ 1 ;
4Ax(x − 1)3
(a2 ) f6 (x, 1, 1) ≥ for 1 ≤ x ≤ 4.
27

4.1.2. Inequalities of degree seven and eight


A symmetric and homogeneous polynomial of degree seven has the highest poly-
nomial of the form
A(p, q) = μ1 p, μ1 ∈ ,
while a symmetric and homogeneous polynomial of degree eight has the highest
polynomial of the form

A(p, q) = μ1 p2 + μ2 q, μ1 , μ2 ∈ .

Theorems 1, 1 , 2 and 3 can be extended to these polynomials as follows:
Theorem 4. Let f (x, y, z) be a symmetric homogeneous polynomial of degree seven
or eight which has the highest polynomial A(p, q). The inequality f (x, y, z) ≥ 0
holds for all nonnegative real numbers x, y, z satisfying A(p, q) ≤ 0 if and only if
f (x, 1, 1) ≥ 0 and f (0, y, z) ≥ 0 for all nonnegative real numbers x, y, z such that
A(x + 2, 2x + 1) ≤ 0 and A( y + z, yz) ≤ 0.

Corollary 2. Let f (x, y, z) be a symmetric homogeneous polynomial of degree seven


or eight having the highest polynomial A(p, q). The inequality f (x, y, z) ≥ 0 holds
for all nonnegative real numbers x, y, z satisfying A(p, q) ≤ 0 if f (x, 1, 1) ≥ 0 and
f (0, y, z) ≥ 0 for all nonnegative real numbers x, y, z.

Theorem 5. Let f (x, y, z) be a symmetric homogeneous polynomial of degree seven


or eight having the highest polynomial A(p, q). The inequality f6 (x, y, z) ≥ 0 holds
for all nonnegative x, y, z satisfying A(p, q) ≥ 0 if there exist four real numbers α,
β, γ and δ, and
Eα,β ∈ { fα,β , gα,β , hα,β },
Fγ,δ ∈ { fγ,δ , gγ,δ , hγ,δ },
such that the following three conditions are satisfied:
(a) f (x, 1, 1) ≥ A(x + 2, 2x + 1)Eα,β (x) for 0 ≤ x ≤ 4 and A(x + 2, 2x + 1) ≥ 0 ;
Highest Coefficient Cancellation Method for Nonnegative Variables 369

(b) f (x, 1, 1) ≥ A(x + 2, 2x + 1)Fγ,δ (x) for x ≥ 4 and A(x + 2, 2x + 1) ≥ 0 ;


(c) f (0, y, z) ≥ A( y + z, yz) F̂γ,δ ( y, z) for y, z ≥ 0 and A( y + z, yz) ≥ 0 .

For Fγ,δ = g0,0 , we get the following corollaries:

Corollary 3. Let f (x, y, z) be a symmetric homogeneous polynomial of degree degree


seven or eight having the highest polynomial A(p, q) . The inequality f6 (x, y, z) ≥ 0
holds for all nonnegative x, y, z satisfying A(p, q) ≥ 0 if there exist two real numbers
α and β, and
Eα,β ∈ { fα,β , gα,β , hα,β },
such that the following three conditions are satisfied:
(a) f (x, 1, 1) ≥ A(x + 2, 2x + 1)Eα,β (x) for 0 ≤ x ≤ 4 and A(x + 2, 2x + 1) ≥ 0 ;
(b) f (x, 1, 1) ≥ A(x + 2, 2x + 1)x 2 for x ≥ 4 and A(x + 2, 2x + 1) ≥ 0 ;
(c) f (0, y, z) ≥ 0 for y, z ≥ 0 and A( y + z, yz) ≥ 0 .

Theorem 6. Let f (x, y, z) be a symmetric homogeneous polynomial of degree seven


or eight having the highest polynomial A(p, q). The inequality f6 (x, y, z) ≥ 0 holds
for all nonnegative x, y, z satisfying A(p, q) ≥ 0 if
(a) f (x, 1, 1) ≥ 0 for 0 ≤ x ≤ 1 and A(x + 2, 2x + 1) ≥ 0 ;
4A(x + 2, 2x + 1)x(x − 1)3
(b) f (x, 1, 1) ≥ for x ≥ 1 and A(x +2, 2x +1) ≥ 0 ;
27
(c) f (0, y, z) ≥ 0 for y, z ≥ 0 and A( y + z, yz) ≥ 0 .
370 Vasile Cîrtoaje
Highest Coefficient Cancellation Method for Nonnegative Variables 371

4.2 Applications

4.1. If x, y, z are nonnegative real numbers, then



x( y + z)(x − y)(x − z)(x − 3 y)(x − 3z) ≥ 0.

4.2. If x, y, z are nonnegative real numbers, then



x(2x + y +z)(x − y)(x −z)(2x −11 y)(2x −11z)+102(x − y)2 ( y −z)2 (z− x)2 ≥ 0.

4.3. If x, y, z are nonnegative real numbers, then



x(2x + y + z)(x − y)(x − z)(x − 3 y)(x − 3z) + 8(x − y)2 ( y − z)2 (z − x)2 ≥ 0.

4.4. If x, y, z are nonnegative real numbers, no two of which are zero, then
1 1 1 9
+ + ≥ 2 .
4x 2 + yz 4 y 2 + z x 4z 2 + x y (x + 4 yz) + ( y 2 + 4z x) + (z 2 + 4x y)

4.5. If x, y, z are nonnegative real numbers, then

2x 2 − yz 2 y2 − z x 2z 2 − x y 1
+ + 2 ≤ .
4x + y + z
2 2 2 4y + z + x
2 2 2 4z + x 2 + y 2 2

4.6. If x, y, z are real numbers, then

21x 2 + 4 yz 21 y 2 + 4z x 21z 2 + 4x y
+ + ≥ 15.
x 2 + 2 y 2 + 2z 2 y 2 + 2z 2 + 2x 2 z 2 + 2x 2 + 2 y 2

4.7. If x, y, z are real numbers, then


x y − yz + z x yz − z x + x y z x − x y + yz 3
+ + ≤ .
x 2 + 3 y 2 + 3z 2 y 2 + 3z 2 + 3x 2 z 2 + 3x 2 + 3 y 2 7

4.8. If x, y, z are nonnegative real numbers, then



x 3 (2x + y + z)(x − y)(x − z) ≥ 13(x − y)2 ( y − z)2 (z − x)2 .
372 Vasile Cîrtoaje

11
4.9. Let x, y, z be nonnegative real numbers. If k ≤ , then
2

x(2x + y +z)(x − y)(x −z)(x −k y)(x −kz)+(7k−13)(x − y)2 ( y −z)2 (z− x)2 ≥ 0.

4.10. If x, y, z are nonnegative real numbers, then


 12(x − y)2 ( y − z)2 (z − x)2
x 2 (x − y)(x − z) ≥ .
(x + y + z)2

4.11. Let x, y, z be nonnegative real numbers. If k is a real number, then



x( y + z)(x − y)(x − z)(x − k y)(x − kz) + (k − 3)(x − y)2 ( y − z)2 (z − x)2 ≥ 0.

4.12. Let x, y, z be nonnegative real numbers. If k ≤ 4, then



x 2 (x − y)(x − z)(x − k y)(x − kz) + (3k − 5)(x − y)2 ( y − z)2 (z − x)2 ≥ 0.

4.13. Let x, y, z be nonnegative real numbers. If k is a real numbers, then



yz(x − y)(x − z)(x − k y)(x − kz) ≥ 0.

4.14. If x, y, z are nonnegative real numbers, then


 3(x − y)2 ( y − z)2 (z − x)2
x 2 (x − y)(x − z) ≥ .
x y + yz + z x

4.15. If x, y, z are nonnegative real numbers, then


 9(x − y)2 ( y − z)2 (z − x)2
(x − y)(x − z)(x + 2 y)(x + 2z) ≥ .
x y + yz + z x

4.16. If x, y, z are nonnegative real numbers, then


 6(x − y)2 ( y − z)2 (z − x)2
(x − y)(x − z)(x − 3 y)(x − 3z) ≥ .
x y + yz + z x
Highest Coefficient Cancellation Method for Nonnegative Variables 373

4.17. Let x, y, z be nonnegative real numbers, and let


,
3(1 − k), k ≤ 0
αk = .
3 + k, k≥0

Then,
 αk (x − y)2 ( y − z)2 (z − x)2
(x − y)(x − z)(x − k y)(x − kz) ≥ .
x y + yz + z x

4.18. If x, y, z are nonnegative real numbers, then


 4(x − y)2 ( y − z)2 (z − x)2
x 2 ( y + z)(x − y)(x − z) ≥ .
x + y +z

4.19. Let x, y, z be nonnegative real numbers. If k is a real numbers, then


 (2 + |k|)2 (x − y)2 ( y − z)2 (z − x)2
( y + z)(x − y)(x − z)(x − k y)(x − kz) ≥ .
x + y +z

4.20. If x, y, z are nonnegative real numbers, then


 12(x − y)2 ( y − z)2 (z − x)2
x(x 2 − y 2 )(x 2 − z 2 ) ≥ .
x + y +z

4.21. Let x, y, z be nonnegative real numbers, and let



⎨ 4(k − 2), k ≤ 6
αk = .
⎩ (k + 2) , k ≥ 6
2

4
Then,
 αk (x − y)2 ( y − z)2 (z − x)2
x(x − y)(x − z)(x − k y)(x − kz) + ≥ 0.
x + y +z

4.22. If x, y, z are nonnegative real numbers, then


 5(x − y)2 ( y − z)2 (z − x)2
(x 2 + yz)(x − y)(x − z) ≥ .
x y + yz + z x
374 Vasile Cîrtoaje

4.23. If x, y, z are nonnegative real numbers, then


 16(x − y)2 ( y − z)2 (z − x)2
(4x 2 + yz)(x − y)(x − z) ≥ .
x y + yz + z x

4.24. Let x, y, z be nonnegative real numbers. If k ≥ 0, then




(3 + 2 k )(x − y)2 ( y − z)2 (z − x)2


(x 2 + k yz)(x − y)(x − z) ≥ .
x y + yz + z x

4.25. If x, y, z are nonnegative real numbers, then




(x 2 − yz)2 (x − y)(x − z) ≥ 4( 2 + 1)(x − y)2 ( y − z)2 (z − x)2 .

4.26. If x, y, z are nonnegative real numbers, then


 1 9
≥ .
4x 2 + y 2 + z 2 4(x 2 + y 2 + z 2 ) + 2(x y + yz + z x)

4.27. If x, y, z are nonnegative real numbers, no two of which are zero, then
2 2 2 45
+ + ≥ .
x 2 + y 2 y 2 + z2 z2 + x 2 4(x 2 + y 2 + z 2 ) + x y + yz + z x

4.28. If x, y, z are nonnegative real numbers, no two of which are zero, then
 1 18
≥ .
2 y 2 + yz + 2z 2 5(x 2 + y 2 + z 2 + x y + yz + z x)

4.29. If x, y, z are nonnegative real numbers, no two of which are zero, then
 x − (2 +
2)( y + z)

9(3 + 2 2)
+ ≥ 0.
( y + z)2 4(x + y + z)

4.30. If x, y, z are nonnegative real numbers, no two of which are zero, then
6x − y − z 6 y − z − x 6z − x − y 18
+ + ≥ .
y 2 + z2 z2 + x 2 x2 + y2 x + y +z
Highest Coefficient Cancellation Method for Nonnegative Variables 375

4.31. If x, y, z are nonnegative real numbers, no two of which are zero, then
 2x − 3 y − 3z 6
+ ≥ 0.
y 2 + 4 yz + z 2 x + y + z

4.32. If x, y, z are nonnegative real numbers, no two of which are zero, then
 7x + 4 y + 4z 27
≥ .
4x 2 + yz x + y +z

4.33. If x, y, z are nonnegative real numbers, then


 9x − 2 y − 2z 3
≤ .
7x 2 + 8 yz x + y +z

4.34. If x, y, z are nonnegative real numbers, then


 y +z 2
≥ .
7x 2 + y 2 + z 2 x + y +z

4.35. If x, y, z are nonnegative real numbers, then


 7x − 2 y − 2z 3
≥ .
x 2 + 4 y 2 + 4z 2 x + y +z

4.36. If x, y, z are nonnegative real numbers, no two of which are zero, then
 2x 2 + yz 9 31(x − y)2 ( y − z)2 (z − x)2
≥ + .
y +z
2 2 2 2(x 2 + y 2 )( y 2 + z 2 )(z 2 + x 2 )

4.37. If x, y, z are nonnegative real numbers, no two of which are zero, then
 2x 2 − yz 9(x − y)2 ( y − z)2 (z − x)2
≥3+ 2 .
y 2 − yz + z 2 (x − x y + y 2 )( y 2 − yz + z 2 )(z 2 − z x + x 2 )

4.38. If x, y, z are nonnegative real numbers, no two of which are zero, then
 x y − yz + z x 3 5(x − y)2 ( y − z)2 (z − x)2
≥ + .
y +z
2 2 2 2(x 2 + y 2 )( y 2 + z 2 )(z 2 + x 2 )
376 Vasile Cîrtoaje

4.39. If x, y, z are nonnegative real numbers, no two of which are zero, then
 x y − 2 yz + z x 3(x − y)2 ( y − z)2 (z − x)2
≥ 2 .
y − yz + z
2 2 (x − x y + y 2 )( y 2 − yz + z 2 )(z 2 − z x + x 2 )

4.40. If x, y, z are nonnegative real numbers, no two of which are zero, then
 x + 3 y + 3z 7(x + y + z)
≥ .
( y + 2z)(2 y + z) 3(x y + yz + z x)

4.41. If x, y, z are nonnegative real numbers, no two of which are zero, then
 9x − 5 y − 5z 3(x + y + z)
+ ≥ 0.
2 y 2 − 3 yz + 2z 2 x y + yz + z x

4.42. If x, y, z are nonnegative real numbers, no two of which are zero, then
 3x − y − z 3(x + y + z)
≥ .
y 2 + z2 2(x y + yz + z x)

4.43. Let x, y, z be nonnegative real numbers, no two of which are zero. If



1 − 17 1 + 17
k ∈ [a, b], a= ≈ −1.56155, b = ≈ 2.56155 ,
2 2
then  (3 − k)x + (k − 1)( y + z) 3(k + 1) x + y +z
≥ · .
y 2 + k yz + z 2 k+2 x y + yz + z x

4.44. If x, y, z are nonnegative real numbers, no two of which are zero, then
 x + 13 y + 13z 27(x + y + z)
≥ .
y 2 + 4 yz + z 2 2(x y + yz + z x)

4.45. If x, y, z are nonnegative real numbers, no two of which are zero, then
 −x + y + z x + y +z
≥ .
2x 2 + yz x y + yz + z x
Highest Coefficient Cancellation Method for Nonnegative Variables 377

4.46. If x, y, z are nonnegative real numbers, no two of which are zero, then
 11x − 3 y − 3z 3(x + y + z)
≤ .
2x 2 + 3 yz x y + yz + z x

4.47. If x, y, z are nonnegative real numbers, no two of which are zero, then
1 1 1 1 2
+ + ≥ + .
2x 2 + yz 2 y 2 + z x 2z 2 + x y x y + yz + z x x 2 + y 2 + z 2

4.48. If x, y, z are nonnegative real numbers, no two of which are zero, then
x( y + z) y(z + x) z(x + y) x 2 + y 2 + z2
+ 2 + 2 ≤ .
x + 5 yz y + 5z x z + 5x y
2 x y + yz + z x

4.49. If x, y, z are nonnegative real numbers, no two of which are zero, then
x( y + z) y(z + x) z(x + y) 15(x y + yz + z x)
+ 2 + 2 +2≥ .
x 2 + yz y + zx z +xy (x + y + z)2

4.50. If x, y, z are nonnegative real numbers, no two of which are zero, then
x( y + z) y(z + x) z(x + y) x y + yz + z x
+ + ≥1+ 2 .
x 2 + 2 yz y 2 + 2z x z 2 + 2x y x + y 2 + z2

4.51. If x, y, z are nonnegative real numbers such that x y + yz + z x = 3, then


 
1 1 1
18 + + + 5(x 2 + y 2 + z 2 ) ≥ 42.
x 2 + y 2 y 2 + z2 z2 + x 2

4.52. If x, y, z are nonnegative real numbers, then


2x y 2 yz 2z x 30(x y + yz + z x)
+ + +7≥ .
x 2 + y 2 y 2 + z2 z2 + x 2 (x + y + z)2

4.53. If x, y, z are nonnegative real numbers, then


2x y 2 yz 2z x x 2 + y 2 + z2 5
+ + + ≥ .
(x + y) 2 ( y + z) 2 (z + x) 2 x y + yz + z x 2

4.54. If x, y, z are nonnegative real numbers, no two of which are zero, then
2 2 2 8 1
+ + ≥ 2 + .
x 2 + y 2 y 2 + z2 z2 + x 2 x + y 2 + z 2 x y + yz + z x

4.55. If x, y, z are nonnegative real numbers, then


2x y 2 yz 2z x 4(x y + yz + z x)
+ + +1≥ .
x 2 + y 2 y 2 + z2 z2 + x 2 x 2 + y 2 + z2
378 Vasile Cîrtoaje
Highest Coefficient Cancellation Method for Nonnegative Variables 379

4.3 Solutions

P 4.1. If x, y, z are nonnegative real numbers, then



x( y + z)(x − y)(x − z)(x − 3 y)(x − 3z) ≥ 0.

(Vasile C., 2008)

Solution. Write the inequality as f6 (x, y, z) ≥ 0, where



f6 (x, y, z) = x(x + y)(x − y)(x − z)(x − 3 y)(x − 3z).

Since
x( y + z) = x(p − x),
(x − y)(x − z) = x 2 + 2 yz − q,
(x − 3 y)(x − 3z) = x 2 + 12 yz − 3q,
f6 (x, y, z) has the same highest coefficient A as

P1 (x, y, z) = −x 2 (x 2 + 2 yz)(x 2 + 12 yz),

that is
A = P1 (1, 1, 1) = −3(1 + 2)(1 + 12) < 0.
By Theorem 1, we only need to show that f6 (x, 1, 1) ≥ 0 and f6 (0, y, z) ≥ 0 for
x, y, z ≥ 0. We have

f6 (x, 1, 1) = 2x(x − 1)2 (x − 3)2 ≥ 0,

f6 (0, y, z) = yz( y − z)4 ≥ 0.

The equality holds for x = y = z, for x/3 = y = z (or any cyclic permutation),
for x = 0 and y = z (or any cyclic permutation), and for y = z = 0 (or any cyclic
permutation).
Observation. Similarly, we can prove the following generalization:
• Let x, y, z be nonnegative real numbers. If

k ∈ (−∞, −2 − 2 3] ∪ [−2 + 2 3, ∞),

then

x( y + z)(x − y)(x − z)(x − k y)(x − kz) + (k − 3)(x − y)2 ( y − z)2 (z − x)2 ≥ 0,

with equality

for x = y = z, for x/k = y = z (or any cyclic permutation) if k ≥
−2 + 2 3, for x = 0 and y = z (or any cyclic permutation), and for y = z = 0 (or
any cyclic permutation).
380 Vasile Cîrtoaje

We have

A = −3(1 + 2)(1 + k + k2 ) + (k − 3)(−27) = 9(8 − 4k − k2 ) ≤ 0,

f6 (x, 1, 1) = 2x(x − 1)2 (x − k)2 ≥ 0,


f6 (0, y, z) = yz( y − z)4 ≥ 0.

P 4.2. If x, y, z are nonnegative real numbers, then



x(2x + y +z)(x − y)(x −z)(2x −11 y)(2x −11z)+102(x − y)2 ( y −z)2 (z− x)2 ≥ 0.

(Vasile C., 2011)

Solution. Write the inequality as f6 (x, y, z) ≥ 0, where

f6 (x, y, z) = f (x) + 102(x − y)2 ( y − z)2 (z − x)2 ,



f (x) = x(2x + y + z)(x − y)(x − z)(2x − 11 y)(2x − 11z).
Since
x(2x + y + z) = x(x + p),
(x − y)(x − z) = x 2 + 2 yz − q,
(2x − 11 y)(2x − 11z) = 4x 2 + 143 yz − 22q,
f (x, y, z) has the same highest coefficient A1 as

P1 (x, y, z) = x 2 (x 2 + 2 yz)(4x 2 + 143 yz),

that is
A1 = P1 (1, 1, 1) = 3(1 + 2)(147) = 1323.
Therefore, f6 (x, y, z) has the highest coefficient

A = 1323 + 102(−27) = −1431.

By Theorem 1, we only need to show that f6 (x, 1, 1) ≥ 0 and f6 (0, y, z) ≥ 0 for


x, y, z ≥ 0. We have

f6 (x, 1, 1) = 2x(x + 1)(x − 1)2 (2x − 11)2 ≥ 0,

f6 (0, y, z) = ( y − z)[8( y 5 − z 5 ) − 40 yz( y 3 − z 3 ) − 22 y 2 z 2 ( y − z)] + 102 y 2 z 2 ( y − z)2


= 8( y − z)6 ≥ 0.
Highest Coefficient Cancellation Method for Nonnegative Variables 381

The equality holds for x = y = z, for 2x = 11 y = 11z (or any cyclic permuta-
tion), and for x = 0 and y = z (or any cyclic permutation).
Observation. Similarly, we can prove the following generalization:
• Let x, y, z be nonnegative real numbers. If

4k2 + 12k + 17 11
αk = , k≥ ,
8 2
then

x(2x + y + z)(x − y)(x − z)(x − k y)(x − kz) + αk (x − y)2 ( y − z)2 (z − x)2 ≥ 0,

with equality for x = y = z, for x/k = y = z (or any cyclic permutation), for x = 0
y z 2k − 3
and y = z (or any cyclic permutation), and for x = 0 and + = (or any
z y 4
cyclic permutation).
Denoting the left side of the inequality by f6 (x, y, z), we have

4k2 + 12k + 17
A = 3(1 + 2)(1 + k + k2 ) + (−27)
8
−9
= (4k2 + 28k + 387) ≤ 0,
8
f6 (x, 1, 1) = 2x(x + 1)(x − 1)2 (x − k)2 ≥ 0,
1
f6 (0, y, z) = ( y − z)2 [4 y 2 + 4z 2 − (2k − 3) yz]2 ≥ 0.
8

P 4.3. If x, y, z are nonnegative real numbers, then



x(2x + y + z)(x − y)(x − z)(x − 3 y)(x − 3z) + 8(x − y)2 ( y − z)2 (z − x)2 ≥ 0.

(Vasile C., 2011)

Solution. Write the inequality as f6 (x, y, z) ≥ 0, where

f6 (x, y, z) = f (x) + 8(x − y)2 ( y − z)2 (z − x)2 ,



f (x) = x(2x + y + z)(x − y)(x − z)(x − 3 y)(x − 3z).
Since
x(2x + y + z) = x(x + p),
(x − y)(x − z) = x 2 + 2 yz − q,
(x − 3 y)(x − 3z) = x 2 + 12 yz − 3q,
382 Vasile Cîrtoaje

f (x, y, z) has the same highest coefficient A1 as



P1 (x, y, z) = x 2 (x 2 + 2 yz)(x 2 + 12 yz),

that is
A1 = P1 (1, 1, 1) = 3(1 + 2)(1 + 12) = 117.
Therefore, f6 (x, y, z) has the highest coefficient

A = 117 + 8(−27) = −99.

By Theorem 1, we only need to show that f6 (x, 1, 1) ≥ 0 and f6 (0, y, z) ≥ 0 for


x, y, z ≥ 0. We have

f6 (x, 1, 1) = 2x(x + 1)(x − 1)2 (x − 3)2 ≥ 0,

f6 (0, y, z) = ( y − z)[2( y 5 − z 5 ) − 5 yz( y 3 − z 3 ) − 3 y 2 z 2 ( y − z)] + 8 y 2 z 2 ( y − z)2


= ( y − z)4 (2 y 2 + 2 y 2 + yz) ≥ 0.

The equality holds for x = y = z, for x/3 = y = z (or any cyclic permutation),
and for x = 0 and y = z (or any cyclic permutation).
Observation. Similarly, we can prove the following generalization:
• Let x, y, z be nonnegative real numbers. If

11
10 − 2 15 ≤ k ≤ ,
then 2

x(2x + y +z)(x − y)(x −z)(x −k y)(x −kz)+(7k−13)(x − y)2 ( y −z)2 (z− x)2 ≥ 0,

with equality for x = y = z, for x/k = y = z (or any cyclic permutation), and for
x = 0 and y = z (or any cyclic permutation).
We have

A = 3(1 + 2)(1 + k + k2 ) + (7k − 13)(−27) = 9(k2 − 20k + 40) ≤ 0,

f6 (x, 1, 1) = 2x(x + 1)(x − 1)2 (x − k)2 ≥ 0,


f6 (0, y, z) = ( y − z)4 [2 y 2 + 2z 2 + (7 − 2k) yz] ≥ 0.

P 4.4. If x, y, z are nonnegative real numbers, no two of which are zero, then
1 1 1 9
+ + ≥ 2 .
4x 2 + yz 4 y 2 + z x 4z 2 + x y (x + 4 yz) + ( y 2 + 4z x) + (z 2 + 4x y)
(Vasile C., 2008)
Highest Coefficient Cancellation Method for Nonnegative Variables 383

Solution. Write the inequality as f6 (x, y, z) ≥ 0, where


 (
f6 (x, y, z) = [x 2 + y 2 +z 2 +4(x y+ yz+z x)] (4 y 2 +z x)(4z 2 +x y)−9 (4x 2 + yz).

The highest coefficient A of f6 (x, y, z) is equal to the highest coefficient of the prod-
uct (
P3 (x, y, z) = −9 (4x 2 + yz),
that is
A = P3 (1, 1, 1) = −1125.
By Theorem 1, we only need to prove the original inequality for y = z = 1, and for
x = 0. Thus, we need to show that
1 2 9
+ ≥ 2
4x 2 + 1 x + 4 x + 8x + 6
and
1 1 1 9
+ + ≥ 2 .
yz 4 y 2 4z 2 y + z 2 + 4 yz
These inequalities are respectively equivalent to

x(x − 1)2 (8x + 45) ≥ 0

and
( y − z)2 ( y 2 + z 2 + 10 yz) ≥ 0.
The equality holds for x = y = z, and for x = 0 and y = z (or any cyclic
permutation).
Observation. Similarly, we can prove the following generalization:
• Let x, y, z be nonnegative real numbers, no two of which are zero. If

1 ≤ k ≤ 3 + 5,

then
 1 9(k + 2)
≥ ,
k x 2 + yz (−k2 + 6k − 2)(x 2 + y 2 + z 2 ) + (2k2 − 3k + 4)(x y + yz + z x)
with equality for x = y = z, and for x = 0 and y = z (or any cyclic permutation).

P 4.5. If x, y, z are nonnegative real numbers, then

2x 2 − yz 2 y2 − z x 2z 2 − x y 1
+ + ≤ .
4x 2 + y 2 + z 2 4 y 2 + z 2 + x 2 4z 2 + x 2 + y 2 2
(Vasile C., 2008)
384 Vasile Cîrtoaje

Solution. Write the inequality as f6 (x, y, z) ≥ 0, where


( 
f6 (x, y, z) = (4x 2 + y 2 + z 2 ) − 2 (2x 2 − yz)(4 y 2 + z 2 + x 2 )(4z 2 + x 2 + y 2 ).

Since
4x 2 + y 2 + z 2 = 3x 2 + p2 − 2q,
the highest coefficient A of f6 (x, y, z) is equal to the highest coefficient of
(
(3x 2 ) − 2P2 (x, y, z),

where 
P2 (x, y, z = (2x 2 − yz)(3 y 2 )(3z 2 ),
that is
A = 27 − 2P2 (1, 1, 1) = 27 − 54 = −27.
By Theorem 1, we only need to prove the original inequality for y = z = 1, and for
x = 0. Thus, we need to show that

2x 2 − 1 2(2 − x) 1
+ 2 ≤
2(2x 2 + 1) x +5 2

and
− yz 2 y2 2z 2 1
+ + ≤ .
y 2 + z 2 4 y 2 + z 2 4z 2 + y 2 2
These inequalities are respectively equivalent to

(x − 1)2 (4x + 1) ≥ 0

and
− yz ( y 2 + z 2 )2 + 6 y 2 z 2 1
+2 ≤ .
y +z
2 2 4( y 2 + z 2 )2 + 9 y 2 z 2 2
For yz = 0, the last inequality is an equality. For yz = 0, using the substitution

y 2 + z2
t= , t ≥ 2,
yz

the inequality becomes


−1 2(t 2 + 6) 1
+ ≤ ,
t 4t 2 + 9 2
8t 2 − 15t + 18 ≥ 0,
t(8t − 15) + 18 > 0.

The equality holds for x = y = z, and for y = z = 0 (or any cyclic permutation).

Observation. Similarly, we can prove the following generalization:


Highest Coefficient Cancellation Method for Nonnegative Variables 385

• Let x, y, z be nonnegative real numbers. If k ≥ 2, then


 3k x 2 − 2(k − 1) yz
≤ 3,
k x 2 + y 2 + z2

with equality for x = y = z, and for y = z = 0 (or any cyclic permutation). If k = 2,


then the equality holds also for x = 0 and y = z (or any cyclic permutation).

P 4.6. If x, y, z are real numbers, then

21x 2 + 4 yz 21 y 2 + 4z x 21z 2 + 4x y
+ + ≥ 15.
x 2 + 2 y 2 + 2z 2 y 2 + 2z 2 + 2x 2 z 2 + 2x 2 + 2 y 2

(Vasile C., 2012)

Solution. Write the inequality as f6 (x, y, z) ≥ 0, where



f6 (x, y, z) = (21x 2 + 4 yz)( y 2 + 2z 2 + 2x 2 )(z 2 + 2x 2 + 2 y 2 )
− 15(x 2 + 2 y 2 + 2z 2 )( y 2 + 2z 2 + 2x 2 )(z 2 + 2x 2 + 2 y 2 ).

Since
x 2 + 2 y 2 + 2z 2 = −x 2 + 2(p2 − 2q),
f6 (x, y, z) has the same highest coefficient A as
(
P2 (x, y, z) − 15 (−x 2 ),

where 
P2 (x, y, z) = (21x 2 + 4 yz)(− y 2 )(−z 2 ),
that is
A = P2 (1, 1, 1) + 15 = 75 + 15 = 90.
Since

f6 (x, 1, 1) =(21x 2 + 4)(2x 2 + 3)2 + 2(4x + 21)(2x 2 + 3)(x 2 + 4)


− 15(2x 2 + 3)2 (x 2 + 4)
=4x(2x 2 + 3)(x − 1)2 (3x + 8),

we apply Theorem 3. The condition (a) in Theorem 3 is clearly satisfied.


4Ax(x − 1)3
The condition (b) in Theorem 3 is satisfied if f6 (x, 1, 1) ≥ for x ≥ 1.
27
We have
4Ax(x − 1)3 40x(x − 1)3
= ,
27 3
386 Vasile Cîrtoaje

4Ax(x − 1)3 4x(x − 1)2 f (x)


f6 (x, 1, 1) − = ,
27 3
where

f (x) = 3(2x 2 + 3)(3x + 8) − 10(x − 1) ≥ 15(3x + 8) − 10(x − 1) = 5(7x + 26) > 0.

The condition (c) in Theorem 3 is satisfied if the original inequality holds for
x = 0. Thus, we need to show that

2 yz ( y 2 + z 2 )2 − y 2 z 2
+ 42 ≥ 15.
y 2 + z2 2( y 2 + z 2 )2 + y 2 z 2

Using the substitution


y 2 + z2
t= , t ≥ 2,
yz
we may write the inequality as follows:

2 42(t 2 − 1)
+ ≥ 15,
t 2t 2 + 1

12t 3 + 4t 2 − 57t + 2 ≥ 0,

(t − 2)(12t 2 + 28t − 1) ≥ 0.

The equality holds for x = y = z, and for x = 0 and y = z (or any cyclic
permutation).

Observation. Similarly, we can prove the following generalization:


• Let x, y, z be nonnegative real numbers. If 0 ≤ k ≤ 2, k = 1, then

 (1+k)(4−k) 2
x + 2 yz 3(3 − k)
1−k
≥ ,
k x 2 + y 2 + z2 1−k

with equality for x = y = z, and for x = 0 and y = z (or any cyclic permutation). If
k = 2, then the equality holds also for y = z = 0 (or any cyclic permutation).

P 4.7. If x, y, z are real numbers, then

x y − yz + z x yz − z x + x y z x − x y + yz 3
+ + ≤ .
x 2 + 3 y 2 + 3z 2 y 2 + 3z 2 + 3x 2 z 2 + 3x 2 + 3 y 2 7

(Vasile C., 2012)


Highest Coefficient Cancellation Method for Nonnegative Variables 387

Solution. Write the inequality as f6 (x, y, z) ≥ 0, where

f6 (x, y, z) =3(x 2 + 3 y 2 + 3z 2 )( y 2 + 3z 2 + 3x 2 )(z 2 + 3x 2 + 3 y 2 )



−7 (x y − yz + z x)( y 2 + 3z 2 + 3x 2 )(z 2 + 3x 2 + 3 y 2 ).

Since

x 2 + 3 y 2 + 3z 2 = −2x 2 + 3(p2 − 2q), x y − yz + z x = −2 yz + q

f6 (x, y, z) has the same highest coefficient A as



3(−2x 2 )(−2 y 2 )(−2z 2 ) − 7 (−2 yz)(−2 y 2 )(−2z 2 ),

that is
A = −24 + 168 = 144.
Since

f6 (x, 1, 1) =3(x 2 + 6)(3x 2 + 4)2 − 7(2x − 1)(3x 2 + 4)2 − 14(x 2 + 6)(3x 2 + 4)


= (3x 2 + 4)(9x 4 − 42x 3 + 73x 2 − 56x + 16)
= (3x 2 + 4)(x − 1)2 (3x − 4)2 ,

we apply Corollary 1 for


(x − 1)4 (3x − 4)2
Eα,β (x) = f4/3,−2 (x) = .
81 · 25

The condition (a) of Corollary 1 is satisfied if f6 (x, 1, 1) ≥ Af4/3,−2 (x) for x ∈


[0, 4]. We have
16(x − 1)4 (3x − 4)2
Af4/3,−2 (x) = ,
225
(x − 1)2 (3x − 4)2 f (x)
f6 (x, 1, 1) − Af4/3,−2 (x) = ,
225
where

f (x) = 225(3x 2 +4)−16(x −1)2 > 16(3x 2 +4)−16(x −1)2 = 16(2x 2 +2x +3) > 0.

The condition (b) of Corollary 1 is satisfied if f6 (x, 1, 1) ≥ Ax 2 for x ≥ 4. We


have

f6 (x, 1, 1) − Ax 2 = (3x 2 + 4)(x − 1)2 (3x − 4)2 − 144x 2


> 3x 2 [(x − 1)2 (3x − 4)2 − 48] > 3x 2 (9 · 64 − 48) > 0.

The condition (c) of Corollary 1 is satisfied if the original inequality holds for
x = 0. Thus, we need to show that
− yz yz yz 3
+ + ≤ ,
3( y 2 + z 2 ) y 2 + 3z 2 z 2 + 3 y 2 7
388 Vasile Cîrtoaje

which can be rewritten as

− yz 4 yz( y 2 + z 2 ) 3
+ ≤ .
3( y 2 + z 2 ) 3( y 2 + z 2 )2 + 4 y 2 z 2 7

For the nontrivial case yz = 0, using the substitution

y 2 + z2
t= , t ≥ 2,
yz

we may write the inequality as follows:

−1 4t 3
+ 2 ≤ ,
3t 3t + 4 7

9t 2 − 4 3
≤ .
3t(3t 2 + 4 7
It suffices to show that
9t 2 3
≤ ,
3t(3t 2 + 4) 8
which reduces to
(t − 2)(3t − 2) ≥ 0.
3x
The equality holds for x = y = z, and for = y = z (or any cyclic permuta-
4
tion).

Observation. Similarly, we can prove the following generalization:


• Let x, y, z be nonnegative real numbers, and let

3k(k − 1) 3k2 − 6k + 1
α= , β= ,
3k2 − 1 3k2 − 1
1
where k > 0, k =
. Then,
3
 αx( y + z) − β yz 3(2α − β)
≤ ,
3k2 x 2 + y 2 + z 2 3k2 + 2

with equality for x = y = z, and for x/γ = y = z (or any cyclic permutation), where

2
γ=k+ − 1.
3k
Highest Coefficient Cancellation Method for Nonnegative Variables 389

P 4.8. If x, y, z are nonnegative real numbers, then



x 3 (2x + y + z)(x − y)(x − z) ≥ 13(x − y)2 ( y − z)2 (z − x)2 .

(Vasile C., 2011)

Solution. Write the inequality as f6 (x, y, z) ≥ 0, where

f6 (x, y, z) = f (x) − 13(x − y)2 ( y − z)2 (z − x)2 ,



f (x) = x 3 (2x + y + z)(x − y)(x − z).
Since
2x + y + z) = x + p, (x − y)(x − z) = x 2 + 2 yz − q,
f (x, y, z) has the same highest coefficient A1 as

P1 (x, y, z) = x 4 (x 2 + 2 yz),

that is
A1 = P1 (1, 1, 1) = 3(1 + 2) = 9.
Therefore, f6 (x, y, z) has the highest coefficient

A = 9 − 13(−27) = 360.

Since
f6 (x, 1, 1) = 2x 3 (x + 1)(x − 1)2 ,
we apply Corollary 1 for

x 4 (x − 1)4
Eα,β (x) = f0,0 (x) = .
36(x + 2)2

The condition (a) of Corollary 1 is satisfied if f6 (x, 1, 1) ≥ Af0,0 (x) for x ∈ [0, 4].
We have

10x 4 (x − 1)4
f6 (x, 1, 1) − Af0,0 (x) = 2x 3 (x + 1)(x − 1)2 −
(x + 2)2
2x (x − 1) (4 − x)(4x + x + 1)
3 2 2
= ≥ 0.
(x + 2)2

The condition (b) of Corollary 1 is satisfied if f6 (x, 1, 1) ≥ Ax 2 for x ≥ 4. We


have

f6 (x, 1, 1) − Ax 2 = 2x 2 [x(x + 1)(x − 1)2 − 180]


≥ 2x 2 [4 · 5 · 9 − 180] = 0.
390 Vasile Cîrtoaje

The condition (c) of Corollary 1 is satisfied if f6 (0, y, z) ≥ 0 for y, z ≥ 0. We have

f6 (0, y, z) = ( y − z)[2( y 5 − z 5 ) + yz( y 3 − z 3 )] − 13 y 2 z 2 ( y − z)2


= ( y − z)2 [2( y 2 + z 2 )2 + 3 yz( y 2 + z 2 ) − 14 y 2 z 2 ]
= ( y − z)4 (2 y 2 + 2z 2 + 7 yz) ≥ 0.

The equality holds for x = y = z, and for x = 0 and y = z (or any cyclic
permutation).

11
P 4.9. Let x, y, z be nonnegative real numbers. If k ≤ , then
2

x(2x + y +z)(x − y)(x −z)(x −k y)(x −kz)+(7k−13)(x − y)2 ( y −z)2 (z− x)2 ≥ 0.

(Vasile C., 2011)

Solution. Write the inequality as f6 (x, y, z) ≥ 0, where

f6 (x, y, z) = f (x) + (7k − 13)(x − y)2 ( y − z)2 (z − x)2 ,



f (x) = x(2x + y + z)(x − y)(x − z)(x − k y)(x − kz).
Since 2x + y + z = x + p,

(x − y)(x − z) = x 2 + 2 yz − q, (x − k y)(x − kz) = x 2 + (k + k2 ) yz − kq,

f (x, y, z) has the same highest coefficient A1 as



P1 (x, y, z) = x 2 (x 2 + 2 yz)[x 2 + (k + k2 ) yz],

that is
A1 = P1 (1, 1, 1) = 9(k2 + k + 1).
Therefore, f6 (x, y, z) has the highest coefficient

A = 9(k2 + k + 1) + (7k − 13)(−27) = 9(k2 − 20k + 40).

We have
f6 (x, 1, 1) = 2x(x + 1)(x − 1)2 (x − k)2 ≥ 0,

f6 (0, y, z) =( y − z)[2( y 5 − z 5 ) − (2k − 1) yz( y 3 − z 3 ) − k y 2 z 2 ( y − z)]


+ (7k − 13) y 2 z 2 ( y − z)2
=( y − z)4 [2 y 2 + 2 y 2 + (7 − 2k) yz]
=( y − z)4 [2( y − z)2 + (11 − 2k) yz] ≥ 0
Highest Coefficient Cancellation Method for Nonnegative Variables 391

Case 1: k ∈ [10 − 2 15, 11/2]. Since A ≤ 0, we only need to show that


f6 (x, 1, 1) ≥ 0 and f6 (0, y, z) ≥ 0 for x, y, z ≥ 0 (Theorem 1). Both conditions are
satisfied.

Case 2: k < 10 − 2 15. We apply Corollary 1 for

4x 2 (x − 1)4 (x − k)2
Eα,β (x) = f k,0 (x) = .
9(4 − k)2 (x + 2)2

The condition (a) in Corollary 1 is satisfied if f6 (x, 1, 1) ≥ Af k,0 (x) for x ∈ [0, 4].
We have
4(k2 − 20k + 40)x 2 (x − 1)4 (x − k)2
Af k,0 (x) = ,
(4 − k)2 (x + 2)2
2x(x − 1)2 (x − k)2 g(x)
f6 (x, 1, 1) − Af k,0 (x) = ,
(4 − k)2 (x + 2)2
where
g(x) = (4 − k)2 (x + 1)(x + 2)2 − 2(k2 − 20k + 40)x(x − 1)2 .
Since
(x + 1)(x + 2)2 − 5x(x − 1)2 = (4 − x)(1 + x + 4x 2 ) ≥ 0,
we get

g(x) ≥ 5(4 − k)2 x(x − 1)2 − 2(k2 − 20k + 40)x(x − 1)2 = 3k2 x(x − 1)2 ≥ 0.

The condition (b) in Corollary 1 is satisfied if f6 (x, 1, 1) ≥ Ax 2 for x ≥ 4. We


have
f6 (x, 1, 1) − Ax 2 = x g(x),
where
g(x) = 2(x + 1)(x − 1)2 (x − k)2 − 9(k2 − 20k + 40)x.
Since
4(x − 1)2 − 9x = (x − 4)(4x − 1) ≥ 0,
we get

2g(x) ≥ 9x(x + 1)(x − k)2 − 18(k2 − 20k + 40)x


= 9x[(x + 1)(x − k)2 − 2(k2 − 20k + 40)]
≥ 9x[5(4 − k)2 − 2(k2 − 20k + 40)] = 18k2 x ≥ 0.

The condition (c) in Corollary 1 is satisfied because f6 (0, y, z) ≥ 0 for y, z ≥ 0.


The equality holds for x = y = z, and for x/k = y = z (or any cyclic permuta-
tion) if k = 0. If k = 0, then the equality holds also for x = 0 and y = z.
Observation 1. Having in view the inequality in Observation from P 4.2 and the
inequality in P 4.9, we can formulate the following statement:
392 Vasile Cîrtoaje

• Let x, y, z be nonnegative real numbers. If



⎪ 11
⎨ 7k − 13, k≤
2
αk = ,

⎩ 4k 2
+ 12k + 17 11
, k≥
8 2
then

x(2x + y + z)(x − y)(x − z)(x − k y)(x − kz) + αk (x − y)2 ( y − z)2 (z − x)2 ≥ 0.

Observation 2. The coefficient αk in the inequality from Observation 1 is the best


possible.
Setting x = 0, the inequality becomes

( y − z)[2( y 5 − z 5 ) − (2k − 1) yz( y 3 − z 3 ) − k y 2 z 2 ( y − z)] + αk y 2 z 2 ( y − z)2 ,

( y − z)2 f ( y, z) ≥ 0,
where

f ( y, z) = 2( y 2 + z 2 )2 − (2k − 3) yz( y 2 + z 2 ) + (αk − 3k − 1) y 2 z 2 .

From the necessary condition f (1, 1) ≥ 0, we get αk ≥ 7k − 13.


Assume now that k ≥ 11/2. Since (2k − 3)/4 ≥ 2, there exist y > 0 and z > 0
such that
2k − 3
y 2 + z2 = yz.
4
For this case, we have

f ( y, z) 1 1
= (2k − 3)2 − (2k − 3)2 + (αk − 3k − 1)
y 2z2 8 4
4k2 + 12k + 17
= αk − .
8
4k2 + 12k + 17
From the necessary condition f ( y, z) ≥ 0, we get αk ≥ .
8

P 4.10. If x, y, z are nonnegative real numbers, then


 12(x − y)2 ( y − z)2 (z − x)2
x 2 (x − y)(x − z) ≥ .
(x + y + z)2

(Vasile C., 2010)


Highest Coefficient Cancellation Method for Nonnegative Variables 393

Solution. Write the inequality as f6 (x, y, z) ≥ 0, where



f6 (x, y, z) = (x + y + z)2 x 2 (x − y)(x − z) − 12(x − y)2 ( y − z)2 (z − x)2

has the highest coefficient

A = −12(−27) = 324.

Since
f6 (x, 1, 1) = x 2 (x + 2)2 (x − 1)2 ,

we apply Corollary 1 for

x 2 (x − 1)4
Eα,β (x) = f0,−2 (x) = .
81

Since
Af0,−2 (x) = 4x 2 (x − 1)4 ,

f6 (x, 1, 1) − Af0,−2 (x) = 3x 3 (x − 1)2 (4 − x) ≥ 0,

the condition (a) from Corollary 1 is satisfied.


The condition (b) from Corollary 1 is satisfied if f6 (x, 1, 1) ≥ Ax 2 for x ≥ 4. This
is true since
 
f6 (x, 1, 1) − Ax 2 = x 2 (x + 2)2 (x − 1)2 − 324 ≥ x 2 (36 · 9 − 324) = 0.

The condition (c) from Corollary 1 is also satisfied because

f6 (0, y, z) = ( y + z)2 ( y − z)2 ( y 2 + z 2 + yz) − 12 y 2 z 2 ( y − z)2


= ( y − z)4 ( y 2 + z 2 + 5 yz) ≥ 0.

The equality occurs for x = y = z, and for x = 0 and y = z (or any cyclic
permutation).

P 4.11. Let x, y, z be nonnegative real numbers. If k is a real number, then



x( y + z)(x − y)(x − z)(x − k y)(x − kz) + (k − 3)(x − y)2 ( y − z)2 (z − x)2 ≥ 0.

(Vasile C., 2011)


394 Vasile Cîrtoaje

Solution. Write the inequality as f6 (x, y, z) ≥ 0, where

f6 (x, y, z) = f (x) + (k − 3)(x − y)2 ( y − z)2 (z − x)2 ,



f (x) = x( y + z)(x − y)(x − z)(x − k y)(x − kz).
Since y + z = −x + p,

(x − y)(x − z) = x 2 + 2 yz − q, (x − k y)(x − kz) = x 2 + (k + k2 ) yz − kq,

f (x, y, z) has the same highest coefficient A1 as



P1 (x, y, z) = − x 2 (x 2 + 2 yz)[x 2 + (k + k2 ) yz],

that is
A1 = P1 (1, 1, 1) = −9(k2 + k + 1).
Therefore, f6 (x, y, z) has the highest coefficient

A = −9(k2 + k + 1) + (k − 3)(−27) = 9(8 − 4k − k2 ).

We have
f6 (x, 1, 1) = 2x(x − 1)2 (x − k)2 ≥ 0,

f6 (0, y, z) = yz( y − z)[ y 3 − z 3 − k yz( y − z)] + (k − 3) y 2 z 2 ( y − z)2 ]


= yz( y − z)4 ≥ 0

Case 1: k ∈ (−∞, −2 − 2 3] ∪ [−2 + 2 3, ∞). Since A ≤ 0, we only need


to show that f6 (x, 1, 1) ≥ 0 and f6 (0, y, z) ≥ 0 for x, y, z ≥ 0 (Theorem 1). Both
conditions are satisfied.

Case 2: k ∈ (−2 − 2 3, −2 + 2 3). We apply Corollary 1 for

4x 2 (x − 1)4 (x − k)2
Eα,β (x) = f k,0 (x) = .
9(4 − k)2 (x + 2)2

The condition (a) of Corollary 1 is satisfied if f6 (x, 1, 1) ≥ Af k,0 (x) for x ∈ [0, 4].
We have
4(8 − 4k − k2 )x 2 (x − 1)4 (x − k)2
Af k,0 (x) = ,
(4 − k)2 (x + 2)2
2x(x − 1)2 (x − k)2 g(x)
f6 (x, 1, 1) − Af k,0 (x) = ,
(4 − k)2 (x + 2)2
where
g(x) = (4 − k)2 (x + 2)2 − 2(8 − 4k − k2 )x(x − 1)2 .
Since
(x + 2)2 − x(x − 1)2 = (4 − x)(1 + x + x 2 ) ≥ 0,
Highest Coefficient Cancellation Method for Nonnegative Variables 395

we get

g(x) ≥ (4 − k)2 x(x − 1)2 − 2(8 − 4k − k2 )x(x − 1)2 = 3k2 x(x − 1)2 ≥ 0.

The condition (b) of Corollary 1 is satisfied if f6 (x, 1, 1) ≥ Ax 2 for x ≥ 4. We


have
f6 (x, 1, 1) − Ax 2 ) = x g(x),
where
g(x) = 2(x − 1)2 (x − k)2 − 9(8 − 4k − k2 )x.
Since
4(x − 1)2 − 9x = (x − 4)(4x − 1) ≥ 0,
we get

2g(x) ≥ 9x(x − k)2 − 18(8 − 4k − k2 )x


= 9x[(x − k)2 − 2(8 − 4k − k2 )]
≥ 9x[(4 − k)2 − 2(8 − 4k − k2 )] = 27k2 x ≥ 0.

The condition (c) of Corollary 1 is satisfied since f6 (0, y, z) ≥ 0.


The equality holds for x = y = z, and for x/k = y = z (or any cyclic permuta-
tion) if k = 0, for x = 0 and y = z (or any cyclic permutation, and for y = z (or
any cyclic permutation.
Observation. The coefficient of the product (x − y)2 ( y − z)2 (z − x)2 is the best
possible.
Setting x = 0, the inequality

x( y + z)(x − y)(x − z)(x − k y)(x − kz) + αk (x − y)2 ( y − z)2 (z − x)2 ≥ 0

becomes
yz( y − z)[ y 3 − z 3 − k yz( y − z)] + αk y 2 z 2 ( y − z)2 ] ≥ 0,
yz( y − z)2 [ y 2 + z 2 + (1 − k + αk ) yz] ≥ 0.
The necessary condition

y 2 + z 2 + (1 − k + αk ) yz ≥ 0

leads to αk ≥ k − 3 for y = z = 1.

P 4.12. Let x, y, z be nonnegative real numbers. If k ≤ 4, then



x 2 (x − y)(x − z)(x − k y)(x − kz) + (3k − 5)(x − y)2 ( y − z)2 (z − x)2 ≥ 0.

(Vasile C., 2011)


396 Vasile Cîrtoaje

Solution. Denote

f (x, y, z) = x 2 (x − y)(x − z)(x − k y)(x − kz),

and write the desired inequality as f6 (x, y, z) ≥ 0, where

f6 (x, y, z) = f (x, y, z) + (3k − 5)(x − y)2 ( y − z)2 (z − x)2 .

From

(x − y)(x − z) = x 2 + 2 yz − q, (x − k y)(x − kz) = x 2 + (k + k2 ) yz − kq,

it follows that f (x, y, z) has the same highest coefficient A1 as



P1 (x, y, z) = x 2 (x 2 + 2 yz)[x 2 + (k + k2 ) yz],

that is,
A1 = P1 (1, 1, 1) = 9(k2 + k + 1).
Since the highest coefficient of the product (x − y)2 ( y − z)2 (z − x)2 is equal to −27,
f6 (x, y, z) has the highest coefficient

A = A1 + (3k − 5)(−27) = 9(4 − k)2 .

On the other hand, we have

f6 (x, 1, 1) = x 2 (x − 1)2 (x − k)2 ,

f6 (0, y, z) = ( y − z)[ y 5 − z 5 − k yz( y 3 − z 3 )] + (3k − 5) y 2 z 2 ( y − z)2


= ( y − z)2 [( y 2 + z 2 )2 − (k − 1) yz( y 2 + z 2 ) + 2(k − 3) y 2 z 2 ]
= ( y − z)4 [ y 2 + z 2 − (k − 3) yz]
= ( y − z)2 [( y − z)2 + (5 − k) yz] ≥ 0.

For k = 4, we have A = 0. According to Theorem 1, the desired inequality is true


since f6 (x, 1, 1) ≥ 0 and f6 (0, y, z) ≥ 0 for all x, y, z ≥ 0.
For k < 4, we have A > 0. To prove the desired inequality, we will apply Corollary
1 for
4x 2 (x − 1)4 (x − k)2
Eα,β (x) = f k,0 (x) = .
9(4 − k)2 (x + 2)2
The condition (a) in Corollary 1 is satisfied if f6 (x, 1, 1) ≥ Af k,0 (x) for x ∈ [0, 4].
We have
4x 2 (x − 1)4 (x − k)2
Af k,0 (x) = ,
(x + 2)2
3x 3 (x − 1)2 (x − k)2 (4 − x)
f6 (x, 1, 1) − Af k,0 (x) = ≥ 0.
(x + 2)2
Highest Coefficient Cancellation Method for Nonnegative Variables 397

The condition (b) in Corollary 1 is satisfied if f6 (x, 1, 1) ≥ Ax 2 for x ≥ 4. We


have
f6 (x, 1, 1) − Ax 2 = x 2 g(x),
where
g(x) = (x − 1)2 (x − k)2 − 9(4 − k)2 .
Since
(x − 1)2 ≥ 9,
we get
g(x) ≥ 9[(x − k)2 − (4 − k)2 ] ≥ 9[(4 − k)2 − (4 − k)2 ] = 0.

The condition (c) in Corollary 1 is satisfied because f6 (0, y, z) ≥ 0 for y, z ≥ 0.


The equality occurs for x = y = z, for x = 0 and y = z (or any cyclic permutation),
and for x/k = y = z (or any cyclic permutation) if k = 0.
Observation 1. The coefficient of the product (x − y)2 ( y − z)2 (z − x)2 is the best
possible.
Setting x = 0, the inequality

x 2 (x − y)(x − z)(x − k y)(x − kz) + αk (x − y)2 ( y − z)2 (z − x)2 ≥ 0

becomes as follows:

( y − z)[ y 5 − z 5 − k yz( y 3 − z 3 )] + αk y 2 z 2 ( y − z)2 ≥ 0,

( y − z)2 f ( y, z) ≥ 0,
where

f ( y, z) = y 4 + z 4 − (k − 1) yz( y 2 + z 2 ) + (αk − k + 1) y 2 z 2 ≥ 0.

For y = z = 1, the necessary condition f (1, 1) ≥ 0 involves αk ≥ 3k − 5.


Observation 2. For k = 4, the inequality turns into

x 2 (x − y)(x − z)(x − 4 y)(x − 4z) + 7(x − y)2 ( y − z)2 (z − x)2 ≥ 0,

which is equivalent to

(x 2 + y 2 + z 2 − x y − yz − z x)(x 2 + y 2 + z 2 − 2x y − 2 yz − 2z x)2 ≥ 0.


The equality occurs when x = y = z, and when x = y + z (or any cyclic


permutation).
Observation 3. The inequality can be extended for k ≥ 4, as follows:
 20 + 12k − 4k2 − k4
x 2 (x − y)(x −z)(x − k y)(x − kz) ≥ (x − y)2 ( y −z)2 (z − x)2 .
4(k − 1)2
398 Vasile Cîrtoaje

Actually, this inequality is valid for all real x, y, z (see P 3.49 from chapter 3).
Observation 4. Substituting k − 1 for k in P 4.12, and using then the identity

x 2 (x − y)(x − z)[x − (k − 1) y](x − (k − 1)z] =

= x 2 (x − y)(x − z)(x − k y + z)(x − kz + y) + k(x − y)2 ( y − z)2 (z − x)2 ,

we get the following equivalent statement:


• Let x, y, z be nonnegative real numbers. If k ≤ 5, then

x 2 (x − y)(x − z)(x − k y + z)(x − kz + y) ≥ 4(2 − k)(x − y)2 ( y − z)2 (z − x)2 ,

with equality for x = y = z, for x = 0 and y = z (or any cyclic permutation), and for
x/(k − 1) = y = z (or any cyclic permutation) if k = 1.

P 4.13. Let x, y, z be nonnegative real numbers. If k is a real numbers, then



yz(x − y)(x − z)(x − k y)(x − kz) ≥ 0.

(Vasile C., 2010)

Solution. If one of x, y, z is zero, the inequality is trivial. On the other hand, the
inequality remains unchanged by replacing x, y, z and k with 1/x, 1/ y, 1/z and
1/k, respectively. Therefore, it suffices to consider that

k ∈ (−∞, −1] ∪ [0, 1].

Write the inequality as f6 (x, y, z) ≥ 0, where



f6 (x, y, z) = yz(x − y)(x − z)(x − k y)(x − kz).

Since

(x − y)(x − z) = x 2 + 2 yz − q, (x − k y)(x − kz) = x 2 + (k2 + k) yz − kq,

f6 (x, y, z) has the same highest coefficient as



P1 (x, y, z) = yz(x 2 + 2 yz)[x 2 + (k2 + k) yz],

that is,
A = P1 (1, 1, 1) = 9(k2 + k + 1).
On the other hand,
f6 (x, 1, 1) = (x − 1)2 (x − k)2 ,
Highest Coefficient Cancellation Method for Nonnegative Variables 399

f6 (0, y, z) = k2 y 3 z 3 .

Thus, we apply Theorem 2 for

4(x − 1)4 (x − k)2


Eα,β (x) = f k,∞ (x) = .
9(2k + 1)2 (x + 2)2

Since

(x − 1)2 (x − k)2 [(2k + 1)2 (x + 2)2 − 4(k2 + k + 1)(x − 1)2 ]


f6 (x, 1, 1)−Af k,∞ (x) = ,
(2k + 1)2 (x + 2)2

the condition (a) of Theorem 2 is satisfied if

(2k + 1)2 (x + 2)2 ≥ 4(k2 + k + 1)(x − 1)2

for 0 ≤ x ≤ 4. This is true because

(2k + 1)2 ≥ k2 + k + 1, (x + 2)2 ≥ 4(x − 1)2 .

In order to prove the conditions (b) and (c) of Theorem 2, consider two cases:
0 ≤ k ≤ 1 and k ≤ −1.

Case 1: 0 ≤ k ≤ 1. Apply Theorem 2 for


 2
(2x + 1)2
Fγ,δ (x) = g0,δ (x) = x + δ .
x +2

We have
δ2 y 4 z 4
ĝ0,δ ( y, z) = ,
( y + z)2

9(k2 + k + 1)δ2 y 4 z 4 9(k2 + k + 1)δ2 y 3 z 3


Aĝ0,δ ( y, z) = ≤ ,
( y + z)2 4

9(k2 + k + 1)δ2
f6 (0, y, z) − Aĝ0,δ ( y, z) ≥ f (k) y 3 z 3 , f (k) = k2 − .
4
Choosing
−2k
δ= ,
3(k + 1)
we have
k2 (k2 + k + 1)δ2 k3
f (k) = k2 − = ≥ 0,
(k + 1) 2 (k + 1)2
therefore the condition (c) of Theorem 2 is satisfied.
400 Vasile Cîrtoaje

The condition (b) of Theorem 2 is satisfied if f6 (x, 1, 1) ≥ Ag0,δ (x) for x ≥ 4. We


have
 2
(2x + 1)2
Ag0,δ (x) ≤ 9(k + 1)2 x + δ
x +2
 2
2k(2x + 1)2
= 9(k + 1) x −
2
3(k + 1)(x + 2)

2k(2x + 1)2
= 3(k + 1)x +
x +2
[(3 − 5k)x 2 + (6 − 2k)x − 2k]2
=
(x + 2)2
[(3 − 5k)x + (6 − 2k)x − 2k]2
2
≤ ,
36
g1 (x)g2 (x)
f6 (x, 1, 1) − Ag0,δ (x) ≥ ,
36
where

g1 (x) = 6(x − 1)(x − k) − (3 − 5k)x 2 − (6 − 2k)x + 2k


= (3 + 5k)x 2 − (12 + 4k)x + 8k
≥ 4(3 + 5k)x − (12 + 4k)x + 8k
= 11k x + 8k ≥ 0,

g2 (x) = 6(x − 1)(x − k) + (3 − 5k)x 2 + (6 − 2k)x − 2k


= (9 − 5k)x 2 − 8k x + 4k
≥ 4(9 − 5k)x − 8k x + 4k
= (36 − 28k)x + 4k > 0.

Case 2: k ≤ −1. We apply Theorem 2 for

(x − 1)4
Fγ,δ (x) = f∞,∞ (x) = .
9(x + 2)2
The condition (b) of Theorem 2 is satisfied if f6 (x, 1, 1) ≥ Af∞,∞ (x) for x ≥ 4.
We have
(k2 + k + 1)(x − 1)4 k2 (x − 1)4
Af∞,∞ (x) = ≤ ,
(x + 2) 2 (x + 2)2
therefore
(x − 1)2 [(x + 2)2 (x − k)2 − k2 (x − 1)2 ]
f6 (x, 1, 1) − Af∞,∞ (x) ≥ .
(x + 2)2
It suffices to show that

(x + 2)(x − k) ≥ (−k)(x − 1).


Highest Coefficient Cancellation Method for Nonnegative Variables 401

This is true because

x + 2 > x − 1 > 0, x − k > −k > 0.

The condition (c) of Theorem 2 is satisfied if f6 (0, y, z) ≥ A fˆ∞,∞ ( y, z) for y, z ≥


0. Since
y 4z4 y 3z3
fˆ∞,∞ ( y, z) = ≤ ,
9( y + z)2 36
we have
 
9(k2 + k + 1
f6 (0, y, z) − A fˆ∞,∞ ( y, z) ≥ k2 − y 3 z 3 ≥ 0.
36

The equality occurs for x = y = z, for y = z = 0 (or any cyclic permutation), and
for x/k = y = z (or any cyclic permutation) if k = 0. If k = 0, then the equality
holds also for x = 0 and y = z.
Observation 1. The coefficient (zero) of the product (x − y)2 ( y − z)2 (z − x)2 is the
best possible.
Setting x = 0, the inequality

yz(x − y)(x − z)(x − k y)(x − kz) + αk (x − y)2 ( y − z)2 (z − x)2 ≥ 0

becomes as follows:
k2 y 3 z 3 + αk y 2 z 2 ( y − z)2 ≥ 0.
Setting y = 1 and z = 0 in the necessary condition

k2 yz + αk ( y − z)2 ≥ 0,

we get αk ≥ 0.
Observation 2. Substituting k − 1 for k in P 4.13, and using then the identity

yz(x − y)(x − z)[x − (k − 1) y](x − (k − 1)z] =

= yz(x − y)(x − z)(x − k y + z)(x − kz + y) + k(x − y)2 ( y − z)2 (z − x)2 ,

we get the following equivalent statement:


• Let x, y, z be nonnegative real numbers. If k is a real number, then

yz(x − y)(x − z)(x − k y + z)(x − kz + y) + k(x − y)2 ( y − z)2 (z − x)2 ≥ 0,

with equality for x = y = z, for y = z = 0 (or any cyclic permutation), and for
x/(k − 1) = y = z (or any cyclic permutation) if k = 1.
402 Vasile Cîrtoaje

P 4.14. If x, y, z are nonnegative real numbers, then


 3(x − y)2 ( y − z)2 (z − x)2
x 2 (x − y)(x − z) ≥ .
x y + yz + z x

Solution. Write the inequality as f6 (x, y, z) ≥ 0, where



f6 (x, y, z) = (x y + yz + z x) x 2 (x − y)(x − z) − 3(x − y)2 ( y − z)2 (z − x)2 .

We have
A = −3(−27) = 81.
Since
f6 (x, 1, 1) = (2x + 1)x 2 (x − 1)2 ,
we apply Corollary 1 for

x 2 (x − 1)4
Eα,β (x) = f0,−2 (x) = .
81

Condition (a). We need to show that f6 (x, 1, 1) ≥ Af0,−2 (x) for x ∈ [0, 4]. We
have
Af0,−2 (x) = x 2 (x − 1)4 ,

f6 (x, 1, 1) − Af0,−2 (x) = (2x + 1)x 2 (x − 1)2 − x 2 (x − 1)4 = x 3 (x − 1)2 (4 − x) ≥ 0.

Condition (b). This condition is satisfied if f6 (x, 1, 1) ≥ Ax 2 for x ≥ 4. We have

Ax 2 = 81x 2 ,

f6 (x, 1, 1) − Ax 2 = x 2 [(2x + 1)(x − 1)2 − 81] ≥ x 2 (81 − 81) = 0.

Condition (c). This condition is satisfied if f6 (0, y, z) ≥ 0 for y, z ≥ 0. We have

f6 (0, y, z) = yz( y − z)( y 3 − z 3 ) − 3 y 2 z 2 ( y − z)2 = yz( y − z)4 ≥ 0.

The equality occurs for x = y = z, and for x = 0 and y = z (or any cyclic
permutation).

P 4.15. If x, y, z are nonnegative real numbers, then


 9(x − y)2 ( y − z)2 (z − x)2
(x − y)(x − z)(x + 2 y)(x + 2z) ≥ .
x y + yz + z x
Highest Coefficient Cancellation Method for Nonnegative Variables 403

Solution. Write the inequality as f6 (x, y, z) ≥ 0, where



f6 (x, y, z) = (x y+ yz+z x) (x− y)(x−z)(x+2 y)(x+2z)−9(x− y)2 ( y−z)2 (z−x)2 .

We have
A = −9(−27) = 243.
Since
f6 (x, 1, 1) = (2x + 1)(x − 1)2 (x + 2)2 ,

 
f6 (0, y, z) = yz 4 y 2 z 2 + ( y − z)( y 3 − z 3 ) + 2 yz( y − z)2 − 9 y 2 z 2 ( y − z)2
 
= yz 4 y 2 z 2 + ( y − z)4 − 8 yz( y − z)2
 2
= yz ( y − z)2 − 2 yz = yz( y 2 + z 2 − 4 yz)2 ,

y 2 z 2 ( y 2 + z 2 − 4 yz)2
fˆ1,∞ ( y, z) = ,
81( y + z)2
we apply Theorem 2 for

(x − 1)4 (x − 2)2
Eα,β (x) = f2,−2 (x) = ,
324
4(x − 1)6
Fγ,δ (x) = f1,∞ (x) =
81(x + 2)2

Condition (a). We need to show that f6 (x, 1, 1) ≥ Af2,−2 (x) for x ∈ [0, 4]. We
have
3(x − 1)4 (x − 2)2
Af2,−2 (x) = ,
4
(x − 1)2 f (x)
f6 (x, 1, 1)−Af2,−2 (x) = , f (x) = 4(2x +1)(x +2)2 −3(x −1)2 (x −2)2 .
4
Since (x + 2)2 ≥ (x − 2)2 and 2x + 1 ≥ (x − 1)2 , we have f (x) > 0.
Condition (b). This condition is satisfied if f6 (x, 1, 1) ≥ Af1,∞ (x) for x ≥ 4. We
have
12(x − 1)6
Af1,∞ (x) = ,
(x + 2)2
(x − 1)2 f (x)
f6 (x, 1, 1) − Af1,∞ (x) = ,
(x + 2)2
where
f (x) = (2x + 1)(x + 2)4 − 12(x − 1)4 .
The necessary inequality f (x) ≥ 0 can be obtained by multiplying the inequalities

(x + 2)2 > (x − 1)2 ,


404 Vasile Cîrtoaje

2x + 1 > 2(x − 1),


(x + 2)2 > 6(x − 1).

Condition (c). We need to show that f6 (0, y, z) ≥ A fˆ1,∞ ( y, z). We have

3 y 2 z 2 ( y 2 + z 2 − 4 yz)2
A fˆ1,∞ ( y, z) = ,
( y + z)2

3 yz
f6 (0, y, z) − A fˆ1,∞ ( y, z) = yz( y 2 + z 2 − 4 yz)2 1 − ≥ 0.
( y + z)2

The equality occurs for x = y = z, and for x = 0 and y/x + x/ y = 4 (or any
cyclic permutation).

P 4.16. If x, y, z are nonnegative real numbers, then


 6(x − y)2 ( y − z)2 (z − x)2
(x − y)(x − z)(x − 3 y)(x − 3z) ≥ .
x y + yz + z x

Solution. Write the inequality as f6 (x, y, z) ≥ 0, where



f6 (x, y, z) = (x y+ yz+z x) (x− y)(x−z)(x−3 y)(x−3z)−6(x− y)2 ( y−z)2 (z−x)2 .

We have
A = −6(−27) = 162.
Since
f6 (x, 1, 1) = (2x + 1)(x − 1)2 (x − 3)2 ,

 
f6 (0, y, z) = yz 9 y 2 z 2 + ( y − z)( y 3 − z 3 ) − 3 yz( y − z)2 − 6 y 2 z 2 ( y − z)2
 2 2 
= yz 9 y z + ( y − z)4 − 6 yz( y − z)2
 2
= yz ( y − z)2 − 3 yz = yz( y 2 + z 2 − 5 yz)2 ,

y 2 z 2 ( y 2 + z 2 − 5 yz)2
fˆ3/2,∞ ( y, z) = ,
144( y + z)2
we apply Theorem 2 for

4(x − 1)4 (x − 3)2


Eα,β (x) = f3,−2 (x) = ,
2025

(x − 1)4 (2x − 3)2


Fγ,δ (x) = f3/2,∞ (x) =
144(x + 2)2
Highest Coefficient Cancellation Method for Nonnegative Variables 405

Condition (a). We need to show that f6 (x, 1, 1) ≥ Af3,−2 (x) for x ∈ [0, 4]. We
have
8(x − 1)4 (x − 3)2
Af3,−2 (x) = ,
25
(x − 1)2 (x − 3)2 f (x)
f6 (x, 1, 1) − Af3,−2 (x) = ,
25
with
f (x) = 25(2x + 1) − 8(x − 1)2 > 8[2x + 1 − (x − 1)2 ] ≥ 0.
Condition (b). This condition is satisfied if f6 (x, 1, 1) ≥ Af3/2,∞ (x) for x ≥ 4. We
have
9(x − 1)4 (2x − 3)2
Af3/2,∞ (x) = ,
8(x + 2)2
(x − 1)2 f (x)
f6 (x, 1, 1) − Af3/2,∞ (x) = ,
8(x + 2)2
where
f (x) = 8(2x + 1)(x − 3)2 (x + 2)2 − 9(x − 1)2 (2x − 3)2 .
Since 8(2x + 1) ≥ 72 > 64, we have f (x) ≥ 0 if
8(x − 3)(x + 2) ≥ 3(x − 1)(2x − 3).
Indeed,
8(x − 3)(x + 2) − 3(x − 1)(2x − 3) = 2x 2 + 7x − 57 ≥ 32 + 28 − 57 > 0.

Condition (c). We need to show that f6 (0, y, z) ≥ A fˆ3/2,∞ ( y, z). We have


9 y 2 z 2 ( y 2 + z 2 − 5 yz)2
A fˆ3/2,∞ ( y, z) = ,
8( y + z)2

9 yz
f6 (0, y, z) − A fˆ3/2,∞ ( y, z) = yz( y 2 + z 2 − 5 yz)2 1 − ≥ 0.
8( y + z)2

The equality occurs for x = y = z, for x/3 = y = z (or any cyclic permutation),
and for x = 0 and y/x + x/ y = 5 (or any cyclic permutation).

P 4.17. Let x, y, z be nonnegative real numbers, and let


,
3(1 − k), k ≤ 0
αk = .
3 + k, k≥0
Then,
 αk (x − y)2 ( y − z)2 (z − x)2
(x − y)(x − z)(x − k y)(x − kz) ≥ .
x y + yz + z x
(Vasile C., 2010)
406 Vasile Cîrtoaje

Solution. Write the inequality as f6 (x, y, z) ≥ 0, where



f6 (x, y, z) = (x y+ yz+z x) (x− y)(x−z)(x−k y)(x−kz)−αk (x− y)2 ( y−z)2 (z−x)2 .

We have
A = 27αk > 0,
f6 (x, 1, 1) = (2x + 1)(x − 1)2 (x − k)2 ,
 
f6 (0, y, z) = yz k2 y 2 z 2 + ( y − z)( y 3 − z 3 ) − k yz( y − z)2 − αk y 2 z 2 ( y − z)2
   2
= yz k2 y 2 z 2 + ( y − z)4 − 2|k| yz( y − z)2 = yz ( y − z)2 − |k| yz
 2
= yz ( y + z)2 − (4 + |k|) yz ,

y 2z2  2
ĝγ,δ ( y, z) = γ( y + z)2 + δ yz .
( y + z)2
We will apply Theorem 2 for

4(x − 1)4 (x − |k|)2


Eα,β (x) = f|k|,−2 (x) = .
81(2 + |k|)2
Condition (a). Since
f6 (x, 1, 1) − Af|k|,−2 (x) 3(2 + |k|)2 (2x + 1)(x − k)2 − 4αk (x − 1)2 (x − |k|)2
= ,
(x − 1)2 3(2 + |k|)2
the condition (a) of Theorem 2 is satisfied if

3(2 + |k|)2 (2x + 1)(x − k)2 ≥ 4αk (x − 1)2 (x − |k|)2

for 0 ≤ x ≤ 4. Since (x − k)2 ≥ (x − |k|)2 and 2x + 1 ≥ (x − 1)2 , it suffices to show


that
3(2 + |k|)2 ≥ 4αk .
If k ≤ 0, then
3(2 + |k|)2 − 4αk = 3k2 ≥ 0.
Also, if k ≥ 0, then
3(2 + |k|)2 − 4αk = k(3k + 8) ≥ 0.

Conditions (b) and (c). To prove the conditions (b) and (c) of Theorem 2, we
consider four cases:
56
k ≤ 0, k ≥ 2, 0 ≤ k ≤ 1, 1≤k≤ .
25
In the first three cases, we choose
 2
(2x + 1)2
Fγ,δ (x) = gγ,δ (x) = x + γ(x + 2)(2x + 1) + δ .
x +2
Highest Coefficient Cancellation Method for Nonnegative Variables 407

Having in view the expression of f6 (0, y, z), we need to choose

δ = −(4 + |k|)γ,

to have
γ2 y 2 z 2
ĝγ,δ ( y, z) = [( y + z)2 − (4 + |k|) yz]2 .
( y + z)2
In addition,
 2
(4 + |k|)γ(2x + 1)2
gγ,δ (x) = x + γ(x + 2)(2x + 1) − .
x +2

Case 1: k ≤ 0. Choosing
2
γ=
,
27 1 − k
−2(4 − k)
δ = −(4 − k)γ =
,
27 1 − k
we have
A = 27αk = 81(1 − k),
f6 (x, 1, 1) − Agγ,δ (x) = (2x + 1)(x − 1)2 (x − k)2
 2
2 2(4 − k) (2x + 1)2
−81(1 − k) x +
(x + 2)(2x + 1) −
· ,
27 1 − k 27 1 − k x +2

9[ f6 (x, 1, 1) − Agγ,δ (x)] = 9(2x + 1)(x − 1)2 (x − k)2


  2
(2x + 1)2
− 27 1 − k x + 2(x + 2)(2x + 1) − 2(4 − k) ,
x +2
The condition (b) is satisfied if f6 (x, 1, 1) ≥ Agγ,δ (x) for x ≥ 4. Since x ≥ 4
involves
9(2x + 1) ≥ 81,
this condition is true if
  2
(2x + 1)2
81(x − 1)2 (x − k)2 − 27 1 − k x + 2(x + 2)(2x + 1) − 2(4 − k) ≥ 0,
x +2
which can be written as
f1 (x) f2 (x) ≥ 0,
where
 2(4 − k)(2x + 1)2
f1 (x) = 9(x − 1)(x − k) − 27 1 − k x − 2(x + 2)(2x + 1) +
x +2
 2(4 − k)(2x + 1)2
= 5x − (9k + 27 1 − k + 19)x + 9k − 4 +
2
,
x +2
408 Vasile Cîrtoaje

 2(4 − k)(2x + 1)2


f2 (x) = 9(x − 1)(x − k) + 27 1 − k x + 2(x + 2)(2x + 1) − .
x +2
Since
 k
1−k ≤1− ,
2
  
k 9k
9k + 27 1 − k + 19 ≤ 9k + 27 1 − + 19 = 46 − ,
2 2
we have
 
9k 2(4 − k)(2x + 1)2
f1 (x) ≥ 5x 2 − 46 − x + 9k − 4 +
2 x +2
x −4
= [2x(5x + 16) + (−k)(7x + 8)] ≥ 0.
2(x + 2)
To show that f2 (x) ≥ 0 for x ≥ 4, it suffices to prove that

2(4 − k)(2x + 1)2


9(x − 1)(x − k) ≥ .
x +2
Since x − k ≥ 4 − k, we only need to show that

2(2x + 1)2
9(x − 1) ≥ .
x +2
We have
2(2x + 1)2 (x − 4)(x + 5)
9(x − 1) − = ≥ 0.
x +2 x +2
The condition (c) of Theorem 2 is satisfied if f6 (0, y, z) ≥ Aĝγ,δ ( y, z) for y, z ≥ 0.
We have
γ2 y 2 z 2
ĝγ,δ ( y, z) = [( y + z)2 − (4 + |k|) yz]2
( y + z)2
4 y 2z2
= [( y + z)2 − (4 − k) yz]2 ,
729(1 − k)( y + z)2

4 y 2z2 yz
Aĝγ,δ ( y, z) =
[( y+z)2 −(4−k) yz]2 ≤
[( y+z)2 −(4−k) yz]2 ,
9 1 − k( y + z)2 9 1−k
 
1
f6 (0, y, z) − Aĝγ,δ ( y, z) ≥ 1 −
yz[( y + z)2 − (4 − k) yz]2 ≥ 0.
9 1−k
Case 2: k ≥ 2. From the condition gγ,δ (k) = 0, where
 2
(4 + k)γ(2x + 1)2
gγ,δ (x) = x + γ(x + 2)(2x + 1) − ,
x +2
Highest Coefficient Cancellation Method for Nonnegative Variables 409

we get
k+2
γ= ,
(2k + 1)(k + 5)
−(k + 2)(k + 4)
δ = −(4 + k)γ = ,
(2k + 1)(k + 5)
therefore
 2
(k + 2)(x + 2)(2x + 1) (4 + k)(k + 2)(2x + 1)2
gγ,δ (x) = x + −
(2k + 1)(k + 5) (2k + 1)(k + 5)(x + 2)
g12 (x)
= ,
(2k + 1)2 (k + 5)2 (x + 2)2

where

g1 (x) = (2k + 1)(k + 5)x(x + 2) + (k + 2)(x + 2)2 (2x + 1) − (k + 2)(k + 4)(2x + 1)2
= (x − k)[2(k + 2)x 2 − (5k + 9)x + k + 4].

Since
A = 27αk = 27(k + 3),
the condition (b), namely f6 (x, 1, 1) ≥ Agγ,δ (x) for x ≥ 4, is true if

(2k +1)2 (k +5)2 (2x +1)(x −1)2 (x +2)2 ≥ 27(k +3)[2(k +2)x 2 −(5k +9)x + k +4]2 .

Since 2x + 1 ≥ 9, it suffices to show that

9(2k + 1)2 (k + 5)2 (x − 1)2 (x + 2)2 ≥ 27(k + 3)[2(k + 2)x 2 − (5k + 9)x + k + 4]2 .

In addition, because of

2(k +2)x 2 −(5k +9)x + k +4 ≥ 8(k +2)x −(5k +9)x + k +4 = (3k +7)x + k +4 > 0,

the inequality is true if



(2k + 1)(k + 5)(x − 1)(x + 2) ≥ 3(k + 3) [2(k + 2)x 2 − (5k + 9)x + k + 4].

Having in view that



(2k + 1)(k + 5) > (2k + 4)(k + 2) > (2k + 4) 3(k + 3),

it suffices to show that

(2k + 4)(x − 1)(x + 2) ≥ 2(k + 2)x 2 − (5k + 9)x + k + 4,

which is equivalent to
7(k + 7)x − 5k − 12 ≥ 0.
410 Vasile Cîrtoaje

The condition (c) of Theorem 2 is satisfied if f6 (0, y, z) ≥ Aĝγ,δ ( y, z) for y, z ≥ 0.


We have
γ2 y 2 z 2
ĝγ,δ ( y, z) = [( y + z)2 − (4 + |k|) yz]2
( y + z)2
(k + 2)2 y 2 z 2
= [( y + z)2 − (4 + k) yz]2 ,
(2k + 1)2 (k + 5)2 ( y + z)2

27(k + 3)(k + 2)2 y 2 z 2


Aĝγ,δ ( y, z) = [( y + z)2 − (4 + k) yz]2
(2k + 1)2 (k + 5)2 ( y + z)2
27(k + 3)(k + 2)2 yz
≤ [( y + z)2 − (4 + k) yz]2 ,
4(2k + 1)2 (k + 5)2

27(k + 3)(k + 2)2
f6 (0, y, z) − Aĝγ,δ ( y, z) ≥ 1 − yz[( y + z)2 − (4 + k) yz]2 .
4(2k + 1)2 (k + 5)2
It suffices to show that
7(k + 3)(k + 2)2
≤ 1.
(2k + 1)2 (k + 5)2
This is true because
(k + 2)2
<1
(2k + 1)(k + 5)
and
7(k + 3)
≤ 1.
(2k + 1)(k + 5)
Case 3: 0 ≤ k ≤ 1. Since

f6 (0, y, z) = yz[( y + z)2 − (4 + k) yz]2 ,

γ2 y 2 z 2
ĝγ,δ ( y, z) = [( y + z)2 − (4 + k) yz]2 ,
( y + z)2

27(k + 3)γ2 y 2 z 2
Aĝγ,δ ( y, z) = [( y + z)2 − (4 + k) yz]2
( y + z)2
27(k + 3)γ2 yz
≤ [( y + z)2 − (4 + k) yz]2
4
 
27(k + 3)γ2
f6 (0, y, z) − Aĝγ,δ ( y, z) ≥ 1 − yz[( y + z)2 − (4 + k) yz]2 ,
4
we choose
2
γ= 
3 3(k + 3)
to have f6 (0, y, z) − Aĝγ,δ ( y, z) ≥ 0 for y, z ≥ 0. Thus, the condition (c) of Theorem
2 is satisfied.
Highest Coefficient Cancellation Method for Nonnegative Variables 411

The condition (b) of Theorem 2 is satisfied if f6 (x, 1, 1) ≥ Agγ,δ (x) for x ≥ 4.


We have  2
(4 + k)γ(2x + 1)2
gγ,δ (x) = x + γ(x + 2)(2x + 1) − ,
x +2
 2
(4 + k)γ(2x + 1)2
Agγ,δ (x) = 27(k + 3) x + γ(x + 2)(2x + 1) − = g 2 (x).
x +2
where
2(4 + k)(2x + 1)2
g(x) = 3 3(k + 3) x + 2(x + 2)(2x + 1) − .
x +2
For k ∈ [0, 1], we get

10(2x + 1)2
g(x) > 9x + 2(x + 2)(2x + 1) −
x +2
4x 3 − 13x 2 + 2x − 2
=
x +2
4x 2 (x − 4) + 3x 2 + 2x − 2
= > 0.
x +2
According to the AM-GM inequality

3 + (k + 3) ≥ 2 3(k + 3),

it follows that g(x) ≤ g1 (x), where

3(k + 6)x 2(4 + k)(2x + 1)2


g1 (x) = + 2(x + 2)(2x + 1) − .
2 x +2
Thus it suffices to show that f6 (x, 1, 1) ≥ g12 (x), which is equivalent to g2 (k, x) ≥ 0,
where
g2 (k, x) = f6 (x, 1, 1) − g1 (x)

3(k + 6)x 2(4 + k)(2x + 1)2
= (x − k)(x − 1) 2x + 1 − − 2(x + 2)(2x + 1) + .
2 x +2
For k = 0, we have

8(2x + 1)2
g2 (0, x) = x(x − 1) 2x + 1 − 9x − 2(x + 2)(2x + 1) +
x +2

x(4x 2 − 5x + 10)
= x(x − 1) 2x + 1 − .
x +2
The inequality g2 (0, x) ≥ 0 is equivalent to


4x 2 − 5x + 10
(x − 1) 2x + 1 ≥ . (*)
x +2
412 Vasile Cîrtoaje

It is true if
(x − 1)2 (2x + 1)(x + 2)2 ≥ (4x 2 − 5x + 10)2 ,
which is equivalent to (x − 4)g3 (x) ≥ 0, where

g3 (x) = x 3 (2x − 3) + x(24x − 20) + 24 > 0.

Having in view (*), to show that g2 (k, x) ≥ 0, it suffices to prove that

(x − k)(4x 2 − 5x + 10) 3(k + 6)x 2(4 + k)(2x + 1)2


− − 2(x + 2)(2x + 1) + ≥ 0.
x +2 2 x +2
This inequality reduces to

k(5x 2 + 20x − 16) ≥ 0,

which is true.
Case 4: 1 ≤ k ≤ 3. We choose
4(x − 1)4 (x − γ)2
Fγ,δ (x) = fγ,∞ (x) = .
9(1 + 2γ)2 (x + 2)2

Having in view the expression of f6 (0, y, z), we will choose

k
γ=
2
to have
y 2 z 2 [( y + z)2 − (4 + k) yz]2
fˆk/2,∞ ( y, z) = ,
9(1 + k)2 ( y + z)2
3(k + 3) y 2 z 2 [( y + z)2 − (4 + k) yz]2
A fˆk/2,∞ ( y, z) = ,
(1 + k)2 ( y + z)2
therefore,
  2
3(k + 3) yz
f6 (0, y, z) − A fˆk/2,∞ ( y, z) = 1 − yz ( y + z)2 − (4 + k) yz ≥ 0.
(1 + k)2 ( y + z)2

Thus, the condition (c) of Theorem 2 is satisfied. With regard to the condition (b),
we have
(x − 1)4 (2x − k)2
f k/2,∞ (x) = ,
9(1 + k)2 (x + 2)2
3(k + 3)(x − 1)4 (2x − k)2
Af k/2,∞ (x) = ,
(1 + k)2 (x + 2)2
(x − 1)2 f (x)
f6 (x, 1, 1) − Af k/2,∞ (x) = ,
(1 + k)2 (x + 2)2
f (x) = (1 + k)2 (2x + 1)(x + 2)2 (x − k)2 − 3(k + 3)(x − 1)2 (2x − k)2 .
Highest Coefficient Cancellation Method for Nonnegative Variables 413

Since

(2x + 1)(x + 2)2 − 12(x + 1)2 ≥ 6(2x + 1)(x + 2) − 12(x + 1)2 = 6x > 0,

we get

f (x) > 12(1 + k)2 (x + 1)2 (x − k)2 − 3(k + 3)(x − 1)2 (2x − k)2 .

Therefore, it suffices to show that



2(1 + k)(x + 1)(x − k) ≥ k + 3 (x − 1)(2x − k).

In addition, since 
4 + (k + 3) ≥ 4 k + 3,
it suffices to show that

8(1 + k)(x + 1)(x − k) ≥ (k + 7)(x − 1)(2x − k).

This inequality can be written as

6(k − 1)x 2 + (22 + 9k − 7k2 )x − 7 − 9k − 8k2 ≥ 0.

Since
6(k − 1)x 2 ≥ 24(k − 1)x,
it suffices to show that

24(k − 1)x + (22 + 9k − 7k2 )x − 7 − 9k − 8k2 ≥ 0,

which is
(−2 + 33k − 7k2 )x − 7 − 9k − 8k2 ≥ 0.
Since
−2 + 33k − 7k2 > −9 + 30k − 9k2 = 3(3 − k)(3k − 1) ≥ 0,
we have

(−2 + 33k − 7k2 )x − 7 − 9k − 8k2 ≥ 4(−2 + 33k − 7k2 ) − 7 − 9k − 8k2

= −15 + 123k − 36k2 > −27 + 117k − 36k2 = 9(3 − k)(4k − 1) ≥ 0.

The equality holds for x = y = z, for x = 0 and y/z + z/ y = 2 + |k| (or any cyclic
permutation), and for x/k = y = z (or any cyclic permutation) if k > 0

Observation. The coefficient αk of the product (x − y)2 ( y − z)2 (z − x)2 is the best
possible. Setting x = 0, the inequality turns into

( y 2 + z 2 )2 − (1 + k + αk ) yz( y 2 + z 2 ) + (k2 + 2k − 2 + 2αk ) y 2 z 2 ≥ 0.


414 Vasile Cîrtoaje

For k > 0, choosing y and z such that y 2 + z 2 = (2 + k) yz, we get

k(αk − k − 3) y 2 z 2 ≤ 0,

which involves αk ≤ k + 3.
For k < 0, choosing y 2 + z 2 = (2 − k) yz, we get

(−k)(αk + 3k − 3) y 2 z 2 ≤ 0,

which provides αk ≤ 3(1 − k).


For k = 0, we get

( y − z)2 [( y − z)2 + (3 − α0 ) yz] ≥ 0,

which yields α0 ≤ 3.

P 4.18. If x, y, z are nonnegative real numbers, then


 4(x − y)2 ( y − z)2 (z − x)2
x 2 ( y + z)(x − y)(x − z) ≥ .
x + y +z
(Vasile C., 2010)

Solution. Write the inequality as f6 (x, y, z) ≥ 0, where



f6 (x, y, z) = (x + y + z) x 2 ( y + z)(x − y)(x − z) − 4(x − y)2 ( y − z)2 (z − x)2 .

We have
A = −4(−27) = 108.
Since
f6 (x, 1, 1) = 2x 2 (x + 2)(x − 1)2 ,
we apply Corollary 1 for

x 2 (x − 1)4
Eα,β (x) = f0,−2 (x) = .
81

Since
2x 2 (x − 1)2 (2x + 1)(4 − x)
f6 (x, 1, 1) − Af0,−2 (x) = ≥0
3
for 0 ≤ x ≤ 4, the condition (a) in Corollary 1 is satisfied.
The condition (b) in Corollary 1 is satisfied if f6 (x, 1, 1) ≥ Ax 2 for x ≥ 4. This is
true since
 
f6 (x, 1, 1) − Ax 2 = x 2 2(x + 2)(x − 1)2 − 108 ≥ x 2 (2 · 6 · 9 − 108) = 0.
Highest Coefficient Cancellation Method for Nonnegative Variables 415

The condition (c) in Corollary 1 is also satisfied because

f6 (0, y, z) = yz( y + z)( y − z)( y 2 − z 2 ) − 4 y 2 z 2 ( y − z)2 = yz( y − z)4 ≥ 0.

The equality occurs for x = y = z, for x = 0 and y = z (or any cyclic permutation),
and for y = z = 0 (or any cyclic permutation).

P 4.19. Let x, y, z be nonnegative real numbers. If k is a real numbers, then


 (2 + |k|)2 (x − y)2 ( y − z)2 (z − x)2
( y + z)(x − y)(x − z)(x − k y)(x − kz) ≥ .
x + y +z
(Vasile C., 2010)

Solution. Write the inequality as f6 (x, y, z) ≥ 0, where



f6 (x, y, z) =(x + y + z) ( y + z)(x − y)(x − z)(x − k y)(x − kz)
− (2 + |k|)2 (x − y)2 ( y − z)2 (z − x)2 .

We have
A = 27(2 + |k|)2 > 0,
f6 (x, 1, 1) = 2(x + 2)(x − 1)2 (x − k)2 ,
 
f6 (0, y, z) = ( y + z) k2 y 2 z 2 ( y + z) + yz( y − z)( y 2 − z 2 ) − (2 + |k|)2 y 2 z 2 ( y − z)2
 
= k2 y 2 z 2 ( y + z)2 + yz( y − z)2 ( y + z)2 − (2 + |k|)2 yz
  
= k2 y 2 z 2 ( y + z)2 + yz ( y + z)2 − 4 yz ( y + z)2 − (2 + |k|)2 yz
= yz( y + z)4 − 4(2 + |k|) y 2 z 2 ( y + z)2 + 4(2 + |k|)2 y 3 z 3
 2
= yz ( y + z)2 − 2(2 + |k|) yz ,

y 2z2  2
ĝγ,δ ( y, z) = γ( y + z)2 + δ yz .
( y + z)2
We will apply Theorem 2 for

4(x − 1)4 (x − |k|)2


Eα,β (x) = f|k|,−2 (x) = .
81(2 + |k|)2
Condition (a). Since

4(x − 1)4 (x − |k|)2


Af|k|,−2 (x) = ,
3
f6 (x, 1, 1) − Af|k|,−2 (x) 3(x + 2)(x − k)2 − 2(x − 1)2 (x − |k|)2
= ,
(x − 1)2 3
416 Vasile Cîrtoaje

the condition (a) of Theorem 2, namely f6 (x, 1, 1) − Af|k|,−2 (x) ≥ 0 for 0 ≤ x ≤ 4,


is satisfied if
3(x + 2)(x − k)2 ≥ 2(x − 1)2 (x − |k|)2 .
This is true since 3(x + 2) ≥ 2(x − 1)2 and (x − k)2 ≥ (x − |k|)2 . Indeed,

3(x + 2) − 2(x − 1)2 = (4 − x)(1 + 2x) ≥ 0,

(x − k)2 − (x − |k|)2 = 2(|k| − k)x ≥ 0.

Conditions (b) and (c). To prove the conditions (b) and (c) of Theorem 2, we
consider two cases:
|k| ≥ 1, |k| ≤ 1.

Case 1: |k| ≥ 1. We choose

4(x − 1)4 (x − γ)2


Fγ,δ (x) = fγ,∞ (x) = .
9(1 + 2γ)2 (x + 2)2
Having in view the expression of f6 (0, y, z), we will choose

γ = |k|

to have
y 2 z 2 [( y + z)2 − 2(2 + |k|) yz]2
fˆ|k|,∞ ( y, z) = ,
9(1 + 2|k|)2 ( y + z)2
3(2 + |k|)2 y 2 z 2 [( y + z)2 − 2(2 + |k|) yz]2
A fˆ|k|,∞ ( y, z) = ,
(1 + 2|k|)2 ( y + z)2
therefore,
 
3(2 + |k|)2 yz  2
f6 (0, y, z) − A fˆ|k|,∞ ( y, z) = 1 − yz ( y + z)2 − (2 + |k|)2 yz .
(1 + 2|k|)2 ( y + z)2

Thus, the condition (c) of Theorem 2 is satisfied if

(1 + 2|k|)2 ( y + z)2 ≥ 3(2 + |k|)2 yz.

Since ( y + z)2 ≥ 4 yz, it suffices to show that


2(1 + 2|k|) ≥ 3(2 + |k|).

Indeed,




2(1 + 2|k|) − 3(2 + |k|) = (4 − 3)|k| + 2(1 − 3) ≥ (4 − 3) + 2(1 − 3) > 0.

With regard to the condition (b), we have

4(x − 1)4 (x − |k|)2


f|k|,∞ (x) = ,
9(1 + 2|k|)2 (x + 2)2
Highest Coefficient Cancellation Method for Nonnegative Variables 417

12(2 + |k|)2 (x − 1)4 (x − |k|)2


Af|k|,∞ (x) = ,
(1 + 2|k|)2 (x + 2)2
(x − 1)2 f (x)
f6 (x, 1, 1) − Af|k|,∞ (x) = ,
(1 + 2|k|)2 (x + 2)2
f (x) = (1 + 2|k|)2 (x + 2)3 (x − k)2 − 6(2 + |k|)2 (x − 1)2 (x − |k|)2 .
Since (x − k)2 ≥ (x − |k|)2 , it suffices to show that

(1 + 2|k|)2 (x + 2)3 ≥ 6(2 + |k|)2 (x − 1)2 .

Since x + 2 > 6, this is true if

(1 + 2|k|)2 (x + 2)2 ≥ (2 + |k|)2 (x − 1)2 ,

which is equivalent to

(1 + 2|k|)(x + 2) ≥ (2 + |k|)(x − 1).

This is true for x ≥ 4 and |k| ≥ 1 because 1 + 2|k| ≥ 2 + |k| and x + 2 > x − 1.

Case 2: |k| ≤ 1. We choose


 2
(2x + 1)2
Fγ,δ (x) = gγ,δ (x) = x + γ(x + 2)(2x + 1) + δ .
x +2

Having in view the expression of f6 (0, y, z), we need to set

δ = −2(2 + |k|)γ,

to have
γ2 y 2 z 2
ĝγ,δ ( y, z) = [( y + z)2 − 2(2 + |k|) yz]2 .
( y + z)2
In addition, by choosing
1
γ= ,
3(2 + |k|)
we have

y 2z2
ĝγ,δ ( y, z) = [( y + z)2 − 2(2 + |k|) yz]2 ,
9(2 + |k|)2 ( y + z)2

3 y 2z2
Aĝγ,δ ( y, z) = [( y + z)2 − 2(2 + |k|) yz]2 ,
( y + z)2
  2
3 yz
f6 (0, y, z) − Aĝγ,δ ( y, z) = 1 − yz ( y + z)2 − 2(2 + |k|)2 yz ≥ 0.
( y + z)2
Thus, the condition (c) of Theorem 2 is satisfied.
418 Vasile Cîrtoaje

With regard to the condition (b), we have


 2
(x + 2)(2x + 1) 2(2x + 1)2
gγ,δ (x) = x + −
3(2 + |k|) 3(x + 2)
 2
(x + 2)(2x + 1) 5x 2 + 2x + 2
= − ,
3(2 + |k|) 3(x + 2)
 2
(2 + |k|)(5x + 2x + 2)
2
Agγ,δ (x) = 3 (x + 2)(2x + 1) − ,
x +2
f6 (x, 1, 1) = 2(x + 2)(x − 1) (x − k) ≥ 12(x − 1) (x − |k|)2 ,
2 2 2

f6 (x, 1, 1) − Agγ,δ (x) ≥ 12(x − 1)2 (x − |k|)2 − Agγ,δ (x) = 3g1 (x)g2 (x),
where
(2 + |k|)(5x 2 + 2x + 2)
g1 (x) = 2(x − 1)(x − |k|) − (x + 2)(2x + 1) +
x +2
3[(1 + |k|)x 2 − 4x + 2|k|] 3(x 2 − 4x)
= ≥ ≥ 0.
x +2 x +2
(2 + |k|)(5x 2 + 2x + 2)
g2 (x) = 2(x − 1)(x − k) + (x + 2)(2x + 1) −
x +2
3(5x 2 + 2x + 2)
≥ 2(x − 1)(x − 1) + (x + 2)(2x + 1) −
x +2
5x 2 + 2x + 2
≥ 2(x − 1) + (x + 2)(2x + 1) −
2
2
5x 2 + 2x + 2 3(x 2 + 2)
= 4x + x + 4 −
2
= > 0.
2 2
The equality occurs for x = y = z, for x/k = y = z (or any cyclic permutation)
if k = 0, for y = z = 0 (or any cyclic permutation), and for x = 0 and y/z + z/ y =
2 + 2|k| (or any cyclic permutation).
Observation. The coefficient of the product (x − y)2 ( y − z)2 (z − x)2 is the best
possible. Setting x = 0, the inequality
 αk (x − y)2 ( y − z)2 (z − x)2
( y + z)(x − y)(x − z)(x − k y)(x − kz) ≥
x + y +z
becomes
 
yz ( y 2 + z 2 )2 − (αk − k2 ) yz( y 2 + z 2 ) + 2(αk + k2 − 2) y 2 z 2 ≥ 0.

For y 2 + z 2 = 2(1 + |k|) yz, this inequality leads to

|k|[αk − (2 + |k|)2 ] y 2 z 2 ≤ 0,

which implies the necessary condition αk ≤ (2 + |k|)2 .


Highest Coefficient Cancellation Method for Nonnegative Variables 419

P 4.20. If x, y, z are nonnegative real numbers, then


 12(x − y)2 ( y − z)2 (z − x)2
x(x 2 − y 2 )(x 2 − z 2 ) ≥ .
x + y +z

(Vasile C., 2010)

Solution. Write the inequality as f6 (x, y, z) ≥ 0, where



f6 (x, y, z) = (x + y + z) x(x 2 − y 2 )(x 2 − z 2 ) − 12(x − y)2 ( y − z)2 (z − x)2

has the highest coefficient

A = −12(−27) = 324.

Since
f6 (x, 1, 1) = x(x + 2)(x 2 − 1)2 ,
we apply Corollary 1 for

4x 2 (x − 1)4 (x + 1)2
Eα,β (x) = f0,−1 (x) = .
225(x + 2)2

Since
x(x − 1)2 (x + 1)2 f (x)
f6 (x, 1, 1) − Af0,−1 (x) = ,
225(x + 2)2
where

f (x) = 225(x + 2)3 − 1296x(x − 1)2 > 216[(x + 2)3 − 6x(x − 1)2 ],

the condition (a) in Corollary 1 is satisfied if (x + 2)3 ≥ 6x(x − 1)2 for 0 ≤ x ≤ 4.


This is true since

2(x + 2) ≥ 3x, (x + 2)2 ≥ 4(x − 1)2 .

The condition (b) in Corollary 1 is satisfied if f6 (x, 1, 1) ≥ Ax 2 for x ≥ 4. This is


true if
(x + 2)(x 2 − 1)2 ≥ 324x.
It suffices to show that
2(x + 2)(x 2 − 1)2 ≥ 675x,
which follows by multiplying the inequalities

2(x + 2)(x − 1) ≥ 9x, (x − 1)(x + 1)2 ≥ 75.

Indeed, we have

2(x + 2)(x − 1) − 9x = (x − 4)(2x + 1) ≥ 0, (x − 1)(x + 1)2 − 75 ≥ 3 · 25 − 75 = 0.


420 Vasile Cîrtoaje

The condition (c) in Corollary 1 is also satisfied because

f6 (0, y, z) = ( y +z)( y 2 −z 2 )( y 3 −z 3 )−12 y 2 z 2 ( y −z)2 = ( y −z)4 ( y 2 +z 2 +5 yz) ≥ 0.

The equality occurs for x = y = z, for −x = y = z (or any cyclic permutation),


and for x = 0 and y = z (or any cyclic permutation).

P 4.21. Let x, y, z be nonnegative real numbers, and let



⎨ 4(k − 2), k ≤ 6
αk = .
⎩ (k + 2) , k ≥ 6
2

4
Then,
 αk (x − y)2 ( y − z)2 (z − x)2
x(x − y)(x − z)(x − k y)(x − kz) + ≥ 0.
x + y +z
(Vasile C., 2010)
Solution. Write the inequality as f6 (x, y, z) ≥ 0, where

f6 (x, y, z) = (x + y+z) x(x − y)(x −z)(x −k y)(x −kz)+αk (x − y)2 ( y−z)2 (z−x)2 .

Since the product (x − y)2 ( y − z)2 (z − x)2 has the highest coefficient equal to −27,
f6 (x, y, z) has the highest coefficient

A = −27αk .

Also, we have
f6 (x, 1, 1) = x(x + 2)(x − 1)2 (x − k)2 ,
f6 (0, y, z) = ( y − z)2 [( y + z)4 − (k + 2) yz( y + z)2 + αk y 2 z 2 ].
There are three cases to consider.
Case 1: k ≥ 6. Since
αk = (k + 2)2 ,
−27(k + 2)2
A = −27αk = < 0,
4
the desired inequality is true if f6 (x, 1, 1) ≥ 0 and f6 (0, y, z) ≥ 0 for x, y, z ≥ 0
(Theorem 1). The first condition is clearly true and
(k + 2)2 2 2
f6 (0, y, z) = ( y − z)2 [( y + z)4 − (k + 2) yz( y + z)2 + y z ]
4
 2
k+2
= ( y − z)2 ( y + z)2 − yz ≥ 0.
2
Highest Coefficient Cancellation Method for Nonnegative Variables 421

Case 2: 2 ≤ k ≤ 6. Since
αk = 4(k − 2),
A = −27αk = −108(k − 2) ≤ 0,
the desired inequality is true if f6 (x, 1, 1) ≥ 0 and f6 (0, y, z) ≥ 0 for x, y, z ≥ 0
(Theorem 1). The first condition is true and

f6 (0, y, z) = ( y − z)2 [( y + z)4 − (k + 2) yz( y + z)2 + 4(k − 2) y 2 z 2 ]


= ( y − z)4 [( y + z)2 − (k − 2) yz] ≥ ( y − z)6 ≥ 0.

Case 3: k ≤ 2. We have
αk = 4(k − 2),
A = −27αk = 108(2 − k) ≥ 0.
We will apply Corollary 1 for

4x 2 (x − 1)4 (x − k)2
Eα,β (x) = f k,0 (x) = .
9(4 − k)2 (x + 2)2

The condition (a) in Corollary 1 is satisfied if f6 (x, 1, 1) ≥ Af k,0 (x) for x ∈ [0, 4].
We have
48(2 − k)x 2 (x − 1)4 (x − k)2
Af k,0 (x) = ,
(4 − k)2 (x + 2)2
x(x − 1)2 (x − k)2 [(4 − k)2 (x + 2)3 − 48(2 − k)x(x − 1)2 ]
f6 (x, 1, 1) − Af k,0 (x) = .
(4 − k)2 (x + 2)2
The condition (a) is true if

(4 − k)2 (x + 2)3 ≥ 48(2 − k)x(x − 1)2

for 0 ≤ x ≤ 4. This inequality follows by multiplying the inequalities

(4 − k)2 ≥ 8(2 − k)

and
(x + 2)3 ≥ 6x(x − 1)2 ,
which are equivalent to k2 ≥ 0 and (4 − x)(2 + 2x + 5x 2 ) ≥ 0, respectively.
The condition (b) in Corollary 1 is satisfied if f6 (x, 1, 1) ≥ Ax 2 for x ≥ 4. It
suffices to show that

(x + 2)(x − 1)2 (x − k)2 ≥ 108(2 − k)x.

This inequality follows from


4(x − 1)2 ≥ 9x
and
(x + 2)(x − k)2 ≥ 48(2 − k).
422 Vasile Cîrtoaje

Indeed, we have
4(x − 1)2 − 9x = (x − 4)(4x − 1) ≥ 0,

(x + 2)(x − k)2 − 48(2 − k) ≥ 6(4 − k)2 − 48(2 − k) = 6k2 ≥ 0.

The condition (c) in Corollary 1 is satisfied if f6 (0, y, z) ≥ 0 for y, z ≥ 0. Indeed,

f6 (0, y, z) = ( y − z)4 [( y + z)2 + (2 − k) yz] ≥ 0.

The equality occurs for x = y = z, for x = 0 and y = z (or any cyclic permutation),
and for x/k = y = z (or any cyclic permutation) if k = 0, and for x = 0 and
y/z + z/ y = (k − 2)/2 (or any cyclic permutation) if k > 6

Observation. The coefficient αk of the product (x − y)2 ( y − z)2 (z − x)2 is the best
possible. Setting x = 0, the inequality becomes

( y − z)2 [( y 2 + z 2 )2 − (k − 2) yz( y 2 + z 2 ) + (αk − 2k) y 2 z 2 ] ≥ 0.

For y = z = 1, the necessary condition

( y 2 + z 2 )2 − (k − 2) yz( y 2 + z 2 ) + (αk − 2k) y 2 z 2 ≥ 0

involves αk ≥ 4(k − 2).


Also, for
k−2
y 2 + z2 = yz, k ≥ 6,
2
the necessary condition

( y 2 + z 2 )2 − (k − 2) yz( y 2 + z 2 ) + (αk − 2k) y 2 z 2 ≥ 0

becomes
[4αk − (k + 2)2 ] y 2 z 2 ≥ 0,

which involves αk ≥ (k + 2)2 /4.

P 4.22. If x, y, z are nonnegative real numbers, then

 5(x − y)2 ( y − z)2 (z − x)2


(x 2 + yz)(x − y)(x − z) ≥ .
x y + yz + z x

(Vasile C., 2010)


Highest Coefficient Cancellation Method for Nonnegative Variables 423

Solution. Write the inequality as f6 (x, y, z) ≥ 0, where



f6 (x, y, z) = (x y + yz + z x) (x 2 + yz)(x − y)(x − z) − 5(x − y)2 ( y − z)2 (z − x)2 .

We have
A = −5(−27) = 135,
f6 (x, 1, 1) = (2x + 1)(x 2 + 1)(x − 1)2 ,

f6 (0, y, z)
= y 2 z 2 + ( y − z)2 ( y 2 + z 2 + yz) − 5 yz( y − z)2
yz
 
= y 2 z 2 + ( y 2 + z 2 − 2 yz) y 2 + z 2 − 4 yz
= ( y 2 + z 2 )2 − 6( y 2 + z 2 ) yz + 9 y 2 z 2
= ( y 2 + z 2 − 3 yz)2 ,

y 2 z 2 ( y 2 + z 2 − 3 yz)2
fˆ1/2,∞ ( y, z) = .
36( y + z)2

Thus, we apply Theorem 2 for

Eα,β (x) = Fγ,δ = f1/2,∞ (x).

The conditions (a) and (b) in Theorem 2 are satisfied if f6 (x, 1, 1) ≥ Af1/2,∞ (x)
for x ≥ 0. Since
15(x − 1)4 (2x − 1)
Af1/2,∞ (x) = ,
4(x + 2)2
(x − 1)2 f (x)
f6 (x, 1, 1) − Af1/2,∞ (x) = ,
4(x + 2)2
where
f (x) = 4(x + 2)2 (2x + 1)(x 2 + 1) − 15(x − 1)2 (2x − 1)2 ,
we need to show that f (x) ≥ 0 for x ≥ 0. This is true if

(x + 2)2 (2x + 1)(x 2 + 1) ≥ 4(x − 1)2 (2x − 1)2 .

Since x 2 + 1 ≥ (x − 1)2 , we only need to prove that

(x + 2)2 (2x + 1) ≥ 4(2x − 1)2 ,

that is
x(2x 2 − 7x + 28) ≥ 0.

The condition (c) in Theorem 2 is also satisfied because

15 y 2 z 2 ( y 2 + z 2 − 3 yz)2
A fˆ1/2,∞ ( y, z) = ,
4( y + z)2
424 Vasile Cîrtoaje


15 yz
f6 (0, y, z) − A fˆ1/2,∞ ( y, z) = yz( y 2 + z 2 − 3 yz)2 1 −
4( y + z)2
 
15
≥ yz( y 2 + z 2 − 3 yz)2 1 − ≥ 0.
16

The equality occurs for x = y = z, and for x = 0 and y/z + z/ y = 3 (or any
cyclic permutation).

P 4.23. If x, y, z are nonnegative real numbers, then


 16(x − y)2 ( y − z)2 (z − x)2
(4x 2 + yz)(x − y)(x − z) ≥ .
x y + yz + z x
(Vasile C., 2010)

Solution. Write the inequality as f6 (x, y, z) ≥ 0, where



f6 (x, y, z) = (x y + yz +z x) (4x 2 + yz)(x − y)(x −z)−16(x − y)2 ( y −z)2 (z − x)2 .

We have
A = −16(−27) = 432,
f6 (x, 1, 1) = (2x + 1)(4x 2 + 1)(x − 1)2 ,

f6 (0, y, z)
= y 2 z 2 + 4( y − z)2 ( y 2 + z 2 + yz) − 16 yz( y − z)2
yz
 
= y 2 z 2 + 4( y 2 + z 2 − 2 yz) y 2 + z 2 − 3 yz
= 4( y 2 + z 2 )2 − 20( y 2 + z 2 ) yz + 25 y 2 z 2
= (2 y 2 + 2z 2 − 5 yz)2 .
Apply Theorem 2 for

4(x − 1)4 (2x − 1)2


Eα,β (x) = f1/2,−2 (x) = ,
2025

Condition (a). Since

64(x − 1)4 (2x − 1)2


Af1/2,−2 (x) = ,
75
(x − 1)2 f (x)
f6 (x, 1, 1) − Af1/2,−2 (x) = ,
75
where
f (x) = 75(2x + 1)(4x 2 + 1) − 64(x − 1)2 (2x − 1)2 ,
Highest Coefficient Cancellation Method for Nonnegative Variables 425

the condition (a) of Theorem 2 is satisfied if f (x) ≥ 0 for 0 ≤ x ≤ 4. It suffices to


show that
(2x + 1)(4x 2 + 1) ≥ (x − 1)2 (2x − 1)2 .
This is true because
2x + 1 ≥ (x − 1)2 ,
4x 2 + 1 ≥ (2x − 1)2 .
Indeed,
2x + 1 − (x − 1)2 = x(4 − x) ≥ 0,
4x 2 + 1 − (2x − 1)2 = 4x ≥ 0.
Conditions (b) and (c). Having in view the expression of f6 (0, y, z), we will apply
Theorem 2 for

Fγ,δ (x) = gγ,δ (x), δ=− ,
2
which leads to
γ2 y 2 z 2
ĝγ,δ ( y, z) = (2 y 2 + 2z 2 − 5 yz)2 .
4( y + z)2
Since
108γ2 y 2 z 2
Aĝγ,δ ( y, z) = (2 y 2 + 2z 2 − 5 yz)2 ,
( y + z)2
f6 (0, y, z) − Aĝγ,δ ( y, z) = yz(2 y 2 + 2z 2 − 5 yz)2 g( y, z),
where
108γ2 yz
g( y, z) = 1 − ≥ 1 − 27γ2 ,
( y + z)2
we choose
1
γ=

3 3
to have g( y, z) ≥ 0. Thus, the condition (c) in Theorem 2 is satisfied.
The condition (b) is satisfied if f6 (x, 1, 1) ≥ Agγ,δ (x) for x ≥ 4, where
 2
9γ(2x + 1)2
gγ,δ (x) = x + γ(x + 2)(2x + 1) − .
2(x + 2)
Since

2


27 3γ(2x + 1)2
Agγ,δ (x) = 4 6 3x + 6 3γ(x + 2)(2x + 1) − = 4 f 2 (x),
x +2

9(2x + 1)2
f (x) = 6 3x + 2(x + 2)(2x + 1) − ,
x +2
we need to show that

(2x + 1)(4x 2 + 1)(x − 1)2 ≥ 4 f 2 (x)


426 Vasile Cîrtoaje

for x ≥ 4. Since

9(2x + 1)2
f (x) > 9x + 2(x + 2)(2x + 1) −
x +2
x 2 (4x − 9) + 6x − 1
= > 0,
x +2
it suffices to show that

(2x + 1)(4x 2 + 1)(x − 1)2 ≥ 4 f12 (x),

where
9(2x + 1)2
f1 (x) = 11x + 2(x + 2)(2x + 1) − > f (x).
x +2
Since
(2x + 1)(4x 2 + 1)(x − 1)2 > (2x + 1)(4x 2 )(x − 1)2
and
4x 3 − 7x 2 + 10x − 1 x(4x 2 − 7x + 10)
f1 (x) = < ,
x +2 x +2
it suffices to show that

(2x + 1)(x − 1)2 (x + 2)2 ≥ (4x 2 − 7x + 10)2 ,

which can be rewritten as

(2x + 1)2 (x − 1)2 (x + 2)2 ≥ (2x + 1)(4x 2 − 7x + 10)2 .

Since
(x + 5)2
2x + 1 ≤ ,
9
it suffices to show that

9(2x + 1)2 (x − 1)2 (x + 2)2 ≥ (x + 5)2 (4x 2 − 7x + 10)2 ,

which is equivalent to

3(2x + 1)(x − 1)(x + 2) ≥ (x + 5)(4x 2 − 7x + 10),

x 3 − 2x 2 + 8x − 28 ≥ 0,
(x − 4)(x 2 + 2x + 16) + 36 ≥ 0.

The equality occurs for x = y = z, and for x = 0 and y/z + z/ y = 5/2 (or any
cyclic permutation).
Highest Coefficient Cancellation Method for Nonnegative Variables 427

P 4.24. Let x, y, z be nonnegative real numbers. If k ≥ 0, then




(3 + 2 k )(x − y)2 ( y − z)2 (z − x)2


(x + k yz)(x − y)(x − z) ≥
2
.
x y + yz + z x

(Vasile C., 2010)

Solution. Write the inequality as f6 (x, y, z) ≥ 0, where


 
f6 (x, y, z) = q (x 2 + k yz)(x − y)(x − z) − (3 + 2 k)(x − y)2 ( y − z)2 (z − x)2 .

We have 
A = 27(3 + 2 k),
f6 (x, 1, 1) = (2x + 1)(x 2 + k)(x − 1)2 ,

f6 (0, y, z) 
= k y 2 z 2 + ( y − z)2 ( y 2 + z 2 + yz) − (3 + 2 k) yz( y − z)2
yz
"  #
= k y 2 z 2 + ( y 2 + z 2 − 2 yz) y 2 + z 2 − 2(1 + k) yz
 
= ( y 2 + z 2 )2 − 2(2 + k)( y 2 + z 2 ) yz + (2 + k)2 y 2 z 2
"  #2
= y 2 + z 2 − (2 + k) yz ,

We will apply Theorem 2 for



2
4(x − 1)4 x − k
Eα,β (x) = f
k,−2 (x) = 
2 .
81 2 + k

Condition (a). Since



 
2
4 3 + 2 k (x − 1)4 x − k
Af
k,−2 (x) = 
2 ,
3 2+ k

f6 (x, 1, 1) − Af
k,−2 (x) = (x − 1)2 f (x),
where 

4 3+2 k  2
f (x) = (2x + 1)(x 2 + k) − 
2 (x − 1) x − k ,
2

3 2+ k
the condition (a) of Theorem 2 is satisfied if f (x) ≥ 0 for 0 ≤ x ≤ 4. This is true
because 

4 3+2 k
1≥ 
2 ,
3 2+ k
2x + 1 ≥ (x − 1)2 ,
428 Vasile Cîrtoaje

 2
x2 + k ≥ x − k .
Indeed,
2x + 1 − (x − 1)2 = x(4 − x) ≥ 0,
 2 
x 2 + k − x − k = 2 k x ≥ 0.

With regard to the conditions (b) and (c), we consider two cases: 0 ≤ k ≤ 1 and
k ≥ 1.
Case 1: k ≥ 1. Having in view the expression of f6 (0, y, z), we will apply Theorem
2 for 
2
(x − 1)4 2x − k
Fγ,δ (x) = f k/2,∞ (x) = 


2 ,
9 1 + k (x + 2)2

2
y 2 z 2 y 2 + z 2 − (2 + k) yz
ˆ
f k/2,∞ ( y, z) =


.
9(1 + k )2 ( y + z)2
The condition (c) is satisfied because
"  #2
f6 (0, y, z) − A fˆ
k/2,∞ ( y, z) = yz y 2 + z 2 − (2 + k) yz f ( y, z),

where

3(3 + 2 k ) yz
f ( y, z) = 1 −

(1 + k )2 ( y + z)2

3(3 + 2 k ) 4k + 2 k − 5
≥1−
=
> 0.
4(1 + k )2 4(1 + k )2

The condition (b) is satisfied if f6 (x, 1, 1) ≥ Af


k/2,∞ (x) for x ≥ 4. Since


2
3(3 + 2 k )(x − 1)4 2x − k
Af
k/2,∞ (x) = 
2 ,
1 + k (x + 2)2

we need to show that




2
3(3 + 2 k )(x − 1)2 2x − k
(2x + 1)(x + k) ≥
2

2 .
1 + k (x + 2)2

Since

3(3 + 2 k )

2 < 4
1+ k
and  2
2x − k ≤ 4x 2 + k,
Highest Coefficient Cancellation Method for Nonnegative Variables 429

it suffices to show that


4(x − 1)2 (4x 2 + k)
(2x + 1)(x 2 + k) ≥ .
(x + 2)2
Since
x2 + k x2 + 1 3(k − 1)x 2
− 2 = ≥ 0,
4x + k 4x + 1 (4x + k)(4x 2 + 1)
2 2

we only need to prove that

4(x − 1)2 (4x 2 + 1)


(2x + 1)(x 2 + 1) ≥ .
(x + 2)2

This is true because x 2 + 1 > (x − 1)2 and

(2x + 1)(x + 2)2 ≥ 4(4x 2 + 1).

The last inequality is equivalent to

x(2x 2 − 7x + 12) ≥ 0.

Case 2: 0 ≤ k ≤ 1. Having in view the expression of f6 (0, y, z), we will apply


Theorem 2 for 
Fγ,δ (x) = gγ,δ (x), δ = −(4 + k )γ,
which leads to
γ2 y 2 z 2 " 2  #2
ĝγ,δ ( y, z) = y + z 2 − (2 + k ) yz .
( y + z) 2

Since

27(3 + 2 k )γ2 y 2 z 2 " 2  #2


Aĝγ,δ ( y, z) = y + z 2 − (2 + k ) yz ,
( y + z) 2

"  #2
f6 (0, y, z) − Aĝγ,δ ( y, z) = yz y 2 + z 2 − (2 + k ) yz g( y, z),
where

27(3 + 2 k )γ2 yz 27(3 + 2 k)γ2


g( y, z) = 1 − ≥ 1 − ,
( y + z)2 4
we choose
2
γ=

3 3(3 + 2 k )
to have g( y, z) ≥ 0. Thus, the condition (c) in Theorem 2 is satisfied.
The condition (b) is satisfied if f6 (x, 1, 1) ≥ Agγ,δ (x) for x ≥ 4, where
  2
(2x + 1)2
gγ,δ (x) = x + γ(x + 2)(2x + 1) − (4 + k )γ .
x +2
430 Vasile Cîrtoaje

Since
4
Agγ,δ (x) =
gγ,δ (x) = f 2 (x),
γ2
2x  (2x + 1)2
f (x) = + 2(x + 2)(2x + 1) − 2(4 + k) ,
γ x +2
we need to show that

(2x + 1)(x 2 + k)(x − 1)2 ≥ f 2 (x).


Since 2/γ ≥ 9 and 4 + k ≤ 5, we have

10(2x + 1)2
f (x) ≥ 9x + 2(x + 2)(2x + 1) −
x +2
x 2 (4x − 13) + 2(x − 1)
= > 0.
x +2
On the other hand, by the AM-GM inequality,
-   
2 3
= 3 3(3 + 2 k) ≤ [3 + (3 + 2 k)] = 3(3 + k).
γ 2

Therefore,
  (2x + 1)2
f (x) ≤ 3(3 + k)x + 2(x + 2)(2x + 1) − 2(4 + k)

x +2
x(4x 2 − 5x + 10) − k(5x 2 + 2x + 2)
=
x +2
x(4x 2 − 5x + 10)
≤ .
x +2
In addition,
(2x + 1)(x 2 + k)(x − 1)2 ≥ (2x + 1)x 2 (x − 1)2 .
Thus, it suffices to prove that

(2x + 1)(x − 1)2 (x + 2)2 ≥ (4x 2 − 5x + 10)2 ,

which may be rewritten as

(2x + 1)2 (x − 1)2 (x + 2)2 ≥ (2x + 1)(4x 2 − 5x + 10)2 .

According to
(x + 5)2
2x + 1 ≤ ,
9
it suffices to show that
1
(2x + 1)(x − 1)(x + 2) ≥ (x + 5)(4x 2 − 5x + 10).
3
Highest Coefficient Cancellation Method for Nonnegative Variables 431

This is equivalent to the obvious inequality

(x − 4)(x 2 + x + 7) ≥ 0.

The equality occurs for x = y = z, and for x = 0 and y/z + z/ y = 2 + k (or


any cyclic permutation).
Observation. The coefficient of the product (x − y)2 ( y − z)2 (z − x)2 is the best
possible. Setting x = 0, the inequality
 αk (x − y)2 ( y − z)2 (z − x)2
(x 2 + k yz)(x − y)(x − z) ≥
x y + yz + z x
reduces to
"  #2 
yz y 2 + z 2 − (2 + k) yz + (3 + 2 k − αk ) y 2 z 2 ( y − z)2 ≥ 0.

In addition, for 
y 2 + z 2 = (2 + k) yz,
we get the necessary condition
 
3 + 2 k − αk y 2 z 2 ( y − z)2 ≥ 0,

which involves αk ≤ 3 + 2 k.

P 4.25. If x, y, z are nonnegative real numbers, then




(x 2 − yz)2 (x − y)(x − z) ≥ 4( 2 + 1)(x − y)2 ( y − z)2 (z − x)2 .

(Vasile C., 2014)


Solution. Denote

C = 4( 2 + 1)
and write the inequality as f6 (x, y, z) ≥ 0, where

f6 (x, y, z) = (x 2 − yz)2 (x − y)(x − z) − C(x − y)2 ( y − z)2 (z − x)2 .
!
Since (x − y)(x − z) = x 2 + 2 yz − q, the sum (x 2 − yz)2 (x − y)(x − z) has the
same highest coefficient A1 as

P1 (x, y, z) = (x 2 − yz)2 (x 2 + 2 yz)2 ,

that is A1 = P1 (1, 1, 1) = 0. Therefore, f6 (x, y, z) has the highest coefficient


A = A1 − C(−27) = 108( 2 + 1).


432 Vasile Cîrtoaje

We have
f6 (x, 1, 1) = (x + 1)2 (x − 1)4 ,

f6 (0, y, z) = y 3 z 3 + ( y − z)( y 5 − z 5 ) − C y 2 z 2 ( y − z)2


 
= y 3 z 3 + ( y − z)2 y 4 + z 4 + yz( y 2 + z 2 ) + (1 − C) y 2 z 2
 
= y 2 z 2 + ( y 2 + z 2 − 2 yz) ( y 2 + z 2 )2 + ( y 2 + z 2 ) yz − (1 + C) y 2 z 2
= ( y 2 + z 2 )3 − ( y 2 + z 2 )2 yz − (3 + C)( y 2 + z 2 ) y 2 z 2 + (3 + 2C) y 3 z 3

= ( y 2 + z 2 )3 − ( y 2 + z 2 )2 yz − (4 2 + 7)( y 2 + z 2 ) y 2 z 2 + (8 2 + 11) y 3 z 3


2
= y 2 + z 2 + (2 2 + 1) yz y 2 + z 2 − ( 2 + 1) yz .

In addition, for

2−1
γ= ,
2
we have

y 2 z 2 [ y 2 + z 2 − ( 2 + 1) yz]2
fˆγ,∞ ( y, z) = .
18( y + z)2

We apply Theorem 2 for

Eα,β (x) = f2/3,−2 , Fγ,δ (x) = fγ,∞ (x).

Condition (a). We need to show that f6 (x, 1, 1) ≥ Af2/3,−2 (x) for 0 ≤ x ≤ 4. We


have
(x − 1)4 (3x − 2)2
f2/3,∞ (x) = ,
1296

( 2 + 1)(x − 1)4 (3x − 2)2


Af2/3,∞ (x) = ,
12
(x − 1)4 f (x)
f6 (x, 1, 1) − Af2/3,−2 (x) = ,
12

f (x) = 12(x + 1)2 − ( 2 + 1)(3x − 2)2 .


Since

3 5
2+1< +1= ,
2 2
we have

5(3x − 2)2
f (x) > 12(x + 1)2 −
2
5(3x − 2)2
> 10(x + 1) −
2
2
25x(4 − x)
= ≥ 0.
2
Highest Coefficient Cancellation Method for Nonnegative Variables 433

2−1
Condition (b). We need to show that f6 (x, 1, 1) ≥ Afγ,∞ (x) for γ = and
2
x ≥ 4. Since

4(x − 1)4 (x − γ)2 (x − 1)4 (2x − 2 + 1)2


fγ,∞ (x) = = ,
9(1 + 2γ)2 (x + 2)2 18(x + 2)2

6( 2 + 1)(x − 1)4 (2x − 2 + 1)2


Afγ,∞ (x) = ,
(x + 2)2
(x − 1)4 f (x)
f6 (x, 1, 1) − Afγ,∞ (x) = ,
(x + 2)2
where

f (x) = (x + 1)2 (x + 2)2 − 6( 2 + 1)(2x − 2 + 1)2 ,


we need to show that f (x) ≥ 0 for x ≥ 4. Since

6( 2 + 1) < 15,
it suffices to show that

(x + 1)(x + 2) ≥ 15(2x − 2 + 1),


which is equivalent to g(x) ≥ 0, where


g(x) = x 2 − (2 15 − 3)x + 2 + 30 − 15.


We will show that
g(x) ≥ g(4) > 0.
We have

g(x) − g(4) = (x − 4)(x + 7 − 2 15) ≥ (x − 4)(11 − 2 15) ≥ 0,





g(4) = 15( 60 + 2 − 9) > 0.

Condition (c). We need to show that f6 (0, y, z) ≥ A fˆγ,∞ ( y, z) for y, z ≥ 0. This


condition is satisfied because

6( 2 + 1) y 2 z 2 [ y 2 + z 2 − ( 2 + 1) yz]2
A fˆγ,∞ ( y, z) = ,
( y + z)2
 2
 2
f6 (0, y, z) − A fˆγ,∞ ( y, z) = y + z 2 − ( 2 + 1) yz f0 ( y, z),
where


6( 2 + 1) y 2 z 2
f0 ( y, z) = y 2 + z 2 + (2 2 + 1) yz −
( y + z)2


3( 2 + 1) yz ( 2 + 3) yz
≥ 2 yz + (2 2 + 1) yz − = ≥ 0.
2 2

The equality occurs for x = y = z, and for x = 0 and y/z + z/ y = 2 + 1 (or


any cyclic permutation).
434 Vasile Cîrtoaje

P 4.26. If x, y, z are nonnegative real numbers, then


 1 9
≥ .
4x 2 + y 2 + z 2 4(x 2 + y 2 + z 2 ) + 2(x y + yz + z x)

(Vasile C., 2011)

Solution. Write the inequality as f6 (x, y, z) ≥ 0, where


 (
f6 (x, y, z) = P (4 y 2 + z 2 + x 2 )(4z 2 + x 2 + y 2 ) − 9 (4x 2 + y 2 + z 2 ),

P = 4(x 2 + y 2 + z 2 ) + 2(x y + yz + z x).


coefficient A of f6 (x, y, z) is equal with the highest coefficient of the
The highest+
product −9 (4x 2 + y 2 + z 2 ). Since

4x 2 + y 2 + z 2 = 3x 2 + p2 − 2q,

we have
A = −9(3)3 = −243.
By Theorem 1, we only need to prove the original inequality for y = z = 1 and for
x = 0.
Case 1: y = z = 1. We need to show that
1 2 9
+ ≥ ,
4x 2 + 2 x 2 + 5 4x 2 + 4x + 10
which is equivalent to
x(x − 1)2 ≥ 0.

Case 2: x = 0. We need to show that


1 1 1 9
+ + ≥ ,
y 2 + z 2 4 y 2 + z 2 4z 2 + y 2 4( y 2 + z 2 ) + 2 yz
which is equivalent to

1 5( y 2 + z 2 ) 9
+ ≥ ,
y2 +z 2 4( y + z 4 ) + 17 y 2 z 2
4 4( y 2 + z 2 ) + 2 yz
For yz = 0, the inequality is an equality. For yz = 0, using the substitution

y 2 + z2
t= , t ≥ 2,
yz

we may write the inequality as follows:

yz 5 yz( y 2 + z 2 9 yz
+ ≥ ,
y2 +z 2 4( y + z 2 )2 + 9 y 2 z 2
2 4( y 2 + z 2 ) + 2 yz
Highest Coefficient Cancellation Method for Nonnegative Variables 435

1 5t 9
+ ≥ ,
t 4t 2 + 9 4t + 2
(t − 2)(2t − 1) ≥ 0,
The equality holds for x = y = z, for x = 0 and y = z (or any cyclic permutation),
and for y = z = 0 (or any cyclic permutation).

Observation 1. Similarly, we can prove the following generalization:


• Let x, y, z be nonnegative real numbers. If k ≥ 4, then
 1 9(2k + 1)
≥ ,
k x 2 + y 2 + z2 9k(x 2 + y 2 + z 2 ) + 2(k − 1)2 (x y + yz + z x)

with equality for x = y = z, and for y = z = 0 (or any cyclic permutation). If k = 4,


then the equality holds also for x = 0 and y = z (or any cyclic permutation).
For
 (
f6 (x, y, z) = P (k y 2 + z 2 + x 2 )(kz 2 + x 2 + y 2 ) − 9(2k + 1) (k x 2 + y 2 + z 2 ),

P = 9k(x 2 + y 2 + z 2 ) + 2(k − 1)2 (x y + yz + z x),


we have
A = −9(2k + 1)(k − 1)3 .
Since A < 0, it suffices to show that the original inequality holds for y = z = 1 and
for x = 0. In these cases, the inequality respectively reduces to

(k − 1)2 (x − 1)2 [(2k + 1)x + k − 4] ≥ 0

and f (t) ≥ 0, where

f (t) = 2(2k + 1)t 2 − 9(k + 1)t + 2(k − 1)2 , t ≥ 2.

We have
f (t) ≥ f (2) ≥ 0
because

f (t) − f (2) = (t − 2)[2(2k + 1)t − k − 5] ≥ (t − 2)[4(2k + 1) − k − 5] ≥ 0,

f (2) = 2(k − 4)(k + 1) ≥ 0.

Observation 2. Also, the following generalization is valid:


• Let x, y, z be nonnegative real numbers. If 1 < k ≤ 4, then
 1 9(k + 5)
≥ ,
k x 2 + y 2 + z2 (8 + 11k − k2 )(x 2 + y 2 + z 2 ) + 2(k − 1)2 (x y + yz + z x)
436 Vasile Cîrtoaje

with equality for x = y = z, and for x = 0 and y = z (or any cyclic permutation). If
k = 4, then the equality holds also for y = z = 0 (or any cyclic permutation).
For
 (
f6 (x, y, z) = P (k y 2 + z 2 + x 2 )(kz 2 + x 2 + y 2 ) − 9(k + 5) (k x 2 + y 2 + z 2 ),

P = (8 + 11k − k2 )(x 2 + y 2 + z 2 ) + 2(k − 1)2 (x y + yz + z x),


we have
A = −9(k + 5)(k − 1)3 .
Since A < 0, it suffices to show that the original inequality holds for y = z = 1 and
for x = 0. In these cases, the inequality respectively reduces to

(k − 1)2 x(x − 1)2 [(4 − k)x + 2k + 10] ≥ 0

and
(k − 1)2 (t − 2)[2(4 − k)t 2 + 18t − (k − 1)2 ] ≥ 0,
where t ≥ 2.

P 4.27. If x, y, z are nonnegative real numbers, no two of which are zero, then
2 2 2 45
+ + ≥ .
x 2 + y 2 y 2 + z2 z2 + x 2 4(x 2 + y 2 + z 2 ) + x y + yz + z x

(Vasile C., 2011)

Solution. Write the inequality as f6 (x, y, z) ≥ 0, where


 (
f6 (x, y, z) = 2P (x 2 + y 2 )(x 2 + z 2 ) − 45 ( y 2 + z 2 ),

P = 4(x 2 + y 2 + z 2 ) + x y + yz + z x.
The highest coefficient
+ A of f6 (x, y, z) is equal with the highest coefficient of the
product −45 ( y 2 + z 2 ). Since

y 2 + z 2 = −x 2 + p2 − 2q,

we have
A = −45(−1)3 = 45.
In addition,

f6 (x, 1, 1) = 2(4x 2 + 2x + 9)[(x 2 + 1)2 + 4(x 2 + 1)] − 90(x 2 + 1)2


= 4x(x 2 + 1)(2x 3 + x 2 − 8x + 5) = 4x(x 2 + 1)(x − 1)2 (2x + 5) ≥ 0
Highest Coefficient Cancellation Method for Nonnegative Variables 437

and
f6 (0, y, z) ≥ 0.
The last inequality is true if the original inequality holds for x = 0. Thus, we need
to show that
2 2 2 45
+ + ≥ ,
y 2 z2 y 2 + z2 4( y 2 + z 2 ) + yz
which can be written as

2( y 2 + z 2 ) 2 45
+ 2 ≥ ,
y 2z2 y + z2 4( y 2 + z 2 ) + yz

2( y 2 + z 2 ) 2 yz 45 yz
+ 2 ≥ .
yz y + z2 4( y 2 + z 2 ) + yz
Using the notation
y 2 + z2
t= , t ≥ 2,
yz
the inequality becomes
2 45
2t + ≥ ,
t 4t + 1
(t − 2)(8t + 18t − 1) ≥ 0.

First Solution. Since the inequality is an equality for x = y = z = 1 and for x = 0


and y = z, we apply Corollary 1 for

x 2 (x − 1)4
Eα,β (x) = f0,−2 (x) = .
81

Since
x(x − 1)2 f (x)
f6 (x, 1, 1) − Af0,−2 (x) = ,
9
where

f (x) =36(x 2 + 1)(2x + 5) − 5x(x − 1)2


≥ 36(x − 1)2 (2x + 5) − 5x(x − 1)2
= (x − 1)2 (67x + 180) > 0.

the condition (a) in Corollary 1 is satisfied.


The condition (b) in Corollary 1 is satisfied if f6 (x, 1, 1) ≥ 45x 2 for x ≥ 4. This
is true if
4(x 2 + 1)(x − 1)2 (2x + 5) ≥ 45x.
Since x 2 + 1 ≥ 2x, it suffices to show that

8(x − 1)2 (2x + 5) ≥ 45.


438 Vasile Cîrtoaje

which is clearly true for x ≥ 4.


The condition (c) in Corollary 1 is satisfied because f6 (0, y, z) ≥ 0 for y, z ≥ 0.
The equality holds for x = y = z, and for x = 0 and y = z (or any cyclic
permutation).

Second Solution. Apply Theorem 3. Since the conditions (a) and (c) are satisfied,
we only need to show that

4Ax(x − 1)3
f6 (x, 1, 1) ≥
27
for x ≥ 1. We have

4Ax(x − 1)3 4x(x − 1)2 f (x)


f6 (x, 1, 1) − = ,
27 3
where

f (x) = 3(x 2 + 1)(2x + 5) − 5(x − 1) ≥ 6(2x + 5) − 5(x − 1) = 7(x + 5) > 0.

Observation 1. Similarly, we can prove the following generalization:

• Let x, y, z be nonnegative real numbers. If 0 ≤ k < 1, then


 1 9(k + 5)
≥ ,
k x 2 + y 2 + z2 (8 + 11k − k2 )(x 2 + y 2 + z 2 ) + 2(k − 1)2 (x y + yz + z x)

with equality for x = y = z, and for x = 0 and y = z (or any cyclic permutation).

As shown in Observation 2 from the preceding P 4.26, for


 (
f6 (x, y, z) = P(x, y, z) (k y 2 +z 2 + x 2 )(kz 2 + x 2 + y 2 )−9(k +5) (k x 2 + y 2 +z 2 ),

P(x, y, z) = (8 + 11k − k2 )(x 2 + y 2 + z 2 ) + 2(k − 1)2 (x y + yz + z x),


we have
A = 9(k + 5)(1 − k)3 > 0,
P(x, 1, 1) = (8 + 11k − k2 )x 2 + 4(k − 1)2 x + 18(k + 1),

f6 (x, 1, 1)
= P(x, 1, 1)[x 2 + k + 1 + 2(k x 2 + 2)] − 9(k + 5)(x 2 + k + 1)(k x 2 + 2)
x2 + k + 1
= 2(1 − k)2 x(x − 1)2 [(4 − k)x + 2k + 10] ≥ 0,

and
f6 (0, y, z) ≥ 0.
Highest Coefficient Cancellation Method for Nonnegative Variables 439

Since the conditions (a) and (c) in Theorem 3 are satisfied, it suffices to show that

4Ax(x − 1)3
f6 (x, 1, 1) ≥
27
for x ≥ 1. We have
4Ax(x − 1)3 2(1 − k)2 x(x − 1)2 f (x)
f6 (x, 1, 1) − = ,
27 3
where

f (x) = 3(x 2 + k + 1)[(4 − k)x + 2k + 10] − 2(k + 5)(1 − k)(x − 1)


≥ 3(k + 2)[(4 − k)x + 2k + 10] − 2(k + 5)(1 − k)(x − 1)
= (14 + 14k − k2 )x + 70 + 34k + 4k2
≥ (14 + 14k − k2 ) + 70 + 34k + 4k2
= 3(28 + 16k + k2 ) > 0.

Observation 2. Having in view Observation 1 above and Observation 2 from the


preceding P 4.26, it follows that the concerned inequality holds for 0 ≤ k ≤ 4.

P 4.28. If x, y, z are nonnegative real numbers, no two of which are zero, then
 1 18
≥ .
2 y 2 + yz + 2z 2 5(x 2 + y 2 + z 2 + x y + yz + z x)

(Vasile C., 2009)

Solution. Write the inequality as f6 (x, y, z) ≥ 0, where


 (
f6 (x, y, z) = 5P(x, y, z) (2x 2 + x y +2 y 2 )(2x 2 + xz+2z 2 )−18 (2 y 2 + yz+2z 2 ),

P(x, y, z) = x 2 + y 2 + z 2 + x y + yz + z x.
Since
2 y 2 + yz + 2z 2 = −2x 2 + yz + 2(p2 − 2q),
f6 (x, y, z) has the same highest coefficient A as
(
P3 (x, y, z) = −18 (−2x 2 + yz),

that is
A = P3 (1, 1, 1) = −18(−2 + 1)3 = 18.
We have
P(x, 1, 1) = x 2 + 2x + 3,
440 Vasile Cîrtoaje

f6 (x, 1, 1)
= 5(x 2 + 2x + 3)[(2x 2 + x + 2) + 10] − 90(2x 2 + x + 2)
2x 2 + x + 2
= 5x(2x 3 + 5x 2 − 16x + 9) = 5x(x − 1)2 (2x + 9),

f6 (x, 1, 1) = 5x(x − 1)2 (2x 2 + x + 2)(2x + 9) ≥ 0.


Also,
f6 (0, y, z) ≥ 0.
This inequality is true if the original inequality holds for x = 0. Thus, we need to
show that
1 1 1 18
+ + ≥ ,
2 y 2 + yz + 2z 2 2 y 2 2z 2 5( y 2 + z 2 + yz)
which can be rewritten as
yz y 2 + z2 18 yz
+ ≥ .
2( y 2 + z ) + yz
2 2 yz 5( y 2 + z 2 + yz)

Using the substitution


y 2 + z2
t= , t ≥ 2,
yz
the inequality becomes
1 t 18
+ ≥ ,
2t + 1 2 5(t + 1)
10t 3 + 15t 2 − 57t − 26 ≥ 0,
(t − 2)(10t 2 + 35t + 13) ≥ 0.

First Solution. We apply Corollary 1 for

(x − 1)4 x 2
Eα,β (x) = f0,−2 (x) = .
81

Condition (a). Since


2(x − 1)4 x 2
Af0,−2 (x) = ,
9
x(x − 1)2 f (x)
f6 (x, 1, 1)−Af0,−2 (x) = , f (x) = 45(2x 2 +x+2)(2x+9)−2x(x−1)2 ,
9
we need to show that f (x) ≥ 0 for x ∈ [0, 4]. This follows immediately from

2x 2 + x + 2 ≥ 2(x − 1)2 ,

45(2x 2 + x + 2) > x.

Condition (b). Since

f6 (x, 1, 1) − Ax 2 = x g(x), g(x) = 5(x − 1)2 (2x 2 + x + 2)(2x + 9) − 18x,


Highest Coefficient Cancellation Method for Nonnegative Variables 441

we need to show that g(x) ≥ 0 for x ≥ 4, which is trivial.


Condition (c). This condition is satisfied because f6 (0, y, z) ≥ 0 for y, z ≥ 0.
The equality holds for x = y = z, and for x = 0 and y = z (or any cyclic
permutation).

Second Solution. Apply Theorem 3. Since the conditions (a) and (c) are satisfied,
we only need to show that the condition (b) is satisfied. Thus, we need to prove
that
4Ax(x − 1)3
f6 (x, 1, 1) ≥
27
for x ≥ 1. We have

4Ax(x − 1)3 x(x − 1)2 f (x)


f6 (x, 1, 1) − = ,
27 3
where

f (x) = 15(2x 2 + x + 2)(2x + 9) − 8(x − 1) ≥ 75(2x + 9) − 8(x − 1) = 7(x + 5) > 0.

Observation. Similarly, we can prove the following generalization:

• Let x, y, z be nonnegative real numbers, no two of which are zero. If −1 ≤ k ≤ 2,


then
 k+2 9(2k + 5)
≥ ,
y 2 + k yz + z 2 2(2 − k)(x 2 + y 2 + z 2 ) + (4k + 1)(x y + yz + z x)

with equality for x = y = z, and for x = 0 and y = z (or any cyclic permutation).

For
 (
f6 (x, y, z) = P(x, y, z) (x 2 +k x y+ y 2 )(x 2 +k xz+z 2 )−9(2k+5) ( y 2 +k yz+z 2 ),

P(x, y, z)
= 2(2 − k)(x 2 + y 2 + z 2 ) + (4k + 1)(x y + yz + z x),
k+2
we have
A = −9(2k + 5)(k − 1)3 ,
P(x, 1, 1)
= 2(2 − k)x 2 + 2(4k + 1)x + 9,
k+2
f6 (x, 1, 1) = 2(k + 2)(x 2 + k x + 1)x(x − 1)2 [(2 − k)x + (k + 1)(5 − k)].

Case 1: −1 ≤ k < 1. Since A > 0, we apply Corollary 1 for

(x − 1)4 x 2
Eα,β (x) = f0,−2 (x) = .
81
442 Vasile Cîrtoaje

Condition (a). We have


(2k + 5)(1 − k)3 (x − 1)4 x 2
Af0,−2 (x) = ,
9
x(x − 1)2 f (x)
f6 (x, 1, 1) − Af0,−2 (x) = ,
9
f (x) = 18(k + 2)(x 2 + k x + 1)[(2 − k)x + (k + 1)(5 − k)] − (2k + 5)(1 − k)3 x(x − 1)2 .
Since
(2 − k)x + (k + 1)(5 − k) ≥ (2 − k)x,
it suffices to show that x ≥ 4 involves

18(k + 2)(2 − k)(x 2 + k x + 1) ≥ (2k + 5)(1 − k)3 (x − 1)2 .

This is true because

x 2 + k x + 1 ≥ x 2 − x + 1 > (x − 1)2 ,

3(k + 2) ≥ 2k + 5,
4 ≥ (1 − k)2
and
3(2 − k) > 2(1 − k).
Condition (b). Since
f6 (x, 1, 1) − Ax 2 ≥ x 2 g(x),
g(x) = 2(k + 2)(2 − k)(x 2 + k x + 1)(x − 1)2 − 9(2k + 5)(1 − k)3 ,
we need to show that g(x) ≥ 0 for x ≥ 4. It suffices to show that g(4) ≥ 0, which
is true.
Condition (c). For x = 0, the original inequality becomes
 
k+2 1 1 9(2k + 5)
+ (k + 2) + ≥ ,
y 2 + k yz + z 2 y 2 z2 2(2 − k)( y 2 + z 2 ) + (4k + 1) yz
k+2 9(2k + 5)
+ (k + 2)t ≥ ,
t+k 2(2 − k)t + 4k + 1
(t − 2)h(t) ≥ 0,
where
h(t) = 2(4 − k2 )t 2 + (k + 2)(9 + 4k − 2k2 )t + 7k2 + 18k − 1,
h(t) ≥ h(2) = 67 + 52k − k2 − 4k3 > 0.

Case 2: 1 ≤ k ≤ 2. Since A ≤ 0, according to Theorem 1, we only need to show


that f6 (x, 1, 1) ≥ 0 and f6 (0, y, z) ≥ 0 for x, y, z ≥ 0, which are true.
Highest Coefficient Cancellation Method for Nonnegative Variables 443

P 4.29. If x, y, z are nonnegative real numbers, no two of which are zero, then
 x − (2 +
2)( y + z)

9(3 + 2 2)
+ ≥ 0.
( y + z)2 4(x + y + z)

(Vasile C., 2014)

Solution. Write the inequality as f6 (x, y, z) ≥ 0, where



(
f6 (x, y, z) = 4(x + y + z) f (x, y, z) + 9(3 + 2 2) ( y + z)2 ,


f (x, y, z) = x − (2 + 2)( y + z) (x + y)2 (x + z)2 .
Since
( y + z)2 = (p − x)2 = x 2 − 2p x + p2 ,
+
the product ( y + z)2 has the same highest coefficient A1 as x 2 y 2 z 2 , that is

A1 = 1.

Therefore, f6 (x, y, z) has the highest coefficient



A = 9(3 + 2 2) = 9(1 + 2)2 .

We have
f (x, 1, 1)



= (x − 4 − 2 2)(x + 1)2 − 8 (2 + 2)x + 1 + 2
(x + 1)2


= x 3 − 2(1 + 2)x 2 − (23 + 12 2)x − 12 − 10 2,

f6 (x, 1, 1) = 4(x + 2) f (x, 1, 1) + 36(3 + 2 2)(x + 1)4


= 4(x + 1)2 g(x),





g(x) =(x + 2) x 3 − 2(1 + 2)x 2 − (23 + 12 2)x − 12 − 10 2

+ 9(3 + 2 2)(x + 1)2





=x 4 − 2 2x 3 + 2 2x 2 − 2(2 − 2)x + 3 − 2 2

=(x − 1)2 (x − 2 + 1)2 ,

therefore

f6 (x, 1, 1) = 4(x + 1)2 (x − 1)2 (x − 2 + 1)2 .


Thus, we apply Theorem 2 for

4(x − 1)4 (x − 2 + 1)2


Eα,β (x) = Fγ,δ (x) = f
2−1,−2 (x) =
.
81(1 + 2)2
444 Vasile Cîrtoaje

The conditions (a) and (b) in Theorem 2 are satisfied if f6 (x, 1, 1) ≥ Af


2−1,−2 (x)
for x ≥ 0. We have

4(x − 1)4 (x − 2 + 1)2


Af
2−1,−2 (x) = ,
9

16(x − 1)2 (x − 2 + 1)2 (x + 2)(2x + 1)


f6 (x, 1, 1) − Af
2−1,−2 (x) = ≥ 0.
9
The condition (c) in Theorem 2 is satisfied if f6 (0, y, z) ≥ A fˆ
2−1,−2 ( y, z) for
y, z ≥ 0. We have



f (0, y, z) = −(2 + 2)( y + z) y 2 z 2 + ( y + z)2 y 3 + z 3 − (2 + 2) yz( y + z) ,

f (0, y, z)


= −(2 + 2) y 2 z 2 + ( y 2 + z 2 + 2 yz) y 2 + z 2 − (3 + 2) yz
y +z

= ( y 2 + z 2 )2 − (1 + 2)( y 2 + z 2 ) yz − (8 + 3 2) y 2 z 2 ,

f6 (0, y, z) = 4( y + z) f (0, y, z) + 9(3 + 2 2) y 2 z 2 ( y + z)2 ,

f6 (0, y, z) 


=4 ( y 2 + z 2 )2 − (1 + 2)( y 2 + z 2 ) yz − (8 + 3 2) y 2 z 2
( y + z) 2

+ 9(3 + 2 2) y 2 z 2

=4( y 2 + z 2 )2 − 4(1 + 2)( y 2 + z 2 ) yz + (6 2 − 5) y 2 z 2

and 
2
( y + z)2 2( y 2 + z 2 )2 − (3 + 2) yz

2−1,−2 ( y, z) =
,
81(1 + 2)2

2
( y + z)2 2( y 2 + z 2 )2 − (3 + 2) yz
A fˆ
2−1,−2 ( y, z) = .
9
Therefore,
8( y + z)2 g( y, z)
f6 (0, y, z) − A fˆ
2−1,−2 ( y, z) = ,
9
where

g( y, z) = 4( y 2 + z 2 )2 − (3 + 4 2) yz( y 2 + z 2 ) + (6 2 − 7) y 2 z 2



= ( y 2 + z 2 − 2 yz) 4( y 2 + z 2 ) − (4 2 − 5) yz + (3 − 2 2) y 2 z 2 ≥ 0.

x
The equality occurs for x = y = z, and for
= y = z (or any cyclic
2−1
permutation).
Highest Coefficient Cancellation Method for Nonnegative Variables 445

P 4.30. If x, y, z are nonnegative real numbers, no two of which are zero, then

6x − y − z 6 y − z − x 6z − x − y 18
+ + ≥ .
y 2 + z2 z2 + x 2 x2 + y2 x + y +z

(Vasile C., 2014)

Solution. Write the inequality as f6 (x, y, z) ≥ 0, where


(
f6 (x, y, z) = (x + y + z) f (x, y, z) − 18 ( y 2 + z 2 ),

f (x, y, z) = (6x − y − z)(x 2 + y 2 )(x 2 + z 2 ).
Since
y 2 + z 2 = −x 2 + p2 − 2q,
+
the product ( y 2 + z 2 ) has the highest coefficient A1 as (−x 2 )(− y 2 )(−z 2 ), that is

A1 = −1.

Therefore, f6 (x, y, z) has the highest coefficient

A = −18A1 = 18.

We have
f (x, 1, 1)
= 2(3x − 1)(x 2 + 1) + 4(5 − x) = 2(3x 3 − x 2 + x + 9),
x2 + 1

f6 (x, 1, 1) = (x + 2) f (x, 1, 1) − 36(x 2 + 1)2


 
= 2(x 2 + 1) (x + 2)(3x 3 − x 2 + x + 9) − 18(x 2 + 1)
= 2(x 2 + 1)x(3x 3 + 5x 2 − 19x + 11)
= 2x(x 2 + 1)(x − 1)2 (3x + 11) ≥ 0.

Also, we have
f6 (0, y, z) ≥ 0
for y, z ≥ 0. This is true if the original inequality holds for x = 0. Thus, we need
to show that
− y − z 6 y − z 6z − y 18
+ + ≥ ,
y 2 + z2 z2 y2 y +z
which is equivalent to

− y − z 6( y 3 + z 3 ) − yz( y + z) 18
+ ≥ ,
y 2 + z2 y 2z2 y +z

−1 6( y 2 + z 2 ) − 7 yz 18
+ ≥ ,
y2 +z 2 y 2z2 ( y + z)2
446 Vasile Cîrtoaje

− yz 6( y 2 + z 2 ) − 7 yz 18 yz
+ ≥ .
y +z
2 2 yz ( y + z)2
Using the substitution
y 2 + z2
t= , t ≥ 2,
yz
the inequality can be written as follows:
−1 18
+ 6t − 7 ≥ ,
t t +2
6t 3 + 5t 2 − 33t − 2 ≥ 0,
(t − 2)(6t 2 + 17t + 1) ≥ 0.

First Solution. We apply Theorem 3. The conditions (a) and (c) are clearly satis-
fied. The condition (b) is satisfied if
4Ax(x − 1)3
f6 (x, 1, 1) ≥
27
for x ≥ 1. We have
4Ax(x − 1)3 2x(x − 1)2 f (x)
f6 (x, 1, 1) − = ,
27 3
where

f (x) = 3(x 2 + 1)(3x + 11) − 4(x − 1) ≥ 6(3x + 11) − 4(x − 1) = 14(x + 5) > 0.

The equality occurs for x = y = z, and for x = 0 and y = z (or any cyclic
permutation).
Second Solution. We apply Corollary 1 for

(x − 1)4 x 2
Eα,β (x) = f0,−2 (x) = .
81
The condition (a) in Corollary 1 are satisfied if f6 (x, 1, 1) ≥ Af0,−2 (x) for x ∈
[0, 4]. We have
2(x − 1)4 x 2
Af0,−2 (x) = ,
9
2x(x − 1)2 g(x)
f6 (x, 1, 1) − Af0,−2 (x) = ≥ 0,
9
where
g(x) = 9(x 2 + 1)(3x + 11) − x(x − 1)2 .
Since x 2 + 1 ≥ (x − 1)2 , it suffices to show that

9(3x + 11) ≥ x,
Highest Coefficient Cancellation Method for Nonnegative Variables 447

which is clearly true.


The condition (b) in Corollary 1 is satisfied if f6 (x, 1, 1) ≥ Ax 2 for x ≥ 4. We
have
f6 (x, 1, 1) − Ax 2 = 2x g(x),
where

g(x) = (x 2 + 1)(x − 1)2 (3x + 11) − 9x ≥ x[2(x − 1)2 (3x + 11) − 9] ≥ 0.

The condition (c) in Corollary 1, namely f6 (0, y, z) ≥ 0 for y, z ≥ 0, is satisfied.


Observation 1. Similarly, we can prove the following generalization:
• Let x, y, z be nonnegative real numbers, no two of which are zero. If
3
−2 < k ≤ ,
2
then  (6 − 4k)x + (4k − 1)( y + z) 36(k + 1)
≥ ,
y 2 + k yz + z 2 (k + 2)(x + y + z)
with equality for x = y = z, and for x = 0 and y = z (or any cyclic permutation).
For
(
f6 (x, y, z) = (k + 2)(x + y + z) f (x, y, z) − 36(k + 1) ( y 2 + k yz + z 2 ),

where

f (x, y, z) = [(6 − 4k)x + (4k − 1)( y + z)](x 2 + k x y + y 2 )(x 2 + k xz + z 2 ),

we get
A = 36(k + 1)(1 − k)3 .
We have
f (x, 1, 1)
= [(3 − 2k)x + 4k − 1](x 2 + k x + 1) + (k + 2)[(4k − 1)x + 5]
2(x 2 + k x + 1)
= (3 − 2k)x 3 − (2k2 − 7k + 1)x 2 + (8k2 + 4k + 1)x + 9k + 9,

f6 (x, 1, 1) = (k + 2)(x + 2) f (x, 1, 1) − 36(k + 1)(k + 2)(x 2 + k x + 1)2


= 2(k + 2)x(x 2 + k x + 1)g(x),

g(x) = (3 − 2k)x 3 − (2k2 − 3k − 5)x 2 + (4k2 − 19)x + 11 − k − 2k2


= (x − 1)2 [(3 − 2k)x + 11 − k − 2k2 ],
therefore

f6 (x, 1, 1) = 2(k + 2)x(x 2 + k x + 1)(x − 1)2 [(3 − 2k)x + 11 − k − 2k2 ].


448 Vasile Cîrtoaje

Since 3 − 2k ≥ 0 and 11 − k − 2k2 > 0 for −2 < k ≤ 3/2, we have f6 (x, 1, 1) ≥ 0


for all x ≥ 0. Also, we have
f6 (0, y, z) ≥ 0
for y, z ≥ 0. To show this, we only need to prove the original inequality for x = 0.
Using the substitution
y 2 + z2
t= , t ≥ 2,
yz
the original inequality can be written as

(t − 2)(B1 t 2 + B2 t + B3 ) ≥ 0,

where

B1 = (k + 2)(6 − 4k), B2 = (k + 2)(17 − 2k − 4k2 ), B3 = 2 + 25k + 5k2 − 8k3 .

Since B1 ≥ 0 and

B3 ≥ 2 + 25k − 4k2 − 8k3 = (k + 2)(1 + 12k − 8k2 ),

we have

B1 t 2 + B2 t + B3 ≥ (2B1 + B2 )t + B3 = (k + 2)(29 − 10k − 4k2 )t + B3


≥ 2(k + 2)(29 − 10k − 4k2 ) + (k + 2)(1 + 12k − 8k2 )
= (k + 2)(59 − 8k − 16k2 ) > 8(k + 2)(6 − k − 2k2 )
= 8(k + 2)2 (3 − 2k) ≥ 0.

Case 1: k ∈ (−2, −1] ∪ [1, 3/2]. Since A ≤ 0, f6 (x, 1, 1) ≥ 0 and f6 (0, y, z) ≥ 0 for
all x, y, z ≥ 0, the conclusion follows by Theorem 1.
Case 2: k ∈ [−1, 1]. Since A ≥ 0, we apply Corollary 1 for

(x − 1)4 x 2
Eα,β (x) = f0,−2 (x) = .
81

The condition (a) in Corollary 1, namely f6 (x, 1, 1) ≥ Af0,−2 (x) for x ∈ [0, 4], is
satisfied if

9(k + 2)(x 2 + k x + 1)[(3 − 2k)x + 11 − k − 2k2 ] ≥ 2(k + 1)(1 − k)3 x(x − 1)2 .

Since
9 > 8 ≥ 2(1 − k)2 ,
k + 2 > k + 1,
x + k x + 1 ≥ x 2 − x + 1 ≥ (x − 1)2 ,
2

(3 − 2k)x + 11 − k − 2k2 > (3 − 2k)x ≥ (1 − k)x,


Highest Coefficient Cancellation Method for Nonnegative Variables 449

the conclusion follows.


The condition (b) in Corollary 1, namely f6 (x, 1, 1) ≥ Ax 2 for x ≥ 4, is satisfied
if

(k + 2)(x 2 + k x + 1)(x − 1)2 [(3 − 2k)x + 11 − k − 2k2 ] ≥ 18(k + 1)(1 − k)3 x.

Since
k + 2 > k + 1,
x + k x + 1 ≥ x 2 − x + 1 ≥ x,
2

(x − 1)2 [(3 − 2k)x + 11 − k − 2k2 ] ≥ 9[4(3 − 2k) + 11 − k − 2k2 ] = 9(23 − 9k − 2k2 ),


it suffices to show that
23 − 9k − 2k2 ≥ 2(1 − k)3 .
Indeed,

23 − 9k − 2k2 − 2(1 − k)3 ≥ 23 − 9k − 2k2 − 8(1 − k) = 15 − k − 2k2 > 0.

The condition (c) in Corollary 1 is satisfied because f6 (0, y, z) ≥ 0 for y, z ≥ 0.

Observation 2. For k = −1, k = 0, k = 1/4, k = 1 and k = 3/2, we get the


following particular inequalities from the inequality in Observation 1:

 2x − y − z
≥ 0,
y 2 − yz + z 2
 6x − y − z 18
≥ ,
y 2 + z2 x + y +z
 x 1
≥ ,
4 y 2 + yz + z 2 x + y +z
 2x + 3 y + 3z 24
≥ ,
y 2 + yz + z 2 x + y +z
 y +z 18
≥ .
2 y 2 + 3 yz + 2z 2 7(x + y + z)

P 4.31. If x, y, z are nonnegative real numbers, no two of which are zero, then
 2x − 3 y − 3z 6
+ ≥ 0.
y 2 + 4 yz + z 2 x + y + z

(Vasile C., 2014)


450 Vasile Cîrtoaje

Solution. Write the inequality as f6 (x, y, z) ≥ 0, where


(
f6 (x, y, z) = (x + y + z) f (x, y, z) + 6 ( y 2 + 4 yz + z 2 ),

f (x, y, z) = (2x − 3 y − 3z)(x 2 + 4x y + y 2 )(x 2 + 4xz + z 2 ).
Since
y 2 + 4 yz + z 2 = 4 yz − x 2 + p2 − 2q,
+
the product ( y + 4 yz + z 2 ) has the same highest coefficient as
2

(
P3 (x, y, z) = (4 yz − x 2 ),

that is
A1 = P3 (1, 1, 1) = 27.
Therefore, f6 (x, y, z) has the highest coefficient

A = 6A1 = 162.

We have
f (x, 1, 1)
= 2(x − 3)(x 2 + 4x + 1) − 12(3x + 1) = 2(x 3 + x 2 − 29x − 9),
x 2 + 4x + 1
f6 (x, 1, 1) = (x + 2) f (x, 1, 1) + 36(x 2 + 4x + 1)2
 
= 2(x 2 + 4x + 1) (x + 2)(x 3 + x 2 − 29x − 9) + 18(x 2 + 4x + 1)
= 2(x 2 + 4x + 1)x(x 3 + 3x 2 − 9x + 5)
= 2x(x − 1)2 (x + 5)(x 2 + 4x + 1) ≥ 0.

Also, we have
f6 (0, y, z) ≥ 0
for y, z ≥ 0. This is true if the original inequality holds for x = 0. Thus, we need
to show that
−3( y + z) 2 y − 3z 2z − 3 y 6
+ + + ≥ 0,
y 2 + 4 yz + z 2 z2 y2 y +z

which is equivalent to

−3( y + z) 2( y 3 + z 3 ) − 3 yz( y + z) 6
+ + ≥ 0,
y2 + z + 4 yz
2 y 2z2 y +z

−3 2( y + z)2 − 9 yz 6
+ + ≥ 0,
( y + z) + 2 yz
2 y 2z2 ( y + z)2
−3 yz 2( y + z)2 − 9 yz 6 yz
+ + ≥ 0.
( y + z) + 2 yz
2 yz ( y + z)2
Highest Coefficient Cancellation Method for Nonnegative Variables 451

Using the substitution


( y + z)2
t= , t ≥ 4,
yz
the inequality can be written as follows:

−3 6
+ 2t − 9 + ≥ 0,
t +2 t

2t 3 − 5t 2 − 15t + 12 ≥ 0,
(t − 4)(2t 2 + 3t − 3) ≥ 0.

First Solution. We apply Theorem 3. The conditions (a) and (c) are satisfied. The
condition (b) is satisfied if

4Ax(x − 1)3
f6 (x, 1, 1) ≥
27
for x ≥ 1. We have

4Ax(x − 1)3
f6 (x, 1, 1) − = 2x(x − 1)2 f (x),
27
where

f (x) = (x+5)(x 2 +4x+1)−12(x−1) ≥ 6(x 2 +4x+1)−12(x−1) = 6(x 2 +2x+3) > 0.

The equality occurs for x = y = z, and for x = 0 and y = z (or any cyclic
permutation).

Second Solution. We apply Corollary 1 for

(x − 1)4 x 2
Eα,β (x) = f0,−2 (x) = .
81

The condition (a) in Corollary 1 are satisfied if f6 (x, 1, 1) ≥ Af0,−2 (x) for x ∈
[0, 4]. We have
2(x − 1)4 x 2
Af0,−2 (x) = ,
9
f6 (x, 1, 1) − Af0,−2 (x) = 2x(x − 1)2 g(x),
where
g(x) = (x 2 + 4x + 1)(x + 5) − x(x − 1)2 .
Since x 2 + 4x + 1 ≥ (x − 1)2 and x + 5 > x, it follows that g(x) > 0.
The condition (b) in Corollary 1 is satisfied if f6 (x, 1, 1) ≥ Ax 2 for x ≥ 4. We
have
f6 (x, 1, 1) − Ax 2 = 2x g(x),
452 Vasile Cîrtoaje

where

g(x) = (x 2 + 4x + 1)(x − 1)2 (x + 5) − 81x > x[x(x − 1)2 (x + 5) − 81]


≥ x(4 · 9 · 9 − 81) > 0.

The condition (c) in Corollary 1 is satisfied since f6 (0, y, z) ≥ 0 for y, z ≥ 0.

Observation. Similarly, we can prove the following generalization:

• Let x, y, z be nonnegative real numbers, no two of which are zero. If


89 − 1
k≥ ≈ 2.1085,
4
then  (4k − 6)x + (1 − 4k)( y + z) 36(k + 1)
+ ≥ 0,
y 2 + k yz + z 2 (k + 2)(x + y + z)
with equality for x = y = z, and for x = 0 and y = z (or any cyclic permutation).

Write the inequality as f6 (x, y, z) ≥ 0, where


(
f6 (x, y, z) = (k + 2)(x + y + z) f (x, y, z) + 36(k + 1) ( y 2 + k yz + z 2 ),

f (x, y, z) = [(4k − 6)x + (1 − 4k)( y + z)](x 2 + k x y + y 2 )(x 2 + k xz + z 2 ).
Since f6 (x, y, z) is the opposite of f6 (x, y, z) in Observation 1 from the preceding P
4.30, we have
A = 36(k + 1)(k − 1)3 ,
f6 (x, 1, 1) = 2(k + 2)x(x 2 + k x + 1)(x − 1)2 [(2k − 3)x + 2k2 + k − 11].
Since A > 0, we apply Corollary 1 for

(x − 1)4 x 2
Eα,β (x) = f0,−2 (x) = .
81

The condition (a) in Corollary 1, namely f6 (x, 1, 1) ≥ Af0,−2 (x) for x ∈ [0, 4], is
satisfied if

9(k + 2)(x 2 + k x + 1)[(2k − 3)x + 2k2 + k − 11] ≥ 2(k + 1)(k − 1)3 x(x − 1)2 .

Since
9 ≥ (x − 1)2 ,
k + 2 > k + 1,
x + k x + 1 > k x,
2

(2k − 3)x + 2k2 + k − 11 ≥ 4(2k − 3) + 2k2 + k − 11 = 2k2 + 9k − 23,


Highest Coefficient Cancellation Method for Nonnegative Variables 453

it suffices to show that

k(2k2 + 9k − 23) ≥ 2(k − 1)3 .

Indeed,

k(2k2 + 9k − 23) − 2(k − 1)3 = 15k2 − 29k + 2 > 15k2 − 30k = 15k(k − 2) > 0.

The condition (b) in Corollary 1, namely f6 (x, 1, 1) ≥ Ax 2 for x ≥ 4, is satisfied


if

(k + 2)(x 2 + k x + 1)(x − 1)2 [(2k − 3)x + 2k2 + k − 11] ≥ 18(k + 1)(k − 1)3 x.

Since
k + 2 > k + 1,
x + k x + 1 > k x,
2

(x − 1)2 [(2k − 3)x + 2k2 + k − 11] ≥ 9[4(3 − 2k) + 11 − k − 2k2 ] = 9(2k2 + 9k − 23),
it suffices to show that

k(2k2 + 9k − 23) ≥ 2(k − 1)3 .

Indeed,

k(2k2 + 9k − 23) − 2(k − 1)3 = 15k2 − 29k + 2 > 15k2 − 30k = 15k(k − 2) > 0.

The condition (c) in Corollary 1, namely f6 (0, y, z) ≥ 0 for y, z ≥ 0, is satisfied if


the original inequality holds for x = 0. Thus, we need to show that

(t − 2)(B1 t 2 + B2 t + B3 ) ≥ 0,

where
y 2 + z2
t= ≥ 2,
yz
B1 = (k + 2)(4k − 6), B2 = (k + 2)(4k2 + 2k − 17), B3 = 8k3 − 5k2 − 25k − 2.
Since t ≥ 2, B1 > 0 and B2 ≥ 0, we have

B1 t 2 + B2 t + B3 ≥ 4B1 + B3 = (8k3 − 50) + k(11k − 17) > 0.

P 4.32. If x, y, z are nonnegative real numbers, no two of which are zero, then
 7x + 4 y + 4z 27
≥ .
4x 2 + yz x + y +z
(Vasile C., 2014)
454 Vasile Cîrtoaje

Solution. Write the inequality as f6 (x, y, z) ≥ 0, where


(
f6 (x, y, z) = (x + y + z) f (x, y, z) + 27 (4x 2 + yz),

f (x, y, z) = (7x + 4 y + 4z)(4 y 2 + z x)(4z 2 + x y).
The product (
P3 (x, y, z) = (4x 2 + yz)
has the highest coefficient

A1 = P3 (1, 1, 1) = 125.

Therefore, f6 (x, y, z) has the highest coefficient

A = −27A1 < 0.

Since A < 0, we only need to show that f6 (x, 1, 1) ≥ 0 and f6 (0, y, z) ≥ 0 for
x, y, z ≥ 0. The first inequality is true if the original inequality holds for y = z = 1.
So, we need to show that

7x + 8 2(4x + 11) 27
+ ≥ ,
4x 2 + 1 x +4 x +2
which is equivalent to

x(32x 3 + 51x 2 − 198x + 115) ≥ 0,

x(x − 1)2 (32x + 115) ≥ 0.


The second inequality is true if the original inequality holds for x = 0. We need to
show that
4( y + z) 7 y + 4z 7z + 4 y 27
+ + ≥ ,
yz 4 y2 4z 2 y +z
which can be rewritten as follows:

4( y + z) 4( y 3 + z 3 ) + 7 yz( y + z) 27
+ ≥ ,
yz 4 y 2z2 y +z

4 4( y 2 + z 2 ) + 3 yz 27
+ ≥ ,
yz 4 y 2z2 ( y + z)2
4( y + z)2 − 5 yz 27 yz
4+ ≥ .
4 yz ( y + z)2
Substituting
( y + z)2
t= , t ≥ 4,
yz
Highest Coefficient Cancellation Method for Nonnegative Variables 455

the inequality becomes


4t − 5 27
4+ ≥ ,
4 t
(t − 4)(4t + 27) ≥ 0.

The equality occurs for x = y = z, and for x = 0 and y = z (or any cyclic
permutation).

P 4.33. If x, y, z are nonnegative real numbers, then


 9x − 2 y − 2z 3
≤ .
7x 2 + 8 yz x + y +z

(Vasile C., 2014)

Solution. Write the inequality as f6 (x, y, z) ≥ 0, where


(
f6 (x, y, z) = 3 (7x 2 + 8 yz) − (x + y + z) f (x, y, z),

f (x, y, z) = (9x − 2 y − 2z)(7 y 2 + 8z x)(7z 2 + 8x y).
The product (
P3 (x, y, z) = (7x 2 + 8 yz)
has the highest coefficient

A1 = P3 (1, 1, 1) = 153 .

Therefore, f6 (x, y, z) has the highest coefficient

A = 3A1 = 3 · 153 .

We have
f (x, 1, 1)
= (9x − 4)(8x + 7) + 2(2x − 7)(7x 2 + 8)
8x + 7
= −(28x 3 − 170x 2 + x − 84),

f6 (x, 1, 1) = 3(7x 2 + 8)(8x + 7)2 − (x + 2) f (x, 1, 1)


 
= (8x + 7) 3(7x 2 + 8)(8x + 7) + (x + 2)(28x 3 − 170x 2 + x − 84)
= 2x(8x + 7)(14x 3 + 27x 2 − 96x + 55)
= 2x(x − 1)2 (8x + 7)(14x + 55) ≥ 0.

Also, we have
f6 (0, y, z) ≥ 0
456 Vasile Cîrtoaje

for y, z ≥ 0. This is true if the original inequality holds for x = 0. Thus, we need
to show that
y + z 2z − 9 y 2 y − 9z 3
+ + + ≥ 0,
4 yz 7 y2 7z 2 y +z
which is equivalent to

y + z 2( y 3 + z 3 ) − 9 yz( y + z) 3
+ + ≥ 0,
4 yz 7 y 2z2 y +z

1 2( y 2 + z 2 ) − 11 yz 3
+ + ≥ 0,
4 yz 7 y 2z2 ( y + z)2
1 2( y + z)2 − 15 yz 3 yz
+ + ≥ 0,
4 7 yz ( y + z)2
Using the substitution
( y + z)2
t= , t ≥ 4,
yz
the inequality can be written as follows:
1 2t − 15 3
+ + ≥ 0,
4 7 t
8t 2 − 53t + 84 ≥ 0,
(t − 4)(8t − 21) ≥ 0.

First Solution. We apply Theorem 3. The conditions (a) and (c) are satisfied. The
condition (b) is satisfied if
4Ax(x − 1)3
f6 (x, 1, 1) ≥
27
for x ≥ 1. We have
4Ax(x − 1)3
f6 (x, 1, 1) − = 2x(x − 1)2 f (x),
27
where

f (x) = (8x + 7)(14x + 55) − 750(x − 1) = 112x 2 − 212x + 1135


> 106x 2 − 212x + 106 = 106(x − 1)2 ≥ 0.

The equality occurs for x = y = z, and for x = 0 and y = z (or any cyclic
permutation).
Second Solution. We apply Corollary 1 for

(x − 1)4 x 2
Eα,β (x) = f0,−2 (x) = .
81
Highest Coefficient Cancellation Method for Nonnegative Variables 457

The condition (a) in Corollary 1 are satisfied if f6 (x, 1, 1) ≥ Af0,−2 (x) for x ∈
[0, 4]. We have
Af0,−2 (x) = 125(x − 1)4 x 2 ,
f6 (x, 1, 1) − Af0,−2 (x) = x(x − 1)2 g(x),
where

g(x) = 2(8x + 7)(14x + 55) − 125x(x − 1)2


≥ 2(8x + 7)(14x + 55) − 500(x − 1)2
> 2(8x + 6)(15x + 50) − 500(x − 1)2
 
= 20 (4x + 3)(3x + 10) − 25(x − 1)2
= 20[5 + x(99 − 13x)] > 0.

The condition (b) in Corollary 1 is satisfied if f6 (x, 1, 1) ≥ Ax 2 for x ≥ 4. We


have
f6 (x, 1, 1) − Ax 2 = x g(x),
where
g(x) = 2(8x + 7)(x − 1)2 (14x + 55) − 3 · 153 x.
Since
2(8x + 7)(x − 1)2 > 15x(x − 1)2 ≥ 135x,
we get

g(x) > 135x(14x + 55) − 3 · 153 x = 135x(14x + 55 − 75) > 0.

The condition (c) in Corollary 1 is satisfied because f6 (0, y, z) ≥ 0 for y, z ≥ 0.

P 4.34. If x, y, z are nonnegative real numbers, then


 y +z 2
≥ .
7x 2 + y 2 + z 2 x + y +z
(Vasile C., 2014)
Solution. Write the inequality as f6 (x, y, z) ≥ 0, where
 (
f6 (x, y, z) = (x + y +z) ( y +z)(7 y 2 +z 2 + x 2 )(7z 2 + x 2 + y 2 )−2 (7x 2 + y 2 +z 2 ).

Since
7x 2 + y 2 + z 2 = 6x 2 + p2 − 2q,
+
the product (x 2 +4 y 2 +4z 2 ) has the same highest coefficient A1 as (6x 2 )(6 y 2 )(6z 2 ),
that is A1 = 216. Therefore, f6 (x, y, z) has the highest coefficient

A = −2A1 = −432.
458 Vasile Cîrtoaje

According to Theorem 1, we only need to show that f6 (x, 1, 1) ≥ 0 and f6 (0, y, z) ≥


0 for x, y, z ≥ 0. The first condition is true if the original inequality holds for
y = z = 1. Thus, we need to show that
2 2(x + 1) 2
+ 2 ≥ ,
7x 2 + 2 x +8 x +2
which is equivalent to
(x − 1)2 (11x + 2) ≥ 0.
The second condition is true if the original inequality holds for x = 0. Thus, we
need to show that
y +z z y 2
+ + ≥ ,
y 2 + z 2 7 y 2 + z 2 7z 2 + y 2 y +z
which is equivalent to
y +z 7( y 3 + z 3 ) + yz( y + z) 2
+ ≥ ,
y 2 + z2 7( y 4 + z 4 ) + 50 y 2 z 2 y +z
1 7( y 2 + z 2 ) − 6 yz 2
+ ≥ .
+z
y2 2 7( y 4 + z 4 ) + 50 y 2 z 2 ( y + z)2
For yz = 0, the inequality is an equality. For yz = 0, we write the inequality as
yz 7 yz( y 2 + z 2 ) − 6 y 2 z 2 2 yz
+ ≥ 2 .
y2 +z 2 7( y 2 + z 2 )2 + 36 y 2 z 2 y + z 2 + 2 yz
Substituting
y 2 + z2
t= , t ≥ 2,
yz
the inequality becomes as follows:
1 7t − 6 2
+ ≥ ,
t 7t 2 + 36 t +2
11t 2 − 24t + 36 ≥ 0.
We have
11t 2 − 24t + 36 > 6t 2 − 24t + 24 = 6(t − 2)2 ≥ 0.
The equality occurs for x = y = z.

P 4.35. If x, y, z are nonnegative real numbers, then


 7x − 2 y − 2z 3
≥ .
x 2 + 4 y 2 + 4z 2 x + y +z
(Vasile C., 2014)
Highest Coefficient Cancellation Method for Nonnegative Variables 459

Solution. Write the inequality as f6 (x, y, z) ≥ 0, where


(
f6 (x, y, z) = (x + y + z) f (x, y, z) − 3 (x 2 + 4 y 2 + 4z 2 ),

f (x, y, z) = (7x − 2 y − 2z)( y 2 + 4z 2 + 4x 2 )(z 2 + 4x 2 + 4 y 2 ).
Since
x 2 + 4 y 2 + 4z 2 = −3x 2 + 4(p2 − 2q),
+
the product (x +4 y +4z 2 ) has the same highest coefficient A1 as (−3x 2 )(−3 y 2 )(−3z 2 ),
2 2

that is A1 = −27. Therefore, f6 (x, y, z) has the highest coefficient

A = −3A1 = 81.

We have
f (x, 1, 1)
= (7x − 4)(4x 2 + 5) + 2(5 − 2x)(x 2 + 8)
4x 2 + 5
= 3(8x 3 − 2x 2 + x + 20),

f6 (x, 1, 1) = (x + 2) f (x, 1, 1) − 3(x 2 + 8)(4x 2 + 5)2


 
= 3(4x 2 + 5) (x + 2)(8x 3 − 2x 2 + x + 20) − (x 2 + 8)(4x 2 + 5)
= 6x(4x 2 + 5)(2x 3 + 7x 2 − 20x + 11)
= 6x(4x 2 + 5)(x − 1)2 (2x + 11) ≥ 0.

Also, we have
f6 (0, y, z) ≥ 0
for y, z ≥ 0. This is true if the original inequality holds for x = 0. Thus, we need
to show that
−( y + z) 7 y − 2z 7z − 2 y 3
+ + ≥ ,
2( y 2 + z 2 ) y 2 + 4z 2 z 2 + 4 y 2 y +z
which is equivalent to

−( y + z) 26( y 3 + z 3 ) − yz( y + z) 3
+ ≥ ,
2( y + z )
2 2 4( y 4 + z 4 ) + 17 y 2 z 2 y +z

−1 26( y 2 + z 2 ) − 27 yz 3
+ ≥ .
2( y + z ) 4( y 2 + z 2 )2 + 9 y 2 z 2
2 2 ( y + z)2
If yz = 0, then the inequality is an equality. For yz = 0, write the inequality as

− yz 26 yz( y 2 + z 2 ) − 27 y 2 z 2 3 yz
+ ≥ 2 .
2( y + z )
2 2 4( y 2 + z 2 )2 + 9 y 2 z 2 y + z 2 + 2 yz
Using the substitution
y 2 + z2
t= , t ≥ 2,
yz
460 Vasile Cîrtoaje

the inequality becomes


−1 26t − 27 3
+ ≥ ,
2t 4t 2 + 9 t +2
8t 3 + 14t 2 − 57t − 6 ≥ 0,
(t − 2)(8t 2 + 30t + 3) ≥ 0.

First Solution. We apply Theorem 3. The conditions (a) and (c) are satisfied. The
condition (b) is satisfied if

4Ax(x − 1)3
f6 (x, 1, 1) ≥
27
for x ≥ 1. We have
4Ax(x − 1)3
f6 (x, 1, 1) − = 6x(x − 1)2 f (x),
27
where

f (x) = (4x 2 + 5)(2x + 11) − 2(x − 1)


≥ 9(2x + 11) − 2(x − 1) = 16x + 101 > 0.

The equality occurs for x = y = z, and for x = 0 and y = z (or any cyclic
permutation).
Second Solution. We apply Corollary 1 for

(x − 1)4 x 2
Eα,β (x) = f0,−2 (x) = .
81

The condition (a) in Corollary 1 are satisfied if f6 (x, 1, 1) ≥ Af0,−2 (x) for x ∈
[0, 4]. We have
Af0,−2 (x) = (x − 1)4 x 2 ,
f6 (x, 1, 1) − Af0,−2 (x) = x(x − 1)2 g(x),
where
g(x) = 6(4x 2 + 5)(2x + 11) − x(x − 1)2 .
We have g(x) > 0 since 4x 2 + 5 > (x − 1)2 and 2x + 11 > x.
The condition (b) in Corollary 1 is satisfied if f6 (x, 1, 1) ≥ Ax 2 for x ≥ 4. We
have
f6 (x, 1, 1) − Ax 2 = 3x g(x),
where
g(x) = 2(4x 2 + 5)(x − 1)2 (2x + 11) − 27x > 0.

The condition (c) in Corollary 1, namely f6 (0, y, z) ≥ 0 for y, z ≥ 0, is satisfied.


Highest Coefficient Cancellation Method for Nonnegative Variables 461

P 4.36. If x, y, z are nonnegative real numbers, no two of which are zero, then
 2x 2 + yz 9 31(x − y)2 ( y − z)2 (z − x)2
≥ + .
y +z
2 2 2 2(x 2 + y 2 )( y 2 + z 2 )(z 2 + x 2 )

(Vasile C., 2014)

Solution. Write the inequality as f6 (x, y, z) ≥ 0, where


 ( (
f6 (x, y, z) = 2 (2x 2 + yz)(x 2 + y 2 )(x 2 + z 2 ) − 9 ( y 2 + z 2 ) − 31 ( y − z)2 .

Since
y 2 + z 2 = −x 2 + p2 − 2q,
f6 (x, y, z) has the same highest coefficient A as

2 (2x 2 + yz)(−z 2 )(− y 2 ) − 9(−x 2 )(− y 2 )(−z 2 ) − 31(x − y)2 ( y − z)2 (z − x)2 ,

that is
A = 2(6 + 3) + 9 − 31(−27) = 32 · 27.
Also,

f6 (x, 1, 1) =2[(2x 2 + 1)(x 2 + 1)2 + 4(2 + x)(x 2 + 1)] − 18(x 2 + 1)2


= 4x(x 2 + 1)(x 3 − 3x + 2) = 4x(x 2 + 1)(x − 1)2 (x + 2).

Thus, we apply Corollary 1 for

(x − 1)4 x 2
Eα,β (x) = f0,−2 (x) = .
81

The condition (a) in Corollary 1 are satisfied if f6 (x, 1, 1) ≥ Af0,−2 (x) for x ∈
[0, 4]. We have
32(x − 1)4 x 2
Af0,−2 (x) = ,
3
4x(x − 1)2 g(x)
f6 (x, 1, 1) − Af0,−2 (x) = ,
3
where
g(x) = 3(x 2 + 1)(x + 2) − 8x(x − 1)2 .
For 0 ≤ x ≤ 1, we have x + 2 ≥ 3x, hence

g(x) ≥ 9x(x 2 + 1) − 8x(x − 1)2 ≥ 8x(x 2 + 1) − 8x(x − 1)2 = 16x 2 ≥ 0.

For 1 ≤ x ≤ 4, we have

3(x 2 + 1)(x + 2) = 3x 3 + 6x 2 + 3x + 6 > 3x 3 + 6x 2 ,


462 Vasile Cîrtoaje

hence
g(x) > x[3x 2 + 6x − 8(x − 1)2 ] = x(5x − 2)(4 − x) ≥ 0.

The condition (b) in Corollary 1 is satisfied if f6 (x, 1, 1) ≥ Ax 2 for x ≥ 4. We


have
f6 (x, 1, 1) − Ax 2 = 4x g(x),

where
g(x) = (x 2 + 1)(x − 1)2 (x + 2) − 216x.

Since 2(x + 2) ≥ 3x, we have

2g(x) ≥ 3x(x 2 + 1)(x − 1)2 − 432x


= 3x[(x 2 + 1)(x − 1)2 − 144]
≥ 3x(17 · 9 − 144] = 27x > 0.

The condition (c) in Corollary 1 is satisfied if the original inequality holds for
x = 0. Thus, we need to show that
 
yz y 2 z2 9 31( y − z)2
+2 + 2 ≥ + .
y 2 + z2 z2 y 2 2( y 2 + z 2 )

Using the substitution


y 2 + z2
t= , t ≥ 2,
yz
we may write the inequality as follows:

1 9 31(t − 2)
+ 2(t 2 − 2) ≥ + ,
t 2 2t

(t − 2)2 (t + 4) ≥ 0.

The equality occurs for x = y = z, and for x = 0 and y = z (or any cyclic
permutation).

P 4.37. If x, y, z are nonnegative real numbers, no two of which are zero, then
 2x 2 − yz 9(x − y)2 ( y − z)2 (z − x)2
≥3+ 2 .
y − yz + z
2 2 (x − x y + y 2 )( y 2 − yz + z 2 )(z 2 − z x + x 2 )

(Vasile C., 2014)


Highest Coefficient Cancellation Method for Nonnegative Variables 463

Solution. Write the inequality as f6 (x, y, z) ≥ 0, where


 ( (
f6 (x, y, z) = (2x 2 − yz)(x 2 −x y+ y 2 )(x 2 −xz+z 2 )−3 ( y 2 − yz+z 2 )−9 ( y−z)2 .

Since
y 2 − yz + z 2 = −x 2 − yz + p2 − 2q,
f6 (x, y, z) has the same highest coefficient A as

P2 (x, y, z) − 3P3 (x, y, z) − 9(x − y)2 ( y − z)2 (z − x)2 ,

where
 (
P2 (x, y, z) = (2x 2 − yz)(−z 2 − x y)(− y 2 − xz), P3 (x, y, z) = (−x 2 − yz),

that is

A = P2 (1, 1, 1)−3P3 (1, 1, 1)−9(−27) = 3(2−1)(−1−1)2 −3(−1−1)3 −9(−27) = 279.

Also,

f6 (x, 1, 1) =[(2x 2 − 1)(x 2 − x + 1)2 + 2(2 − x)(x 2 − x + 1)] − 3(x 2 − x + 1)2


= 2x(x 2 − x + 1)(x 3 − x 2 − x + 1) = 2x(x 2 − x + 1)(x − 1)2 (x + 1).

Thus, we apply Corollary 1 for

(x − 1)4 x 2
Eα,β (x) = f0,−2 (x) = .
81

The condition (a) in Corollary 1 are satisfied if f6 (x, 1, 1) ≥ Af0,−2 (x) for x ∈
[0, 4]. We have
31(x − 1)4 x 2
Af0,−2 (x) = ,
9
x(x − 1)2 g(x)
f6 (x, 1, 1) − Af0,−2 (x) = ,
9
where
g(x) = 18(x 2 − x + 1)(x + 1) − 31x(x − 1)2 .
For 0 ≤ x ≤ 1, we have x + 1 ≥ 2x, hence

g(x) ≥ 36x(x 2 − x + 1) − 31x(x − 1)2 ≥ 31x(x 2 − x + 1) − 31x(x − 1)2 = 31x 2 ≥ 0.

For 1 ≤ x ≤ 4, we have

18(x 2 − x + 1)(x + 1) = 18x 3 + 18 > 18x 3 ,

hence

g(x) > x[18x 2 − 31(x − 1)2 ] ≥ x[18x 2 − 32(x − 1)2 ] = 2x(7x − 4)(4 − x) ≥ 0.
464 Vasile Cîrtoaje

The condition (b) in Corollary 1 is satisfied if f6 (x, 1, 1) ≥ Ax 2 for x ≥ 4. We


have
f6 (x, 1, 1) − Ax 2 = x g(x),
where
g(x) = 2(x 2 − x + 1)(x − 1)2 (x + 1) − 279x.
Since
4(x 2 − x + 1) − 13x = (x − 4)(4x − 1) ≥ 0,
we have

2g(x) ≥ 13x(x − 1)2 (x + 1) − 558x


 
= x 13(x − 1)2 (x + 1) − 558
≥ x(13 · 9 · 5 − 558) = 27x > 0.

The condition (c) in Corollary 1 is satisfied if the original inequality holds for
x = 0. Thus, we need to show that
 2 
− yz y z2 9( y − z)2
+ 2 + ≥3+ 2 .
y − yz + z
2 2 z 2 y 2 y − yz + z 2
Using the substitution
y 2 + z2
t= , t ≥ 2,
yz
we may write the inequality as
−1 9(t − 2)
+ 2(t 2 − 2) ≥ 3 + ,
t −1 t −1
(t − 2)2 (t + 3) ≥ 0.
The equality occurs for x = y = z, and for x = 0 and y = z (or any cyclic
permutation).

Observation 1. Similarly, we can prove the following generalization:


• Let x, y, z be nonnegative real numbers, no two of which are zero. If

−2 < k ≤ 2,
then
 2x 2 + (2k + 1) yz 3(2k + 3) (8k2 + 30k + 31)(x − y)2 ( y − z)2 (z − x)2
≥ + + .
y2 + k yz + z2 k+2 (k + 2) ( y 2 + k yz + z 2 )
with equality for x = y = z, and for x = 0 and y = z (or any cyclic permutation).

For

f6 (x, y, z) = (k + 2) [2x 2 + (2k + 1) yz](x 2 + k x y + y 2 )(x 2 + k xz + z 2 )
Highest Coefficient Cancellation Method for Nonnegative Variables 465
(
−3(2k + 3) ( y 2 + k yz + z 2 ) − (8k2 + 30k + 31)(x − y)2 ( y − z)2 (z − x)2 ,
we have
A = 9(k + 2)(2k2 + 19k + 48),
f6 (x, 1, 1) = 2(k + 2)x(x 2 + k x + 1)(x − 1)2 (x + k + 2).

Observation 2. For k = 0 and k = −1, the inequality in Observation 1 leads to the


particular inequalities in P 4.36 and P 4.37. For k = −1/2 and k = 1, we get to the
particular inequalities
 x2 24(x − y)2 ( y − z)2 (z − x)2
≥1+ + ,
2 y 2 − yz + 2z 2 (2 y 2 − yz + 2z 2 )
 2x 2 + 3 yz 23(x − y)2 ( y − z)2 (z − x)2
≥5+ + .
y + yz + z
2 2 ( y 2 + yz + z 2 )

P 4.38. If x, y, z are nonnegative real numbers, no two of which are zero, then
 x y − yz + z x 3 5(x − y)2 ( y − z)2 (z − x)2
≥ + .
y 2 + z2 2 2(x 2 + y 2 )( y 2 + z 2 )(z 2 + x 2 )

(Vasile C., 2014)

Solution. Write the inequality as f6 (x, y, z) ≥ 0, where


 ( (
f6 (x, y, z) = 2 (x y − yz + z x)(x 2 + y 2 )(x 2 + z 2 ) − 3 ( y 2 + z2) − 5 ( y − z)2 .

Since
x y − yz + z x = −2 yz + q, y 2 + z 2 = −x 2 + p2 − 2q,
f6 (x, y, z) has the same highest coefficient A as

2 (−2 yz)(−z 2 )(− y 2 ) − 3(−x 2 )(− y 2 )(−z 2 ) − 5(x − y)2 ( y − z)2 (z − x)2 ,

that is
A = −12 + 3 − 5(−27) = 126.
Also,

f6 (x, 1, 1) =2[(2x − 1)(x 2 + 1)2 + 4(x 2 + 1)] − 6(x 2 + 1)2


= 4x(x 2 + 1)(x − 1)2 .

Thus, we may apply Corollary 1 (for Eα,β = f0,−2 ) or Theorem 3. Since the last
method is more simple, we will apply it.
466 Vasile Cîrtoaje

The condition (a) in Theorem 3, namely f6 (x, 1, 1) ≥ 0 for 0 ≤ x ≤ 1, is clearly


satisfied.
4Ax(x − 1)3
The condition (b) in Theorem 3 is satisfied if f6 (x, 1, 1) ≥ for x ≥ 1.
27
We have
4Ax(x − 1)3 56x(x − 1)3
= ,
27 3
4Ax(x − 1)3 4x(x − 1)2 f (x)
f6 (x, 1, 1) − = ,
27 3
where
 
7 2 2
f (x) = 3(x 2 + 1) − 14(x − 1) = 3x 2 − 14x + 17 = 3 x − + > 0.
3 3

The condition (c) in Theorem 3 is satisfied if the original inequality holds for
x = 0. Thus, we need to show that

− yz y 2 + z2 3 5( y − z)2
+ ≥ + .
y 2 + z2 yz 2 2( y 2 + z 2 )

Using the substitution


y 2 + z2
t= , t ≥ 2,
yz
we may write the inequality as follows:

−1 3 5(t − 2)
+t≥ + ,
t 2 2t

(t − 2)2 ≥ 0.

The equality occurs for x = y = z, and for x = 0 and y = z (or any cyclic
permutation).

P 4.39. If x, y, z are nonnegative real numbers, no two of which are zero, then
 x y − 2 yz + z x 3(x − y)2 ( y − z)2 (z − x)2
≥ 2 .
y − yz + z
2 2 (x − x y + y 2 )( y 2 − yz + z 2 )(z 2 − z x + x 2 )

(Vasile C., 2014)

Solution. Write the inequality as f6 (x, y, z) ≥ 0, where


 (
f6 (x, y, z) = (x y − 2 yz + z xz)(x 2 − x y + y 2 )(x 2 − xz + z 2 ) − 3 ( y − z)2 .
Highest Coefficient Cancellation Method for Nonnegative Variables 467

Since

x y − 2 yz + z xz = −3 yz + q, x 2 − x y + y 2 = −z 2 − x y + p2 − 2q,

f6 (x, y, z) has the same highest coefficient A as



(−3 yz)(−z 2 − x y)(− y 2 − xz) − 3(x − y)2 ( y − z)2 (z − x)2 ,

that is
A = 3(−3)(−1 − 1)2 − 3(−27) = 45.
Also,

f6 (x, 1, 1) =[(2x − 2)(x 2 − x + 1)2 + 2(1 − x)(x 2 − x + 1)]


= 2x(x 2 − x + 1)(x − 1)2 .

Thus, we apply Corollary 1 for

(x − 1)4 x 2
Eα,β (x) = f0,−2 (x) = .
81
The condition (a) in Corollary 1 are satisfied if f6 (x, 1, 1) ≥ Af0,−2 (x) for x ∈
[0, 4]. We have
5(x − 1)4 x 2
Af0,−2 (x) = ,
9
x(x − 1)2 g(x)
f6 (x, 1, 1) − Af0,−2 (x) = ,
9
where
g(x) = 18(x 2 − x + 1) − 5x(x − 1)2 .
For 0 ≤ x ≤ 1, we have 18(x 2 − x + 1) ≥ 18(1 − x) ≥ 5(1 − x), hence

g(x) ≥ 5(1 − x) − 5x(1 − x) = 5(1 − x)2 ≥ 0.

For 1 ≤ x ≤ 4, we have 18(x 2 − x + 1) ≥ 18x(x − 1) ≥ 15x(x − 1), hence

g(x) ≥ 15x(x − 1) − 5x(x − 1)2 = 5x(x − 1)(4 − x) ≥ 0.

The condition (b) in Corollary 1 is satisfied if f6 (x, 1, 1) ≥ Ax 2 for x ≥ 4. We


have
f6 (x, 1, 1) − Ax 2 = x g(x),
where
g(x) = 2(x 2 − x + 1)(x − 1)2 − 45x.
We have

g(x) ≥ 2(x 2 − x)(x − 1)2 − 45x = x[2(x − 1)3 − 45]


≥ x(54 − 45) > 0.
468 Vasile Cîrtoaje

The condition (c) in Corollary 1 is satisfied if the original inequality holds for
x = 0. Thus, we need to show that
−2 yz y 2 + z2 3( y − z)2
+ ≥ 2 .
y 2 − yz + z 2 yz y − yz + z 2
Using the substitution
y 2 + z2
t= , t ≥ 2,
yz
we may write the inequality as
−2 3(t − 2)
+t≥ ,
t −1 t −1
(t − 2)2 ≥ 0.
The equality occurs for x = y = z, and for x = 0 and y = z (or any cyclic
permutation).

Observation 1. Similarly, we can prove the following generalization:


• Let x, y, z be nonnegative real numbers, no two of which are zero. If
−2 < k ≤ 2,
then
 x( y + z) + (k − 1) yz 3(k + 1) (k2 + 3k + 5)(x − y)2 ( y − z)2 (z − x)2
≥ + + .
y + k yz + z
2 2 k+2 (k + 2) ( y 2 + k yz + z 2 )
with equality for x = y = z, and for x = 0 and y = z (or any cyclic permutation).

For

f6 (x, y, z) =(k + 2) [x( y + z) + (k − 1) yz](x 2 + k x y + y 2 )(x 2 + k xz + z 2 )
(
− 3(k + 1) ( y 2 + k yz + z 2 ) − (k2 + 3k + 5)(x − y)2 ( y − z)2 (z − x)2 ,
we have
A = 9(k + 2)(2k + 7),
f6 (x, 1, 1) = 2(k + 2)x(x 2 + k x + 1)(x − 1)2 .

Observation 2. For k = 0 and k = −1, the inequality in Observation 1 leads to the


particular inequalities in P 4.38 and P 4.39. For k = 1 and k = 2, we get to the
particular inequalities
 x( y + z) 3(x − y)2 ( y − z)2 (z − x)2
≥2+ 2 ,
y 2 + yz + z 2 (x + x y + y 2 )( y 2 + yz + z 2 )(z 2 + z x + x 2 )
   
 1 9 15 x − y 2 y − z 2  x − x 2
(x y + yz + z x) ≥ + .
(x + y) 2 4 4 x+y y +z z+x
Highest Coefficient Cancellation Method for Nonnegative Variables 469

P 4.40. If x, y, z are nonnegative real numbers, no two of which are zero, then
 x + 3 y + 3z 7(x + y + z)
≥ .
( y + 2z)(2 y + z) 3(x y + yz + z x)

(Vasile C., 2014)

Solution. Write the inequality as f7 (x, y, z) ≥ 0, where



f7 (x, y, z) =3(x y + yz + z x) (x + 3 y + 3z)(2x 2 + 5x y + 2 y 2 )(2x 2 + 5xz + 2z 2 )
(
− 7(x + y + z) (2 y 2 + 5 yz + 2z 2 ).

Since
2 y 2 + 5 yz + 2z 2 = 5 yz − 2x 2 + 2(p2 − 2q),
+
the product (2 y 2 + 5 yz + 2z 2 ) has the same highest coefficient A1 as
(
P3 (x, y, z) = (5 yz − 2x 2 ),

that is
A1 = P3 (1, 1, 1) = 27.
Therefore, f7 (x, y, z) ≥ 0 has the highest polynomial

A(p, q) = −7(x + y + z)(27) = −189p ≤ 0.

According to Corollary 2, we only need to show that

f7 (x, 1, 1) ≥ 0

and
f7 (0, y, z) ≥ 0
for x, y, z ≥ 0. The first inequality is true if the original inequality holds for y =
z = 1. So, we need to show that

x +6 2(3x + 4) 7(x + 2)
+ ≥ ,
9 (x + 2)(2x + 1) 3(2x + 1)

which is equivalent to
x(x − 1)2 ≥ 0.
The second inequality is true if the original inequality holds for x = 0. Thus, we
need to prove that

3( y + z) y + 3z 3 y + z 7( y + z)
+ + ≥ ,
( y + 2z)(2 y + z) 2z 2 2 y2 3 yz
470 Vasile Cîrtoaje

which is equivalent to

3( y + z) ( y + z)3 7( y + z)
+ ≥ ,
( y + 2z)(2 y + z) 2 y 2z2 3 yz

3 ( y + z)2 7
+ ≥ ,
( y + 2z)(2 y + z) 2 y 2z2 3 yz
3 3( y 2 + z 2 ) − 8 yz
+ ≥ 0,
2( y 2 + z 2 ) + 5 yz 6 y 2z2
6( y 2 + z 2 )2 − ( y 2 + z 2 ) yz − 22 y 2 z 2 ≥ 0,
( y − z)2 [6( y 2 + z 2 ) + 11 yz] ≥ 0.

The equality occurs for x = y = z, and for x = 0 and y = z (or any cyclic
permutation).

P 4.41. If x, y, z are nonnegative real numbers, no two of which are zero, then
 9x − 5 y − 5z 3(x + y + z)
+ ≥ 0.
2 y 2 − 3 yz + 2z 2 x y + yz + z x

(Vasile C., 2014)

Solution. Write the inequality as f7 (x, y, z) ≥ 0, where



f7 (x, y, z) =3(x y + yz + z x) (9x − 5 y − 5z)(2x 2 − 3x y + 2 y 2 )(2x 2 − 3xz + 2z 2 )
(
+ 3(x + y + z) (2 y 2 − 3 yz + 2z 2 ).

Since
2 y 2 − 3 yz + 2z 2 = −3 yz − 2x 2 + 2(p2 − 2q),
+
the product (2 y 2 − 3 yz + 2z 2 ) has the same highest coefficient A1 as
(
P3 (x, y, z) = (−3 yz − 2x 2 ),

that is
A1 = P3 (1, 1, 1) = −125.
Therefore, f7 (x, y, z) ≥ 0 has the highest polynomial

A(p, q) = 3(x + y + z)(−125) = −375p < 0.

According to Corollary 2, we only need to prove the original inequality for y = z = 1


and for x = 0.
Highest Coefficient Cancellation Method for Nonnegative Variables 471

Case 1: y = z = 1. We need to show that

2(4 − 5x) 3(x + 2)


9x − 10 + + ≥ 0,
2x 2 − 3x + 2 2x + 1
which is equivalent to
x(x − 1)2 ≥ 0.

Case 2: x = 0. We need to show that


−5( y + z) 9 y − 5z 9z − 5 y 3( y + z)
+ + + ≥ 0,
2 y 2 − 3 yz + 2z 2 2z 2 2 y2 yz

which is equivalent to

−5( y + z) ( y + z)(9 y 2 + 9z 2 − 14 yz) 3( y + z)


+ + ≥ 0,
2 y2− 3 yz + 2z 2 2 y 2z2 yz

−5 9 y 2 + 9z 2 − 14 yz 3
+ + ≥ 0,
2 y2 − 3 yz + 2z 2 2 y 2z2 yz
−5 9( y 2 + z 2 ) − 8 yz
+ ≥ 0,
2( y 2 + z 2 ) − 3 yz 2 y 2z2
18( y 2 + z 2 )2 − 43( y 2 + z 2 ) + 14 y 2 z 2 ≥ 0,
( y − z)2 (18 y 2 + 18z 2 − 7 yz) ≥ 0.

The equality occurs for x = y = z, and for x = 0 and y = z (or any cyclic
permutation).

Observation 1. Similarly, we can prove the following generalization:


• Let x, y, z be nonnegative real numbers, no two of which are zero. If

1 − 17 1 + 17
k ∈ [a, −1] ∪ [1, b], a= ≈ −1.56155, b = ≈ 2.56155 ,
2 2
then  (3 − k)x + (k − 1)( y + z) 3(k + 1) x + y +z
≥ · ,
y 2 + k yz + z 2 k+2 x y + yz + z x
with equality for x = y = z, and for x = 0 and y = z (or any cyclic permutation).

For
(
f7 (x, y, z) = (k + 2)(x y + yz + z x) f (x, y, z) − 3(k + 1)(x + y + z) ( y 2 + k yz + z 2 ),

where

f (x, y, z) = [(3 − k)x + (k − 1)( y + z)](x 2 + k x y + y 2 )(x 2 + k xz + z 2 ),
472 Vasile Cîrtoaje

we have
A = −3(k + 1)(k − 1)3 p ≤ 0,
f7 (x, 1, 1) = 2(k + 2)x(x − 1)2 (x 2 + k x + 1)[(3 − k)x − k2 + k + 4] ≥ 0,
f7 (0, y, z) = yz( y + z)( y − z)2 [(k + 2)(3 − k)( y − z)2 + (13 + 7k − k2 − k3 ) yz] ≥ 0.

Observation 2. For k = 2, the inequality from Observation 1 turns into the well-
known Iran inequality:
 1 9
≥ .
( y + z)2 4(x y + yz + z x)

P 4.42. If x, y, z are nonnegative real numbers, no two of which are zero, then
 3x − y − z 3(x + y + z)
≥ .
y 2 + z2 2(x y + yz + z x)

(Vasile C., 2014)

Solution. Write the inequality as f7 (x, y, z) ≥ 0, where


(
f7 (x, y, z) = 2(x y + yz + z x) f (x, y, z) − 3(x + y + z) ( y 2 + z 2 ),

f (x, y, z) = (3x − y − z)(x 2 + y 2 )(x 2 + z 2 ).
Since
y 2 + z 2 = −x 2 + p2 − 2q,
+
the product ( y + z ) has the same highest coefficient A1 as (−x 2 )(− y 2 )(−z 2 ),
2 2

that is A1 = −1. Therefore, f7 (x, y, z) ≥ 0 has the highest polynomial

A(p, q) = −3pA1 = 3p ≥ 0, A(x + 2, 2x + 1) = 3(x + 2).

We have
 
f7 (x, 1, 1) = 2(2x + 1) (3x − 2)(x 2 + 1)2 + 4(2 − x)(x 2 + 1) − 6(x + 2)(x 2 + 1)2
= 4x(x − 1)2 (x 2 + 1)(3x + 4).

On the other hand, for x = 0, the original inequality becomes

− y − z 3 y − z 3z − y 3( y + z)
+ + ≥ ,
y 2 + z2 z2 y2 2 yz

− yz 3( y 2 + z 2 ) − 4 yz 3
+ ≥ .
y +z
2 2 yz 2
Highest Coefficient Cancellation Method for Nonnegative Variables 473

Using the substitution


y 2 + z2
t= , t ≥ 2,
yz
the inequality becomes
1 3
− + 3t − 4 ≥ ,
t 2
(t − 2)(6t + 1) ≥ 0.

First Solution. Apply Theorem 6. The conditions (a) and (c) are satisfied. In what
concerns the condition (b), we have

4A(x + 2, 2x + 1)x(x − 1)3 4(x + 2)x(x − 1)3


= ,
27 9

4A(x + 2, 2x + 1)x(x − 1)3 4x(x − 1)2 g(x)


f7 (x, 1, 1) − = ,
27 9
where
g(x) = 9(x 2 + 1)(3x + 4) − (x + 2)(x − 1).
For x ≥ 1, we get

g(x) > (x 2 + 1)(3x + 4) − (x + 2)(x − 1)


≥ 7(x 2 + 1) − (x + 2)(x − 1) = 6x 2 − x + 3 > 0.

The equality occurs for x = y = z, and for x = 0 and y = z (or any cyclic
permutation).
Second Solution. Apply Corollary 3 for

x 2 (x − 1)4
Eα,β (x) = f0,−2 (x) = .
81

Condition (a). It suffices to show that f7 (x, 1, 1) ≥ A(x + 2, 2x + 1) f0,−2 (x) for
0 ≤ x ≤ 4. We have

x 2 (x − 1)4 (x + 2)
A(x + 2, 2x + 1) f0,−2 (x) = ,
27
x(x − 1)2 f (x)
f7 (x, 1, 1) − A(x + 2, 2x + 1) f0,−2 (x) = ,
27
f (x) = 108(x 2 + 1)(3x + 4) − x(x − 1)2 (x + 2).
Since x 2 + 1 ≥ (x − 1)2 and 3x + 4 > 3x, we get

f (x) > 108(x − 1)2 (3x) − x(x − 1)2 (x + 2)


= x(x − 1)2 (322 − x) ≥ 0.
474 Vasile Cîrtoaje

Condition (b). It suffices to show that f7 (x, 1, 1) ≥ A(x + 2, 2x + 1)x 2 for x ≥ 4.


Since
f7 (x, 1, 1) ≥ 4x(x − 1)2 (x 2 + 1)(3x),
we get

f7 (x, 1, 1) − A(x + 2, 2x + 1)x 2 ≥ 4x(x − 1)2 (x 2 + 1)(3x) − 3(x + 2)x 2


= 3x 2 [4(x − 1)2 (x 2 + 1) − x − 2] > 0.

Condition (c). This condition is satisfied because the original inequality holds for
x = 0.

P 4.43. Let x, y, z be nonnegative real numbers, no two of which are zero. If



1 − 17 1 + 17
k ∈ [a, b], a= ≈ −1.56155, b = ≈ 2.56155 ,
2 2
then  (3 − k)x + (k − 1)( y + z) 3(k + 1) x + y +z
≥ · .
y 2 + k yz + z 2 k+2 x y + yz + z x
(Vasile C., 2014)
Solution. Write the inequality as f7 (x, y, z) ≥ 0, where
(
f7 (x, y, z) = (k + 2)(x y + yz + z x) f (x, y, z) − 3(k + 1)(x + y + z) ( y 2 + k yz + z 2 ),

f (x, y, z) = [(3 − k)x + (k − 1)( y + z)](x 2 + k x y + y 2 )(x 2 + k xz + z 2 ).
Since
y 2 + k yz + z 2 = −x 2 + k yz + p2 − 2q,
+
the product ( y + k yz + z 2 ) has the same highest coefficient A1 as
2

(
P3 (x, y, z) = (−x 2 + k yz),

that is
A1 = P3 (x1, 1, 1) = (k − 1)3 .
Therefore, f7 (x, y, z) has the highest polynomial

A(p, q) = −3(k + 1)pA1 = 3(1 + k)(1 − k)3 p.

We have
f (x, 1, 1)
= [(3 − k)x + 2k − 2](x 2 + k x + 1) + 2(k + 2)[(k − 1)x + 2]
x2 + kx + 1
= (3 − k)x 3 − (k2 − 5k + 2)x 2 + (2k2 − 3k + 3)x + 2k − 2,
Highest Coefficient Cancellation Method for Nonnegative Variables 475

f7 (x, 1, 1) = (k + 2)(2x + 1) f (x, 1, 1) − 3(k + 1)(k + 2)(x + 2)(x 2 + k x + 1)2


= (k + 2)(x 2 + k x + 1)g(x),
f (x, 1, 1)
g(x) = (2x + 1) 2 − 3(k + 1)(x + 2)(x 2 + k x + 1)
 x + kx + 1 
= 2x (3 − k)x 3 − (k2 − 3k + 2)x 2 + (2k2 − 3k − 5)x − k2 + k + 4
 
= 2x(x − 1)2 (3 − k)x − k2 + k + 4 ,
 
f7 (x, 1, 1) = 2(k + 2)x(x − 1)2 (x 2 + k x + 1) (3 − k)x − k2 + k + 4 ≥ 0.
and
 
f (0, y, z) = (k−1)( y+z) y 2 z 2 +( y 2 +k yz+z 2 ) (3 − k)( y 3 + z 3 ) + (k − 1) yz( y + z) ,

f (0, y, z)  
= (k − 1) y 2 z 2 + ( y 2 + z 2 + k yz) (3 − k)( y 2 + z 2 ) + (2k − 4) yz
y +z
= (3 − k)( y 2 + z 2 )2 − (k2 − 5k + 4)( y 2 + z 2 ) yz + (2k2 − 3k − 1) y 2 z 2 ,

f7 (0, y, z) f (0, y, z)
=(k + 2) − 3(k + 1) yz( y 2 + z 2 + k yz)
yz( y + z) y +z
=(k + 2)(3 − k)( y 2 + z 2 )2 − (k + 2)(k2 − 5k + 4)( y 2 + z 2 ) yz
+ (k + 2)(2k2 − 3k − 1) y 2 z 2 − 3(k + 1) yz( y 2 + z 2 + k yz)
=(k + 2)(3 − k)( y 2 + z 2 )2 − (k3 − 3k2 − 3k + 11)( y 2 + z 2 ) yz
+ 2(k3 − k2 − 5k − 1) y 2 z 2
 
=( y 2 + z 2 − 2 yz) (k + 2)(3 − k)( y 2 + z 2 ) − (k3 − k2 − 5k − 1) y 2 z 2
 
=( y − z)2 (k + 2)(3 − k)( y − z)2 + (13 + 7k − k2 − k3 ) yz ≥ 0.

Case 1: k ∈ [a, −1]∪[1, b]. Since A(p, q) ≤ 0, it suffices to show that f7 (x, 1, 1) ≥ 0
and f7 (0, y, z) ≥ 0 for x, y, z ≥ 0 (see Corollary 2). These condition are satisfied.
Case 2: k ∈ [−1, 1]. Since A(p, q) ≥ 0, we apply Corollary 3 for

x 2 (x − 1)4
Eα,β (x) = f0,−2 (x) = .
81
Condition (a). It suffices to show that f7 (x, 1, 1) ≥ A(x + 2, 2x + 1) f0,−2 (x) for
0 ≤ x ≤ 4, where

A(x + 2, 2x + 1) = 3(1 + k)(1 − k)3 (x + 2).

We have
(1 + k)(1 − k)3 x 2 (x − 1)4 (x + 2)
A(x + 2, 2x + 1) f0,−2 (x) = ,
27
476 Vasile Cîrtoaje

x(x − 1)2 f (x)


f7 (x, 1, 1) − A(x + 2, 2x + 1) f0,−2 (x) = ,
27
f (x) = 54(k +2)(x 2 + k x +1)[(3− k)x − k2 + k +4]−(1+ k)(1− k)3 x(x −1)2 (x +2).
Since
k + 2 > k + 1,

x + k x + 1 ≥ (x − 1)2 ,
2

(3 − k)x − k2 + k + 4 > (3 − k)x,

6 ≥ x + 2,
it suffices to show that
9 ≥ (1 − k)3 .
Actually, we have 8 ≥ (1 − k)3 , which is equivalent to 2 ≥ 1 − k.

Condition (b). It suffices to show that f7 (x, 1, 1) ≥ A(x + 2, 2x + 1)x 2 for x ≥ 4.


Since
A(x + 2, 2x + 1)x 2 = 3(1 + k)(1 − k)3 (x + 2)x 2 ,
we need to prove that

2(k + 2)(x 2 + k x + 1)(x − 1)2 [(3 − k)x − k2 + k + 4] ≥ 3(1 + k)(1 − k)3 (x + 2)x.

Since
k + 2 > k + 1,

x + k x + 1 > x + 2,
2

(x − 1)2 ≥ 9,

(3 − k)x − k2 + k + 4 > (3 − k)x,


it suffices to show that
6(3 − k) ≥ (1 − k)3 .
Indeed, we have

6(3 − k) ≥ 6(3 − 1) = 12 > 8 ≥ (1 − k)3 .

Condition (c). This condition is satisfied because f7 (0, y, z) ≥ 0 for all y, z ≥ 0.

The equality occurs for x = y = z, and for x = 0 and y = z (or any cyclic
permutation).
Highest Coefficient Cancellation Method for Nonnegative Variables 477

P 4.44. If x, y, z are nonnegative real numbers, no two of which are zero, then
 x + 13 y + 13z 27(x + y + z)
≥ .
y 2 + 4 yz + z 2 2(x y + yz + z x)
(Vasile C., 2014)

Solution. Write the inequality as f7 (x, y, z) ≥ 0, where


(
f7 (x, y, z) = 2(x y + yz + z x) f (x, y, z) − 27(x + y + z) ( y 2 + 4 yz + z 2 ),

f (x, y, z) = (x + 13 y + 13z)(x 2 + 4x y + y 2 )(x 2 + 4xz + z 2 ).
Since
y 2 + 4 yz + z 2 = −x 2 + 4 yz + p2 − 2q,
+
the product ( y + 4 yz + z 2 ) has the same highest coefficient A1 as
2

(
P3 (x, y, z) = (−x 2 + 4 yz),

that is
A1 = P3 (1, 1, 1) = 27.
Therefore, f7 (x, y, z) has the highest polynomial

A(p, q) = −27pA1 = −729p.

Since A(p, q) ≤ 0, according to Corollary 2, we only need to show that the original
inequality holds for y = z = 1, and for x = 0. For y = z = 1, the original inequality
becomes
x + 26 2(13x + 14) 27(x + 2)
+ 2 ≥ ,
6 x + 4x + 1 2(2x + 1)
x 4 − 10x 3 + 33x 2 − 40x + 16 ≥ 0,
(x − 1)2 (x − 4)2 ≥ 0.
Also, for x = 0, the original inequality becomes

13( y + z) y + 13z z + 13 y 27( y + z)


+ + ≥ ,
y 2 + 4 yz + z 2 z2 y2 2 yz

13( y + z) y 3 + z 3 + 13 yz( y + z) 27( y + z)


+ ≥ ,
y 2 + 4 yz + z 2 y 2z2 2 yz
13 yz y 2 + z 2 + 12 yz 27
+ ≥ ,
y 2 + z 2 + 4 yz yz 2
13 27
+ t + 12 ≥ ,
t +4 2
2t 2 + 5t + 14 ≥ 0,
478 Vasile Cîrtoaje

where
y 2 + z2
t= .
yz
The equality occurs for x = y = z, and for x/4 = y = z (or any cyclic permutation).

P 4.45. If x, y, z are nonnegative real numbers, no two of which are zero, then
 −x + y + z x + y +z
≥ .
2x 2 + yz x y + yz + z x

(Vasile C., 2014)

Solution. Write the inequality as f7 (x, y, z) ≥ 0, where


 (
f7 (x, y, z) = q (−x + y + z)(2 y 2 + z x)(2z 2 + x y) − (x + y + z) (2x 2 + yz).

Since (
P3 (x, y, z) = (2x 2 + yz)
has the highest coefficient

A1 = P3 (1, 1, 1) = 27,

f7 (x, y, z) has the highest polynomial

A(p, q) = −A1 (x + y + z) = −27p < 0.

According to Corollary 2, we only need to show that f7 (x, 1, 1) ≥ 0 and f7 (0, y, z) ≥


0 for x, y, z ≥ 0. The first condition is true if the original inequality holds for
y = z = 1. Thus, we need to show that
−x + 2 2x x +2
+ ≥ ,
2x 2 + 1 x + 2 2x + 1
which is equivalent to
x(x + 1)(x − 1)2 ) ≥ 0.
The second condition is true if the original inequality holds for x = 0. Thus, we
need to show that
y +z z− y y −z y +z
+ + ≥ ,
yz 2 y2 2z 2 yz
which is equivalent to
( y + z)( y − z)2 ≥ 0.

The equality occurs for x = y = z, and for x = 0 and y = z (or any cyclic
permutation).
Highest Coefficient Cancellation Method for Nonnegative Variables 479

Observation. Similarly, we can prove the following generalization:


• Let x, y, z be nonnegative real numbers, no two of which are zero. If k ≥ 1, then
 (k2 − 4k + 1)x + (2k − 1)( y + z) 3(k − 1)(x + y + z)
≥ ,
k x 2 + yz x y + yz + z x

with equality for x = y = z, and for x = 0 and y = z (or any cyclic permutation).

For k = 1, the following particular inequality holds:

2x − y − z 2 y − z − x 2z − x − y
+ + 2 ≤ 0.
x 2 + yz y2 + z x z +xy

P 4.46. If x, y, z are nonnegative real numbers, no two of which are zero, then
 11x − 3 y − 3z 3(x + y + z)
≤ .
2x 2 + 3 yz x y + yz + z x

(Vasile C., 2014)

Solution. Write the inequality as f7 (x, y, z) ≥ 0, where


(
f7 (x, y, z) = 3(x + y + z) (2x 2 + 3 yz) − (x y + yz + z x) f (x, y, z),

f (x, y, z) = (11x − 3 y − 3z)(2 y 2 + 3z x)(2z 2 + 3x y).
The product (
P3 (x, y, z) = (2x 2 + 3 yz)

has the highest coefficient

A1 = P3 (1, 1, 1) = 125.

Therefore, f6 (x, y, z) has the highest polynomial

A(p, q) = 3A1 (x + y + z) = 375p, A(x + 2, 2x + 1) = 375(x + 2).

We have

f (x, 1, 1)
= (11x − 6)(3x + 2) + 2(8 − 3x)(2x 2 + 3)
3x + 2
= −(12x 3 − 65x 2 + 14x − 36),
480 Vasile Cîrtoaje

f7 (x, 1, 1) = 3(x + 2)(2x 2 + 3)(3x + 2)2 − (2x + 1) f (x, 1, 1)


 
= (3x + 2) 3(x + 2)(2x 2 + 3)(3x + 2) + (2x + 1)(12x 3 − 65x 2 + 14x − 36)
= 14x(3x + 2)(3x 3 − 5x 2 + x + 1)
= 14x(3x + 2)(x − 1)2 (3x + 1).

For x = 0, the original inequality becomes

− y − z 11 y − 3z 11z − 3 y 3( y + z)
+ + ≤ ,
yz 2 y2 2z 2 yz
which is equivalent to

4( y + z) 3( y 3 + z 3 ) − 11 yz( y + z)
+ ≥ 0,
yz 2 y 2z2

( y + z)( y − z)2 ≥ 0.

First Solution. Apply Theorem 6. The conditions (a) and (c) are satisfied. In what
concerns the condition (b), we have

4A(x + 2, 2x + 1)x(x − 1)3 500(x + 2)x(x − 1)3


= ,
27 9
4A(x + 2, 2x + 1)x(x − 1)3 2x(x − 1)2 g(x)
f7 (x, 1, 1) − = ,
27 9
where
g(x) = 63(3x + 2)(3x + 1) − 250(x + 2)(x − 1).
For x ≥ 1, we get

g(x) > 50(3x + 2)(3x + 1) − 250(x + 2)(x − 1)


= 50(4x 2 + 4x + 12) > 0.

The equality occurs for x = y = z, and for x = 0 and y = z (or any cyclic
permutation).
Second Solution. Apply Corollary 3 for

(x − 1)4 x 2
Eα,β (x) = f0,−2 (x) = .
81

The condition (a) in Corollary 3 are satisfied if f7 (x, 1, 1) ≥ A(x+2, 2x+1) f0,−2 (x)
for x ∈ [0, 4]. We have

125(x + 2)x 2 (x − 1)2


A(x + 2, 2x + 1) f0,−2 (x) = ≤ 5(x + 2)x 2 (x − 1)2 ,
27
Highest Coefficient Cancellation Method for Nonnegative Variables 481

f7 (x, 1, 1) − A(x + 2, 2x + 1) f0,−2 (x) ≤ x(x − 1)2 g(x),


where
g(x) = 14(3x + 2)(3x + 1) − 5x(x + 2)(x − 1)2 .
Since
(3x + 2)(3x + 1) > 5x(x + 2),
we get

g(x) ≥ 70x(x + 2) − 5x(x + 2)(x − 1)2 = 5x(x + 2)[14 − (x − 1)2 ]


≥ 5x(x + 2)[9 − (x − 1)2 ] = 5x(x + 2)2 (4 − x) ≥ 0.

The condition (b) in Corollary 3 is satisfied if f7 (x, 1, 1) ≥ A(x + 2, 2x + 1)x 2 for


x ≥ 4. We have
f6 (x, 1, 1) − A(x + 2, 2x + 1)x 2 = x g(x),
where
g(x) = 14(3x + 2)(x − 1)2 (3x + 1) − 375x(x + 2).
Since
3x + 2 > x + 2, 3x + 1 > 3x,
we get
 
g(x) ≥ 3x(x + 2) 14(x − 1)2 − 125 ≥ 3x(x + 2)(14 · 9 − 125) > 0.

The condition (c) in Corollary 3 is satisfied if f7 (0, y, z) ≥ 0 for y, z ≥ 0. This is


true because the original inequality holds for x = 0.
Observation 1. Similarly, we can prove the following generalization:
1
• Let x, y, z be nonnegative real numbers, no two of which are zero. If ≤ k ≤ 1,
2
then
 (k2 − 4k + 1)x + (2k − 1)( y + z) 3(k − 1)(x + y + z)
≥ ,
k x 2 + yz x y + yz + z x
with equality for x = y = z, and for x = 0 and y = z (or any cyclic permutation).
For k = 1/2, the following particular inequality holds (see P 1.101 in Volume 2):
x y z x + y +z
+ + ≤ .
x 2 + 2 yz y 2 + 2z x z 2 + 2x y x y + yz + z x

Observation 2. Having in view Observation 1 above and Observation from the


1
preceding P 4.45, it follows that the concerned inequality holds for k ≥ .
2
482 Vasile Cîrtoaje

P 4.47. If x, y, z are nonnegative real numbers, no two of which are zero, then

1 1 1 1 2
+ + ≥ + .
2x 2 + yz 2 y 2 + z x 2z 2 + x y x y + yz + z x x 2 + y 2 + z 2

Solution. Write the inequality as f8 (x, y, z) ≥ 0, where


 (
f8 (x, y, z) = q(p2 − 2q) (2 y 2 + z x)(2z 2 + x y) − p2 (2x 2 + yz).

The polynomial of degree eight f8 (x, y, z) has the highest polynomial

A(p, q) = −A1 p2 ,

where A1 is the highest coefficient of the polynomial of degree six


(
P3 (x, y, z) = (2x 2 + yz).

Since
A1 = P3 (1, 1, 1) = 27,
we have
A(p, q) = −27p2 < 0.
According to Corollary 2, we only need to prove the original inequality for y = z = 1
and for x = 0. For y = z = 1, the original inequality becomes

1 2 1 2
+ ≥ + ,
2x 2 + 1 x + 2 2x + 1 x 2 + 2
which is equivalent to

2x(x − 1) 2x(x − 1)
≥ ,
(x + 2)(x 2 + 2) (2x + 1)(2x 2 + 1)

x(x 2 + x + 1)(x − 1)2 ≥ 0.


For x = 0, the original inequality becomes

1 1 2
+ ≥ 2 ,
2 y 2 2z 2 y + z2

which is equivalent to
( y 2 − z 2 )2 ≥ 0.

The equality occurs for x = y = z, and for x = 0 and y = z (or any cyclic
permutation).

Observation 1. Similarly, we can prove the following generalization:


• Let x, y, z be nonnegative real numbers, no two of which are zero.
Highest Coefficient Cancellation Method for Nonnegative Variables 483

(a) If k ≥ 2, then
 1 4(k + 1) 5k − 4
k(k + 1) ≥ + ;
x 2 + k yz x y + yz + z x x 2 + y 2 + z 2
1
(b) If ≤ k ≤ 2, then
8
 1 4k2 − 3k + 2 2(−2k2 + 6k − 1)
k(k + 1) ≥ + .
x2 + k yz x y + yz + z x x 2 + y 2 + z2

Observation 2. From the inequalities in Observation 1, we get the following par-


ticular inequalities:
2 2 2 1 1
+ + ≥ + ,
x 2 + 8 yz y 2 + 8z x z 2 + 8x y x y + yz + z x x 2 + y 2 + z 2
1 1 1 2 1
+ + ≥ + ,
x 2 + 2 yz y 2 + 2z x z 2 + 2x y x y + yz + z x x 2 + y 2 + z 2
2 2 2 3 6
+ + ≥ + ,
x 2 + yz y 2 + z x z 2 + x y x y + yz + z x x 2 + y 2 + z 2
1 1 1 1 2
+ + ≥ + ,
2x 2 + yz 2 y 2 + z x 2z 2 + x y x y + yz + z x x 2 + y 2 + z 2
5 5 5 6 3
+ + ≥ + .
4x 2 + yz 4 y 2 + z x 4z 2 + x y x y + yz + z x x 2 + y 2 + z 2

P 4.48. If x, y, z are nonnegative real numbers, no two of which are zero, then

x( y + z) y(z + x) z(x + y) x 2 + y 2 + z2
+ 2 + 2 ≤ .
x + 5 yz y + 5z x z + 5x y
2 x y + yz + z x
(Vasile C., 2012)

Solution. Write the inequality as f8 (x, y, z) ≥ 0, where


( 
f8 (x, y, z) = (x 2 + y 2 +z 2 ) (x 2 +5 yz)−(x y+ yz+z x) x( y+z)( y 2 +5z x)(z 2 +5x y).

Since x( y + z) = − yz + q, the polynomial of degree eight f8 (x, y, z) has the highest


polynomial
A(p, q) = A1 (p2 − 2q) + A2 q,
where A1 and A2 are the highest coefficients of the polynomials of degree six
(
P3 (x, y, z) = (x 2 + 5 yz)
484 Vasile Cîrtoaje

and 
P2 (x, y, z) = yz( y 2 + 5z x)(z 2 + 5x y),
respectively. Therefore, we have

A(p, q) = P3 (1, 1, 1)(p2 − 2q) + P2 (1, 1, 1)q = 63 (p2 − 2q) + 3 · 62 q = 108(2p2 − 3q).

On the other hand,

f8 (x, 1, 1) =(x 2 + 2)(x 2 + 5)(5x + 1)2


− (2x + 1)[2x(5x + 1)2 + 2(x + 1)(x 2 + 5)(5x + 1)]
=x(5x + 1)(5x 4 − 3x 3 + 9x 2 − 29x + 18)
=x(5x + 1)(x − 1)2 (5x 2 + 7x + 18).

Since A(p, q) > 0, we apply Corollary 3 for

x 2 (x − 1)4
Eα,β (x) = f0,−2 (x) = .
81
Condition (a). It suffices to show that f8 (x, 1, 1)(x) ≥ A(x + 2, 2x + 1) f0,−2 (x) for
0 ≤ x ≤ 4, where
A(x + 2, 2x + 1) = 108(2x 2 + 2x + 5).
We have
4x 2 (x − 1)4 (2x 2 + 2x + 5)
A(x + 2, 2x + 1) f0,−2 (x) = ,
3
x(x − 1)2 f (x)
f8 (x, 1, 1)(x) − A(x + 2, 2x + 1) f0,−2 (x) = ,
3
where

f (x) = 3(5x + 1)(5x 2 + 7x + 18) − 4x(2x 2 + 2x + 5)(x − 1)2 .

Since 5x + 1 > 5x and 2(5x 2 + 7x + 18) ≥ 5(2x 2 + 2x + 5), we get

2 f (x) > 75x(2x 2 + 2x + 5) − 8x(2x 2 + 2x + 5)(x − 1)2


= x(2x 2 + 2x + 5)[75 − 8(x − 1)2 ] ≥ x(2x 2 + 2x + 5)(75 − 72) ≥ 0.

Condition (b). It suffices to show that f8 (x, 1, 1)(x) ≥ A(x + 2, 2x + 1)x 2 for
x ≥ 4. The inequality is equivalent to

(5x + 1)(x − 1)2 (5x 2 + 7x + 18) ≥ 108(2x 2 + 2x + 5)x,

which can be obtained by multiplying the inequalities

5x + 1 > 5x,
5
5x 2 + 7x + 18 > (2x 2 + 2x + 5),
2
Highest Coefficient Cancellation Method for Nonnegative Variables 485

25(x − 1)2 ≥ 216.

Condition (c). It suffices to show that f8 (0, y, z) ≥ 0 for y, z ≥ 0. This is true if


the original inequality holds for x = 0. Thus, we need to show that

z y y 2 + z2
+ ≤ ,
y z yz

which is an identity.
The equality occurs for x = y = z, and also for x = 0 (or any cyclic permutation).

Observation 1. Similarly, we can prove the following generalization:


1
• Let x, y, z be nonnegative real numbers, no two of which are zero. If ≤ k ≤ 5,
4
then
x( y + z) y(z + x) z(x + y) 5−k x 2 + y 2 + z2
+ 2 + 2 ≤ + .
x + k yz y + kz x z + k x y
2 1 + k x y + yz + z x

Observation 2. From the inequalities in Observation 1, we get the following par-


ticular inequalities:

2x( y + z) 2 y(z + x) 2z(x + y) x 2 + y 2 + z2


+ + ≤3+ ,
2x 2 + yz 2 y2 + z x 2z 2 + x y x y + yz + z x

x( y + z) y(z + x) z(x + y) (x + y + z)2


+ + ≤ ,
x 2 + yz y2 + z x z2 + x y x y + yz + z x

x( y + z) y(z + x) z(x + y) x 2 + y 2 + z2
+ + ≤ 1 + ,
x 2 + 2 yz y 2 + 2z x z 2 + 2x y x y + yz + z x

x( y + z) y(z + x) z(x + y) 1 x 2 + y 2 + z2
+ + ≤ + ,
x 2 + 4 yz y 2 + 4z x z 2 + 4x y 5 x y + yz + z x

x( y + z) y(z + x) z(x + y) x 2 + y 2 + z2
+ + ≤ .
x 2 + 5 yz y 2 + 5z x z 2 + 5x y x y + yz + z x

P 4.49. If x, y, z are nonnegative real numbers, no two of which are zero, then

x( y + z) y(z + x) z(x + y) 15(x y + yz + z x)


+ 2 + 2 +2≥ .
x 2 + yz y + zx z +xy (x + y + z)2

(Vasile C., 2012)


486 Vasile Cîrtoaje

Solution. Write the inequality as f8 (x, y, z) ≥ 0, where


 (
f8 (x, y, z) = p2 x( y + z)( y 2 + z x)(z 2 + x y) − (15q − 2p2 ) (x 2 + yz).

Consider further the nontrivial case

15q − 2p2 ≥ 0.

Since x( y + z) = − yz + q, the polynomial of degree eight f8 (x, y, z) has the highest


polynomial
A(p, q) = −A1 p2 − A2 (15q − 2p2 ),
where A1 and A2 are the highest coefficients of the polynomials of degree six

P2 (x, y, z) = yz( y 2 + z x)(z 2 + x y)

and (
P3 (x, y, z) = (x 2 + yz),
respectively. Therefore, we have

A(p, q) = −P2 (1, 1, 1)p2 − P3 (1, 1, 1)(15q − 2p2 ) = −12p2 − 8(15q − 2p2 ) < 0.

According to Theorem 4, it suffices to prove the original inequality for y = z = 1


and for x = 0.
Case 1: y = z = 1. We need to show that

2x 15(2x + 1)
+4≥ ,
x2 + 1 (x + 2)2

which is equivalent to
(x − 1)2 (2x − 1)2 ≥ 0.

Case 2: x = 0. We need to show that


z y 15 yz
+ +2≥ ,
y z ( y + z)2

which is equivalent to
( y + z)4 ≥ 15 y 2 z 2 ,
( y + z)4 − 16 y 2 z 2 + y 2 z 2 ≥ 0,
( y − z)2 [( y + z)2 + 4 yz] + y 2 z 2 ≥ 0.
x
The equality occurs for x = y = z, and for = y = z (or any cyclic permutation).
2
Highest Coefficient Cancellation Method for Nonnegative Variables 487

P 4.50. If x, y, z are nonnegative real numbers, no two of which are zero, then

x( y + z) y(z + x) z(x + y) x y + yz + z x
+ + ≥1+ 2 .
x 2 + 2 yz y 2 + 2z x z 2 + 2x y x + y 2 + z2

Solution. Write the inequality as f8 (x, y, z) ≥ 0, where


 (
f8 (x, y, z) = (x 2 + y 2 +z 2 ) x( y +z)( y 2 +2z x)(z 2 +2x y)−(p2 −q) (x 2 +2 yz).

Since x( y + z) = − yz + q, the polynomial of degree eight f8 (x, y, z) has the highest


polynomial
A(p, q) = −A1 (p2 − 2q) − A2 (p2 − q),
where A1 and A2 are the highest coefficients of the polynomials of degree six

P2 (x, y, z) = yz( y 2 + 2z x)(z 2 + 2x y)

and (
P3 (x, y, z) = (x 2 + 2 yz),
respectively. Therefore, we have

A(p, q) = −P2 (1, 1, 1)(p2 − 2q) − P3 (1, 1, 1)(p2 − q)


= −27(p2 − 2q) − 27(p2 − q) < 0.

According to Theorem 4, we only need to prove the original inequality for y = z = 1


and for x = 0.
Case 1: y = z = 1. We need to show that

2x 2(x + 1) 2x + 1
+ ≥1+ 2 ,
x2 + 2 2x + 1 x +2
which is equivalent to
(x − 1)2 ≥ 0.

Case 2: x = 0. We need to show that


z y yz
+ ≥1+ 2 ,
y z y + z2

which is equivalent to

( y 2 + z 2 − yz)2 + yz( y − z)2 ≥ 0.

The equality occurs for x = y = z.


Observation 1. Similarly, we can prove the following generalization:
• Let x, y, z be nonnegative real numbers, no two of which are zero.
488 Vasile Cîrtoaje

(a) If 0 ≤ k ≤ 2, then

k  x( y + z) 2k − 1 x y + yz + z x
≥ + 2 ;
2 x 2 + k yz k+1 x + y 2 + z2

(b) If k ≥ 2, then

k  x( y + z) 2k − 1 x y + yz + z x
≥1+ · 2 .
2 x 2 + k yz k+1 x + y 2 + z2

Observation 2. From the inequalities in Observation 1, we get the following par-


ticular inequalities:

x( y + z) y(z + x) z(x + y) 1 7 x y + yz + z x
+ + ≥ + · ,
x 2 + 4 yz y 2 + 4z x z 2 + 4x y 2 10 x 2 + y 2 + z 2

x( y + z) y(z + x) z(x + y) x y + yz + z x
+ + ≥1+ 2 ,
x 2 + 2 yz y 2 + 2z x z 2 + 2x y x + y 2 + z2
x( y + z) y(z + x) z(x + y) (x + y + z)2
+ 2 + 2 ≥ 2 ,
x 2 + yz y + zx z +xy x + y 2 + z2
x( y + z) y(z + x) z(x + y) 2(x y + yz + z x)
+ + ≥ .
2x 2 + yz 2 y 2 + z x 2z 2 + x y x 2 + y 2 + z2

P 4.51. If x, y, z are nonnegative real numbers such that x y + yz + z x = 3, then


 
1 1 1
18 + + + 5(x 2 + y 2 + z 2 ) ≥ 42.
x 2 + y 2 y 2 + z2 z2 + x 2

(Vasile C., 2012)

Solution. Write the inequality in the homogeneous form


 
1 1 1 5(x 2 + y 2 + z 2 )
2(x y + yz + z x) + + + ≥ 14.
x 2 + y 2 y 2 + z2 z2 + x 2 x y + yz + z x

which is equivalent to f8 (x, y, z) ≥ 0, where



f8 (x, y, z) = 2q2 (x 2 + y 2 )(x 2 + z 2 ) + (5p2 − 24q)(x 2 + y 2 )( y 2 + z 2 )(z 2 + x 2 ).

Since y 2 + z 2 = −x 2 + p2 − 2q, the polynomial of degree eight f8 (x, y, z) has the


same highest polynomial A(p, q) as

(5p2 − 24q)(−z 2 )(−x 2 )(− y 2 ),


Highest Coefficient Cancellation Method for Nonnegative Variables 489

that is
A(p, q) = −5p2 + 24q.
We have

f8 (x, 1, 1) = 2(2x + 1)2 [(x 2 + 1)2 + 4(x 2 + 1)] + 2(5x 2 − 28x − 4)(x 2 + 1)2
= 2(x 2 + 1)(9x 4 − 24x 3 + 22x 2 − 8x + 1)
= 2(x 2 + 1)(x − 1)2 (3x − 1)2 ,

f8 (0, y, z) = 2 y 2 z 2 [ y 2 z 2 + ( y 2 + z 2 )2 ] + [5( y 2 + z 2 ) − 14 yz] y 2 z 2 ( y 2 + z 2 )


= 7 y 2 z 2 ( y 2 + z 2 )( y − z)2 + 2 y 4 z 4 .

Case 1: −5p2 + 24q ≤ 0. Since A(p, q) ≤ 0, it suffices to show that f8 (x, 1, 1) ≥ 0


and f8 (0, y, z) ≥ 0 for x, y, z ≥ 0 (see Corollary 2). These conditions are clearly
satisfied.
Case 2: −5p2 + 24q ≥ 0. Since A(p, q) ≥ 0, we apply Corollary 3 for

4(x − 1)4 (3x − 1)2


Eα,β (x) = f1/3,−2 (x) = .
3969

The condition (a) in Corollary 3 are satisfied if f8 (x, 1, 1) ≥ A(x+2, 2x+1) f1/3,−2 (x)
for x ∈ [0, 4], where

A(x + 2, 2x + 1) = −5x 2 + 28x + 4.

We have
2(x − 1)2 (3x − 1)2 g(x)
f8 (x, 1, 1) − A(x + 2, 2x + 1) f1/3,−2 (x) = ,
3969
where

g(x) = 3969(x 2 + 1) − 2(−5x 2 + 28x + 4)(x − 1)2


≥ 3969(x − 1)2 − 2(−5x 2 + 28x + 4)(x − 1)2
≥ 98(x − 1)2 − 2(−5x 2 + 30x + 4)(x − 1)2
= 10(x − 1)2 (x − 3)2 ≥ 0.

The condition (b) in Corollary 3 is satisfied if f8 (x, 1, 1) ≥ A(x + 2, 2x + 1)x 2 for


x ≥ 4. The inequality is equivalent to

2(x 2 + 1)(x − 1)2 (3x − 1)2 ≥ (−5x 2 + 28x + 4)x 2 .

For the nontrivial case −5x 2 + 28x + 4 ≥ 0, since x 2 + 1 > x 2 and 2(x − 1)2 > 1, it
suffices to show that
(3x − 1)2 ≥ −5x 2 + 28x + 4.
490 Vasile Cîrtoaje

Indeed,

(3x − 1)2 − (−5x 2 + 28x + 4) = 14x 2 − 34x − 3 ≥ 56x − 34x − 3 = 22x − 3 > 0.

The condition (c) in Corollary 3 is satisfied because f8 (0, y, z) ≥ 0 for y, z ≥ 0.


1 3
The equality occurs for x = y = z = 1, and for x =
and y = z =
(or any
5 5
cyclic permutation).

P 4.52. If x, y, z are nonnegative real numbers, then

2x y 2 yz 2z x 30(x y + yz + z x)
+ + +7≥ .
x 2 + y 2 y 2 + z2 z2 + x 2 (x + y + z)2

(Vasile C., 2012)

Solution. Write the inequality as f8 (x, y, z) ≥ 0, where



f8 (x, y, z) = 2p2 yz(x 2 + y 2 )(x 2 + z 2 ) + (7p2 − 30q)(x 2 + y 2 )( y 2 + z 2 )(z 2 + x 2 ).

Since y 2 + z 2 = −x 2 + p2 − 2q, the polynomial of degree eight f8 (x, y, z) has the


same highest polynomial A(p, q) as

2p2 yz(−z 2 )(− y 2 ) + (7p2 − 30q)(−z 2 )(−x 2 )(− y 2 ),

that is
A(p, q) = 6p2 − (7p2 − 30q) = 30q − p2 .
We have

f8 (x, 1, 1) = 2(x + 2)2 [(x 2 + 1)2 + 4x(x 2 + 1)] + 2(7x 2 − 32x − 2)(x 2 + 1)2
= 4(x 2 + 1)(4x 4 − 12x 3 + 13x 2 − 6x + 1)
= 4(x 2 + 1)(x − 1)2 (2x − 1)2 ,

f8 (0, y, z) = 2( y + z)2 y 3 z 3 + [7( y 2 + z 2 ) − 16 yz] y 2 z 2 ( y 2 + z 2 )


= 7 y 2 z 2 ( y 2 + z 2 )( y − z)2 + 4 y 4 z 4 .

Case 1: 30q − p2 ≤ 0. Since A(p, q) ≤ 0, it suffices to show that f8 (x, 1, 1) ≥ 0 and


f8 (0, y, z) ≥ 0 for x, y, z ≥ 0 (see Corollary 2). These conditions are satisfied.
Case 2: 30q − p2 ≥ 0. Since A(p, q) ≥ 0, we apply Corollary 3 for

4(x − 1)4 (2x − 1)2


Eα,β (x) = f1/2,−2 (x) = .
2025
Highest Coefficient Cancellation Method for Nonnegative Variables 491

The condition (a) in Corollary 3 are satisfied if f8 (x, 1, 1) ≥ A(x+2, 2x+1) f1/2,−2 (x)
for x ∈ [0, 4], where

A(x + 2, 2x + 1) = −x 2 + 56x + 26.

We have
4(x − 1)2 (2x − 1)2 g(x)
f8 (x, 1, 1) − A(x + 2, 2x + 1) f1/2,−2 (x) = ,
2025
where
g(x) = 2025(x 2 + 1) − (−x 2 + 56x + 26)(x − 1)2
≥ 2025(x − 1)2 − (−x 2 + 56x + 26)(x − 1)2
≥ 234(x − 1)2 − (−x 2 + 56x + 26)(x − 1)2
= (x − 1)2 (4 − x)(52 − x) ≥ 0.

The condition (b) in Corollary 3 is satisfied if f8 (x, 1, 1) ≥ A(x + 2, 2x + 1)x 2 for


x ≥ 4. The inequality is equivalent to

4(x 2 + 1)(x − 1)2 (2x − 1)2 ≥ (−x 2 + 56x + 26)x 2 .

For the nontrivial case −x 2 + 56x + 26 ≥ 0, since x 2 + 1 > x 2 and (x − 1)2 > 2, it
suffices to show that
8(2x − 1)2 ≥ −x 2 + 56x + 26.
Indeed,

8(2x − 1)2 − (−x 2 + 56x + 26) = 33x 2 − 88x − 18 > 32x − 88x − 24
= 8(x − 3)(4x + 1) > 0.

The condition (c) in Corollary 3 is satisfied because f8 (0, y, z) ≥ 0 for y, z ≥ 0.


The equality occurs for x = y = z, and for 2x = y = z (or any cyclic permutation).

Observation. Similarly, we can prove the following generalization:

• Let x, y, z be nonnegative real numbers. If −2 < k ≤ 1, then


 
(4 − k2 )(k + 2)  xy 3 3(x y + yz + z x)
− +1− ≥ 0,
8(5 − 2k) x2 + kx y + y2 k + 2 (x + y + z)2
(4 − k)x
with equality for x = y = z, and also for = y = z (or any cyclic permuta-
2 − 2k
tion).

For k = −1 and k = 1, we get the inequalities:


 xy 47 56(x y + yz + z x)
+ ≥ ;
x2 − x y + y2 3 (x + y + z)2
492 Vasile Cîrtoaje

 xy 8(x y + yz + z x)
+7≥ .
x2 + x y + y2 (x + y + z)2

P 4.53. If x, y, z are nonnegative real numbers, then

2x y 2 yz 2z x x 2 + y 2 + z2 5
+ + + ≥ .
(x + y) 2 ( y + z) 2 (z + x) 2 x y + yz + z x 2

(Vasile C., 2012)

Solution. Write the inequality as f8 (x, y, z) ≥ 0, where



f8 (x, y, z) = 4q yz(x + y)2 (x + z)2 + (2p2 − 9q)(x + y)2 ( y + z)2 (z + x)2 .

Since ( y +z)2 = (p− x)2 = x 2 + p(p−2x), the polynomial of degree eight f8 (x, y, z)
has the same highest polynomial A(p, q) as

4q yz(z 2 )( y 2 ) + (2p2 − 9q)(z 2 )(x 2 )( y 2 ),

that is
A(p, q) = 12q − (2p2 − 9q) = 21q − 2p2 .
We have
 
f8 (x, 1, 1) = 4(2x + 1) (x + 1)4 + 8x(x + 1)2 + 4(2x 2 − 10x − 1)(x + 1)4
= 8x 2 (x + 1)2 (x − 1)2 ,
 
f8 (0, y, z) = 4 y 4 z 4 + 2( y + z)2 − 9 yz y 2 z 2 ( y 2 + z 2 )
= y 2 z 2 ( y − z)2 (2 y 2 + 2z 2 + 3 yz).

Case 1: 21q − 2p2 ≤ 0. Since A(p, q) ≤ 0, it suffices to show that f8 (x, 1, 1) ≥ 0


and f8 (0, y, z) ≥ 0 for x, y, z ≥ 0 (see Corollary 2). These conditions are clearly
satisfied.

Case 2: 21q − 2p2 ≥ 0. Since A(p, q) ≥ 0, we may apply Corollary 3 for

(x − 1)4 x 2
Eα,β (x) = f0,−2 (x) = .
81

The condition (a) in Corollary 3 are satisfied if f8 (x, 1, 1) ≥ A(x+2, 2x+1) f0,−2 (x)
for x ∈ [0, 4], where

A(x + 2, 2x + 1) = −2x 2 + 34x + 13.


Highest Coefficient Cancellation Method for Nonnegative Variables 493

We have
(x − 1)2 x 2 g(x)
f8 (x, 1, 1) − A(x + 2, 2x + 1) f0,−2 (x) = ,
81
where

g(x) = 648(x + 1)2 − (−2x 2 + 34x + 13)(x − 1)2


≥ 648(x − 1)2 − (−2x 2 + 34x + 13)(x − 1)2
≥ 117(x − 1)2 − (−2x 2 + 34x + 13)(x − 1)2
= 2(x − 1)2 (4 − x)(13 − x) ≥ 0.

The condition (b) in Corollary 3 is satisfied if f8 (x, 1, 1) ≥ A(x + 2, 2x + 1)x 2 for


x ≥ 4. The inequality is true if

8(x + 1)2 (x − 1)2 ≥ −2x 2 + 34x + 13.

For the nontrivial case −2x 2 + 34x + 13 ≥ 0, since (x − 1)2 > 1, it suffices to show
that
8(x + 1)2 ≥ −2x 2 + 34x + 13.
Indeed,

8(x + 1)2 − (−2x 2 + 34x + 13) = 10x 2 − 18x − 5 ≥ 40x − 18x − 5


= 22x − 5 > 0.

The condition (c) in Corollary 3 is satisfied since f8 (0, y, z) ≥ 0 for all y, z ≥ 0.


The equality occurs for x = y = z, and for x = 0 and y = z (or any cyclic
permutation).

Observation 1. In the case 2, we can give a more simple solution based on The-
orem 6. The conditions (a) and (c) are satisfied. Thus, it suffices to show that
4x(x − 1)3
f8 (x, 1, 1) ≥ A(x + 2, 2x + 1) for x ≥ 1. We have
27

4x(x − 1)3 4x(x − 1)2 g(x)


f8 (x, 1, 1) − A(x + 2, 2x + 1) = ,
27 27
where

g(x) = 54x(x + 1)2 − (−2x 2 + 34x + 13)(x − 1)


> 13(x − 1)(x + 1)2 − (−2x 2 + 34x + 13)(x − 1)
= x(x − 1)(15x − 8) ≥ 0.

Observation 2. Similarly, we can prove the following generalization:


494 Vasile Cîrtoaje

• Let x, y, z be nonnegative real numbers. If −2 < k ≤ 2, then


    2 
xy 3 x + y 2 + z2
(k+2) k − 1 + 5 − 2k − +4 − 1 ≥ 0,
x2 + kx y + y2 k + 2 x y + yz + z x

2x
with equality for x = y = z, and also for
= y = z (or any cyclic
5 − 2k − 1
permutation).

For k = −1, k = 0, k = 1/2, k = 1 and k = 2, we get the inequalities:



 
xy 4( 7 + 2) x 2 + y 2 + z 2
−3+ − 1 ≥ 0;
x2 − x y + y2 3 x y + yz + z x

 2x y  2 

x + y 2 + z2
− 3 + ( 5 + 1) − 1 ≥ 0;
x2 + y2 x y + yz + z x
  
5x y 8 x 2 + y 2 + z2
− 3 + − 1 ≥ 0;
2x 2 + x y + 2 y 2 3 x y + yz + z x
  2 
3x y 4 x + y 2 + z2
− 3 +
− 1 ≥ 0;
x2 + x y + y2 3 x y + yz + z x
 4x y  
x 2 + y 2 + z2
− 3 + 2 − 1 ≥ 0.
(x + y)2 x y + yz + z x

P 4.54. If x, y, z are nonnegative real numbers, no two of which are zero, then

2 2 2 8 1
+ + ≥ 2 + .
x 2 + y 2 y 2 + z2 z2 + x 2 x + y 2 + z 2 x y + yz + z x

(Vasile C., 2012)

Solution. Write the inequality as f8 (x, y, z) ≥ 0, where


 (
f8 (x, y, z) = 2q(p2 − 2q) (x 2 + y 2 )(x 2 + z 2 ) − (p2 + 6q) (x 2 + y 2 ).

Since x 2 + y 2 = −z 2 + p2 − 2q, the polynomial of degree eight f8 (x, y, z) has the


same highest polynomial A(p, q) as

−(p2 + 6q)(−z 2 )(−x 2 )(− y 2 ),

that is
A(p, q) = p2 + 6q, A(x + 2, 2x + 1) = x 2 + 16x + 10.
Highest Coefficient Cancellation Method for Nonnegative Variables 495

We have
 
f8 (x, 1, 1) = 2(2x + 1)x 2 + 2) (x 2 + 1)2 + 4(x 2 + 1) − 2(x 2 + 16x + 10)(x 2 + 1)2
= 4x(x 2 + 1)(x 4 − x 2 − 2x + 2)
= 4x(x 2 + 1)(x − 1)2 (x 2 + 2x + 2).

On the other hand, for x = 0, the desired inequality becomes


2 2 2 8 1
+ + ≥ 2 + ,
y 2 z2 y 2 + z2 y + z 2 yz

which is equivalent to

( y − z)2 (2 y 2 + 2z 2 + 3 yz) ≥ 0.

First Solution. Apply Theorem 6. The conditions (a) and (c) are satisfied. In what
concerns the condition (b), we have

4A(x + 2, 2x + 1)x(x − 1)3 4(x 2 + 16x + 10)x(x − 1)3


= ,
27 27
4A(x + 2, 2x + 1)x(x − 1)3 4x(x − 1)2 g(x)
f7 (x, 1, 1) − = ,
27 27
where
g(x) = 27(x 2 + 1)(x 2 + 2x + 2) − (x 2 + 16x + 10)(x − 1).
For x ≥ 1, we get

g(x) > 27(x − 1)(x 2 + 2x + 2) − (x 2 + 16x + 10)(x − 1)


≥ 6(x − 1)(x 2 + 2x + 2) − (x 2 + 16x + 10)(x − 1)
= (x − 1)(5x 2 − 4x + 2) ≥ 0.

The equality occurs for x = y = z, and for x = 0 and y = z (or any cyclic
permutation).
Second Solution. Apply Corollary 3 for

(x − 1)4 x 2
Eα,β (x) = f0,−2 (x) = .
81

The condition (a) in Corollary 3 is satisfied if f8 (x, 1, 1) ≥ A(x +2, 2x +1) f0,−2 (x)
for x ∈ [0, 4]. We have

x(x − 1)2 g(x)


f8 (x, 1, 1) − A(x + 2, 2x + 1) f0,−2 (x) = ,
81
where
496 Vasile Cîrtoaje

g(x) = 324(x 2 + 1)(x 2 + 2x + 2) − x(x 2 + 16x + 10)(x − 1)2


≥ 324(x − 1)2 (x 2 + 2x + 2) − 4(x 2 + 16x + 10)(x − 1)2
= 4(x − 1)2 [81(x 2 + 2x + 2) − (x 2 + 16x + 10)]
≥ 4(x − 1)2 [6(x 2 + 2x + 2) − (x 2 + 16x + 10)]
= 4(x − 1)2 [3x 2 + 2(x − 1)2 ] ≥ 0.

The condition (b) in Corollary 3 is satisfied if f8 (x, 1, 1) ≥ A(x + 2, 2x + 1)x 2 for


x ≥ 4. This is true if

4(x 2 + 1)(x − 1)2 (x 2 + 2x + 2) ≥ x(x 2 + 16x + 10).

Since 4 ≥ x and (x 2 + 1)(x − 1)2 > 5, it suffices to show that

5(x 2 + 2x + 2) ≥ x 2 + 16x + 10,

which is equivalent to
2x(2x − 3) ≥ 0.
The condition (c) in Corollary 3 is satisfied since the original inequality holds for
x = 0.
Observation. Similarly, we can prove the following generalization:

• Let x, y, z be nonnegative real numbers. If



10 + 136
1− 2≤ k ≤ ,
9
then  k+2 8 − 4k 4k + 1
≥ 2 + ,
x2 + kx y + y2 x + y 2 + z 2 x y + yz + z x

with equality for x = y = z, and also for x = 0 and y = z (or any cyclic permutation).

For k = −1/4, k = 0, k = 1 and k = 2, we get the inequalities:


 1 9
≥ ,
4x 2 − x y + 4 y 2 7(x 2 + y 2 + z 2 )
 2 8 1
≥ 2 + ,
x2 + y2 x + y 2 + z 2 x y + yz + z x
 3 4 5
≥ 2 + ,
x2 + x y + y2 x + y 2 + z 2 x y + yz + z x
 4 9
≥ .
(x + y)2 x y + yz + z x
Highest Coefficient Cancellation Method for Nonnegative Variables 497

P 4.55. If x, y, z are nonnegative real numbers, then

2x y 2 yz 2z x 4(x y + yz + z x)
+ + +1≥ .
x 2 + y 2 y 2 + z2 z2 + x 2 x 2 + y 2 + z2

(Vasile C., 2014)

Solution. Write the inequality as f8 (x, y, z) ≥ 0, where



f8 (x, y, z) = 2(p2 −2q) yz(x 2 + y 2 )(x 2 +z 2 )+(p2 −6q)(x 2 + y 2 )( y 2 +z 2 )(z 2 + x 2 ).

Since
y 2 + z 2 = −x 2 + p2 − 2q,
the polynomial of degree eight f8 (x, y, z) has the same highest polynomial A(p, q)
as 
2(p2 − 2q) yz(−z 2 )(− y 2 ) + (p2 − 6q)(−z 2 )(−x 2 )(− y 2 ),
that is

A(p, q) = 6(p2 − 2q) − (p2 − 6q) = 5p2 − 6q = 2(p2 − 3q) + 3p2 > 0,

A(x + 2, 2x + 1) = 5x 2 + 8x + 14 > 0.
For x = 0, the original inequality becomes

2 yz 4 yz
+1≥ 2 ,
y 2 + z2 y + z2

which is equivalent to
( y − z)2 ≥ 0.
Also, we have
 
f8 (x, 1, 1) = 2(x 2 + 2) (x 2 + 1)2 + 4x(x 2 + 1) + 2(x 2 − 8x − 2)(x 2 + 1)2
= 4x 2 (x 2 + 1)(x − 1)2 .

First Solution. Apply Theorem 6. The conditions (a) and (c) are satisfied. In what
concerns the condition (b), we have

4A(x + 2, 2x + 1)x(x − 1)3 4x(x − 1)3 (5x 2 + 8x + 14)


= ,
27 27

4A(x + 2, 2x + 1)x(x − 1)3 4x(x − 1)2 g(x)


f8 (x, 1, 1) − = ,
27 9
where
g(x) = 27x(x 2 + 1) − (x − 1)(5x 2 + 8x + 14).
498 Vasile Cîrtoaje

For x ≥ 1, we get
g(x) > 27(x − 1)(x 2 + 1) − (x − 1)(5x 2 + 8x + 14)
= (x − 1)(27x 2 − 8x + 13) ≥ 0.

The equality holds for x = y = z, and also for x = 0 and y = z (or any cyclic
permutation).
Second Solution. Apply Corollary 3 for
x 2 (x − 1)4
Eα,β (x) = f0,−2 (x) = .
81
Condition (a). It suffices to show that f8 (x, 1, 1) ≥ A(x + 2, 2x + 1) f0,−2 (x) for
0 ≤ x ≤ 4. We have
x 2 (x − 1)4 (5x 2 + 8x + 14)
A(x + 2, 2x + 1) f0,−2 (x) = ,
81
x 2 (x − 1)2 f (x)
f8 (x, 1, 1) − A(x + 2, 2x + 1) f0,−2 (x) = ,
81
f (x) = 324(x + 1) − (x − 1) (5x + 8x + 14).
2 2 2

Since x 2 + 1 ≥ (x − 1)2 , we get


f (x) ≥ 324(x − 1)2 − (x − 1)2 (5x 2 + 8x + 14)
= (x − 1)2 (310 − 5x 2 − 8x) ≥ 0.

Condition (b). It suffices to show that f8 (x, 1, 1) ≥ A(x + 2, 2x + 1)x 2 for x ≥ 4.


We have
f8 (x, 1, 1) − A(x + 2, 2x + 1)x 2 = x 2 [4(x 2 + 1)(x − 1)2 − 5x 2 − 8x − 14]
≥ x 2 [36(x 2 + 1) − 5x 2 − 8x − 14]
= x 2 (31x 2 − 8x + 22) > 0.

Condition (c). This condition is satisfied because the original inequality holds for
x = 0.

Observation. Similarly, we can prove the following generalization:

• Let x, y, z be real numbers. If −2 < k ≤ 0, then


 (k + 2) yz  
8 x y + yz + z x
− 3 +
1 − ≥ 0,
y 2 + k yz + z 2 3 − 2k2 + 1 x 2 + y 2 + z2
kx
with equality for x = y = z, and also for
= y = z (or any cyclic
1− 2k2 + 1
permutation).
Chapter 5

On Popoviciu’s Inequality

5.1 Theoretical Basis


In 1965, the Romanian mathematician T. Popoviciu proved the following inequality:
      c + a
a+b+c a+b b+c
f (a) + f (b) + f (c) + 3 f ≥ 2f + 2f + 2f ,
3 2 2 2

where f is a convex function on a real interval , and a, b, c ∈ .


In 2002, we gave the following proof for the variant below of Popoviciu’s in-
equality for n variables:
Theorem 1. If f is a convex function on a real interval  and a1 , a2 , . . . , an ∈ , then

f (a1 ) + f (a2 ) + · · · + f (an ) + n(n − 2) f (a) ≥ (n − 1)[ f (b1 ) + f (b2 ) + · · · + f (bn )],

1! n
1 !
where a = a j and bi = a j for all i.
n j=1 n − 1 j =i

Proof. Assume that n ≥ 3 and a1 ≤ a2 ≤ · · · ≤ an . There exists an integer m,


1 ≤ m ≤ n − 1, such that

a1 ≤ · · · ≤ am ≤ a ≤ am+1 ≤ · · · ≤ an ,

b1 ≥ · · · ≥ bm ≥ a ≥ bm+1 ≥ · · · ≥ bn .
We can get the desired inequality by summing the inequalities

f (a1 ) + · · · + f (am ) + n(n − m − 1) f (a) ≥ (n − 1)[ f (bm+1 ) + · · · + f (bn )],

f (am+1 ) + · · · + f (an ) + n(m − 1) f (a) ≥ (n − 1)[ f (b1 ) + · · · + f (bm )].


Let
a1 + · · · + am + (n − m − 1)a am+1 + · · · + an + (m − 1)a
b= , c= .
n−1 n−1

499
500 Vasile Cîrtoaje

Using Jensen’s inequalities

f (a1 ) + · · · + f (am ) + (n − m − 1) f (a) ≥ (n − 1) f (b),

f (am+1 ) + · · · + f (an ) + (m − 1) f (a) ≥ (n − 1) f (c),


it suffices to show that

(n − m − 1) f (a) + f (b) ≥ f (bm+1 ) + · · · + f (bn ), (*)

f (c) + (m − 1) f (a) ≥ f (b1 ) + · · · + f (bm ). (**)


Since
a ≥ bm+1 ≥ · · · ≥ bn , (n − m − 1)a + b = bm+1 + · · · + bn ,
it follows that the decreasingly ordered sequence An−m = (a, · · · , a, b) majorizes the
decreasingly ordered sequence Bn−m = (bm+1 , bm+2 · · · , bn ). Similarly, since

b1 ≥ · · · ≥ bm ≥ a, c + (m − 1)a = b1 + · · · + bm ,

the decreasingly ordered sequence Cm = (c, a, · · · , a) majorizes the decreasingly


ordered sequence Dm = (b1 , b2 · · · , bm ). Therefore, the inequalities (∗) and (∗∗)
are consequences of Karamata’s inequality.
Another variant of Popoviciu’s inequality for n variables is the following:
Theorem 2. If f is a convex function on a real interval  and a1 , a2 , . . . , an ∈ , then
a + a + ··· + a    
1 2 n
ai + a j
(n−2)[ f (a1 )+ f (a2 )+· · ·+ f (an )]+n f ≥ 2 f .
n 1≤i< j≤n
2

Proof. We use the induction method. For n = 2, the equality occurs. Suppose that
the inequality holds for n − 1 numbers, n ≥ 3, and show that it also holds for n
numbers. Let
a1 + a2 + · · · + an a1 + a2 + · · · + an−1
a= , b= .
n n−1
By the induction hypothesis,
  
ai + a j
(n − 3)[ f (a1 ) + f (a2 ) + · · · + f (an−1 )] + (n − 1) f (b) ≥ 2 f .
1≤i< j≤n−1
2

Thus, it suffices to show that



n−1 a + a 
i n
f (a1 )+ f (a2 )+· · ·+ f (an−1 )+(n−2) f (an )+n f (a) ≥ (n−1) f (b) + 2 f .
i=1
2

Since
f (a1 ) + f (a2 ) + · · · + f (an−1 ) ≥ (n − 1) f (b)
On Popoviciu’s Inequality 501

(by Jensen’s inequality), it suffices to show that



n−1 a + a 
i n
(n − 2) f (an ) + n f (a) ≥ 2 f . (***)
i=1
2

Since

n−1
ai + an
(n − 2)an + na = 2 ,
i=1
2
we will apply Karamata’s inequality.
Case 1: a1 ≤ a2 ≤ · · · ≤ an , 2a ≤ a1 + an . Since
a + a a + a an−1 + an  an−1 + an
1 n 2 n
an ≥ max , ,..., =
2 2 2 2
and a + a a + a
1 n 2 n an−1 + an  a1 + an
a ≤ min , ,..., = ,
2 2 2 2
the inequality (***) follows from Karamata’s inequality.
Case 2: a1 ≥ a2 ≥ · · · ≥ an , 2a ≥ a1 + an . Since
a + a a + a an−1 + an  a1 + an
1 n 2 n
a ≥ max , ,..., = ,
2 2 2 2
a + a a + a an−1 + an  an−1 + an
1 n 2 n
an ≤ min , ,..., = ,
2 2 2 2
the inequality (***) follows from Karamata’s inequality.

For n = 4, the inequality in Theorem 2 has the elegant form


    
a+b+c+d a+b
f (a) + f (b) + f (c) + f (d) + 2 f ≥ f .
4 s ym
2

Actually, the following generalization holds:


Popoviciu’s Theorem. If f is a convex function on a real interval , a1 , a2 , . . . , an ∈ 
and 2 ≤ k ≤ n − 1, then
   n   k 
1 n−2 n−k  1  1
n
f (x i ) + n f xi ≥ f xij .
k k − 2 k − 1 i=1 n i=1 1≤i <···<i ≤n
k j=1
1 k

Note. We can rewrite Popoviciu’s inequality in Theorem 1 as

En (a1 , a2 , . . . , an ) ≥ n − 1,

where
f (a1 ) + f (a2 ) + · · · + f (an ) − n f (a)
En (a1 , a2 , . . . , an ) = .
f (b1 ) + f (b2 ) + · · · + f (bn ) − n f (a)
502 Vasile Cîrtoaje

For some convex functions, the minimum (greatest lower bound) of En is just n − 1,
but it is greater for other functions. Thus, for f (x) = x 2 , we have

a12 + a22 + · · · + an2 − na2


= (n − 1)2 ,
b12 + b22 + · · · + bn2 − na2

1! n
1 !
where a = a j and bi = a j for all i. Also, for f (x) = x 3 , x ≥ 0, if
n j=1 n − 1 j =i
a1 , a2 , . . . , an are nonnegative numbers, then

a13 + a23 + · · · + an3 − na3 (2n − 1)(n − 1)3


≥ ,
b13 + b23 + ··· + bn3 − na3 3n2 − 5n + 1

with equality for a1 = 0 and a2 = a3 = · · · = an (or any cyclic permutation). On


the assumption that a1 + a2 + · · · + an = n, this inequality can be rewritten as

(n − 1)(a13 + a23 + · · · + an3 ) + n2 ≥ (2n − 1)(a12 + a22 + · · · + an2 ),

which is the inequality in P 1.7 from Volume 4 (for k = 3), and the inequality in P
1.2 from Volume 5 (for n = 6.)
On Popoviciu’s Inequality 503

5.2 Applications

5.1. If a1 , a2 , . . . , an are positive real numbers such that

a1 a2 · · · an = 1,

then
 
1 1 1
a1n−1 + a2n−1 + · · · + ann−1 + n(n − 2) ≥ (n − 1) + + ··· + .
a1 a2 an

5.2. If a1 , a2 , . . . , an are positive real numbers such that

a1 a2 · · · an = 1,

then
 
n−1 1 1 1
a1n−1 + a2n−1 + · · ·+ ann−1 + n(n− 2) ≥ a1 + a2 + · · · + an + + + ··· + .
2 a1 a2 an

5.3. If a1 , a2 , . . . , an are positive real numbers such that

a1 a2 · · · an = 1,

then
(n − 1)(a12 + a22 + · · · + an2 ) + n ≥ (a1 + a2 + · · · + an )2 .

5.4. If a, b, c, d are positive real numbers such that

a b + bc + cd + d a = 4,

then     
a b  c d
1+ 1+ 1+ 1+ ≥ (a + b + c + d)2 .
b c d a

5.5. If a1 , a2 , . . . , an are positive real numbers such that

a1 + a2 + · · · + an = n,

then     
1 − a1 1 − a2 1 − an 
1+ 1+ ··· 1 + ≥ n−1 a1 a2 · · · an .
n−1 n−1 n−1
504 Vasile Cîrtoaje

5.6. If a1 , a2 , . . . , an (n ≥ 3) are positive real numbers such that

a1 + a2 + · · · + an = 1,

then       n
1 1 1 1
a1 + −2 a2 + − 2 · · · an + −2 ≥ n+ −2 .
a1 a2 an n

5.7. If a1 , a2 , . . . , an are positive real numbers, then

b1 b2 bn a1 a2 an
+ + ··· + ≥ + + ··· + ,
a1 a2 an b1 b2 bn

where
1 
bi = aj
n − 1 j =i

for all i.

5.8. If a1 , a2 , . . . , an (n ≥ 3) are positive real numbers such that

1 1 1
a1 + a2 + · · · + an = + + ··· + ,
a1 a2 an

then
1 1 1
(a) + + ··· + ≥ 1;
1 + (n − 1)a1 1 + (n − 1)a2 1 + (n − 1)an

1 1 1
(b) + + ··· + ≤ 1.
n − 1 + a1 n − 1 + a2 n − 1 + an

5.9. If a1 , a2 , . . . , an are positive real numbers such that

1 1 1
a 1 + a2 + · · · + a n = + + ··· + = ns,
a1 a2 an

then
1 1 1 1 1 1
+ +· · ·+ ≥ + +· · ·+ .
a1 + n − 1 a2 + n − 1 an + n − 1 1 + ns − a1 1 + ns − a2 1 + ns − an
On Popoviciu’s Inequality 505

5.10. If a, b, c are nonnegative real numbers such that a + b + c = 3, then


 



a + b
b + c c + a
4 a + b + c + 15 ≤ 9 + + .
2 2 2

5.11. If a, b, c, d, e are positive real numbers such that a bcd e = 1, then


1 1 1 1 1

+
+
+
+
≤ 1.
2+ 4 + 5a 2 + 4 + 5b 2 + 4 + 5c 2 + 4 + 5d 2 + 4 + 5e

5.12. Let a1 , a2 , . . . , an (n ≥ 3) be positive real numbers such that a1 a2 · · · an = 1.


If
2n − 1
0<p≤ ,
(n − 1)2
then
1 1 1 n
 + + ≤ .
1 + pa1 1 + pa2 1 + pan 1+p

5.13. Let f be a convex function on a real interval . If a1 , a2 , . . . , an ∈ , then


n  ai − ai+1 
2 f (a1 ) + 2 f (a2 ) + · · · + 2 f (an ) + n(n − 2) f (a) ≥ n f a+ ,
i=1
n

where
1
a= (a1 + a2 + · · · + an ).
n

5.14. If a1 , a2 , . . . , an (n ≥ 3) are positive real numbers such that

a1 a2 · · · an = 1,

then  
a1 a2 an
2(a1n + a2n + · · · + ann ) + n(n − 2) ≥ n + + ··· + .
a2 a3 a1

5.15. If a1 , a2 , . . . , an (n ≥ 3) are positive real numbers such that

a1 + a2 + · · · + an = n,

then  a1 − a2   a2 − a3   a n − a1  2
1+ 1+ ··· 1 + ≥ (a1 a2 · · · an ) n .
n n n
506 Vasile Cîrtoaje

5.16. If a1 , a2 , . . . , an (n ≥ 3) are positive real numbers such that

a1 + a2 + · · · + an = n,

then
 
2 1 1 1 1 1 1
+ + ··· + +n−2≥ + + ··· + .
n a1 a 2 an a1 − a2 a2 − a3 an − a1
1+ 1+ 1+
n n n

5.17. Let f be a convex function on (0, ∞). If a1 , a2 , . . . , an are positive real num-
bers, then      
1 1 1
f a1 + + f a2 + + · · · + f an + ≥
a2 a3 a1
     
1 1 1
≥ f a1 + + f a2 + + · · · + f an + .
a1 a2 an
On Popoviciu’s Inequality 507

5.3 Solutions

P 5.1. If a1 , a2 , . . . , an are positive real numbers such that


a1 a2 · · · an = 1,
then
 
1 1 1
a1n−1 + a2n−1 + · · · + ann−1 + n(n − 2) ≥ (n − 1) + + ··· + .
a1 a2 an

Solution. Let x 1 , x 2 , . . . , x n be real numbers such that


x 1 + x 2 + · · · + x n = 0.
Applying Popoviciu’s inequality from Theorem 1 to the convex function
f (x) = e x , x ∈ ,
we get
−x1 −x 2 −x n

e x 1 + e x 2 + · · · + e x n + n(n − 2) ≥ (n − 1) e n−1 + e n−1 + · · · + e n−1 .

Using the substitution


x 1 = (n − 1) ln a1 , x 2 = (n − 1) ln a2 , . . . , x n = (n − 1) ln an ,
gives the desired inequality. For n ≥ 3, the equality holds if and only if
a1 = a2 = · · · = an = 1.
Remark. For n = 3, using the substitution
a1 = x 3 , a2 = y 3 , a3 = z 3 ,
we get the known homogeneous inequality
x 6 + y 6 + z 6 + 3x 2 y 2 z 2 ≥ 2(x 3 y 3 + y 3 z 3 + z 3 x 3 ),
which holds for any real numbers x, y, z.

P 5.2. If a1 , a2 , . . . , an are positive real numbers such that


a1 a2 · · · an = 1,
then
 
n−1 1 1 1
a1n−1 + a2n−1 + · · ·+ ann−1 + n(n− 2) ≥ a1 + a2 + · · · + an + + + ··· + .
2 a1 a2 an

(Bin Zhao, 2005)


508 Vasile Cîrtoaje

Solution. We can get this inequality by adding the inequality in P 5.1 and the
inequality

a1n−1 + a2n−1 + · · · + ann−1 + n(n − 2) ≥ (n − 1)(a1 + a2 + · · · + an ).

This inequality follows by adding the AM-GM inequalities

ain−1 + n − 2 ≥ (n − 1)ai , i = 1, 2, . . . , n.

For n ≥ 3, the equality holds if and only if a1 = a2 = · · · = an = 1.

P 5.3. If a1 , a2 , . . . , an are positive real numbers such that

a1 a2 · · · an = 1,

then
(n − 1)(a12 + a22 + · · · + an2 ) + n ≥ (a1 + a2 + · · · + an )2 .
(F. Shleifer, 1979)

Solution. Let x 1 , x 2 , . . . , x n be real numbers such that

x 1 + x 2 + · · · + x n = 0.

Applying the inequality from Theorem 2 to the convex function f (x) = e x , we get
 x i +x j
(n − 2) (e x 1 + e x 2 + · · · + e x n ) + n ≥ 2 e 2 .
1≤i< j≤n

Using the substitution

x 1 = 2 ln a1 , x 2 = 2 ln a2 , . . . , x n = 2 ln an ,

this inequality becomes



(n − 2)(a12 + a22 + · · · + an2 ) + n ≥ 2 ai a j .
1≤i< j≤n

Since 
2 ai a j = (a1 + a2 + · · · + an )2 − (a12 + a22 + · · · + an2 ),
1≤i< j≤n

the conclusion follows. For n ≥ 3, the equality holds if and only if

a1 = a2 = · · · = an = 1.
On Popoviciu’s Inequality 509

P 5.4. If a, b, c, d are positive real numbers such that


a b + bc + cd + d a = 4,
then     
a b  c d
1+ 1+ 1+ 1+ ≥ (a + b + c + d)2 .
b c d a

Solution. Applying the inequality from Theorem 2 to the convex function


f (x) = − ln x, x > 0,
we get
(a + b)(b + c)(c + d)(d + a)(a + c)(b + d) ≥ 4a bcd(a + b + c + d)2 .
Since
(a + c)(b + d) = a b + bc + cd + d a = 4,
the inequality is equivalent to
(a + b)(b + c)(c + d)(d + a) ≥ 4a bcd(a + b + c + d)2 ,
    
a b  c d
1+ 1+ 1+ 1+ ≥ (a + b + c + d)2 .
b c d a
The equality holds for a = b = c = d = 1.

P 5.5. If a1 , a2 , . . . , an are positive real numbers such that


a1 + a2 + · · · + an = n,
then     
1 − a1 1 − a2 1 − an 
1+ 1+ ··· 1 + ≥ n−1 a1 a2 · · · an .
n−1 n−1 n−1

Solution. Applying Popoviciu’s inequality from Theorem 1 to the convex function


f (x) = − ln x, x > 0,
gives
 n−a n − a2 n − an 
1
(n − 1) ln + ln + · · · + ln ≥ ln a1 + ln a2 + · · · + ln an ,
n−1 n−1 n−1
n − a n − a  n − a  
1 2 1
··· ≥ n−1 a1 a2 · · · an ,
n−1 n−1 n−1
    
1 − a1 1 − a2 1 − an 
1+ 1+ ··· 1 + ≥ n−1 a1 a2 · · · an .
n−1 n−1 n−1
For n ≥ 3, the equality holds if and only if a1 = a2 = · · · = an = 1.
510 Vasile Cîrtoaje

P 5.6. If a1 , a2 , . . . , an (n ≥ 3) are positive real numbers such that


a1 + a2 + · · · + an = 1,
then       n
1 1 1 1
a1 + −2 a2 + − 2 · · · an + −2 ≥ n+ −2 .
a1 a2 an n

Solution. Write the inequality as


(1 − a1 )2 (1 − a2 )2 · · · (1 − an )2 (n − 1)2n
≥ ,
a1 a2 · · · an nn
(n − 1)n 
(1 − a1 )(1 − a2 ) · · · (1 − an ) ≥
a1 a2 · · · an .
nn/2
Applying Popoviciu’s inequality from Theorem 1 to the convex function
f (x) = − ln x, x > 0,
gives
 a + a + · · · + a n(n−2)
1 2 n
(b1 b2 · · · bn )n−1 ≥ (a1 a2 · · · an ) ,
n
where
1 
bi = aj, i = 1, 2, . . . , n.
n − 1 j =i
Under the hypothesis a1 + a2 + · · · + an = 1, this inequality becomes
(n − 1)n(n−1)
(1 − a1 )n−1 (1 − a2 )n−1 · · · (1 − an )n−1 ≥ a1 a2 · · · an ,
nn(n−2)
(n − 1)n n−1 
(1 − a1 )(1 − a2 ) · · · (1 − an ) ≥ n(n−2)
a1 a2 · · · an .
n n−1

Thus, it suffices to show that


 n(n−3) 
n−1
a1 a2 · · · an ≥ n 2(n−1) a1 a2 · · · an ,
which is equivalent to
1
≥ (a1 a2 · · · an )n−3 .
nn(n−3)
This inequality is valid if
1
≥ a1 a2 · · · an ,
nn
which is just the AM-GM inequality
 a + a + · · · + a n
1 2 n
≥ a1 a2 · · · an .
n
1
The equality holds if and only if a1 = a2 = · · · = an = .
n
On Popoviciu’s Inequality 511

P 5.7. If a1 , a2 , . . . , an are positive real numbers, then


b1 b2 bn a1 a2 an
+ + ··· + ≥ + + ··· + ,
a1 a2 an b1 b2 bn
where
1 
bi = aj
n − 1 j =i
for all i.

Solution. Let
a = a1 + a2 + · · · + an .
Since
bi a ai a
(n − 1)= − 1, = − n + 1, i = 1, 2, . . . , n,
ai ai bi bi
the inequality becomes
 
1 1 1 n−2 1 1 1
+ + ··· + + ≥ (n − 1) + + ··· + ,
a1 a2 an a b1 b2 bn
which is just Popoviciu’s inequality from Theorem 1 applied to the convex function
1
f (x) = , x > 0.
x
For n ≥ 3, the equality holds if and only if a1 = a2 = · · · = an .
Remark. We can also prove this inequality using the Cauchy-Schwarz inequality
n−1 1  1
! ≤ , i ∈ {1, 2, . . . , n}.
aj n − 1 j =i a j
j =i

Setting
1 1 1 1
a = a1 + a2 + · · · + an , = + + ··· + ,
A a1 a2 an
we can write this inequalities as
 
1 1 1 1
≤ − ,
bi n − 1 A ai
ai 1  ai 
≤ −1 .
bi n−1 A
Therefore,
n   
 1 a  1  a
n n
a i bi
≤ −n = −1 = .
i=1
bi n−1 A n − 1 i=1 ai a
i=1 i
512 Vasile Cîrtoaje

P 5.8. If a1 , a2 , . . . , an (n ≥ 3) are positive real numbers such that

1 1 1
a1 + a2 + · · · + an = + + ··· + ,
a1 a2 an

then
1 1 1
(a) + + ··· + ≥ 1;
1 + (n − 1)a1 1 + (n − 1)a2 1 + (n − 1)an

1 1 1
(b) + + ··· + ≤ 1.
n − 1 + a1 n − 1 + a2 n − 1 + an
(Vasile C., 1996)

Solution. (a) We use the contradiction method. For the sake of contradiction, as-
sume that
1 1 1
+ + ··· + < 1.
1 + (n − 1)a1 1 + (n − 1)a2 1 + (n − 1)an

Using the substitution

1 − xi
ai = , i = 1, 2, . . . , n,
(n − 1)x i
we get
x 1 + x 2 + · · · + x n < 1,
which is equivalent to
1 − x i > (n − 1) yi ,
where
1 
yi = x j, i = 1, 2, . . . , n.
n − 1 j =i

Therefore, we have
n
1 − xi n
yi
a1 + a2 + · · · + an = > .
i=1
(n − 1)x i x
i=1 i

Taking account of the inequality from the preceding P 5.7, we get


n
xi
a1 + a2 + · · · + a n > .
y
i=1 i

In addition, since
n
x i  (n − 1)x i
n
1 1 1
> = + + ··· + ,
y
i=1 i i=1
1 − xi a1 a2 an
On Popoviciu’s Inequality 513

we have
1 1 1
a1 + a2 + · · · + an > + + ··· + ,
a1 a2 an
which contradicts the hypothesis

1 1 1
a1 + a2 + · · · + an = + + ··· + .
a1 a2 an

For n ≥ 3, the equality holds if and only if a1 = a2 = · · · = an = 1.


(b) Replacing a1 , a2 , . . . , an respectively with 1/a1 , 1/a2 , . . . , 1/an , the inequal-
ity in (a) becomes
a1 a2 an
+ + ··· + ≥ 1,
n − 1 + a1 n − 1 + a 2 n − 1 + an

which is equivalent to the desired inequality.

P 5.9. If a1 , a2 , . . . , an are positive real numbers such that

1 1 1
a1 + a2 + · · · + an = + + ··· + = ns,
a1 a2 an

then
1 1 1 1 1 1
+ +· · ·+ ≥ + +· · ·+ .
a1 + n − 1 a2 + n − 1 an + n − 1 1 + ns − a1 1 + ns − a2 1 + ns − an

(Gabriel Dospinescu, 2004)

Solution. By the Cauchy-Schwarz inequality, we have


 
1 1 1
(a1 + a2 + · · · + an ) + + ··· + ≥ n2 ,
a1 a2 an

which leads to
s ≥ 1.
Applying Popoviciu’s inequality from Theorem 1 to the convex function

1
f (x) = , x > 0,
1 + (n − 1)x
we get

n
1 n(n − 2) n
1
+ ≥ (n − 1) .
i=1
1 + (n − 1)ai 1 + (n − 1)s i=1
1 + ns − ai
514 Vasile Cîrtoaje

Thus, it suffices to show that



n
1  n
1 n(n − 2)
(n − 1) ≥ + ,
i=1
ai + n − 1 i=1
1 + (n − 1)a i 1 + (n − 1)s

which is equivalent to

n
1 1
≥ .
i=1 (ai + n − 1) 1
+n−1 1 + (n − 1)s
ai

Write this inequality as


1 1 1 1
+ + ··· + ≥ ,
A1 A2 An 1 + (n − 1)s
where
   
1 1
Ai = (ai + n − 1) + n − 1 = (n − 1) ai + + n2 − 2n + 2.
ai ai
By the AM-HM inequality, we have

1 1 1 n2 n
+ + ··· + ≥ = .
A 1 A2 An A1 + A 2 + · · · + A n 2(n − 1)s + n2 − 2n + 2
Consequently, it is enough to prove the inequality
n 1
≥ ,
2(n − 1)s + n2 − 2n + 2 1 + (n − 1)s
which reduces to s ≥ 1. For n ≥ 3, the equality holds if and only if a1 = · · · = an = 1.

P 5.10. If a, b, c are nonnegative real numbers such that a + b + c = 3, then


 



a + b
b + c c + a
4 a + b + c + 15 ≤ 9 + + .
2 2 2

Solution. Applying Popoviciu’s inequality from Theorem 1 to the convex function


f (x) = − x, x ≥ 0,

we get the inequality


  



a + b
b+c c+a
a+ b+ c +3≤2 + + ,
2 2 2
On Popoviciu’s Inequality 515

which is weaker that the desired inequality. To prove the desired inequality, con-
sider the nontrivial case where at least two of a, b, c are positive, and write it in the
homogeneous form
"

#  "
 #
10 3(a + b + c) − a − b − c ≤ 9 2(a + b) − a − b ,

or, equivalently,


2
10 
 2  a− b



a− b ≤9 

.
3(a + b + c) + a + b + c 2(a + b) + a + b
This is true if
10 9



≤

,
3(a + b + c) + a+ b+ c 2(a + b) + a + b
which is equivalent to
 

3

9 − 10 2(a + b) ≥ a + b.
2

This inequality is true because




3

9 − 10 > 1, 2(a + b) ≥ a+ b.
2
The equality holds for a = b = c = 1.
Remark. We can rewrite the inequality as
  


9
a + b
b + c c+a
a+ b+ c−3≤ + + −3 .
4 2 2 2

As shown in P 1.46 from Volume 4, the best inequality of the form


 



a + b
b + c c + a
a+ b+ c−3≤ k + + −3 , k > 0,
2 2 2

is for

k = ( 3 − 1)( 3 + 2) ≈ 2.303.

P 5.11. If a, b, c, d, e are positive real numbers such that a bcd e = 1, then


1 1 1 1 1

+
+
+
+
≤ 1.
2+ 4 + 5a 2 + 4 + 5b 2 + 4 + 5c 2 + 4 + 5d 2 + 4 + 5e
516 Vasile Cîrtoaje

Solution. Use the contradiction method. Assume that


1 1 1 1 1

+
+
+
+
> 1,
2+ 4 + 5a 2 + 4 + 5b 2 + 4 + 5c 2 + 4 + 5d 2 + 4 + 5e
and show that
a bcd e < 1.
Using the substitution

1 5− x 1 5− y 1 5−z

= ,
= ,
= ,
2 + 4 + 5a 30 2 + 4 + 5b 30 2 + 4 + 5c 30

1 5−u 1 5−v

= ,
= ,
2 + 4 + 5d 30 2 + 4 + 5e 30
which involves
16x 16 y 16z 16u 16v
a= , b= , c= , d= , e= ,
(5 − x)2 (5 − y)2 (5 − z)2 (5 − u)2 (5 − v)2

and
0 < x < 5, 0 < y < 5, 0 < z < 5, 0 < u < 5, 0 < v < 5,
we need to show that
x + y +z+u+v <5
implies
 2  2  2  2  2
5− x 5− y 5−z 5−u 5−v
x yzuv < .
4 4 4 4 4

It is easy to see that if x increases, then the left side of this inequality increases,
while the right side decreases. Therefore, it suffices to show that

x + y +z+u+v =5

implies
 2  2  2  2  2
5− x 5− y 5−z 5−u 5−v
≥ x yzuv.
4 4 4 4 4

Popoviciu’s inequality from Theorem 1 applied to the convex function

f (x) = − ln x, x > 0,

gives
 4  4  4  4  4  x + y + z + u + v 15
5− x 5− y 5−z 5−u 5−v
≥ x yzuv ,
4 4 4 4 4 5
On Popoviciu’s Inequality 517

 4  4  4  4  4
5− x 5− y 5−z 5−u 5−v
≥ x yzuv.
4 4 4 4 4
Thus, it suffices to show that x yzuv ≤ 1. By the AM-GM inequality, we have
 x + y + z + u + v 5
x yzuv ≤ = 1.
5
The equality holds for a = b = c = d = e = 1.
Remark. In the same manner, we can prove the following generalization:
• If a1 , a2 , . . . , an (n ≥ 3) are positive real numbers such that

a1 a2 · · · an = 1,

then

n
1 1
 ≤ ,
i=1 n−1+ (n − 1)2 + 4nai 2
with equality for a1 = a2 = · · · = an = 1.

P 5.12. Let a1 , a2 , . . . , an (n ≥ 3) be positive real numbers such that a1 a2 · · · an = 1.


If
2n − 1
0<p≤ ,
(n − 1)2
then
1 1 1 n
 + + ≤ .
1 + pa1 1 + pa2 1 + pan 1+p

(Vasile Cîrtoaje and Gabriel Dospinescu, 2006)

Solution. We will apply the contradiction method. Assume that the reverse in-
equality holds, namely

1 1 1 n
 + + > ,
1 + pa1 1 + pa2 1 + pan 1+p

and show that


a1 a2 · · · an < 1.
Using the substitution

 1+p 
1 + pai = , 0 < xi < p + 1, i = 1, 2, . . . , n,
xi
518 Vasile Cîrtoaje

we need to show that x 1 + x 2 + · · · + x n > n yields


    
1+p 1+p 1+p
− 1 − 1 · · · − 1 < pn.
x 12 x 22 x n2

It suffices to prove that


x1 + x2 + · · · + x n = n
involves     
1+p 1+p 1+p
−1 − 1 ··· − 1 ≤ pn.
x 12 x 22 x n2
Denoting
 n
1 + p = q, 1<q≤ ,
n−1
we need to show that

(q2 − x 12 )(q2 − x 22 ) · · · (q2 − x n2 ) ≤ (q2 − 1)n (x 1 x 2 · · · x n )2 (*)

for all x i ∈ (0, q) satisfying x 1 + x 2 + · · · + x n = n. Applying Popoviciu’s inequality


to the convex function
 n 
f (x) = − ln − x , 0 < x < 1,
n−1
we get

(x 1 x 2 · · · x n )n−1 ≥ [n − (n − 1)x 1 ][n − (n − 1)x 2 ] · · · [n − (n − 1)x n ]. (**)

On the other hand, Jensen’s inequality applied to the convex function

n − (n − 1)x
f (x) = ln
q−x

yields
[n − (n − 1)x 1 ][n − (n − 1)x 2 ] · · · [n − (n − 1)x n ] 1
≥ .
(q − x 1 )(q − x 2 ) · · · (q − x n ) (q − 1)n
Multiplying this inequality and (**) gives

(q − x 1 )(q − x 2 ) · · · (q − x n )
(x 1 x 2 · · · x n )n−1 ≥ .
(q − 1)n

Therefore, in order to prove (*), we still have to show that

(x 1 x 2 · · · x n )n−3 (q + x 1 )(q + x 2 ) · · · (q + x n ) ≤ (q + 1)n .

This is true because, by the AM-GM inequality, we have


 x + x + · · · + x n
1 2 n
x1 x2 · · · x n ≤ =1
n
On Popoviciu’s Inequality 519

and
 x 1 + x 2 + · · · + x n n
(q + x 1 )(q + x 2 ) · · · (q + x n ) ≤ q + = (q + 1)n .
n
The equality occurs for a1 = a2 = · · · = an = 1.
2n − 1
Remark. For p = , we get the following inequality
(n − 1)2

n
1
 ≤ 1,
i=1 (n − 1)2 + (2n − 1)ai
which holds for all positive numbers a1 , a2 , . . . , an (n ≥ 3) satisfying a1 a2 · · · an = 1.

P 5.13. Let f be a convex function on a real interval . If a1 , a2 , . . . , an ∈ , then


n  ai − ai+1 
2 f (a1 ) + 2 f (a2 ) + · · · + 2 f (an ) + n(n − 2) f (a) ≥ n f a+ ,
i=1
n

where
1
a= (a1 + a2 + · · · + an ).
n
(Darij Grinberg and Vasile Cîrtoaje, 2006)
Solution. Let
1 
bi = aj, i = 1, 2, . . . , n.
n − 1 j =i
By Jensen’s inequality, we have
    
ai − ai+1  ai + (n − 1)bi+1 1 1
f a+ =f ≤ f (ai ) + 1 − f (bi+1 ),
n n n n
hence
n  ai − ai+1  
n n
n f a+ ≤ f (ai ) + (n − 1) f (bi ).
i=1
n i=1 i=1

Therefore, it suffices to show that



n 
n
2 f (a1 ) + 2 f (a2 ) + · · · + 2 f (an ) + n(n − 2) f (a) ≥ f (ai ) + (n − 1) f (bi ),
i=1 i=1

which is just Popoviciu’s inequality from Theorem 1:



n
f (a1 ) + f (a2 ) + · · · + f (an ) + n(n − 2) f (a) ≥ (n − 1) f (bi ).
i=1
520 Vasile Cîrtoaje

P 5.14. If a1 , a2 , . . . , an (n ≥ 3) are positive real numbers such that

a1 a2 · · · an = 1,

then  
a1 a2 an
2(a1n + a2n + · · · + ann ) + n(n − 2) ≥ n + + ··· + .
a2 a3 a1

Solution. Let x 1 , x 2 , . . . , x n be real numbers such that

x 1 + x 2 + · · · + x n = 0.

Applying the inequality in the preceding P 5.13 to the convex function

f (x) = e x , x ∈ ,

we get
x1 −x2 x 2 −x 3 x n −x 1 
2e x 1 + 2e x 2 + · · · + 2e x n + n(n − 2) ≥ n e n + e n + · · · + e n .

Using the substitution

x 1 = n ln a1 , x 2 = n ln a2 , . . . , x n = n ln an ,

we get the desired inequality. The equality occurs if and only if

a1 = a2 = · · · = an = 1.

P 5.15. If a1 , a2 , . . . , an (n ≥ 3) are positive real numbers such that

a1 + a2 + · · · + an = n,

then  a1 − a2   a2 − a3   a n − a1  2
1+ 1+ ··· 1 + ≥ (a1 a2 · · · an ) n .
n n n

Solution. Applying the inequality in P 5.13 to the convex function

f (x) = − ln x, x > 0,

we get
.  a1 − a2   a2 − a3   an − a1 /
n ln 1 + + ln 1 + + · · · + ln 1 + ≥
n n n
≥ 2(ln a1 + ln a2 + · · · + ln an ),
which is equivalent to the desired inequality. The equality occurs if and only if
a1 = a2 = · · · = an = 1.
On Popoviciu’s Inequality 521

P 5.16. If a1 , a2 , . . . , an (n ≥ 3) are positive real numbers such that

a1 + a2 + · · · + an = n,

then
 
2 1 1 1 1 1 1
+ + ··· + +n−2≥ + + ··· + .
n a1 a 2 an a1 − a2 a2 − a3 an − a1
1+ 1+ 1+
n n n

Solution. Apply the inequality in P 5.13 to the convex function

1
f (x) = , x > 0.
x
The equality occurs if and only if a1 = a2 = · · · = an = 1.

P 5.17. Let f be a convex function on (0, ∞). If a1 , a2 , . . . , an are positive real num-
bers, then      
1 1 1
f a1 + + f a2 + + · · · + f an + ≥
a2 a3 a1
     
1 1 1
≥ f a1 + + f a2 + + · · · + f an + .
a1 a2 an
(Vasile C., 2009)

Solution. For n = 2, the inequality is


       
1 1 1 1
f a1 + + f a2 + ≥ f a1 + + f a2 + .
a2 a1 a1 a2

Assume that a1 ≥ a2 . Since


       
1 1 1 1
a1 + + a2 + = a1 + + a2 +
a2 a1 a1 a2

and ) *
1 1 1
a1 + ≥ max a1 + , a2 + ,
a2 a1 a2
the inequality for n = 2 follows from Lemma below (which is a consequence of
Karamata’s inequality). To prove the original inequality, consider that

an+1 ≤ min{a1 , a2 , . . . , an },
522 Vasile Cîrtoaje

and use the induction method. Based on the induction hypothesis, we only need
to show that
       
1 1 1 1
f an + + f an+1 + ≥ f an + + f an+1 + .
an+1 a1 a1 an+1
This inequality follows also by Lemma below, since
       
1 1 1 1
an + + an+1 + = an + + an+1 +
an+1 a1 a1 an+1
and ) *
1 1 1
an + ≥ max an + , an+1 + .
an+1 a1 an+1
The equality occurs for a1 = a2 = · · · = an .
Lemma. Let f be a convex function on a real interval . If a, b, c, d ∈  such that
a + b = c + d, a ≥ max{c, d},
then
f (a) + f (b) ≥ f (c) + f (d).
Proof. Without loss of generality, assume that c ≥ d; then,
a ≥ c ≥ d ≥ b.
If a = c, then b = d, and the inequality is an equality. Consider now that a > c,
when
a > c ≥ d > b.
First proof. The desired inequality follows by adding the following Jensen’s inequal-
ities
(c − b) f (a) + (a − c) f (b) ≥ (a − b) f (c),
(a − c) f (a) + (c − b) f (b) ≥ (a − b) f (d).

Second Proof. Since c, d ∈ (b, a), there are p, q ∈ (0, 1) such that
c = pa + (1 − p)b, d = qa + (1 − q)b.
From a + b = c + d, we get
a + b = (p + q)a + (2 − p − q)b,
(a − b)(p + q − 1) = 0,
p + q = 1.
Using Jensen’s inequalities
f (c) ≤ p f (a) + (1 − p) f (b),
f (d) ≤ q f (a) + (1 − q) f (b),
we get
f (c) + f (d) ≤ (p + q) f (a) + (2 − p − q) f (b) = f (a) + f (b).
Appendix A

Glosar

1. AM-GM (ARITHMETIC MEAN-GEOMETRIC MEAN) INEQUALITY

If a1 , a2 , . . . , an are nonnegative real numbers, then



a1 + a2 + · · · + an ≥ n n a1 a2 · · · an ,

with equality if and only if a1 = a2 = · · · = an .

2. WEIGHTED AM-GM INEQUALITY

Let p1 , p2 , . . . , pn be positive real numbers satisfying

p1 + p2 + · · · + pn = 1.

If a1 , a2 , . . . , an are nonnegative real numbers, then


p p
p1 a1 + p2 a2 + · · · + pn an ≥ a1 1 a2 2 · · · anpn ,

with equality if and only if a1 = a2 = · · · = an .

3. AM-HM (ARITHMETIC MEAN-HARMONIC MEAN) INEQUALITY

If a1 , a2 , . . . , an are positive real numbers, then


 
1 1 1
(a1 + a2 + · · · + an ) + + ··· + ≥ n2 ,
a1 a2 an

with equality if and only if a1 = a2 = · · · = an .

523
524 Vasile Cîrtoaje

4. POWER MEAN INEQUALITY

The power mean of order k of positive real numbers a1 , a2 , . . . , an ,


⎧  1
⎪ a k +a k +···+ank k
⎨ 1 2n , k = 0
Mk =
,

⎩ n a1 a2 · · · an , k=0

is an increasing function with respect to k ∈ . For instant, M2 ≥ M1 ≥ M0 ≥ M−1


is equivalent to


a2 + a2 + · · · + a2
1 2 n a1 + a2 + · · · + an  n
≥ ≥ n a1 a2 · · · an ≥ .
n n 1 1 1
+ + ··· +
a1 a2 an

5. BERNOULLI’S INEQUALITY

For any real number x ≥ −1, we have


a) (1 + x) r ≥ 1 + r x for r ≥ 1 and r ≤ 0;
b) (1 + x) r ≤ 1 + r x for 0 ≤ r ≤ 1.
If a1 , a2 , . . . , an are real numbers such that either a1 , a2 , . . . , an ≥ 0 or

−1 ≤ a1 , a2 , . . . , an ≤ 0,

then
(1 + a1 )(1 + a2 ) · · · (1 + an ) ≥ 1 + a1 + a2 + · · · + an .

6. SCHUR’S INEQUALITY

For any nonnegative real numbers a, b, c and any positive number k, the inequality
holds
a k (a − b)(a − c) + b k (b − c)(b − a) + c k (c − a)(c − b) ≥ 0,
with equality for a = b = c, and for a = 0 and b = c (or any cyclic permutation).
For k = 1, we get the third degree Schur’s inequality, which can be rewritten as
follows
a3 + b3 + c 3 + 3a bc ≥ a b(a + b) + bc(b + c) + ca(c + a),
(a + b + c)3 + 9a bc ≥ 4(a + b + c)(a b + bc + ca),
9a bc
a2 + b2 + c 2 + ≥ 2(a b + bc + ca),
a+b+c
Glosar 525

(b − c)2 (b + c − a) + (c − a)2 (c + a − b) + (a − b)2 (a + b − c) ≥ 0.


For k = 2, we get the fourth degree Schur’s inequality, which holds for any real
numbers a, b, c, and can be rewritten as follows

a4 + b4 + c 4 + a bc(a + b + c) ≥ a b(a2 + b2 ) + bc(b2 + c 2 ) + ca(c 2 + a2 ),

a4 + b4 + c 4 − a2 b2 − b2 c 2 − c 2 a2 ≥ (a b + bc + ca)(a2 + b2 + c 2 − a b − bc − ca),
(b − c)2 (b + c − a)2 + (c − a)2 (c + a − b)2 + (a − b)2 (a + b − c)2 ≥ 0,
6a bc p ≥ (p2 − q)(4q − p2 ), p = a + b + c, q = a b + bc + ca.
A generalization of the fourth degree Schur’s inequality, which holds for any
real numbers a, b, c and any real number m, is the following (Vasile Cirtoaje, 2004)

(a − mb)(a − mc)(a − b)(a − c) ≥ 0,

with equality for a = b = c, and also for a/m = b = c (or any cyclic permutation).
This inequality is equivalent to
   
a4 + m(m + 2) a2 b2 + (1 − m2 )a bc a ≥ (m + 1) a b(a2 + b2 ),

(b − c)2 (b + c − a − ma)2 ≥ 0.

7. CAUCHY-SCHWARZ INEQUALITY
If a1 , a2 , . . . , an and b1 , b2 , . . . , bn are real numbers, then

(a12 + a22 + · · · + an2 )(b12 + b22 + · · · + bn2 ) ≥ (a1 b1 + a2 b2 + · · · + an bn )2 ,

with equality for


a1 a an
= 2 = ··· = .
b1 b2 bn
Notice that the equality conditions are also valid for ai = bi = 0, where 1 ≤ i ≤ n.

8. HÖLDER’S INEQUALITY
If x i j (i = 1, 2, · · · , m; j = 1, 2, · · · n) are nonnegative real numbers, then
 n   m
(
m  n 0(
 m


m
xi j xi j .
i=1 j=1 j=1 i=1
526 Vasile Cîrtoaje

9. CHEBYSHEV’S INEQUALITY
Let a1 ≥ a2 ≥ · · · ≥ an be real numbers.

a) If b1 ≥ b2 ≥ · · · bn , then
  

n 
n 
n
n ai bi ≥ ai bi ;
i=1 i=1 i=1

b) If b1 ≤ b2 ≤ · · · ≤ bn , then
  

n 
n 
n
n ai bi ≤ ai bi .
i=1 i=1 i=1

10. REARRANGEMENT INEQUALITY


(1) If (a1 , a2 , . . . , an ) and (b1 , b2 , . . . , bn ) are two increasing (or decreasing) real
sequences, and (i1 , i2 , · · · , in ) is an arbitrary permutation of (1, 2, · · · , n), then

a1 b1 + a2 b2 + · · · + an bn ≥ a1 bi1 + a2 bi2 + · · · + an bin

and

n(a1 b1 + a2 b2 + · · · + an bn ) ≥ (a1 + a2 + · · · + an )(b1 + b2 + · · · + bn ).

(2) If (a1 , a2 , . . . , an ) is decreasing and (b1 , b2 , . . . , bn ) is increasing, then

a1 b1 + a2 b2 + · · · + an bn ≤ a1 bi1 + a2 bi2 + · · · + an bin

and

n(a1 b1 + a2 b2 + · · · + an bn ) ≤ (a1 + a2 + · · · + an )(b1 + b2 + · · · + bn ).

(3) Let b1 , b2 , . . . , bn ) and (c1 , c2 , . . . , cn ) be two real sequences such that

b1 + · · · + bi ≥ c1 + · · · + ci , i = 1, 2, · · · , n.

If a1 ≥ a2 ≥ · · · ≥ an ≥ 0, then

a1 b1 + a2 b2 + · · · + an bn ≥ a1 c1 + a2 c2 + · · · + an cn .

Notice that all these inequalities follow immediately from the identity
 i 
 n 
n  
i
ai (bi − ci ) = (ai − ai+1 ) bj − cj , an+1 = 0.
i=1 i=1 j=1 j=1
Glosar 527

11. SQUARE PRODUCT INEQUALITY

Let a, b, c be real numbers, and let

p = a + b + c, q = a b + bc + ca, r = a bc,
 
s = p2 − 3q = a2 + b2 + c 2 − a b − bc − ca.
From the identity

(a − b)2 (b − c)2 (c − a)2 = −27r 2 + 2(9pq − 2p3 )r + p2 q2 − 4q3 ,

it follows that
 
−2p3 + 9pq − 2(p2 − 3q) p2 − 3q −2p3 + 9pq + 2(p2 − 3q) p2 − 3q
≤r≤ ,
27 27
which is equivalent to

p3 − 3ps2 − 2s3 p3 − 3ps2 + 2s3


≤r≤ .
27 27
Therefore, for constant p and q, the product r is minimum and maximum when
two of a, b, c are equal.
On the other hand, the identity

27(a − b)2 (b − c)2 (c − a)2 = 4(p2 − 3q)3 − (2p3 − 9pq + 27r)2 ,

leads to the inequality

27(a − b)2 (b − c)2 (c − a)2 ≤ 4(p2 − 3q)3 ,

with equality for 2p3 − 9pq + 27r = 0.

12. KARAMATA’S MAJORIZATION INEQUALITY


Let f be a convex function on a real interval . If a decreasingly ordered sequence

A = (a1 , a2 , . . . , an ), ai ∈ ,

majorizes a decreasingly ordered sequence

B = (b1 , b2 , . . . , bn ), bi ∈ ,

then
f (a1 ) + f (a2 ) + · · · + f (an ) ≥ f (b1 ) + f (b2 ) + · · · + f (bn ).
We say that a sequence A = (a1 , a2 , . . . , an ) with a1 ≥ a2 ≥ · · · ≥ an majorizes a
sequence B = (b1 , b2 , . . . , bn ) with b1 ≥ b2 ≥ · · · ≥ bn , and write it as

A  B,
528 Vasile Cîrtoaje

if
a1 ≥ b1 ,
a1 + a2 ≥ b1 + b2 ,
·····················
a1 + a2 + · · · + an−1 ≥ b1 + b2 + · · · + bn−1 ,
a1 + a2 + · · · + an = b1 + b2 + · · · + bn .

13. CONVEX FUNCTIONS


A function f defined on a real interval  is said to be convex if

f (αx + β y) ≤ α f (x) + β f ( y)

for all x, y ∈  and any α, β ≥ 0 with α + β = 1. If the inequality is reversed, then


f is said to be concave.
If f is differentiable on , then f is (strictly) convex if and only if the derivative f
is (strictly) increasing. If f ≥ 0 on , then f is convex on . Also, if f ≥ 0 on (a,
b) and f is continuous on [a, b], then f is convex on [a, b].

Jensen’s inequality. Let p1 , p2 , . . . , pn be positive real numbers. If f is a convex


function on a real interval , then for any a1 , a2 , . . . , an ∈ , the inequality holds
 
p1 f (a1 ) + p2 f (a2 ) + · · · + pn f (an ) p1 a1 + p2 a2 + · · · + pn an
≥f .
p1 + p2 + · · · + pn p1 + p2 + · · · + pn
For p1 = p2 = · · · = pn , Jensen’s inequality becomes
a + a + ··· + a 
1 2 n
f (a1 ) + f (a2 ) + · · · + f (an ) ≥ n f .
n

Right Half Convex Function Theorem (Vasile Cîrtoaje, 2004). Let f be a real
function defined on an interval  and convex on ≥s , where s ∈ int(). The inequality
a + a + ··· + a 
1 2 n
f (a1 ) + f (a2 ) + · · · + f (an ) ≥ n f
n
holds for all a1 , a2 , . . . , an ∈  satisfying

a1 + a2 + · · · + an = ns

if and only if
f (x) + (n − 1) f ( y) ≥ n f (s)
for all x, y ∈  such that x ≤ s ≤ y and x + (n − 1) y = ns.

Left Half Convex Function Theorem (Vasile Cîrtoaje, 2004). Let f be a real function
defined on an interval  and convex on ≤s , where s ∈ int(). The inequality
a + a + ··· + a 
1 2 n
f (a1 ) + f (a2 ) + · · · + f (an ) ≥ n f
n
Glosar 529

holds for all a1 , a2 , . . . , an ∈  satisfying

a1 + a2 + · · · + an = ns

if and only if
f (x) + (n − 1) f ( y) ≥ n f (s)
for all x, y ∈  such that x ≥ s ≥ y and x + (n − 1) y = ns.

Right Partially Convex Function Theorem (Vasile Cîrtoaje, 2012). Let f be a real
function defined on an interval  and convex on [s, s0 ], where s, s0 ∈ , s < s0 . In
addition, f is decreasing on ≤s0 and f (u) ≥ f (s0 ) for u ∈ . The inequality
a + a + ··· + a 
1 2 n
f (a1 ) + f (a2 ) + · · · + f (an ) ≥ n f
n
holds for all a1 , a2 , . . . , an ∈  satisfying

a1 + a2 + · · · + an = ns

if and only if
f (x) + (n − 1) f ( y) ≥ n f (s)
for all x, y ∈  such that x ≤ s ≤ y and x + (n − 1) y = ns.

Left Partially Convex Function Theorem (Vasile Cîrtoaje, 2012). Let f be a real
function defined on an interval  and convex on [s0 , s], where s0 , s ∈ , s0 < s. In
addition, f is increasing on ≥s0 and f (u) ≥ f (s0 ) for u ∈ . The inequality
a + a + ··· + a 
1 2 n
f (a1 ) + f (a2 ) + · · · + f (an ) ≥ n f
n
holds for all a1 , a2 , . . . , an ∈  satisfying

a1 + a2 + · · · + an = ns

if and only if
f (x) + (n − 1) f ( y) ≥ n f (s)
for all x, y ∈  such that x ≥ s ≥ y and x + (n − 1) y = ns.

Equal Variables Theorem for Nonnegative Variables (Vasile Cirtoaje, 2005). Let
a1 , a2 , . . . , an (n ≥ 3) be fixed nonnegative real numbers, and let

0 ≤ x1 ≤ x2 ≤ · · · ≤ x n

such that

x 1 + x 2 + · · · + x n = a1 + a2 + · · · + an , x 1k + x 2k + · · · + x nk = a1k + a2k + · · · + ank ,


530 Vasile Cîrtoaje

where k is a real number (k = 1); for k = 0, assume that

x 1 x 2 · · · x n = a1 a2 · · · an .

Let f be a real-valued function, continuous on [0, ∞) and differentiable on (0, ∞),


such that the associated function
1 
g(x) = f x k−1

is strictly convex on (0, ∞). Then, the sum

Sn = f (x 1 ) + f (x 2 ) + · · · + f (x n )

is maximum for
x 1 = x 2 = · · · = x n−1 ≤ x n ,
and is minimum for
0 < x1 ≤ x2 = x3 = · · · = x n
or
0 = x 1 = · · · = x j ≤ x j+1 ≤ x j+2 = · · · = x n , j ∈ {1, 2, . . . , n − 1}.

Equal Variables Theorem for Real Variables (Vasile Cirtoaje, 2010). Let a1 , a2 , . . . , an
(n ≥ 3) be fixed real numbers, and let

0 ≤ x1 ≤ x2 ≤ · · · ≤ x n
such that
x 1 + x 2 + · · · + x n = a1 + a2 + · · · + an , x 1k + x 2k + · · · + x nk = a1k + a2k + · · · + ank ,

where k is an even positive integer. If f is a differentiable function on  such that the


associated function g :  →  defined by


 k−1
g(x) = f x

is strictly convex on , then the sum

Sn = f (x 1 ) + f (x 2 ) + · · · + f (x n )

is minimum for x 2 = x 3 = · · · = x n , and is maximum for x 1 = x 2 = · · · = x n−1 .

Best Upper Bound of Jensen’s Difference Theorem (Vasile Cirtoaje, 1990). Let
p1 , p2 , . . . , pn (n ≥ 3) be fixed positive real numbers, and let f be a convex function
on  = [a, b]. If a1 , a2 , . . . , an ∈ , then Jensen’s difference
 
p1 f (a1 ) + p2 f (a2 ) + · · · + pn f (an ) p1 a1 + p2 a2 + · · · + pn an
−f
p1 + p2 + · · · + pn p1 + p2 + · · · + pn

is maximum when all ai ∈ {a, b}.


Glosar 531

14. ARITHMETIC MEAN METHOD

Arithmetic Mean Theorem. Let

F (a1 , a2 , . . . , an ) :  → ,  ∈ n ,
be a symmetric continuous function satisfying
a + a a1 + an 
1 n
F (a1 , a2 , . . . , an−1 , an ) ≥ F , a2 , . . . , an−1 ,
2 2
for all a1 , a2 , . . . , an ∈  such that a1 ≤ a2 ≤ · · · ≤ an or a1 ≥ a2 ≥ · · · ≥ an . Then,
for all a1 , a2 , . . . , an ∈ , the following inequality holds:
a1 + a2 + · · · + an
F (a1 , a2 , . . . , an ) ≥ F (a, a, . . . , a), a= .
n

Arithmetic Mean Corollary (Vasile Cîrtoaje, 2005). Let

F (a1 , a2 , . . . , an ) :  → ,  ∈ n ,
be a symmetric continuous function satisfying
a + a a1 + an−1 
1 n−1
F (a1 , a2 , . . . , an−2 , an−1 , an ) ≥ F , a2 , . . . , an−2 , , an
2 2
for all a1 , a2 , . . . , an ∈  such that a1 ≤ a2 ≤ · · · ≤ an (or a1 ≥ a2 ≥ · · · ≥ an ). Then,
for all a1 , a2 , . . . , an ∈  such that a1 ≤ a2 ≤ · · · ≤ an (or a1 ≥ a2 ≥ · · · ≥ an ), the
following inequality holds:
a1 + a2 + · · · + an−1
F (a1 , a2 , . . . , an−1 , an ) ≥ F (t, t, . . . , t, an ), t= .
n−1

15. ARITHMETIC COMPENSATION METHOD

Arithmetic Compensation Theorem (Vasile Cîrtoaje, 2005). Let s > 0 and let F be
a symmetric continuous function on the compact set in n

S = {(a1 , a2 , . . . , an ) : a1 + a2 + · · · + an = s, ai ≥ 0, i = 1, 2, . . . , n}.

If
F (a1 , a2 , a3 , . . . , an ) ≥
 a + a a + a  
1 2 1 2
≥ min F , , a3 , . . . , an , F (0, a1 + a2 , a3 , . . . , an ) (*)
2 2
for all (a1 , a2 , . . . , an ) ∈ S, then
 s s
F (a1 , . . . , an−k , an−k+1 , . . . , an ) ≥ min F 0, . . . , 0, , . . . ,
1≤k≤n k k
for all (a1 , a2 , . . . , an ) ∈ S.
532 Vasile Cîrtoaje

Arithmetic Compensation Corollary (Vasile Cîrtoaje, 2005). Let s > 0 and let F be
a symmetric continuous function on the compact set in n

S = {(a1 , a2 , . . . , an ) : a1 + a2 + · · · + an = s, ai ≥ 0, i = 1, 2, . . . , n}.

If
F (a1 , a2 , a3 , . . . , an ) ≥ F (0, a1 + a2 , a3 , . . . , an )
for all (a1 , a2 , . . . , an ) ∈ S satisfying
a + a a + a 
1 2 1 2
F (a1 , a2 , a3 , . . . , an ) < F , , a3 , . . . , a n , a1 = a2 ,
2 2
then F (a1 , a2 , . . . , an ) is minimum when n − k of the variables a1 , a2 , . . . , an are zero
s
and the other k variables are equal to , where k ∈ {1, 2, . . . , n}.
k

16. pqr METHOD


Theorem 1. If a ≥ b ≥ c are real numbers such that

a + b + c = p, a b + bc + ca = q,

where p and q are fixed real numbers satisfying p2 ≥ 3q, then the product r = a bc is
minimum for a = b, and maximum for b = c.

Theorem 2. If a ≥ b ≥ c are real nonnegative numbers such that

a + b + c = p, a b + bc + ca = q,

where p and q are fixed real numbers satisfying p2 ≥ 3q, then the product r = a bc is
minimum for a = b or c = 0, and maximum for b = c.

Theorem 3. If a ≥ b ≥ c are real nonnegative numbers such that

a + b + c = p, a bc = r,

where p and r are fixed real numbers satisfying p3 ≥ 27r, then q = a b + bc + ca is


minimum for b = c, and maximum for a = b.

pqr Theorem. Let a, b, c be real numbers and

p = a + b + c, q = a b + bc + ca, r = a bc.

For any real β, the following inequality holds




1  3/2
27β r + |(a − b)(b − c)(c − a)| ≤ 9β pq − 2β p + 2 + β 2 p2 − 3q
3
,
27
Glosar 533

with equality for




1  
2β(p2 − 3q)3/2 = + β 2 2p3 − 9pq + 27r .
27

17. SYMMETRIC INEQUALITIES OF DEGREE THREE, FOUR, FIVE AND SIX

Theorem 1. Let f n (a, b, c) be a symmetric homogeneous polynomial of degree n.


(a) The inequality f4 (a, b, c) ≥ 0 holds for all real numbers a, b, c if and only if
f4 (a, 1, 1) ≥ 0 for all real a;
(b) For n ∈ {3, 4, 5}, the inequality f n (a, b, c) ≥ 0 holds for all a, b, c ≥ 0 if and
only if f n (a, 1, 1) ≥ 0 and f n (0, b, c) ≥ 0 for all a, b, c ≥ 0.

A symmetric and homogeneous polynomial of degree six can be written in the


form
f6 (a, b, c) = Ar 2 + g1 (p, q)r + g2 (p, q),
where
p = a + b + c, q = a b + bc + ca, r = a bc,
A is the highest coefficient of f6 (a, b, c), and g1 (p, q) and g2 (p, q) are polynomial
functions.

Theorem 2 (Vasile Cîrtoaje, 2008). Let f6 (a, b, c) be a symmetric homogeneous


polynomial of degree six which has the highest coefficient A ≤ 0. The inequality
f6 (a, b, c) ≥ 0 holds for all real numbers a, b, c if and only if

f6 (a, 1, 1) ≥ 0

for all real a.

Theorem 3 (Vasile Cîrtoaje, 2008). Let f6 (a, b, c) be a sixth degree symmetric homo-
geneous polynomial having the highest coefficient A ≤ 0. The inequality f6 (a, b, c) ≥
0 holds for all nonnegative real numbers a, b, c if and only if f6 (a, 1, 1) ≥ 0 and
f6 (0, b, c) ≥ 0 for all nonnegative real numbers a, b, c.

∗∗∗∗∗

Consider the inequality


f6 (a, b, c) ≥ 0,
where a, b, c are real numbers and f6 (a, b, c) is a symmetric homogeneous poly-
nomial of degree six with the highest coefficient A > 0. The highest coefficient
cancellation method for proving such an inequality uses the above Theorem 2 and
the following three ideas:
534 Vasile Cîrtoaje

1) finding a nonnegative symmetric homogeneous function f¯6 (a, b, c) of the form


 2
q2
f¯6 (a, b, c) = r + A1 pq + A2 p3 + A3 , (A.1)
p

where A1 , A2 , A3 are real numbers chosen such that

f6 (a, b, c) ≥ A f¯6 (a, b, c) ≥ 0

for all real numbers a, b, c;


2) seeing that the difference f6 (a, b, c) − A f¯6 (a, b, c) has the highest coefficient
equal to zero, therefore the inequality

f6 (a, b, c) ≥ A f¯6 (a, b, c)

holds if and only if it holds for b = c = 1 (see Theorem 2);


3) choosing a suitable real number

ξ ∈ (−∞, 0) ∪ (3, ∞)

and treating successively the cases p2 < ξq and p2 ≥ ξq.

∗∗∗∗∗

Consider the inequality


f6 (a, b, c) ≥ 0,
where a, b, c are nonnegative numbers and f6 (a, b, c) is a symmetric homogeneous
polynomial of degree six with the highest coefficient A > 0. The highest coefficient
cancellation method for proving such an inequality uses the above Theorem 3 and
the following three ideas:
1) finding a nonnegative symmetric homogeneous function f¯6 (a, b, c) of the
form  2
q2
f¯6 (a, b, c) = r + A1 pq + A2 p3 + A3 , (A.2)
p
where A1 , A2 , A3 are real numbers chosen such that

f6 (a, b, c) ≥ A f¯6 (a, b, c) ≥ 0

for all nonnegative real numbers a, b, c;


2) seeing that the the difference f6 (a, b, c) − A f¯6 (a, b, c) has the highest coeffi-
cient equal to zero, therefore the inequality

f6 (a, b, c) ≥ A f¯6 (a, b, c)


Glosar 535

holds for all nonnegative real numbers a, b, c if and only if it holds for b = c = 1
and for a = 0 (see Theorem 3);
3) treating successively the cases p2 < 4q and p2 ≥ 4q.

18. POPOVICIU’S INEQUALITY


If f is a convex function on a real interval  and a1 , a2 , . . . , an ∈ , then
a + a + ··· + a 
1 2 n
f (a1 ) + f (a2 ) + · · · + f (an ) + n(n − 2) f ≥
n
≥ (n − 1)[ f (b1 ) + f (b2 ) + · · · + f (bn )],
where
1 
bi = aj, i = 1, 2, · · · , n.
n − 1 j =i

In the same conditions, the following similar inequality holds:


n a + a + ··· + a 
1 2 n
f (a1 ) + f (a2 ) + · · · + f (an ) + f ≥
n−2 n
  
2 ai + a j
≥ f .
n − 2 1≤i< j≤n 2
536 Vasile Cîrtoaje
Appendix B

Index

A
AC-Corollary: 6, 20-27, 30-32, 34-38, 42-44, 46, 47, 51, 53-55, 58, 87-90, 92-96.
adding: 235, 305, 508, 522.
AM-Corollary: 3, 4, 18-22, 35, 36, 43-45, 49-52, 77, 79, 81, 83-86.
AM1-Corollary: 4, 62, 63, 66-71, 74, 75.
AM-GM inequality: 23, 24, 26, 28, 30, 31, 50, 96, 112, 183, 411, 430, 508, 510,
517, 518.
AM-Theorem: 3, 4, 17.
any permutation: 32, 33, 39, 40, 55, 59, 61, 62, 65, 73, 74, 87, 90, 92.
assume: 17, 41, 43, 46, 47, 50, 77, 79, 80, 82, 84-86, 88, 89, 91, 93, 96, 112,
126, 132, 134, 135, 137, 138, 141, 143, 145, 149, 150, 152, 154, 157, 159, 161,
163, 165, 180, 183, 292, 392, 499, 512, 516, 517, 521, 522, 530.
assumption: 2, 51, 53, 502.

C
cancellation: 186-188, 359, 361.
Cauchy-Schwarz inequality: 240, 270, 327, 354, 511, 513, 525.
Chebyshev’s inequality: 526.
claim: 38, 53, 251.
concave: 198, 528.
consequence: 3, 34, 37, 58, 74, 168, 170-172, 254, 341, 500, 521.
consequently: 49, 514.
contradiction: 512, 513, 516, 517.
convex: 499-502, 508-511, 513, 514, 516, 518-522, 527, 528-530, 534.

D
decreasing: 49, 67, 255, 256, 276, 278, 279, 297, 298, 311, 500, 516, 526, 527,
529.

537
538 Vasile Cîrtoaje

decreasing: 49, 67, 255, 256, 276, 278, 279, 297, 298, 311, 500, 516, 526, 527,
529.
degree eight: 197, 198, 356, 368, 482, 483, 486-488, 490, 492, 494, 497.
degree seven: 368, 369.
degree six: 185-188, 192, 193, 359, 361, 364, 365, 367, 482, 483, 486, 487, 533,
534.

G
generalization: 25, 29 32, 139, 175, 217, 221, 222, 233, 241, 248, 253, 262, 263,
279, 288, 292, 294, 314, 317, 327, 329, 334, 335, 379, 381-384, 386, 388, 435,
438, 441, 447, 452, 464, 468, 471, 479, 481, 482, 485, 487, 491, 493, 496, 498,
501, 517, 525.

H
highest coefficient: 186-188, 192, 193, 198, 213-220, 221, 223-234, 236, 237,
239, 242, 246, 250, 254, 257-259, 261, 263-265, 267, 269-271, 273, 275, 277,
280, 282-285, 287, 291, 294, 295, 301, 303, 307, 313, 316, 317, 319, 321-323,
326-328, 331-333, 335-342, 344, 359-362, 364, 365, 367, 368, 379-385, 387, 389,
390, 393-396, 398, 419, 420, 431, 434, 436, 439, 443, 445, 450, 454, 455, 457,
459, 461, 463, 465, 467, 469-474, 477-479, 482, 483, 486, 487, 533-534.
highest polynomial: 197, 198, 346-348, 350, 351, 353, 354, 356, 368, 369, 469,
470, 472, 474, 477-479, 482, 483, 486-488, 490, 492, 494, 497.
homogeneity (homogeneous): 17, 39, 41, 43, 45, 48, 50, 52, 69, 75, 116-118,
127, 129, 131, 132, 134, 135, 137-139, 141, 143, 146, 149, 151, 152, 154, 157,
159, 161, 163, 166, 171, 173, 175, 177, 180, 183, 185, 187-189, 192, 193, 197,
198, 340, 359-361, 364, 365, 367-369, 488, 507, 515, 533, 534.
hypothesis: 1, 6, 21, 23, 24, 26, 28, 30, 31, 64, 72, 113, 138, 193, 500, 510, 513,
522.

I
identity: 48, 194, 195, 227, 231, 234, 235, 253, 256, 300, 305, 306, 312, 366,
398, 401,485, 526, 527.
increasing: 48, 49, 255, 256, 278, 298, 524.
induction: 5, 500, 522.
interval: 2, 499, 500, 501, 505, 519, 522, 527-529, 534.

J
Jensen’s inequality: 500, 501, 518, 519, 522.

K
Karamata’s inequality: 500, 501, 521, 527.
known: 3, 174, 176, 178, 472, 507.
Index 539

L
left inequality: 109, 281, 315, 338, 381, 516.
lemma: 2, 3,6, 100, 101, 185, 187, 198, 359, 360, 521, 522.
loss: 77, 79, 522.

M
maximum: 98-100, 109, 185, 198, 359, 527, 530, 532.
minimum: 5-7, 38, 99, 100, 109-111, 185, 198, 359, 502, 527, 530, 532.
more general: 56, 119, 123, 127, 128, 315, 350, 351.
multiplying: 72, 174, 403, 419, 421, 484, 518.

N
nontrivial: 48, 88, 132, 134, 135, 137, 138, 141, 143, 388, 486, 489, 491, 493,
515.
notation: 437.

O
obvious: 6, 19, 21, 42, 44-46, 78, 80, 81, 83, 85-88, 111, 115, 122, 125, 129,
177, 179, 431.

P
Popoviciu’s inequality: 499-501, 507, 509-511, 513, 514, 516, 518, 519, 535.
power mean: 5244.
proportional to: 118-121, 123-125, 127, 130, 133, 135, 136, 138, 142, 144, 146,
150, 152, 153, 155, 158, 160, 162, 164, 167, 179, 181, 260, 263.

R
real function: 528, 529.
remark: 25, 28, 29, 32, 39, 41, 42, 111, 115, 122, 125, 129, 139, 141, 143, 145,
147, 148, 150, 153, 156, 158, 161, 163, 165, 168, 169, 171, 172, 175, 179, 187,
194, 195, 197-199, 227, 231, 234, 236, 238, 243, 260, 263, 275, 277, 178, 316,
320, 329, 355-368, 507, 511, 515, 517, 519.
replacing: 1, 2, 132, 134, 135, 137, 138, 141, 143, 145, 149, 150, 152, 154, 157,
159, 161, 163, 165, 180, 183, 315, 368, 398, 513.
right inequality: 516, 528, 529.
root: 118-125, 127, 130, 133, 135, 136, 138, 139, 142, 144, 146, 148, 150,
152-155, 158, 160, 162, 164, 167, 175, 179, 181, 182, 184, 260, 263.

S
same manner: 25, 29, 32, 517.
same highest: 187, 213-216, 218, 220, 221, 223-233, 236, 237, 239, 246, 258,
261, 264-267, 269-271, 273, 280, 282, 283, 285, 287, 291, 294, 301, 307, 313,
540 Vasile Cîrtoaje

316, 317, 319, 321-323, 331-333, 335, 336, 339, 346, 347, 350, 351, 353, 354,
356, 379, 380, 382, 385, 387, 389, 390, 394, 396, 398, 431, 439, 443, 450, 457,
459, 461, 463, 465, 467, 469, 470, 472, 474, 477, 488, 490, 492, 494, 497.
Schur’s inequality: 39, 524, 525.
sharper: 111, 115, 122, 125, 129, 156, 168-171.
squaring: 76,167.
substitution (substituting): 18, 36, 48, 59, 119, 120, 123, 124, 126, 128, 146,
149, 151, 155, 157, 160, 162, 164, 166, 183, 191, 235, 253, 256, 300, 304, 306,
312, 384, 386, 388, 398, 401, 434, 440, 446, 448, 451, 454, 456, 458, 459, 462,
464, 466, 468, 473, 507, 508, 512, 516, 517, 520.
summing: 111, 115, 122, 125, 129, 499.

U
unchanged: 132, 134, 135, 137, 138, 141, 143, 145, 149, 150, 152, 154, 157,
159, 161, 163, 165, 180, 183, 398.

V
vicinity: 196, 222, 257, 281, 314, 349, 354
Bibliography

[1] Andreescu T., Cîrtoaje V., Dospinescu G., Lascu M., Old and New Inequalities,
GIL Publishing House, 2004.

[2] Bin X., Boreico I., Can V.Q.B., Bulj A., Lascu M., Opympiad Inequalities, GIL
Publishing House, 2015.

[3] Bin X., Boreico I., Can V.Q.B., Cîrtoaje V., Lascu M., An Introduction to Inequal-
ities, GIL Publishing House, 2015.

[4] Can V.Q.B., Pohoaţă C., Old and New Inequalities, GIL Publishing House, 2008.

[5] Can V.Q.B., Anh T.Q., Su Dung Phuong Phap Cauchy-Schwarz De Chung Minh
Bat Dang Thuc, Nha Xuat Ban Dai Hoc Su Pham, 2010.

[6] Cîrtoaje V., Two Generalizations of Popoviciu’s Inequality, Crux Mathematico-


rum, Issue 5, 2005.

[7] Cîrtoaje V., A Generalization of Jensen’s Inequality, Gazeta Matematica-A, 2,


2005.

[8] Cîrtoaje V., Algebraic Inequalities-Old and New Methods, GIL Publishing House,
2006.

[9] Cîrtoaje V., Arithmetic Compensation Method, Mathematical Reflections, 2,


2006, 1 - 5.

[10] Cîrtoaje V., The Equal Variable Method, Journal of Inequalities In Pure and
Applied Mathematics, Volume 8, Issue 1, 2007.

[11] Cîrtoaje V., On Jensen Type Inequalities with Ordered Variables, Journal of In-
equalities In Pure and Applied Mathematics, Volume 9, Issue 1, 2008.

[12] Cîrtoaje V., The Proof of Three Open Inequalities, Crux Mathematicorum, Vol-
ume 34, Issue 4, 2008.

[13] Cîrtoaje V., Can V.Q.B., Anh T.Q., Inequalities with Beautiful Solutions, GIL
Publishing House, 2009.

541
542 Vasile Cîrtoaje

[14] Cîrtoaje V., On the Cyclic Homogeneous Polynomial Inequalities of Degree Four,
Journal of Inequalities in Pure and Applied Mathematics, Volume 10, Issue 3,
2009.

[15] Cîrtoaje V., The Best Upper Bound for Jensen’s Inequality, Australian Journal of
Mathematical Analysis and Aplications, Volume 7, Issue 2, Art. 22, 2011.

[16] Cîrtoaje V., Baiesu A., An Extension of Jensen’s Discrete Inequality to Half Convex
Functions, Journal of Inequalities and Applications, Volume 2011.

[17] Cîrtoaje V., On the Arithmetic Compensation Methods, International Journal of


Pure and Applied Mathematics, Volume 80, No. 3, 2012.

[18] Cîrtoaje V., Can V.Q.B., On Some Cyclic Homogeneous Polynomial Inequalities
of Degree Four in Real Variables, International Journal of Pure and Applied
Mathematics, Volume 80, No. 3, 2012.

[19] Cîrtoaje V., The Best Lower Bound for Jensen’s Inequality with three fixed ordered
variables Journal, Banach Journal of Mathematical Analysis, Volume 7, Issue
1, 2013.

[20] Cîrtoaje V., An Extension of Jensen’s Discrete Inequality to Partially Convex Func-
tions, Journal of Inequalities and Applications, Volume 2013:54.

[21] Cîrtoaje V., A Strong Method for Symmetric Homogeneous Polynomial Inequali-
ties of Degree Six in Nonnegative Real Variables, British Journal of Mathemati-
cal and Computers Science, 4(5), 2014.

[22] Cîrtoaje V., On the Equal Variables Method Applied to Real Variables, Creative
Mathematics and Informatics, no. 2, 2015.

[23] Cîrtoaje V., Three Extensions of HCF and PCF Theorems, Advances in Inequali-
ties and Applications, no. 2, 2016.

[24] Cîrtoaje V., Extensions of Jensen’s Discrete Inequality with Ordered Variables
to Half and Partially Convex Functions, Journal of Inequalities and Special
Functions, Volume 8, Issue 3, 2017.

[25] Cîrtoaje V., Mathematical Inequalities - Volume 1, Symmetric Polynomial In-


equalities, Lambert Academic Publishing, 2018.

[26] Cîrtoaje V., Mathematical Inequalities - Volume 2, Symmetric Rational and Non-
rational Inequalities, Lambert Academic Publishing, 2018.

[27] Cîrtoaje V., Mathematical Inequalities - Volume 3, Cyclic and Noncyclic Inequal-
ities, Lambert Academic Publishing, 2018.

[28] Cîrtoaje V., Mathematical Inequalities - Volume 4, Extensions and Refinements


of Jensen’s Inequalities, Lambert Academic Publishing, 2018.
Index 543

[29] Cvetkovski, Inequalities: Theorems, Techniques and Selected Problems,


Springer-Verlag Berlin Heidelberg, 2012.

[30] Hung P.K., Secrets in Inequalities, Volume 1: Basic Inequalities, GIL Publishing
House, 2007.

[31] Hung P.K., Secrets in Inequalities, Volume 2: Advanced Inequalities, GIL Pub-
lishing House, 2008.

[32] Littlewood G.H., Polya J.E., Inequalities, Cambridge University Press, 1967.

[33] Mitrinovic̆ D.S., Pecaric̆ J.E., Fink A.M., Classical and New Inequalities in Anal-
ysis, Kluwer, 1993.
View publication stats

You might also like